Adult health and PEDS nclex (copy)

Lakukan tugas rumah & ujian kamu dengan baik sekarang menggunakan Quizwiz!

A nurse has reinforced dietary instructions to a client with a diagnosis of hypoparathyroidism. The nurse instructs the client to include which of the following items in the diet?

*1. Vegetables* 2. Meat 3. Fish 4. Cereals *rationale* The client with hypoparathyroidism is instructed to follow a calcium-rich diet and to restrict the amount of phosphorus in the diet. The client should limit meat, poultry, fish, eggs, cheese, and cereals. Vegetables are allowed in the diet.

A nurse is caring for a client diagnosed with Paget's disease. The nurse plans care, knowing that this condition usually affects which bones?

Axial skeleton including the vertebrae

A nurse is checking the skin on a client who is immobile and notes the presence of a partial-thickness skin loss of the upper layer of the skin in the sacral area. The nurse documents this finding as a pressure ulcer of which stage? 1. Stage 1 2. Stage 2 3. Stage 3 4. Stage 4

2. Stage 2

A client has sustained full-thickness circumferential burns of the trunk. Which of the following should be the priority concern of the nurse? 1. Urine output of 30 to 50 mL/hr 2. Client's increased risk for infection 3. Adequate management of burn pain 4. Client's ability to adequately ventilate

4. Client's ability to adequately ventilate

A client arrives at the health care clinic and tells the nurse that he was just bitten by a tick and would like to be tested for Lyme disease. Which nursing action is appropriate? 1. Tell the client that a blood test is needed immediately. 2. Inform the client that there is no test available for Lyme disease. 3. Tell the client that testing is not necessary unless arthralgia develops. 4. Inform the client that he will need to return in 4 to 6 weeks to be tested, because testing before this time is not reliable.

4. Inform the client that he will need to return in 4 to 6 weeks to be tested, because testing before this time is not reliable.

Which of the following would cause a false-positive result on a pregnancy test? A. The test was performed less than 10 days after an abortion B. The test was performed too early or too late in the pregnancy C. The urine sample was stored too long at room temperature D. A spontaneous abortion or a missed abortion is impending

A - Explanation A false-positive reaction can occur if the pregnancy test is performed less than 10 days after an abortion. Performing the tests too early or too late in the pregnancy, storing the urine sample too long at room temperature, or having a spontaneous or missed abortion impending can all produce false- negative results.

A nurse is reviewing the plan of care developed by a nursing student for a client scheduled for keratoplasty. The nurse discusses the plan with the student if which incorrect intervention is listed in the plan?

Administering medications that will dilate the pupil

A client arrives at the emergency department and complains of severe abdominal pain. The initial diagnosis is acute abdomen, and an x-ray and an abdominal ultrasonogram are prescribed to be obtained immediately. The nurse prepares the client for these diagnostic tests and reviews the health care provider's prescriptions. Which of the following prescriptions would the nurse question if written on the health care provider's prescription form?

Administration of an analgesic

A client has been taught to use a walker to aid in mobility following internal fixation of a hip fracture. The nurse determines that the client is using the walker incorrectly if the client:

Advances the walker with reciprocal motion

A client is diagnosed with hyphema after experiencing a traumatic blow to the eye. The nurse explains to the client that which activity limitations need to be implemented following this type of injury?

Bedrest with the head in semi-Fowler's position

A nurse is performing nasopharyngeal suctioning on a client and suddenly notes the presence of bloody secretions. The nurse would first:

Check the amount of suction pressure being applied.

A nurse is caring for a client with a diagnosis of myocardial infarction (MI). The client reports chest pain. When the administration of a sublingual nitroglycerin tablet as prescribed does not relieve the chest pain, the next nursing action is to:

Check the blood pressure and administer another nitroglycerin tablet.

A client attached to mechanical ventilation suddenly becomes restless and pulls out the tracheostomy tube. The nurse would first:

Check the client for spontaneous breathing

A nurse reviews the health care provider's treatment plan for a client with Guillain-Barré syndrome. Which prescription, if noted in the client's record, should the nurse question?

Clear liquid diet

A client has experienced an episode of pulmonary edema. The nurse determines that the client's respiratory status is improving if which of the following breath sounds are noted?

Crackles in the lung bases

The nurse should assess for vision loss in a client with which of the following conditions?

Diabetes mellitus

An ambulatory clinic nurse is interviewing a client who is complaining of flu-like symptoms. The client suddenly develops chest pain. Which question would best help the nurse to discriminate pain caused by a noncardiac problem?

Does the pain get worse when you breathe in?"

A male client who experienced a myocardial infarction (MI) tells the nurse that he is fearful about not being able to return to a normal life. Which action by the nurse is therapeutic at this time?

Explore the specific concerns with the client.

A nurse has a prescription to place a client with a herniated lumbar intervertebral disk on bedrest to minimize the pain. The nurse plans to put the bed:

In semi-Fowler's position with the knee gatch slightly raised

A client is admitted to the hospital with a diagnosis of carbon dioxide narcosis. In addition to respiratory failure, the nurse plans to monitor the client for which of the following complications of this disorder?

Increased intracranial pressure

A nurse is turning a postoperative client who had extensive back surgery yesterday. What turning intervention or position would be best for repositioning this client?

Logrolling

A nursing student is assisting in caring for a client with a lung tumor; the client will be having a pneumonectomy. The nursing instructor reviews the postoperative plan of care developed by the student and suggests deleting which of the following from the plan?

Monitoring the closed chest tube drainage system

A client is admitted to the emergency department with a C4 spinal cord injury. The nurse performs which intervention first when collecting data on the client?

Monitoring the respiratory rate

A nurse working in an obstetrical-gynecological health care provider's office is instructing a small group of female clients about breast self-examination (BSE). The nurse teaches the clients to perform the exam:

One week after menstruation begins

A nurse is preparing to assist a health care provider with the insertion of a chest tube. The nurse anticipates that which of the following supplies will be required for the chest tube insertion site?

Petrolatum (Vaseline) gauze

A client with a respiratory disorder has anorexia secondary to fatigue and dyspnea while eating. The nurse determines that the client has followed the recommendations to improve intake if the client:

Planned to eat the largest meal of the day at a time when hungry

A nurse should monitor for which of the following laboratory results as indicating an adverse reaction in the client with endometrial cancer who is receiving chemotherapy?

Platelet count 20,000/mm3

A nurse is assisting in caring for a client with a newly inserted tracheostomy. The nurse notes documentation of an airway problem because of thick respiratory secretions. The nurse should monitor for which item as the best indicator of an adequate respiratory status?

Respiratory rate of 18 breaths per minute

A nurse is assisting in caring for a client who has just returned from the postanesthesia care unit after radical neck dissection. The nurse monitors the portable wound suction for which of the following types of drainage expected in the immediate postoperative period?

Serosanguineous

A nurse is evaluating the pin sites of a client in skeletal traction. The nurse would be least concerned with which finding?

Serous drainage

A 24-year-old man seeks medical attention for complaints of claudication in the arch of the foot. The nurse also notes superficial thrombophlebitis of the lower leg. The nurse would next check the client for:

Smoking history

The nurse is reinforcing dietary instructions to a client with systemic lupus erythematosus. Which dietary items should the nurse instruct the client to avoid?

Steak

A nurse is caring for a client at home who has had a tracheostomy tube for several months. The nurse monitors the client for complications associated with the long-term tracheostomy and suspects tracheoesophageal fistula if which of the following is noted in the client?

Abdominal distention

A client has an inoperable abdominal aortic aneurysm (AAA). The nurse teaches the client about the need for:

Antihypertensives

A client with acute pancreatitis is experiencing severe pain from the disorder. The nurse tells the client to avoid which position that could aggravate the pain?

Lying flat

A nurse is suctioning an unconscious client who has a tracheostomy. The nurse should avoid which action during this procedure?

Making sure not to suction for longer than 30 seconds

A nurse is caring for a client with a nasogastric tube. Which observation is reliable in determining that the tube is correctly placed?

The pH of the aspirate is 5.

A nurse is planning care for the client with hemiparesis of the right arm and leg. The nurse incorporates in the care plan placement of objects:

Within the client's reach, on the left side

A nurse should plan to fill which chamber of the chest drainage unit to prevent atmospheric air from re-entering the pleural space? Refer to figure.

2

The nurse is reviewing the medical record of a client with a diagnosis of pyelonephritis. Which disorder noted on the client's record should the nurse identify as a risk factor for this disorder?

Diabetes mellitus

A client with cancer develops white patches on the mucous membranes of the oral cavity. The nurse noting this would:

Report these symptoms, which are consistent with candidiasis.

The nurse is preparing a client scheduled for an intravenous pyelogram (IVP). The nurse should take which important action before the test?

ask about allergies to iodine or shellfish

The nurse is assessing a client who has small groups of vesicles over his chest and upper abdominal area. They are located only on the right side of his body. The client states his pain level is 8/10, and describes the pain as burning in nature. Which question is most appropriate to include in the data collection?

"Did you have chicken pox as a child?"

A nurse is reviewing the list of discharge instructions for a client who underwent left total knee replacement (TKR) with insertion of a metal prosthesis. Which statement by the client indicates the need for further instructions?

"I don't need to be worried if the shape of my knee changes."

A client diagnosed with angina pectoris returns to the nursing unit after experiencing an angioplasty. The nurse reinforces instructions to the client regarding the procedure and home care measures. Which statement by the client indicates an understanding of the instructions?

"I need to adhere to my dietary restrictions."

A nurse is providing discharge instructions to a client following a keratoplasty. Which statement by the client indicates the need for further instruction?

"Sutures are removed in 2 weeks."

Using the rule of nines, calculate the burn percentage for the client. Refer to the figure; the burned area is the darkly shaded area. (figure shows entire torso, chest and genital area)

19

A client is at risk for developing disseminated intravascular coagulopathy (DIC). The nurse should become concerned with which of the following fibrinogen levels?

90 mg/dL

According to Diane, her LMP is November 15, 2002, using the Naegle's rule what is her EDC? A. August 23, 2003 B. August 18, 2003 C. July 22, 2003 D. February 22, 2003

A

A client with peptic ulcer disease is scheduled for a pyloroplasty and the client asks the nurse about the procedure. The nurse bases the response on which of the following?

A pyloroplasty involves an incision and resuturing of the pylorus to relax the muscle and enlarge the opening from the stomach to the duodenum

After a liver biopsy, the nurse places the client in which of the following positions?

A right side-lying position with a small pillow or folded towel under the puncture site

A tuberculin test (Mantoux test) is administered to an individual infected with human immunodeficiency virus (HIV). Seventy-two hours later, the nurse checks the test site and documents the results as positive, indicating that the individual has been exposed to tuberculosis. Which of the following findings did the nurse note to make this interpretation?

An area of induration at the test site measuring 7 mm

A client with pemphigus is being seen in the clinic regularly. The nurse plans care based on which description of this condition?

An autoimmune disease that causes blistering in the epidermis

A client arrives in the emergency department with a bloody nose. What is the initial nursing action?

Assist the client to a sitting position with the head tilted slightly forward.

A client with a diagnosis of myocardial infarction has a new activity prescription allowing the client to have bathroom privileges. As the nurse stands and begins to walk, the client begins to complain of chest pain. The nurse should initially take which action?

Assist the client to get back into bed.

A client is complaining of low back pain, with radiation down the left posterior thigh. The nurse continues to collect data from the client to see if the pain is worsened or aggravated by:

Bending or lifting

A nurse caring for a client following a craniotomy monitors for signs of increased intracranial pressure (ICP). Which of the following indicates an early sign of increased ICP?

Confusion

The nurse is monitoring an older client suspected of having a urinary tract infection (UTI) for signs of the infection. Which sign/symptom should occur first?

Confusion

A nurse documents that a client with a hiatal hernia is complying with the prescribed treatment if the client reports doing which of the following

Consuming low-fat or nonfat foods

During which of the following phase of the menstrual cycle is it ideal for implantation of a fertilized egg to occur? A. ischemic phase B. mentrual phase C. proliferative phase D. secretory phase

D

A client who is paraplegic after spinal cord injury has been taught muscle-strengthening exercises for the upper body. The nurse determines that the client will derive the least muscle-strengthening benefit from which activity?

Doing active range of motion to finger joints

A nurse understands that which of the following is a correct guideline for adult cardiopulmonary resuscitation (CPR) for a health care provider?

Each rescue breath should be given over 1 second and should produce a visible chest rise.

A nurse is assisting in caring for a client with an inoperable lung tumor and helps to develop a plan of care by addressing complications related to the disorder. The nurse includes in the plan to monitor for the early signs of vena cava syndrome. Which of the following should the nurse include in the plan of care as the early sign of this oncological emergency?

Edema of the face and eyes

A client is admitted to the hospital with a diagnosis of suspected Hodgkin's disease. Which of the following findings would the nurse most likely expect to find documented in the client's record?

Enlarged lymph nodes

A client is receiving chemotherapy that carries a risk of phototoxicity as an adverse effect. Which finding indicates that the client experienced this side effect?

Erythema

A nurse is assigned to care for a client with a Sengstaken-Blakemore tube. The nurse should suspect that the client has which diagnosis?

Esophageal varices

A client with a neurological impairment experiences urinary incontinence. Which nursing action should help the client adapt to this alteration?

Establishing a toileting schedule

A nurse determines that a client with a history of which of the following is most at risk of endometrial cancer?

Estrogen replacement therapy

A client is admitted to the hospital with acute viral hepatitis. Which signs or symptoms would the nurse expect to note, based upon this diagnosis?

Fatigue

The nurse is providing instructions to the client regarding the complications of peritoneal dialysis. The nurse instructs the client that which symptom is likely associated with the onset of peritonitis?

Fever

A licensed practical nurse (LPN) is assisting in the care of a client who is having central venous pressure (CVP) measurements taken by the registered nurse (RN). The LPN would assist the RN by placing the bed in which of the following positions for the reading?

Flat

A client is diagnosed with stage I Lyme disease. In addition to the rash, the nurse should check the client for which manifestation?

Flulike symptoms

A nurse is preparing to administer an enteral feeding through a nasogastric tube. The nurse would place the client in which position during and after the feedings

Fowler's

The community health nurse is conducting a research study and is identifying clients in the community who are at risk for latex allergy. Which client population is at risk for developing this type of allergy?

Hairdressers

The nurse is collecting data on a newly admitted client with a diagnosis of bladder cancer. Which sign/symptom should be noted first?

Hematuria

A client arrives at the emergency department after experiencing a traumatic blow to the eye and a hyphema is diagnosed. The nurse immediately positions the client:

In semi-Fowler's position

A client seeks treatment for a complaint of hoarseness that has lasted for 6 weeks. Based on this symptom, the nurse interprets that the client is at risk of having:

Laryngeal cancer

A nurse is assisting in caring for a client with a supratentorial lesion. The nurse monitors which of the following as the critical index of central nervous system (CNS) dysfunction?

Level of consciousness

The nurse is providing discharge instructions to a client after gastrectomy. Which measure will the nurse instruct the client to follow to help prevent dumping syndrome?

Limit the fluids taken with meals.

A nurse is discussing primary prevention measures to clients regarding osteoporosis. The nurse plans to tell the clients that which of the following is a primary prevention measure?

Maintaining body weight at or above minimum recommended levels

A nurse is caring for the client who is at risk for lung cancer because of an extremely long history of heavy cigarette smoking. The nurse tells the client to report which most frequent early symptom of lung cancer?

Nonproductive hacking cough

A nurse is assisting the health care provider in performing a caloric test on a client. Following instillation of cool water into the ear, the nurse notes the presence of nystagmus. The nurse should document the findings of this test as:

Normal

A nurse is monitoring a client with a diagnosis of cancer for signs and symptoms related to vena cava syndrome. The nurse understands that which of the following is an early sign of this oncological emergency?

Periorbital edema

A post-gastrectomy client is at high risk for hyperglycemia related to uncontrolled gastric emptying of fluid and food into the small intestine (dumping syndrome). Because of this risk, the nurse plans to monitor the

Postprandial blood glucose readings

After a routine eye examination, a client has been told there are refractive errors in both eyes. The nurse explains to the client that this problem is primarily treated by

Prescription of corrective lense

A nurse is performing colostomy irrigation on a client. During the irrigation, the client begins to complain of abdominal cramps. Which of the following is the appropriate nursing action?

Stop the irrigation temporarily

A nurse reviews the care plan of a client with cancer and notes that the client has a problem with adequate food intake related to side effects of therapy. In order to enhance appetite and nutrition, the nurse advises the client to avoid:

Strong-smelling foods

The nurse interprets that a client diagnosed with glaucoma needs information about the expected effects of this condition when she states that:

Taking my daily walk right around dusk each evening has proven to be so enjoyable."

A female client who has been receiving radiation therapy for bladder cancer tells the nurse that it feels as if she is voiding through the vagina. The nurse interprets that the client may be experiencing:

The development of a vesicovaginal fistula

A nurse is observing a nursing assistant talking to a client who is hearing impaired. The nurse would intervene if which is performed by the nursing assistant during communication with the client?

The nursing assistant is speaking directly into the impaired ear

A nurse inspects the skin of a client receiving external radiation therapy and documents a finding noted as moist desquamation. The nurse understands that moist desquamation is best described as which of the following?

Weeping of the skin

A client has just returned from the radiology department following an upper gastrointestinal (GI) series. The nurse reviews the health care provider's prescriptions, expecting to note which of the following needed for routine post-procedure care?

laxative

The client with chronic kidney disease who is scheduled for hemodialysis this morning is due to receive a daily dose of enalapril (Vasotec). When should the nurse plan to administer this medication?

on return from dialysis

A nurse is preparing a client for an intravesical instillation of an alkylating chemotherapeutic agent into the bladder for the treatment of bladder cancer. The nurse provides instructions to the client regarding the procedure. Which statement by the client indicates an understanding of this procedure?

"After the instillation is done, I will need to change position every 15 minutes from side to side."

A nurse is assisting in preparing a teaching plan of care for a client being discharged from the hospital following surgery for testicular cancer. Which instruction will the nurse suggest to include in the plan?

"An elevation in temperature should be reported to the health care provider."

A nurse is caring for a client with a diagnosis of multiple sclerosis who has been prescribed oxybutynin (Ditropan). The nurse evaluates the effectiveness of the medication by asking the client which of the following questions?

"Are you getting up at night to urinate?"

A nurse is collecting data from a client suspected of having ovarian cancer. Which question should the nurse ask the client to elicit information specifically related to this disorder?

"Does your abdomen feel as though it is swollen?"

A nurse is assisting in providing a session to community members about the risks associated with laryngeal cancer. Which statement by a client indicates an understanding of the risk factors?

"Exposure to airborne carcinogens can cause this type of cancer."

A nurse has given medication instructions to the client receiving phenytoin (Dilantin). The nurse determines that the client understands the instructions if the client states:

"Good oral hygiene is needed, including brushing and flossing."

Which statement by the client who has received home care instruction following an arthroscopy of the knee indicates a need for further education?

"I can apply heat to my knee if it becomes uncomfortable."

A nurse has provided discharge instructions to a client following right eye cataract surgery about ways to avoid strain on the operative eye. The nurse determines that the client needs further instruction if the client makes which of the following statements?

"I can lie on my right side."

A nurse is reinforcing discharge instructions to a client who has had ocular surgery of the left eye. Which statement by the client indicates a need for further instructions?

"I need to call the doctor if I develop any fever."

A client has just had a cast removed and the underlying skin is yellow-brown and crusted. The nurse determines that further skin care instructions are required when the client states:

"I need to scrub the skin vigorously with soap and water."

A clinic nurse has provided home care instructions to a client with a diagnosis of glaucoma. Which statement by the client indicates an understanding of the treatment plan for glaucoma?

"I need to take my eye drops for the rest of my life."

A nurse caring for a client following a radical neck dissection and creation of a tracheostomy performed for laryngeal cancer is providing discharge instructions to the client. Which statement by the client indicates the need for additional instructions regarding care to the stoma?

"I need to use an air conditioner to provide cool air to assist in breathing."

A nurse is reinforcing instructions to a client following a cataract extraction on the right eye. Which statement by the client indicates a need for further teaching?

"I need to wear an eye shield all the time."

A nurse is obtaining data from a client admitted with a diagnosis of bladder cancer. Which question should the nurse ask the client to determine if the client experienced the common symptom associated with this type of cancer?

"Do you notice any blood in the urine?"

A nurse is preparing to administer an enema to an adult client. Choose the interventions that the nurse would perform for this procedure

Apply disposable gloves,Lubricate the enema tube and insert it approximately 4 inches,Clamp the tubing if the client expresses discomfort during the procedure,Ensure that the temperature of the solution is between 100° F (37.8° C) and 105° F (40.5° C).

A client arrives at the emergency department following a blow to the eye from a softball. Which intervention should be implemented by the nurse initially?

Apply ice to the affected eye.

A client has an endocrine system dysfunction of the pancreas. The nurse anticipates that the client will exhibit impaired secretion of which of the following substances?

1. Amylase 2. Lipase 3. Trypsin *4. Insulin* *rationale* The pancreas produces both endocrine and exocrine secretions as part of its normal function. The organ secretes insulin as a key endocrine hormone to regulate the blood glucose level. Other pancreatic endocrine hormones are glucagon and somatostatin. The exocrine pancreas produces digestive enzymes such as amylase, lipase, and trypsin.

Which nursing action would be appropriate to implement when a client has a diagnosis of pheochromocytoma?

1. Weigh the client. 2. Test the client's urine for glucose. *3. Monitor the client's blood pressure.* 4. Palpate the client's skin to determine warmth. *rationale* Hypertension is the major symptom that is associated with pheochromocytoma. The blood pressure status is monitored by taking the client's blood pressure. Glycosuria, weight loss, and diaphoresis are also clinical manifestations of pheochromocytoma, but hypertension is the major symptom.

A nurse is collecting data on a client who sustained circumferential burns of both legs. The nurse should check which first? 1. Heart rate 2. Temperature 3. Peripheral pulses 4. Blood pressure (BP)

3. Peripheral pulses

A nurse is caring for a client hospitalized with acute exacerbation of chronic obstructive pulmonary disease (COPD). Which of the following would the nurse expect the client to experience?

A hyperinflated chest on x-ray

Acetazolamide is prescribed for a client with a diagnosis of a supratentorial lesion. A nurse monitors the client for effectiveness of this medication, knowing that its primary action is to:

Decrease cerebrospinal fluid production.

The nurse observes the following rhythm on the cardiac monitor. (Refer to the figure.) Which action should the nurse take first?

Evaluate the client for hypotension and assess mental status.

A nurse is assisting in developing a teaching plan for the client with glaucoma. Which instruction should the nurse suggest to include in the plan of care?

Eye medications will need to be administered for the rest of your life.

A client is admitted to the hospital with viral hepatitis and is complaining of a loss of appetite. In order to provide adequate nutrition, the nurse encourages the client to:

Increase intake of fluids.

A client with myasthenia gravis is having difficulty speaking. The client's speech is dysarthric and has a nasal tone. The nurse should use which communication strategies when working with this client? Select all that apply.

Listening attentively Asking yes and no questions when able Using a communication board when necessary Repeating what the client said to verify the message

A nurse is caring for a client with a diagnosis of right (nondominant) hemispheric brain attack (stroke). The nurse notes that the client is alert and oriented to time and place. Based on these findings, the nurse interprets that the client:

May likely have perceptual and spatial disabilities

Immediately following cataract repair, the client's affected conjunctiva and eyelids are edematous. The nurse tells the client that this is:

Normal and should subside within 3 days

A client who sustained an eye injury arrives at the emergency department. The initial nursing action is to

Obtain a history regarding the cause of the injury.

Which data would indicate a potential complication associated with age-related changes in the musculoskeletal system?

Overall sclerotic lesions

A client's blood pressure is 100/78 mm Hg; the client has tachycardia and is cool and pale. The nurse assists the client to which position to promote tissue oxygenation?

Semi-Fowler's

A client is receiving external radiation to the neck for cancer of the larynx. The nurse plans the client's care knowing that the most likely side effect to be expected is:

Sore throat

Which data collection finding supports the possible diagnosis of Bell's palsy?

Speech or chewing difficulties accompanied by facial droop

The nurse has reinforced instructions to the client with a cystocele about Kegel exercises. The nurse determines that the client has not fully understood the directions if the client makes which statement?

"Begin voiding and then stop the stream, holding residual urine for an hour."

A client is somewhat nervous about having magnetic resonance imaging (MRI). Which statement by the nurse would provide reassurance to the client about the procedure?

"Even though you are alone in the scanner, you will be in voice communication with the technologist during the procedure."

The nurse determines that the neutropenic client needs further teaching if which statement is made by the client?

"I will include plenty of fresh fruits in my diet."

The nurse is inspecting the stoma of a client after creation of an ureterostomy. Which appearance should the nurse expect to note?

A red and moist stoma

A client has had surgery to repair a fractured left hip. The nurse plans to use which of the following important items when repositioning the client from side to side in the bed?

Abductor splint

A nurse has reinforced home care instructions to a client who had a permanent pacemaker inserted. Which educational outcome has the greatest impact on the client's long-term cardiac health?

Ability to take an accurate pulse in either the wrist or neck

A client calls the office of his primary care health care provider and tells the nurse that he was just stung by a bumblebee while gardening. The client is afraid of a severe reaction because his neighbor experienced such a reaction just 1 week ago. Which is the appropriate nursing action?

Ask the client if he ever sustained a bee sting in the past

A client in skeletal leg traction with an overbed frame is not allowed to turn from side to side. Which action by the nurse would be most useful in trying to provide good skin care to the client?

Asking the client to pull up on a trapeze to lift the hips off the bed

A nurse is caring for a client who had a total knee replacement. Postoperatively, the nurse monitors for which highest priority item?

Calf pain

A female client has a prescription for a clean-catch urine culture. After providing a sterile specimen cup to the client, the nurse should give which instruction so that the specimen is collected properly?

Cleanse the labia using cleansing towels, begin to void into toilet, and then collect the specimen

A client who had previously undergone cataract surgery tells the nurse that she has begun seeing flashing lights and floaters in the eye. Based on the client's history, the nurse interprets that the client is at risk of:

Detached retina

A nurse reinforces what information to a client who is scheduled for an electromyogram (EMG)?

Electrodes will be injected into the skeletal muscles.

A nurse is developing a plan of care for a client following a radical mastectomy and includes measures that will assist in preventing lymphedema of the affected arm. The nurse should include which of the following to prevent this complication?

Elevate the affected arm on a pillow.

A nurse is reviewing the record of a client with a iagnosis of cervical cancer. Which of the following should the nurse expect to note in the client's record related to a risk factor associated with this type of cancer?

History of human papillomavirus

A nurse is collecting physical data of the musculoskeletal system on an assigned client. The nurse should document the presence of which of the following as a normal finding?

Hypertrophy on the client's dominant side

A pilocarpine ocular system is prescribed for the client with glaucoma. The nurse provides instructions to the client regarding the medication. Which statement by the client indicates an understanding of the use of this medication?

I should check my eye each morning to make sure that the medication system is in place."

A female client arrives at the health care clinic and tells the nurse that she was just bitten by a tick and would like to be tested for Lyme disease. The client tells the nurse that she removed the tick and flushed it down the toilet. Which nursing action is appropriate?

Instruct the client to return in 4 to 6 weeks to be tested, because testing before this time is not reliable

A nurse is providing morning care to a client who has a closed chest tube drainage system to treat a pneumothorax. When the nurse turns the client to the side, the chest tube is accidentally dislodged from the chest. The nurse immediately applies sterile gauze over the chest tube insertion site and next:

Notifies the registered nurse (RN)

A client hospitalized with urolithiasis has a sudden significant decrease in urine output. The nurse should perform which action?

Notify the registered nurse

A client begins to drain small amounts of red blood from a tracheostomy tube 36 hours after a supraglottic laryngectomy. The licensed practical nurse would:

Notify the registered nurse.

A nurse is caring for a client with metastatic lung cancer. The client was medicated 2 hours ago and now reports a new and sudden sharp pain in the back. The nurse appropriately interprets this finding as possibly indicating:

Spinal cord compression

A nurse has given instructions to the family of an older client who seems anxious about being discharged after cardiac surgery. The nurse would need to reinforce the teaching if a family member made which of the following statements?

"A daily half-mile-long brisk walk generally helps people bounce back more quickly and provides more of a sense of control."

Surgery has been recommended for the client with otosclerosis. The client tells the nurse that surgery is not desired and asks the nurse about alternative methods to improve hearing. The nurse makes which appropriate response to the client?

"A hearing aid may improve your hearing."

A nurse is caring for a client who has undergone pelvic exenteration. In addressing psychosocial issues related to the surgery, which statement by the nurse would be therapeutic?

"How do you feel about your body?"

A nurse has reinforced discharge instructions regarding home care to a client following a prostatectomy for cancer of the prostate. Which statement by the client indicates an understanding of the instructions?

"I should not lift anything over 20 pounds."

A nurse is providing discharge instructions to a client following surgical treatment for carpal tunnel syndrome. Which statement by the client would indicate a need for further instruction?

"I should perform pronation and supination exercises of my wrist starting twenty-four hours after surgery."

The nurse reinforces home care instructions to a client with systemic lupus erythematosus and tells the client about methods to manage fatigue. Which statement by the client indicates a need for further teaching?

"I should take hot baths because they are relaxing."

A nurse is reinforcing home-care instructions to a client and family regarding care after cataract removal from the right eye. Which of the following statements, if made by the client, would indicate an understanding of the instructions?

"I will not sleep on my right side."

A postoperative client received a spinal anesthetic during the repair of a right hip fracture. The client has not experienced pain because the anesthetic has not yet worn off. The nurse will monitor the client closely for pain and provide the client with which instruction?

"You will need to let me know when you start to get feeling back in your legs."

A client with Crohn's disease has a prescription to begin taking antispasmodic medication. The nurse should time the medication so that each dose is taken:

30 minutes before meals

A nurse reinforces instructions to a client who is to return to the health care provider's office in 1 week for a patch test to identify the allergen causing the dermatitis. The nurse provides which instruction to the client? 1. "Remain nothing per mouth (NPO) prior to the test." 2. "Consume fluids only on the day of the test." 3. "Shower using an antibacterial soap on the morning of the test." 4. "Discontinue the prescribed antihistamine 2 days before the test."

4. "Discontinue the prescribed antihistamine 2 days before the test."

Several clients arrive simultaneously to the emergency department, after sustaining burn injuries in a house fire. Which client will require the closest observation for signs of respiratory distress?

A client who has singed nasal hairs and worsening hoarseness

It is all right to drive an hour after the test is finished

A nurse has given postprocedure instructions to a client who has undergone a colonoscopy. The nurse determines that the client did not fully understand the directions if the client states that:

A client who has had a total knee replacement tells the nurse that there is pain with extension of the knee. The nurse should:

Administer an analgesic.

A client has a diagnosis of presbycusis. The nurse interprets that which behavior indicates that the client has successfully adapted to this disorder?

Agrees to use a prescribed hearing aid, especially when home alone

A client is scheduled for a cardiac catheterization using a radiopaque dye. The nurse checks which most critical item before the procedure?

Allergy to iodine or shellfish

A client with glaucoma asks the nurse if complete vision will return. The nurse makes which response to the client?

Although some vision has been lost and cannot be restored, further loss may be prevented by adhering to the treatment plan."

A client with chronic kidney disease has been on dialysis for 3 years. The client is receiving the usual combination of medications for the disease, including aluminum hydroxide as a phosphate-binding agent. The client now has mental cloudiness, dementia, and complaints of bone pain. Which does this data indicate?

Aluminum intoxication

The nurse is working with a client newly diagnosed with chronic kidney disease (CKD) to set up a schedule for hemodialysis. The client states, "This is impossible! How can I even think about leading a normal life again if this is what I'm going to have to do?" The nurse determines that the client is exhibiting which reaction?

Anger

A client with Ménière's disease is experiencing severe vertigo. The nurse instructs the client to do which of the following to assist in controlling the vertigo?

Avoid sudden head movements

The nurse is providing information to a client with systemic lupus erythematosus (SLE) about dietary alterations. The nurse should remind the client to avoid which foods? Select all that apply.

Beef Cheese

A nurse is preparing for the admission of a client with a suspected diagnosis of herpes simplex encephalitis. Which diagnostic test will be prescribed to confirm this diagnosis?

Brain biopsy

The health care provider prescribes fluconazole (Diflucan) for a client. When administering this medication the nurse should explain to the client that it is used to treat which opportunistic infection?

Candidiasis

A client with myasthenia gravis is experiencing prolonged periods of weakness. The health care provider prescribes a test dose of edrophonium (Enlon) and the client becomes weaker. The nurse interprets this outcome as:

Cholinergic crisis

A nurse is caring for a client being treated for fat embolus after multiple fractures. Which of the following data indicates to the nurse favorable resolution of the fat embolus?

Clear chest x-ray

A client has several fractures of the lower leg and has been placed in an external fixation device. The client is upset about the appearance of the leg, which is very edematous. The nurse determines that the client is experiencing which problem?

Concerns about appearance

A nurse is working with a client diagnosed with anorexia nervosa. The nurse plans care, focusing on which of the following as the primary problem

Impaired nutritional status

Betaxolol hydrochloride (Betoptic SR) eye drops have been prescribed for the client with glaucoma. Which nursing action is most appropriate related to monitoring for the side effects of this medication?

Monitoring blood pressure

A client has an impairment of cranial nerve II. Specific to this impairment, the nurse would plan to do which of the following to ensure client safety?

Provide a clear path for ambulation without obstacles.

A client with type 1 diabetes mellitus has had a left above-the-knee amputation. The nurse carefully inspects the residual limb for which of the following complications because of the history of diabetes?

Separation of wound edges

A nurse is asked to assist another health care member in providing care to a client who is placed in a modified Trendelenburg's position. The nurse interprets that the client is likely being treated for which condition?

Shock

A nurse is planning adaptations needed for activities of daily living for a client with cardiac disease. The nurse would incorporate which of the following in discussions with the client?

Take in adequate daily fiber to prevent straining during a bowel movement.

A health care provider has given a prescription for dietary iron supplements to the client with osteoporosis who has an iron deficiency anemia. The nurse suggests that the client do which of the following to enhance compliance with therapy?

Take the medication following a meal

A client has had same-day surgery to insert a ventilating tube in the tympanic membrane. The nurse reinforces to the client to be sure to do which of the following until the postoperative assessment by the surgeon?

Use a shower cap to protect the ears if taking a shower.

Which interventions would apply in the care of a client at high risk for an allergic response to a latex allergy? Select all that apply.

Use nonlatex gloves Use medications from glass ampules Do not puncture rubber stoppers with needles Keep a latex-safe supply cart available in the client's area

A client with acquired immunodeficiency syndrome (AIDS) has histoplasmosis. Which one of the following would the nurse expect the client to experience?

Dyspnea

A client with new-onset renal failure is having a first hemodialysis treatment. The nurse is especially careful to monitor the client for which signs/symptoms after the dialysis treatment?

Headache, decreasing level of consciousness, and seizures

A client with a hip fracture asks the nurse why Buck's extension traction is being applied before surgery. The nurse's response is based on the understanding that Buck's extension traction primarily:

Provides comfort by reducing muscle spasms and provides fracture immobilization

A client who has undergone femoropopliteal bypass grafting says to the nurse, "I hope I don't have any more problems that could make me lose my leg. I'm so afraid that I'll have gone through this for nothing." The appropriate nursing response is which of the following?

"You are concerned about losing your leg?"

A client with coronary artery disease has selected guided imagery to help cope with psychological stress. Which statement by the client indicates the best understanding of this stress reduction measure?

"The best thing about this is that I can use it anywhere, anytime."

A nurse is reviewing a chart of a child with a head injury. The nurse notes that the level of consciousness has been documented as obtunded. Which of the following would the nurse expect to note on data collection of the child?

1. Awake, alert, interacting with the environment 2. The ability to think clearly and rapidly is majorly impaired. 3. The ability to recognize place or person is severely affected. *4. Sleeps unless aroused and, once aroused, interacts poorly with the environment* *Rationale:* Obtunded indicates that the child sleeps unless aroused and, once aroused, has limited interaction with the environment. Full consciousness indicates that the child is alert, awake, orientated, and interacts with the environment. Confusion indicates that the ability to think clearly and rapidly is lost, and disorientation indicates that the ability to recognize a place or person is lost.

A nurse is planning to teach a client with peripheral arterial disease about measures to limit disease progression. The nurse should include which items on a list of suggestions to be given to the client? Select all that apply.

1. Cut down on the amount of fats consumed in the diet. 3. Walk each day to increase circulation to the legs. 4. Be careful not to injure the legs or feet. 5. Eat a well-balanced diet every day.

A client has been diagnosed with hypoparathyroidism. The nurse teaches the client to include foods in the diet that are:

1. High in phosphorus and low in calcium 2. Low in phosphorus and low in calcium *3. Low in phosphorus and high in calcium* 4. High in phosphorus and high in calcium *rationale* Hypoparathyroidism results in hypocalcemia. A therapeutic diet for this disorder is one that is high in calcium but low in phosphorus because these two electrolytes have inverse proportions in the body. All of the other options are unrelated to this disorder and are incorrect.

When caring for a client who is having clear drainage from his nares after transsphenoidal hypophysectomy, which action by the nurse is appropriate?

1. Lower the head of the bed. *2. Test the drainage for glucose.* 3. Obtain a culture of the drainage. 4. Continue to observe the drainage. *rationale* After hypophysectomy, the client should be monitored for rhinorrhea, which could indicate a cerebrospinal fluid (CSF) leak. If this occurs, the drainage should be collected and tested for glucose, indicating the presence of CSF. The head of the bed should not be lowered to prevent increased intracranial pressure. Clear nasal drainage would not indicate the need for a culture. Continuing to observe the drainage without taking action could result in a serious complication.

A male client is diagnosed with urethritis caused by chlamydial infection. The unlicensed assistive personnel (UAP) assigned to the client asks the nurse what measures are necessary to prevent a contraction of the infection during care. Which instruction should the nurse give the UAP?

Standard precautions are sufficient because the infection is transmitted sexually

A nurse notes that a client is due in hydrotherapy for a burn dressing change in 30 minutes. The nurse plans to do which of the following next in the care of this client? 1. Get out a robe and slippers for the client. 2. Administer an opioid analgesic last taken 6 hours ago. 3. Immediately place the client on nothing-by-mouth (NPO) status. 4. Gather dressing supplies to send with the client to hydrotherapy.

2. Administer an opioid analgesic last taken 6 hours ago.

A client with end stage kidney disease (ESKD) undergoes a surgical procedure to create an arteriovenous fistula for hemodialysis in the upper extremity. The nurse should take which actions when the client returns from surgery? Select all that apply.

Monitor pain and administer analgesics Monitor bleeding and swelling at the site Check for audible bruit and palpable thrill at the fistula site

Ativan 0.5 mg IM every 1 hour as needed is prescribed for a client experiencing delirium tremens. The medication vial reads 2mg/mL of solution. How many mL should the LPN draw into the syringe for single dose administration?

Possible correct answers: 0.25 mL0.25mL0.25ml0.25 ml Explanation 2mg/mL= 0.5mg/xmL 2x=0.5 x=0.5/2 x=0.25 mL

A client, on the waiting list for a renal transplant, receives a hemodialysis treatment. Which findings indicate to the nurse that the treatment has been effective? Select all that apply.

Serum potassium level is within the normal range The client's weight is 2 kilograms less than predialysis weight Serum blood urea nitrogen (BUN) and creatinine levels are lower than predialysis

A nurse is teaching the client with angina pectoris about disease management and lifestyle changes that are necessary in order to control disease progression. Which statement by the client indicates a need for further teaching?

"It is best to exercise once a week for an hour."

A client arrives at the ambulatory care center complaining of flulike symptoms. On data collection, the client tells the nurse that he was bitten by a tick and is concerned that the bite is causing the sick feelings. The client requests a blood test to determine the presence of Lyme disease. Which question should the nurse ask next?

"When were you bitten by the tick?"

A nurse is collecting data on a client with a diagnosis of angina pectoris who takes nitroglycerin for chest pain. During the admission, the client reports chest pain. The nurse immediately asks the client which of the following questions?

"Where is the pain located?"

A nurse is caring for an infant with a diagnosis of Hirschsprung's disease. The nurse should check for which clinical findings that are consistent with Hirschsprung's disease? *Select all that apply.*

*1. Fever* *2. Constipation* *3. Failure to thrive* 4. Intolerance to wheat *5. Abdominal distention* *6. Explosive, watery diarrhea* *Rationale:* Clinical manifestations of Hirschsprung's disease during infancy include failure to thrive, constipation, abdominal distention, episodes of diarrhea and vomiting, signs of enterocolitis, explosive and watery diarrhea, and fever. The infant appears significantly ill. Intolerance to wheat occurs in celiac disease.

A nurse is collecting data from a child suspected of having juvenile idiopathic arthritis (JIA). Which findings would the nurse expect to note if JIA were present? *Select all that apply.*

*1. Malaise, fatigue, and lethargy* *2. Painful, stiff, and swollen joints* *3. Limited range of motion of the joints* 4. Stiffness that develops later in the day 5. Cool temperature of the skin over the affected joints *6. History of late afternoon temperature, with temperature spiking up to 105° F* *Rationale:* Clinical manifestations associated with JIA include intermittent joint pain that lasts longer than 6 weeks and painful, stiff, and swollen joints that are warm to the touch, with limited range of motion. The child will complain of morning stiffness and may protect the affected joint or refuse to walk. Systemic symptoms include malaise, fatigue and lethargy, anorexia, weight loss, and growth problems. A history of a late afternoon fever with temperature spiking up to 105° F will also be part of the clinical manifestations.

In planning care for a child with contact dermatitis, which concern is the highest priority for the child?

*1. Pain* 2. Skin breaks 3. Infection 4. Parental knowledge about care *Rationale:* In any skin disorder, the goal with children is to offer comfort interventions so that the child can rest. Once pain has decreased, the skin can be assessed for integrity and infection. Although important, teaching is not the priority in this situation.

A 5-year-old child is admitted to the hospital for heart surgery to repair tetralogy of Fallot. The nurse notes that the child has clubbed fingers, and the nurse knows that this symptom is likely a result of:

*1. Peripheral hypoxia* 2. Chronic hypertension 3. Delayed physical growth 4. Destruction of bone marrow *Rationale:* Clubbing, a thickening and flattening of the tips of the fingers and toes, is thought to occur because of a chronic tissue hypoxemia and polycythemia. Options 2, 3, and 4 are not causes of clubbing.

The appropriate child position after a tonsillectomy is which of the following? 1. Supine position 2. Side-lying position 3. High Fowler's position 4. Trendelenburg's position

*2. Side-lying position* *Rationale:* The child should be placed in a semi-prone or side-lying position after tonsillectomy to facilitate drainage. Options 1, 3, and 4 will not achieve this goal.

A nurse is caring for a client after thyroidectomy and monitoring for signs of thyroid storm. The nurse understands that which of the following is a manifestation associated with this disorder?

1. Bradycardia *2. Hypotension* 3. Constipation 4. Hypothermia *rationale* Clinical manifestations associated with thyroid storm include a fever as high as 106° F (41.1° C), severe tachycardia, profuse diarrhea, extreme vasodilation, hypotension, atrial fibrillation, hyperreflexia, abdominal pain, diarrhea, and dehydration. With this disorder, the client's condition can rapidly progress to coma and cardiovascular collapse.

A clinic nurse notes that following several eye examinations the health care provider has documented a diagnosis of legal blindness in the client's chart. Which of the following would the nurse expect to note documented as the result of the Snellen chart test?

20/200 vision

A client arrives at the emergency department and has experienced frostbite to the right hand. Which of the following would the nurse note when performing data collection regarding the client's hand? 1. A pink, edematous hand 2. Fiery red skin with edema in the nail beds 3. Black fingertips surrounded by an erythematous rash 4. A white color of the skin, which is insensitive to touch

4. A white color of the skin, which is insensitive to touch

An adult client with hepatic encephalopathy has a serum ammonia level of 120 mcg/dL and receives treatment with lactulose (Chronulac) syrup. The nurse determines that the client has the best and most optimal response if the level changes to which of the following after medication administration?

70 mcg/dL

A client with spinal cord injury has experienced more than one episode of autonomic dysreflexia. The nurse would avoid which of the following that could trigger an episode of this complication?

Allowing the client's bladder to become distended

A client has just undergone renal biopsy. In planning care for this client, the nurse should avoid which intervention?

Ambulate in the room and hall for short distances

A nurse is providing instructions to a client receiving external radiation therapy. The nurse determines that the client needs further instructions if the client states an intention to:

Apply pressure on the radiated area to prevent bleeding.

A nurse is monitoring a client with a spinal cord injury for signs of spinal shock. Which of the following is indicative of this complication of a spinal cord injury?

Areflexia below the level of injury

A nurse is caring for a client after a modified radical mastectomy. Which of the following findings would indicate that the client is experiencing a complication related to the surgery?

Arm edema on the operative side

A client with pneumonia is experiencing problems with ventilation as a result of accumulated respiratory secretions. The nurse determines that which of the following accurately indicates effectiveness of the treatments prescribed for this problem?

Arterial blood gases indicate a pH of 7.4, Po2 of 80 mm Hg, Pco2 of 40 mm Hg

A nurse collects data from a client with a diagnosis of macular degeneration of the eye. The nurse should expect the client to report which of the following symptoms?

Blurred central vision

A client is scheduled for a dipyridamole (Persantine) thallium scan. The nurse would check to make sure that the client has not had which of the following before the procedure?

Caffeine

The nurse must ambulate a client who has a nephrostomy tube attached to a drainage bag. The nurse plans to do this most safely and effectively by performing which action?

Changing the drainage

The nurse is collecting neurological data on an unconscious client. On application of a central noxious stimulus, the nurse observes this response (refer to figure). How should the nurse document this response on the client's record?

Client demonstrated decerebrate posturing.

A nurse is collecting subjective and objective assessment data from a client admitted to the hospital with tuberculosis (TB). The nurse should expect to note which of the findings?

Complaints of night sweats

A nurse is assisting a client with a closed chest tube drainage system to get out of bed to a chair. During the transfer, the chest tube gets caught in the leg of the chair and accidentally dislodges from the insertion site. The immediate nursing action is to:

Cover the insertion site with sterile Vaseline gauze.

A nurse is assisting in performing a confrontation test on a client seen in the clinic. The nurse understands that this test is performed to check the client's ability to:

Demonstrate effective peripheral vision.

A nurse is talking to a client who underwent a below-the-knee amputation 2 days earlier. The client says to the nurse, "I hate looking at this; I feel that I'm not even myself anymore." The nurse understands that the client is experiencing which problem?

Disturbed body image

A nurse is monitoring a client who sustained a head injury and suspects that the client has a skull fracture. This conclusion is based on which of the following findings? Select all that apply.

Drainage from ear Bruising around the eyes Pink-tinged drainage from the nose

A client with Parkinson's disease is embarrassed about the symptoms of the disorder and is bored and lonely. The nurse would plan which approach as therapeutic in assisting the client to cope with the disease?

Encourage and praise perseverance in exercising and performing ADL.

A nurse is monitoring a confused older client admitted to the hospital with a hip fracture. Which of the following data obtained by the nurse could place the client at increased risk for disturbed thought processes?

Eyeglasses left at home

A client has an Unna boot applied for treatment of a venous stasis leg ulcer. The nurse notes that the client's toes are mottled and cool, and the client verbalizes some numbness and tingling of the foot. The nurse interprets that the boot:

Has been applied too tightly

A nurse is preparing to obtain a sputum specimen from the client. Which nursing action will facilitate obtaining the specimen?

Have the client take three deep breaths.

A nurse is providing instructions to a client regarding the use of a hearing aid. Which statement by the client indicates a need for further instructions?

I should turn the hearing aid off after removing it from my ear."

A nurse is caring for a client with anorexia. The nurse plans care for the client, focusing on which of the following as the primary problem?

Impaired nutritional status

A nurse is caring for a client with a diagnosis of brain attack (stroke) with anosognosia. To meet the needs of the client with this deficit, the nurse plans activities that will:

Increase the client's awareness of the affected side.

A client has undergone a transurethral resection of the prostate (TURP) a few hours ago to treat symptoms of benign prostatic hypertrophy. The nurse notes bright red blood and clots in the urinary catheter drainage bag. Which response should be the nurse's initial action?

Increase the flow rate of the continuous bladder irrifation

A nurse is caring for an older client with a diagnosis of osteoarthritis. Which of the following would be least helpful for the client?

Increasingly vigorous and high-impact exercise

A client arrives in the emergency department with an eye injury caused by metal fragments that hit the eye while drilling into metal. The nurse checks the eye and notes small pieces of metal floating on the eyeball. Which action should the nurse plan to assist with first

Irrigate the eye with sterile saline.

A client arrives in the emergency department with an eye injury resulting from metal fragments that hit the eye while the client was drilling into metal. The nurse assesses the eye and notes small pieces of metal floating on the eyeball. Which action should the nurse take first?

Irrigate the eye with sterile saline.

A client arrives in the emergency department with a chemical eye injury. The nurse immediately:

Irrigates the eye with copious amounts of sterile normal saline

The nurse is assigned to care for a client with systemic lupus erythematosus (SLE). The nurse plans care considering which factor regarding this diagnosis?

It is an inflammatory disease of collagen contained in connective tissue

A client who is human immunodeficiency virus (HIV) positive has had a tuberculin skin test. The results show a 7-mm area of induration. How should the nurse interpret the test?

It is positive

A nurse is reinforcing education to a client who has just obtained a hearing aid about its use and maintenance. The nurse tells the client that it is helpful to

Keep an extra battery readily available.

A client is admitted to the hospital with a diagnosis of pleurisy. The nurse checks the client for which characteristic symptom of this disorder?

Knifelike pain that worsens on inspiration

A client with right-sided weakness needs to learn how to use a cane. The nurse plans to teach the client to position the cane by holding it with the:

Left hand, and 6 inches lateral to the left foot

A nurse is teaching a client with chronic airflow limitation (CAL) about positions that help breathing during dyspneic episodes. The nurse instructs the client to avoid which of the following positions because it will aggravate breathing?

Lying on his or her back in low Fowler's position

A nurse who is assisting in the care of a client within the first 24 hours following a total gastrectomy for gastric cancer should focus interventions on which of the following?

Maintaining a patent nasogastric (NG) tube

A nurse is preparing to provide instructions to a client with glaucoma regarding the prescribed treatment measures for the disorder. The nurse prepares the instructions based on the primary objective of:

Maintaining intraocular pressure at a reduced level

A client who has been diagnosed with multiple myeloma asks the nurse about the diagnosis. The nurse bases the response on which characteristic of the disorder?

Malignant proliferation of plasma cells and tumors within the bone

The client scheduled for a right femoropopliteal bypass graft is at risk for compromised tissue perfusion to the extremity. The nurse takes which action before surgery to address this risk?

Marking the location of the pedal pulses on the right leg

A nurse is taking the nursing history of a client with silicosis. The nurse checks whether the client wears which of the following items during periods of exposure to silica particles?

Mask

A client has undergone fasciotomy to treat compartment syndrome of the leg. The nurse should anticipate that which type of wound care will be prescribed for the fasciotomy site?

Moist, sterile saline dressings

A client with pneumonia is admitted to the hospital, and the health care provider writes prescriptions for the client. Which of the following prescriptions written by the health care provider would the nurse complete first?

Obtain a culture and sensitivity of sputum.

The nurse is caring for a client undergoing peritoneal dialysis. The nurse checks the client and notes that the drainage from the outflow catheter is cloudy. The nurse notifies the registered nurse and plans to take which action?

Obtain a culture and sensitivity of the drainage

A client has experienced an episode of myasthenic crisis. The nurse collects data to determine whether the client has precipitating factors such as:

Omitted doses of medication

A nurse is caring for the restless client who keeps biting down on an orotracheal tube. The nurse uses which of the following to prevent the client from obstructing the airway with the teeth?

Oral airway

Which medications should the nurse administer to reduce nasal edema and rhinorrhea (thin watery discharge from the nose)? Select all that apply.

Oxymetazoline (Dristan) Pseudoephedrine (Sudafed)

A clinic nurse is reviewing the medical record of a client scheduled to be seen in the clinic. The nurse notes that the client is prescribed selegiline hydrochloride (Eldepryl). The nurse understands that this medication is prescribed for which diagnosis?

Parkinson's disease

A client has undergone total hip replacement of the right hip, which was damaged by osteoarthritis. Which of the following should be included in the postoperative plan of care?

Partial weight bearing on the operative leg is usually permitted 72 hours postoperatively.

A nurse is caring for a client who has a cast applied to the left lower leg. On data collection of the client, the nurse notes the presence of skin irritation from the edges of a cast. Which nursing intervention is appropriate?

Petal the cast edges with adhesive tape.

A nurse is caring for a client who sustained multiple fractures in a motor vehicle accident 12 hours ago. The client develops severe dyspnea, tachycardia, and mental confusion, and the nurse suspects fat embolism. The initial action of the nurse is to:

Place the client in a Fowler's position.

A client who is 36 hours post myocardial infarction has ambulated for the first time. The nurse determines that the client best tolerated the activity if which observation was made?

Preactivity pulse rate 86 beats per minute, postactivity pulse rate 94 beats per minute

A nurse is caring for a client with an internal radiation implant. When caring for the client, the nurse should observe which principle?

Pregnant women are not allowed into the client's room.

A nurse notes that a client with glaucoma has vision that is lost due to obstruction to aqueous humor flow by the trabecular meshwork. The nurse interprets that this client is suffering from:

Primary open-angle glaucoma

A nurse is preparing to care for a client following a lumbar puncture. The nurse plans to place the client in which position immediately after the procedure?

Prone with a pillow under the abdomen

A client with acquired immunodeficiency syndrome (AIDS) has difficulty swallowing. The nurse has given the client suggestions to minimize the problem. The nurse determines that the client has understood the instructions if the client verbalizes the intent to increase intake of which food(s)?

Puddings

A client with carcinoma of the lung develops the syndrome of inappropriate antidiuretic hormone (SIADH) as a complication of the cancer. The nurse anticipates that which of the following may be prescribed? Select all that apply.

Radiation, Chemotherapy, Serum sodium blood levels, Medication that is antagonistic to antidiuretic hormone (ADH)

A nurse is caring for a client who is being treated for a pneumothorax with a closed chest tube drainage system. When repositioning the client, the chest tube disconnects. The immediate nursing action is to:

Reattach the chest tube to the drainage system

An adult client has undergone lumbar puncture to obtain cerebrospinal fluid (CSF) for analysis. The nurse checks for which of the following negative values if the CSF is normal?

Red blood cells

A client with acute glomerulonephritis is admitted to the nursing unit. The nurse should plan to do which action immediately on admission?

Remove the water pitcher from the bedside

A client with a cervical spine injury has Crutchfield tongs applied in the emergency department. The nurse should avoid which of the following when planning care for this client?

Removing the weights to reposition the client

A nurse is caring for a client who had an above-the-knee amputation 2 days ago. The residual limb was wrapped with an elastic compression bandage that has fallen off. The nurse immediately:

Rewraps the residual limb with an elastic compression bandage

A client has undergone a right pneumonectomy. The nurse positioning this client following admission from the postanesthesia care unit avoids placing the client in which harmful position?

Right lateral

While at home, a nurse receives a telephone call from a neighbor, who reports that while accidentally breaking a mirror, a piece of glass flew into her eye. What is the appropriate initial nursing action after observing that the large glass shard is protruding from the neighbor's eye?

Secure a cup over the affected eye.

A nurse is caring for a client following craniotomy who has a supratentorial incision. The nurse reviews the client's plan of care, expecting to note that the client should be maintained in which of the following positions?

Semi-Fowler's position

A nurse is collecting data on a client with chronic sinusitis. The nurse interprets that which of the following client manifestations is unrelated to this problem?

Severe evening headache

A nurse is providing discharge instructions to the client with pulmonary sarcoidosis. The nurse knows that the client understands the information if the client verbalizes which early sign of exacerbation?

Shortness of breath

A client is fearful about having an arm cast removed. Which of the following actions by the nurse would be the most helpful?

Showing the client the cast cutter and explaining how it works

A client is about to undergo a lumbar puncture (LP). The nurse tells the client that which of the following positions will be used during the procedure?

Side-lying with the legs pulled up and the head bent down onto the chesT

The nurse prepares to care for a client with inflamed joints and plans to use which item to maintain proper positioning for rest of the inflamed joints?

Small pillows

A client who has undergone a subtotal gastrectomy is being prepared for discharge. Which item concerning ongoing self-management should the nurse reinforce to the client?

Smaller and more frequent meals should be eaten.

A client with a history of angina pectoris tells the nurse that chest pain usually occurs after going up two flights of stairs or after walking four blocks. The nurse interprets that the client is experiencing which of the following types of angina?

Stable

A nurse is suctioning a client through an endotracheal tube. During the suctioning procedure, the nurse notes on the cardiac monitor that the heart rate has dropped 10 beats. The nurse should:

Stop the procedure and oxygenate the client.

A nurse is inserting soft contact lenses into the eyes of a client. The nurse tells the client to look:

Straight ahead

A nurse provides dietary instructions to a client with Ménière's disease. The nurse tells the client that which food or fluid item is acceptable to consume?

Sugar-free Jell-O

When preparing a client for a pericardiocentesis, how does the nurse position the client?

Supine with the head of bed elevated at a 45- to 60-degree angle

A nurse who is providing instructions to a client following gastric resection would include which of the following suggestions

Take action to prevent dumping syndrome.

The nurse's teaching plan for a client with a family history of breast cancer should include which important item?

Teaching breast self-exam technique to be done every month

A nurse is assisting to perform a Romberg test on a client being seen in the clinic. The nurse performs this test to determine

The ability of the vestibular apparatus in the inner ear

A nurse instructs a client on pursed-lip breathing and asks the client to demonstrate the breathing technique. Which observation by the nurse indicates that the client is performing the technique correctly?

The client breathes out slowly through the mouth.

A nurse has completed nutritional counseling with an overweight client about weight reduction to modify the risk for coronary artery disease. The nurse would determine the teaching as successful if the client stated that a safe weight loss goal is:

Two pounds per week

A nurse is caring for a client with a diagnosis of gout. Which laboratory value would the nurse expect to note in the client?

Uric acid level of 8 mg/dL

A client with a diagnosis of congestive heart failure is preparing for discharge to home from the hospital. The nurse determines that the client is ready for discharge to home if the client can:

Verbally describe the daily medications, doses, and times to be administered.

A client with heart failure is scheduled to be discharged to home with digoxin (Lanoxin) and furosemide (Lasix) as ongoing prescribed medications. The nurse teaches the client to report which sign that indicates that the medications are not having the intended effect?

Weight gain of 2 to 3 pounds in a few days

When the nurse documents the results of a Snellen vision test as 20/80 vision, the client asks the nurse to describe what these numbers mean. Which statement is the appropriate response?

You can read at a distance of 20 feet what a client with normal vision can read at 80 feet."

A resident in a long-term care facility prepares to walk out into a rainstorm after saying, "My father is waiting to take me for a ride." Which of the following would be an appropriate response by the nurse?

"I'm glad you told me that. Let's have a cup of coffee and you can tell me about your father."

A client diagnosed with Lyme disease says to the nurse, "I heard this disease can affect the heart. Is this true?" The nurse should make which response to the client?

"It can, but you will be monitored closely for cardiac complications."

A myringotomy is performed on a client in the ambulatory care center. The ambulatory care nurse calls the client 24 hours after the procedure to evaluate the status of the client. The client reports to the nurse that a small amount of brownish drainage has been coming from the ear. Which instruction should the nurse provide to the client?

"Continue to monitor the drainage, because this is normal and may occur for 24 to 48 hours following the surgery."

A nurse is planning to teach the client with a left arm cast about measures to keep the left shoulder from becoming stiff. Which suggestion should the nurse include in the teaching plan?

"Lift the left arm up over the head."

A client is brought to the ambulatory care department by the spouse one day following a cataract extraction procedure. A diagnosis of hyphema is made, which occurred as a result of the surgical procedure. The nurse provides instructions to the client and spouse regarding the treatment for the complication and makes which statement to them?

"Maintain bedrest and patching of both eyes."

A maternity nursing instructor asks a nursing student to identify the hormones that are produced by the ovaries. Which of the following, if identified by the student, indicates an understanding of the hormones produced by this endocrine gland?

1. Oxytocin 2. Luteinizing hormone (LH) *3. Estrogen and progesterone* 4. Follicle-stimulating hormone (FSH) *rationale* The ovaries are the endocrine glands that produce estrogen and progesterone. Oxytocin is produced by the posterior pituitary gland and stimulates the uterus to produce contractions. LH and FSH are produced by the anterior pituitary gland.

A newborn is transferred to the neonatal intensive care unit with an admitting diagnosis of esophageal atresia accompanied by a distal tracheoesophageal fistula (TEF). When assisting to care for the newborn, the priority concern would be:

1. Pain 2. Infection *3. Aspiration* 4. The parents' concerns *Rationale:* Because TEF manifests itself with regurgitation and coughing, the concern that has the highest priority is aspiration. Although the other problems are an important part of care, the one with the highest concern relates to airway.

When reviewing the health care record of a client with ovarian cancer, the nurse recognizes which symptom as being typical of the disease?

Abdominal distention

A nurse teaching a group of adults about cancer warning signs presents to the group a list of the seven warning signs of cancer. The nurse determines that further teaching is necessary if a member of the group states that which of the following is a warning sign?

Absence or decreased frequency of menses

A nurse is reviewing the health care record of a client with a new onset of pleurisy. The nurse notes documentation that the client does not have a pleural friction rub, which was auscultated the previous day. The nurse interprets that this is likely a result of:

Accumulation of pleural fluid in the inflamed area

A client is experiencing an acute exacerbation of bursitis. The nurse encourages the client to avoid which of the following least helpful measures until the current episode is resolved?

Active intermittent range of motion

The nurse is preparing a list of home care instructions for the client who has been hospitalized and treated for tuberculosis. Choose the instructions that the nurse will include on the list. Select all that apply.

Activities should be resumed gradually. A sputum culture is needed every 2 to 4 weeks once medication therapy is initiated. Respiratory isolation is not necessary because family members have already been exposed. Cover the mouth and nose when coughing or sneezing and confine used tissues to plastic bags.

A postcardiac surgery client with a blood urea nitrogen (BUN) level of 45 mg/dL and a serum creatinine level of 2.2 mg/dL has a total 2-hour urine output of 25 mL. The nurse understands that the client is at risk for:

Acute renal failure

A client who suffered a crush injury to the leg has a highly positive urine myoglobin level. The nurse plans to monitor this particular client carefully for signs of which complication?

Acute tubular necrosis

A client with a history of angina pectoris complains of substernal chest pain. The nurse checks the client's blood pressure and administers nitroglycerin grains 1/150 sublingually. Five minutes later, the client is still experiencing chest pain. If the blood pressure is still stable, the nurse should take which action next?

Administer another nitroglycerin tablet.

A client who suffered a cervical spine injury had Crutchfield tongs applied in the emergency department. The nurse would avoid which of the following actions in the care of the client?

Removing the weights when repositioning the client

A nurse is caring for a client following segmental resection of the upper lobe of the left lung. The nurse notes 700 mL of grossly bloody drainage in the chest tube drainage system during the first hour following surgery. The nurse is aware that this finding:

Requires further data collection

An ultrasound of the gallbladder is scheduled for the client with a suspected diagnosis of cholecystitis. The nurse explains to the client that this test:

Requires the client to lie still for short intervals

A nurse checks the water seal chamber of a closed chest drainage system and notes fluctuations in the chamber. Based on this finding, the nurse determines that:

The system is functioning as expected.

A sublingual nitroglycerin (Nitrostat) tablet administered to a client for chest pain has relieved the pain. The nurse ensures that the client has which important item within easy reach before leaving the room?

Call bell

A nurse has given discharge instructions to the client who has had ocular surgery of the right eye. The nurse determines that the client needs further instruction if the client states to:

Call the health care provider if a temperature of 99° F is present.

A client with Parkinson's disease "freezes" while ambulating, increasing the risk for falls. Which suggestion should the nurse include in the client's plan of care to alleviate this problem?

Consciously think about walking over imaginary lines on the floor.

A nurse is caring for a client with pneumonia who has a history of bleeding esophageal varices. Based on this information, the nurse plans care, knowing that it is important to prevent:

Constipation

When reinforcing information to a client regarding how to appropriately care for a new hearing aid, the nurse would tell the client:

To check the battery regularly to ensure that it is working before use

A nurse is assigned to care for a client with a diagnosis of Ménière's disease. The nurse plans care, knowing that this condition is a disorder of the:

Inner ear

A nurse is caring for a client with acute otitis media. The nurse plans care, knowing that the treatment for this problem is likely to include:

Myringotomy

A client is experiencing blockage of the eustachian tubes. The nurse teaches the client that which of the following activities by the client may forcibly open the eustachian tube?

Performing the Valsalva maneuver

A nurse is assisting in caring for a client with a chest tube. The nurse understands that which of the following is an incorrect action for the care of the client?

Pin the tubing to the bed linens.

Which of the following interventions would be contraindicated in the postprocedure care of the client following a bone biopsy of the left arm?

Place the left arm in a dependent position for 24 hours.

A nurse provides medication instructions to a client with peptic ulcer disease. Which statement by the client indicates the best understanding of the medication therapy?

The nizatidine (Axid) will cause me to produce less stomach acid."

A nurse is speaking with a client with a hearing impairment. The nurse refrains from doing which of the following, which is least helpful when communicating with this client?

Using many exaggerated hand gestures while talking

A nurse is orienting a new nurse to the care of a client who has an internal radiation implant. The nurse includes which of the following instructions in discussions with the new nurse?

Pregnant women are not allowed in the client's room.

A client with a peptic ulcer is scheduled for a vagotomy, and the client asks the nurse about the purpose of this procedure. The nurse tells the client that a vagotomy:

Reduces the stimulation of acid secretions

A nurse is caring for a client who had a below-the-knee amputation of the right leg and has a cast on the residual limb. The client calls the nurse and reports that the cast fell off. The nurse immediately:

Wraps the residual limb with an elastic compression bandage

A nurse is assisting in the care of a client with myocardial infarction who should reduce intake of saturated fat and cholesterol. The nurse should help the client comply with diet therapy by selecting which of the following food items from the dietary menu?

Baked haddock, steamed broccoli, herbed rice, sliced strawberries

A nurse is planning a dietary menu for a client with heart failure being treated with digoxin (Lanoxin) and furosemide (Lasix). Which of the following would be the best dinner choice from the daily menu?

Baked pollack, mashed potatoes, and carrot-raisin salad

A nurse is preparing a client scheduled for a bone marrow aspiration, and the client asks the nurse whether the procedure will be painful. The nurse should make which response to the client?

"A local anesthetic will be given."

A nurse has completed counseling about smoking cessation with a client with coronary artery disease. The nurse determines that the client has understood the material best if the client states that:

"A smoker has twice the risk of having a heart attack than a nonsmoker."

A client with Parkinson's disease quickly develops akinesia while ambulating, increasing the risk for falls. Which suggestion should the nurse provide to the client to alleviate this problem?

"Consciously think about walking over imaginary lines on the floor."

A client arrives at the emergency department following an eye injury in which an acid used to clean the brick on the fireplace splashed into the eye. The initial question by the nurse is which of the following?

"Did you flush the eye following the injury?"

A nurse is teaching a group of high school males in a health class about how to perform a testicular self-examination (TSE). The nurse would make which of the following statements during the class?

"Do the examination after a warm bath or shower."

A nurse has instructed the client with myasthenia gravis about ways to manage his or her own health at home. The nurse determines that the client needs more information if the client makes which of the following statements?

"Going to the beach will be a nice, relaxing form of activity."

A client who is performing peritoneal dialysis at home calls the clinic and reports that the outflow from the dialysis catheter seems to be decreasing in amount. The nurse appropriately asks which question first?

"Have you experienced any constipation recently?"

A client reports to the health care clinic for an eye examination, and a diagnosis of primary open-angle glaucoma is suspected. Which data collection question will elicit information regarding the initial clinical manifestations associated with this disorder?

"Have you had difficulty with peripheral vision?"

A nurse is collecting data from a client who is admitted to the hospital for diagnostic studies to rule out the presence of Hodgkin's disease. Which question should the nurse ask the client to elicit information specifically related to this disease?

"Have you noticed any swollen lymph nodes?"

A female client with myasthenia gravis comes to the health care provider's office for a scheduled office visit. The client is very concerned and tells the nurse that her husband seems to be avoiding her because she is very unattractive. The appropriate nursing response is:

"Have you thought about sharing your feelings with your husband?"

A nurse is reinforcing discharge teaching to a client following right eye cataract surgery. The nurse determines that the client needs further instruction about ways to avoid strain on the operative eye when he makes which of the following statements?

"I can lie on my right side."

A nurse provides a list of instructions to a client with glaucoma regarding measures that will prevent an increase in intraocular pressure in the eyes. Which statement by the client indicates a need for further instructions?

"I can tie my shoelaces by bending over slowly."

A nurse is collecting data on a client with myasthenia gravis. The nurse determines that the client may be developing myasthenic crisis if the client states:

"I can't swallow very well today."

A client with a diagnosis of otosclerosis is admitted to the ambulatory care unit for stapedectomy, and the nurse provides instructions to the client regarding home care following the procedure. Which statement by the client indicates a need for further instructions?

"I need to avoid air travel for at least 6 months."

A nurse is providing discharge instructions to a client who has undergone transsphenoidal surgery for a pituitary adenoma. Which statement by the client indicates the client understands the discharge instructions?

"I need to call the doctor if I develop frequent swallowing or postnasal drip."

Indinavir (Crixivan) is prescribed for a client with human immunodeficiency virus (HIV). The nurse has reinforced instructions to the client regarding ways to maximize absorption of the medication. Which statement by the client indicates an adequate understanding of the use of this medication?

"I need to take the medication with water but on an empty stomach."

A nurse is providing client and family instructions for a client who has been recently diagnosed with glaucoma. Which statement indicates that the client's family member needs additional instruction regarding the eye drop application of pilocarpine hydrochloride (Isopto Carpine)? Select all that apply.

"I should apply the eye drops directly over my family member's pupil." I need to wipe off the tip of the eye drop bottle with a tissue between administrations." "I have to contact the prescriber if my family member develops a small pupil."

Which statements indicate an understanding of the necessary dietary modifications of a client diagnosed with chronic kidney disease? Select all that apply.

"I should avoid eggs, and a bagel is preferable." "I should consume approximately 40 g of protein daily."

A nurse provides instructions to a client at risk for thrombophlebitis regarding measures to minimize its occurrence. Which statement by the client indicates an understanding of this information?

"I should avoid sitting in one position for long periods of time."

The nurse provides home care instructions to a client undergoing hemodialysis with regard to care of an arteriovenous (AV) fistula. Which statement by the client indicates an understanding of the instructions?

"I should check the fistula every day by feeling it for a vibration."

A nurse has reinforced discharge teaching with a client who was diagnosed with tuberculosis (TB) and has been on medication for 1½ weeks. The nurse knows that the client has understood the information if the client makes which statement?

"I should not be contagious after 2 to 3 weeks of medication therapy."

The nurse, a Cub Scout leader, is preparing a group of Cub Scouts for an overnight camping trip and instructs them about the methods to prevent Lyme disease. Which statement by one of the Cub Scouts indicates a need for further teaching?

"I should not use insect repellent because it will attract the ticks."

A nurse has provided instructions to a client scheduled for a mammography regarding the procedure. Which statement by the client indicates an understanding of the procedure?

"I should not wear deodorant on the day of the test."

A nurse has reinforced dietary instructions to a client with coronary artery disease. Which statement by the client indicates an understanding of the dietary instructions?

"I should routinely use polyunsaturated oils in my diet."

A clinic nurse is providing instructions to a client regarding the use of a hearing aid. Which statement by the client indicates a need for further instructions?

"I should turn the hearing aid off after removing it from my ear."

A client has just had an application of a nonplaster (fiberglass) leg cast, and the nurse is giving the client instructions on cast care at home. Which statement by the client indicates the need for further instructions?

"I should use a hair dryer set to the hot setting to dry my cast if it gets wet."

A nurse has instructed a client diagnosed with tuberculosis (TB) about how to prevent the spread of infection after discharge. The nurse determines that the client needs further teaching if the client makes which of the following statements?

"I should use disposable plates, forks, and knives."

A nurse is assisting in preparing a client for discharge to home. Daily cold therapy has been prescribed for the client, and the nurse provides instructions about this treatment. Which statement by the client indicates adequate understanding of cold therapy treatment?

"I should wrap the frozen ice pack in a warm towel to help adjust to the cold."

A nurse is providing instructions to the client who has just been fitted for a halo vest. Which statement by the client indicates the need for further instructions?

"I will avoid driving at night because the vest limits the ability to turn the head."

A halo vest is applied to a client following a cervical spine fracture. The nurse provides instructions to the client regarding safety measures related to the vest. Which statement by the client indicates a need for further instructions?

"I will bend at the waist, keeping the halo vest straight to pick up items."

A nursing student prepares to instruct a client to expectorate a sample of sputum that will be sent to the laboratory for Gram stain, culture, and sensitivity and describes the procedure to the licensed practical nurse (LPN), who is the primary nurse. The LPN corrects the student if which incorrect description is provided?

"I will have the client take a shallow breath before coughing."

A nurse teaches skin care to the client who is receiving external radiation therapy. Which of the following statements, if made by the client, would indicate the need for further instruction?

"I will limit sun exposure to 1 hour daily."

Which statement by the client indicates a need for further education regarding the home care instructions for acute sinusitis?

"I will need surgery to drain my sinuses."

A nurse reinforces home care instructions to the postcraniotomy client. Which statement by the client indicates the need for further instruction?

"I will not hear sounds clearly unless they are loud."

A client with chronic atrial fibrillation is being started on amiodarone (Cordarone) as maintenance therapy for dysrhythmia suppression. A nurse provides instructions to the client about the medication. Which statement by the client indicates a need for further instruction?

"I will stop taking the prescribed anticoagulant after starting this new medication."

A nurse is providing discharge instructions for a client who has had a fenestration procedure for the treatment of otosclerosis. Which of the following, if stated by the client, would indicate an understanding of the instructions?

"I will take stool softeners as prescribed by my doctor."

A nurse is providing discharge teaching for a post myocardial infarction (MI) client who will be taking one baby aspirin a day. The nurse determines that the client understands the use of this medication if the client makes which statement?

"I will take this medication every day."

A nurse has given suggestions to the client with trigeminal neuralgia about strategies to minimize episodes of pain. The nurse determines that the client needs additional information if the client made which of the following statements?

"I will try to eat my food either very warm or very cold."

A client has received instructions on self-management of peritoneal dialysis. The nurse determines that the client needs further teaching if the client makes which statement?

"I will use a strong adhexive tape to anchor the catheter dressing."

A nurse determines that a client with coronary artery disease (CAD) has the necessary understanding of disease management if the client makes which statement?

"I will walk for one-half hour daily."

A nurse determines that a client with coronary artery disease (CAD) understands disease management if the client makes which of the following statements?

"I will walk for one-half hour daily."

A client is being discharged from the ambulatory care unit following cataract removal, and the nurse provides instructions regarding home care. Which of the following, if stated by the client, indicates an understanding of the instructions?

"I will wear my eye shield at night and my glasses during the day."

A nurse informs a client that a Papanicolaou smear will be done at the next scheduled clinic visit, and the nurse provides instructions to the client regarding preparation for this test. Which statement by the client indicates an understanding of the procedure?

"If I have my period at the time of my next scheduled visit, I will not be able to have the test done."

A nurse has reinforced instructions with the client with a nonplaster (fiberglass) leg cast about cast care at home. The nurse determines that the client needs further instruction if the client makes which statement?

"If the cast gets wet, I can dry it with a hair dryer turned to the warmest setting."

A nurse is discussing smoking cessation with a client diagnosed with coronary artery disease (CAD). Which statement would the nurse make to the client to try to motivate the client to quit smoking?

"If you quit now, your risk of cardiovascular disease will decrease to that of a nonsmoker in three to four years."

A nurse has reinforced instructions to a client following a right keratoplasty. Which statement by the client indicates a need for further instruction?

"In 1 week I'll return to have the sutures removed."

A client with possible rib fracture has never had a chest x-ray. The nurse would plan to tell the client which of the following items about the procedure?

"It is necessary to remove jewelry and any other metal objects."

A nurse provides discharge instructions to the client who was hospitalized for an acute attack of Ménière's disease. Which statement, if made by the client, indicates a need for further instructions?

"It is not necessary to restrict salt in my diet."

While a nurse is involved in preparing a client for a cardiac catheterization, the client says, "I don't want to talk with you. You're only a nurse. I want my doctor." Which response by the nurse would be therapeutic?

"So you're saying that you want to talk to your health care provider?"

A nurse is caring for a client hospitalized with an acute attack from Ménière's disease. The client verbalizes concern because the client has experienced a hearing loss as a result of the attack. Which response would the nurse make to the client regarding the hearing loss?

"The attack leaves a hearing loss in the involved ear."

A nursing student is caring for a hospitalized client with a diagnosis of lung cancer. The health care provider has prescribed a partial rebreathing face mask for the client, and the nursing instructor asks the student about its purpose. The student correctly responds by stating that:

"The device conserves oxygen by having the client rebreathe her own exhaled air."

A nurse is assisting a client who has just been given a hearing aid to wear for the first time. The nurse provides teaching about the device, including that:

"The hearing aid should not be worn if an ear infection is present."

A miotic medication has been prescribed for the client with glaucoma. The client asks the nurse about the purpose of the medication. The nurse tells the client that:

"The medication causes the pupil to constrict and will lower the pressure in the eye."

Prescriptive glasses are prescribed for a client with bilateral aphakia, and the nurse provides instructions to the client regarding the use of the glasses. Which statement by the client indicates the need for further instructions?

"The prescriptive glasses will correct my visual field of sight."

A client is seen in the health care provider's office for a physical examination after experiencing unusual fatigue over the last several weeks. Height is 5 feet, 8 inches, with a weight of 220 pounds. Vital signs are temperature 98.6° F oral, pulse 86 beats per minute, respirations 18 breaths per minute, and blood pressure 184/96 mm Hg. Random blood glucose is 110 mg/dL. In order to best collect relevant data, which question should the nurse ask the client first?

"When was the last time you had your blood pressure checked?"

A nurse finds a client tensing while lying in bed staring at the cardiac monitor. What is the nurse's best response when the client states, "There sure are a lot of wires around there. I sure hope we don't get hit by lightning!"?

"Yes, this equipment is a little scary. Can we talk about how the cardiac monitor works?"

A client had an aortic valve replacement 2 days ago. This morning, the client says to the nurse, "I don't feel any better than I did before surgery." The appropriate response by the nurse is:

"You are concerned that you don't feel any better after surgery?"

A client in skeletal leg traction with an overbed frame is not allowed to turn from side to side. Which action by the nurse would be most useful in trying to provide good skin care to the client?

"You have concerns about skeletal versus skin traction for your type of fracture?"

A client who is scheduled for surgery and who is to be placed in skeletal traction says to the nurse, "I'm not sure if I want to have this skeletal traction or if the skin traction would be best to stabilize my fracture." Based on the client's statement, the nurse should make which response to the client?

"You have concerns about skeletal versus skin traction for your type of fracture?"

A nurse is reinforcing discharge instructions with a client who is being discharged following a fenestration procedure for the treatment of otosclerosis. Which of the following should be included in the list of instructions prepared for the client?

"You need to avoid air travel."

A mother of a child brings the child to a clinic and reports that the child has a fever and has developed a rash on the neck and trunk. Roseola is diagnosed, and the mother is concerned that her other children will contract the disease. Which instruction should the nurse give to the mother to prevent the transmission of the disease?

*1. "Disease transmission is unknown."* 2. "The disease is transmitted through the urine and feces, so the other children should use a separate bathroom." 3. "The disease is transmitted through the respiratory tract, so the child should be isolated from the other children as much as possible." 4. "The disease is transmitted by contact with body fluids, so any items contaminated with body fluids need to discarded in a separate receptacle." *Rationale:* The method of transmission of roseola is unknown. Options 2, 3, and 4 are not correct transmission routes of roseola.

A nurse is collecting data on an infant with a diagnosis of suspected Hirschsprung's disease. Which of the following questions to the mother will most specifically elicit information regarding this disorder?

*1. "Does your infant have foul-smelling, ribbon-like stools?"* 2. "Is your infant constantly vomiting?" 3. "Does your infant constantly spit up feedings?" 4. "Does your infant have diarrhea?" *Rationale:* Chronic constipation, beginning in the first month of life and resulting in pellet-like or ribbon-like stools that are foul smelling, is a clinical manifestation of Hirschsprung's disease. Delayed passage or absence of meconium stool in the neonatal period is the cardinal sign. Bowel obstruction, especially in the neonatal period, abdominal pain and distention, and failure to thrive are also clinical manifestations. Options 2, 3, and 4 are not specific clinical manifestations of this disorder.

A nurse provides instructions to parents regarding the methods that will decrease the risk of recurrent otitis media in infants. Which of the following should the nurse include in the instructions?

*1. "Feed the infant in an upright position."* 2. "Maintain bottle-feeding as long as possible." 3. "Discontinue breast-feeding as soon as possible." 4. "Allow the infant to have a bottle during nap time." *Rationale:* To decrease the risk of recurrent otitis media, parents should be encouraged to breast-feed during infancy, discontinue bottle-feeding as soon as possible, feed the infant in an upright position, and avoid giving the infant a bottle in bed. Parents should be told not to smoke in the child's presence because passive smoking increases the incidence of otitis media.

A nurse is reviewing discharge teaching with a client who has Cushing's syndrome. Which statement by the client indicates that the instructions related to dietary management were understood?

*1. "I can eat foods that contain potassium."* 2. "I will need to limit the amount of protein in my diet." 3. "I am fortunate that I can eat all the salty foods I enjoy." 4. "I am fortunate that I do not need to follow any special diet." *rationale* A diet that is low in calories, carbohydrates, and sodium but ample in protein and potassium content is encouraged for a client with Cushing's syndrome. Such a diet promotes weight loss, the reduction of edema and hypertension, the control of hypokalemia, and the rebuilding of wasted tissue.

Which statement by the client would cause the nurse to suspect that the thyroid test results drawn on the client this morning may be inaccurate?

*1. "I had a radionuclide test done 3 days ago."* 2. "When I exercise I sweat more than normal." 3. "I drank some water before the blood was drawn." 4. "That hamburger I ate before the test sure tasted good." *rationale* Option 1 indicates that a recent radionuclide scan had been performed. Recent radionuclide scans performed before the test can affect thyroid laboratory results. No food, fluid, or activity restrictions are required for this test, so options 2, 3, and 4 are incorrect.

A nurse is assigned to care for a client at home who has a diagnosis of type 1 diabetes mellitus. When the nurse arrives to care for the client, the client tells the nurse that she has been vomiting and has diarrhea. Which additional statement by the client indicates a need for further teaching?

*1. "I need to stop my insulin."* 2. "I need to increase my fluid intake." 3. "I need to call my health care provider." 4. "I need to monitor my blood glucose every 4 to 6 hours." *rationale* When a client with diabetes is unable to eat normally because of illness, the client should still take the prescribed insulin or oral medication. Additional fluids should be consumed and a call placed to the health care provider. The client should monitor the blood glucose levels every 4 to 6 hours.

A nurse provides instructions to the mother of an infant with cleft palate regarding feeding. Which statement by the mother indicates a need for further instructions?

*1. "I need to use a nipple with a small hole to prevent choking."* 2. "I need to stimulate sucking by rubbing the nipple on the lower lip." 3. "I need to allow my infant time to swallow." 4. "I need to allow my infant to rest frequently to provide time for swallowing what has been placed in the mouth." *Rationale:* The mother should be taught the ESSR method of feeding the child with a cleft palate: ENLARGE the nipple by cross-cutting a hole so that food is delivered to the back of the throat without sucking; STIMULATE sucking by rubbing the nipple on the lower lip; SWALLOW; then REST to allow the infant to finish swallowing what has been placed in the mouth.

A nurse has provided discharge instructions to the mother of an 18-month-old child following surgical repair of hypospadias. Which statement by the mother indicates a need for further instruction?

*1. "I should carry my child by straddling the child on my hip."* 2. "I should use double diapers to hold the surgery site in place." 3. "I should avoid toilet training right now." 4. "I should encourage fluid intake." *Rationale:* Parent teaching following hypospadias repair includes restricting the child from activities that put pressure on the surgical site. Straddling the child on the hip will cause pressure on the surgical site. The parents should be instructed to use double diapers to hold the stent in place and should be instructed how to hold the child during the postoperative period. Fluids should be encouraged to maintain hydration. Toilet training should not be an issue during this stressful period.

A nurse has reinforced home care measures to a client diagnosed with diabetes mellitus regarding exercise and insulin administration. Which statement by the client indicates a need for further instruction?

*1. "I should perform my exercise at peak insulin time."* 2. "I should always carry a quick-acting carbohydrate when I exercise." 3. "I should always wear a Medic-Alert bracelet especially when I exercise." 4. "I should avoid exercising at times when a hypoglycemic reaction is likely to occur." *rationale* The client should be instructed to avoid exercise at peak insulin time because this is when a hypoglycemic reaction is likely to occur. If exercise is performed at this time, the client should be instructed to eat an hour before the exercise and drink a carbohydrate liquid. Options 2, 3, and 4 are correct statements regarding exercise, insulin, and diabetic control.

A nurse reinforces instructions to the mother of a child who has been hospitalized with croup. Which of the following statements, if made by the mother, would indicate the need for further instruction? 1. "I will give my child cough syrup if a cough develops." 2. "During an attack, I will take my child to a cool location." 3. "I will give acetaminophen (Tylenol) if my child develops a fever." 4. "I will be sure that my child drinks at least three to four glasses of fluids every day."

*1. "I will give my child cough syrup if a cough develops."* *Rationale:* Cough syrups and cold medicines are not to be given, because they may dry and thicken secretions. During a croup attack, the child can be taken to a cool basement or garage. Acetaminophen is used if a fever develops. Adequate hydration of 500 to 1000 mL of fluids daily is important for thinning secretions.

A nurse reinforces instructions to the mother of a child with croup about the measures to take if an acute spasmodic episode occurs. Which statement by the mother indicates the need for further instruction? 1. "I will place a steam vaporizer in my child's room." 2. "I will take my child out into the cool, humid night air." 3. "I will place a cool-mist humidifier in my child's room." 4. "I will place my child in a closed bathroom and allow my child to inhale steam from the running water."

*1. "I will place a steam vaporizer in my child's room."* *Rationale:* Steam from warm running water in a closed bathroom and cool mist from a bedside humidifier are effective for reducing mucosal edema. Cool-mist humidifiers are recommended as compared with steam vaporizers, which present a danger of scalding burns. Taking the child out into the cool, humid night air may also relieve mucosal swelling. Remember, however, that a cold mist may precipitate bronchospasm.

A nurse is evaluating the parent's understanding of discharge care regarding the functioning of the infant's ventricular peritoneal shunt. Which statement by a parent indicates an understanding of the shunt complications?

*1. "If my baby has a high-pitched cry, I should call the doctor."* 2. "I should position my baby on the side with the shunt when sleeping." 3. "My baby will pass urine more often now that the shunt is in place." 4. "I should call my doctor if my baby refuses purees." *Rationale:* If the shunt is broken or malfunctioning, the fluid from the ventricle part of the brain will not be diverted to the peritoneal cavity. The cerebrospinal fluid will build up in the cranial area. The result is intracranial pressure, which then causes a high-pitched cry in the infant. The baby should not have pressure when on the shunt side. Skin breakdown and possible compression to the apparatus could result. This type of shunt affects the gastrointestinal system, not the genitourinary system. Option 4 is only a concern if the baby becomes malnourished or dehydrated, which could then raise the body temperature. Otherwise, refusal to eat purees has no direct relationship to the shunt functioning.

A nurse is assisting in collecting data on a child with seizures. The nurse is interviewing the child's parents to establish their adjustment to caring for their child with a chronic illness. Which statement by a parent would indicate a need for further teaching? 1. "Our child sleeps in our bedroom at night." 2. "We worry about injuries when our child has a seizure." 3. "Our child is involved in a swim program with neighbors and friends." 4. "Our babysitter just completed cardiopulmonary resuscitation (CPR) training."

*1. "Our child sleeps in our bedroom at night."* *Rationale:* Parents are especially concerned about seizures that might go undetected at night. The nurse should suggest a baby monitor. Reassurance by the nurse should ensure parental confidence and decrease parental overprotection. Option 2 is a common concern. Options 3 and 4 demonstrate the parents' ability to choose respite care and activities appropriately. The parents need to be reminded that, as the child grows, they cannot always observe their child, but their knowledge of seizure activity and care are appropriate to minimize complications.

A 4-year-old child sustains a fall at home and is brought to the emergency department by the mother. Following x-ray examination, it has been determined that the child has a fractured arm, and a plaster cast is applied. The nurse provides instructions to the mother regarding cast care for the child. Which statement by the mother indicates a need for further instruction?

*1. "The cast will feel warm when it is dried."* 2. "If the cast becomes wet, a fan may be used to dry the cast." 3. "I need to call the health care provider if any blood or drainage appears on the cast." 4. "I can apply ice to the casted area to prevent swelling." *Rationale:* Once the cast dries, the cast will sound hollow and will be cool (not warm) to touch. A fan can be directed toward the cast to facilitate drying. The mother must be instructed to call the health care provider if any blood or drainage appears on the cast. Ice can be applied to the casted area to prevent swelling.

A child with croup is placed in a cool-mist tent. The mother asks if the child may have her security blanket inside the tent. The appropriate response is:

*1. "The child may have the security blanket inside the tent."* 2. "Objects from home are not allowed to be brought to the hospital." 3. "The blanket is not allowed because it will promote the growth of bacteria." 4. "The blanket is not allowed but the child may have a toy from the hospital playroom." *Rationale:* Familiar objects provide a sense of security for children in the strange hospital environment. The child is allowed to have a favorite toy or blanket while in the mist tent. Options 2, 3, and 4 are inappropriate statements.

A nurse is providing instructions to the mother of a child with juvenile idiopathic arthritis regarding measures to take if a painful exacerbation of the disease occurs. Which statement by the mother indicates the need for further instruction?

*1. "The full range-of-motion (ROM) exercises must be performed every day, even during the exacerbations."* 2. "Hot or cold packs will assist in reducing discomfort." 3. "The painful joint should be splinted and positioned in a neutral position." 4. "I should have my child perform simple isometric exercises during exacerbations." *Rationale:* During painful episodes, hot or cold packs and splinting and positioning the affected joint in a neutral position help reduce the pain. Full ROM exercises will cause significant pain during exacerbation and should be avoided during this time. Although resting the extremity is appropriate, it is important to begin simple isometric or tensing exercises as soon as the child is able. These exercises do not involve joint movement.

A 13-year-old child is diagnosed with osteogenic sarcoma of the femur. Following a course of chemotherapy, it is decided that leg amputation is necessary. Following the amputation, the child becomes very frightened because of aching and cramping felt in the missing limb. Which statement made by the nurse will best assist in alleviating the child's fear?

*1. "This aching and cramping is normal and temporary and will subside."* 2. "The pain medication that I give you will take these feelings away." 3. "This always occurs after the surgery and we will teach you ways to deal with it." 4. "This pain is not real pain and relaxation exercises will help it go away." *Rationale:* Following amputation, phantom limb pain is a temporary condition that some children may experience. This sensation of aching or cramping in the missing limb is most distressing to the child. The child needs to be reassured that the condition is normal and only temporary.

A nurse is providing instructions to a child with cystic fibrosis regarding how to perform the "huff" maneuver. The child asks the nurse about the purpose of this type of breathing. The appropriate nursing response is which of the following?

*1. "This type of breathing is used to mobilize secretions so that they can be easily coughed out."* 2. "This type of breathing prolongs inspiration time." 3. "This type of breathing moves air out of the lower lungs." 4. "This type of breathing moves air through the lungs." *Rationale:* The "huff" maneuver (forced expiratory technique) is used to mobilize secretions. This technique reduces the likelihood of bronchial collapse. The child is taught to cough with an open glottis by taking a deep breath, then exhaling rapidly whispering the word, "huff." Options 2, 3, and 4 are not the purpose of this breathing technique.

Antibiotics are prescribed for a child following a myringotomy with insertion of tympanostomy tubes, and the nurse provides instructions to the parents regarding the administration of the antibiotics. Which statement, if made by a parent, would indicate that the instructions were understood?

*1. "We will administer the antibiotics until they are gone."* 2. "We will administer the antibiotics if the child has a fever." 3. "We will administer the antibiotics until the child feels better." 4. "We will begin to taper the antibiotics after 3 days of a full course." *Rationale:* Antibiotics need to be taken as prescribed, and the full course needs to be completed. It is important that parents are instructed regarding the administration of antibiotics. Options 2, 3, and 4 are incorrect. Antibiotics are not tapered but administered until they are completed.

Which statement should the nurse include when providing safety instructions to the parents of an infant with a diagnosis of hydrocephalus?

*1. "When picking up your infant, support the infant's neck and head with the open palm of your hand."* 2. "Feed your infant in a side-lying position." 3. "Place a helmet on your infant when in bed." 4. "Hyperextend your infant's head with a rolled blanket under the neck area." *Rationale:* Hydrocephalus is a condition characterized by an enlargement of the cranium caused by an abnormal accumulation of cerebrospinal fluid within the cerebral ventricular system. This characteristic causes the increase in the weight of the infant's head. The infant's head becomes top heavy. Supporting the infant's head and neck when picking it up prevents hyperextension of the neck area and the infant from falling backward. The infant should be fed with the head elevated for proper motility of food processing. A helmet could suffocate an unattended infant during rest and sleep times, and hyperextension of the infant's head can put pressure on the neck vertebrae, causing injury.

A nurse is monitoring the laboratory values of a child with leukemia who is receiving chemotherapy. The nurse prepares to implement bleeding precautions if the child becomes thrombocytopenic and the platelet count is less than:

*1. 20,000/mm3* 2. 100,000/mm3 3. 120,000/mm3 4. 150,000/mm3 *Rationale:* If a child is severely thrombocytopenic, with a platelet count of less than 20,000/mm3, precautions need to be taken because of the increased risk of bleeding. The precautions include limiting activity that could result in head injury, using soft toothbrushes or Toothettes, checking urine and stools for blood, and administering stool softeners to prevent straining with constipation. In addition, suppositories and rectal temperatures are avoided. The normal platelet count ranges from 150,000 to 400,000/mm3.

A nurse is caring for a child who is scheduled for an appendectomy. When the nurse reviews the health care provider's preoperative prescriptions, which of the following would be questioned?

*1. Administer a Fleet enema.* 2. Maintain nothing per mouth (NPO) status. 3. Maintain intravenous (IV) fluids as prescribed. 4. Administer preoperative medication on call to the operating room. *Rationale:* In the preoperative period, enemas or laxatives should not be administered. No heat should be applied to the abdomen because this may increase the chance of perforation secondary to vasodilation. IV fluids would be started and the child would be NPO. Prescribed preoperative medications most likely would be administered on call to the operating room.

A nurse is assigned to assist in caring for a client admitted to the emergency department with diabetic ketoacidosis (DKA). In the acute phase, the priority nursing action is to plan to prepare to:

*1. Administer intravenous (IV) regular insulin.* 2. Administer IV 5% dextrose. 3. Correct the acidosis. 4. Apply an electrocardiogram (ECG) monitor. *rationale* Lack (absolute or relative) of insulin is the primary cause leading to DKA. Treatment consists of IV fluids (normal saline initially), regular insulin administration, and potassium replacement, followed by correcting the acidosis. An ECG monitor may be applied but is not the priority in this situation.

A client has just been admitted with a diagnosis of myxedema coma. If all of the following interventions were prescribed, the nurse would place highest priority on completing which of the following first?

*1. Administering oxygen* 2. Administering thyroid hormone 3. Warming the client 4. Giving fluid replacement *rationale* As part of maintaining a patent airway, oxygen would be administered first. This would be quickly followed by fluid replacement, keeping the client warm, monitoring vital signs, and administering thyroid hormones.

A nurse obtains a health history from a mother of a 15-month-old child before administering a measles, mumps, and rubella (MMR) vaccine. Which of the following is essential information to obtain before the administration of this vaccine?

*1. Allergy to eggs* 2. A recent cold 3. The presence of diarrhea 4. Any recent ear infections *Rationale:* Before the administration of MMR vaccine, a thorough health history needs to be obtained. MMR is used with caution in a child with a history of allergy to gelatin or eggs because the live measles vaccine is produced by chick embryo cell culture. MMR also contains a small amount of the antibiotic neomycin. Options 2, 3, and 4 are not contraindications to administering this immunization.

An adolescent is seen in the emergency department following an athletic injury, and it is suspected that the child sprained an ankle. X-rays are taken, and a fracture has been ruled out. The nurse provides instructions to the adolescent regarding home care for treatment of the sprain and tells the adolescent which of the following?

*1. Apply ice to the injured area for a period of 30 minutes every 4 to 6 hours for the first 24 hours.* 2. Apply heat to the injured area every 4 hours for the first 48 hours, and then begin to apply ice. 3. Immobilize the extremity and maintain the extremity in a dependent position. 4. Elevate the extremity and maintain strict bedrest for a period of 7 days. *Rationale:* To treat a sprain, the injured area should be wrapped immediately to support the joint and control the swelling. Ice is applied to reduce the swelling and should be applied for no longer than 30 minutes every 4 to 6 hours for the first 24 to 48 hours. The joint should be immobilized and elevated, but strict bedrest for a period of 7 days is not required. A dependent position will cause swelling in the affected area.

An emergency department nurse is gathering initial data on a child suspected of epiglottitis. The nurse's priority would be to:

*1. Assess for a patent airway.* 2. Prepare the child for an x-ray. 3. Prepare the child for tracheotomy. 4. Assist the health care provider with intubation. *Rationale:* When epiglottitis is suspected, the priorities are to maintain a patent airway and to next obtain an x-ray to confirm the diagnosis. If epiglottitis is present, the child is taken promptly to the operating room for tracheal intubation. Although options 2, 3, and 4 may be components of care, they are not the priority.

A 5-year-old child has been transferred to the pediatric unit after a cardiac catheterization. Which of the following interventions has the highest priority in the care of this child immediately following the procedure?

*1. Assess for any bleeding on the dressing.* 2. Position the child's leg so that it is straight. 3. Assess the strength and presence of the distal pulses. 4. Take the vital signs including blood pressure and oxygen saturation. *Rationale:* Bleeding is a primary concern following this procedure. Although options 2, 3, and 4 are correct interventions, they are not the priority.

A nursing student caring for a 6-month-old infant is asked to collect a urine specimen from the infant. The student collects the specimen by:

*1. Attaching a urinary collection device to the infant's perineum for collection* 2. Obtaining the specimen from the diaper by squeezing the diaper after the infant voids 3. Catheterizing the infant using the smallest available French Foley catheter 4. Noting the time of the next expected voiding and preparing to collect the specimen into a cup when the infant voids *Rationale:* Although many methods have been used to collect urine from an infant, the most reliable method is the urine collection device. This device is a plastic bag that has an opening that is lined with adhesive so that it may be attached to the perineum. Urine for certain tests, such as specific gravity, may be obtained from a diaper by collection of the urine with a syringe. Urinary catheterization is not to be done unless specifically prescribed because of the risk of infection. It is not reasonable to try to identify the time of the next voiding to attempt to collect the specimen.

A nurse is assigned to care for a child admitted to the hospital with a diagnosis of suspected bacterial endocarditis. The nurse prepares the child for which of the following diagnostic tests that will confirm the diagnosis?

*1. Blood cultures* 2. Chest x-ray 3. Echocardiogram 4. Transesophageal echocardiography *Rationale:* When endocarditis is suspected, a definitive diagnosis is achieved through blood cultures. A negative blood culture does not rule out the existence of endocarditis; it just indicates a lesser likelihood of its existence. A chest x-ray, echocardiogram, and transesophageal echocardiography are performed to aid in the diagnosis of endocarditis.

A school nurse is preparing a physical education plan for a child with Down syndrome. Before preparing the plan, the nurse obtains a copy of an x-ray report of the child's:

*1. Cervical spine* 2. Hands 3. Heart 4. Chest and lungs *Rationale:* Children with Down syndrome frequently have instability of the space between the first two cervical vertebrae. They require diagnostic studies (an x-ray of the cervical spine) to determine if this is present before participating in activities that put pressure on the head and neck, which could cause spinal cord compression. Options 2, 3, and 4 are not necessary.

A nurse has reviewed the health care provider's prescriptions for a child suspected of a diagnosis of neuroblastoma and is preparing to implement diagnostic procedures that will confirm the diagnosis. The nurse prepares to:

*1. Collect a 24-hour urine sample.* 2. Perform a neurological assessment. 3. Send the child to the radiology department for a chest x-ray. 4. Assist with a bone marrow aspiration. *Rationale:* Neuroblastoma is a solid tumor found only in children. It arises from neural crest cells that develop into the sympathetic nervous system and the adrenal medulla. Typically the tumor infringes on adjacent normal tissue and organs. Neuroblastoma cells may excrete catecholamines and their metabolites. Urine samples will indicate elevated vanillylmandelic acid levels. A bone marrow aspiration will assist in determining marrow involvement. A neurological examination and a chest x-ray may be performed but will not confirm the diagnosis.

A nurse is providing discharge instructions to the mother of a child who had a myringotomy with insertion of tympanostomy tubes. The nurse instructs the mother that if the tubes fall out, she should:

*1. Contact the health care provider.* 2. Bring the child to the emergency department immediately. 3. Replace them immediately. 4. Immediately immerse the tubes in half-strength hydrogen peroxide. *Rationale:* The size and appearance of the tympanostomy tubes should be described to the parents following surgery. They should be reassured that if the tubes fall out, it is not an emergency, but the health care provider should be notified. Therefore options 2, 3, and 4 are incorrect.

A nurse is monitoring a child with a head injury. On data collection, the nurse notes an abnormal flexion of the upper extremities and an extension of the lower extremities. The nurse documents that the child is experiencing:

*1. Decorticate posturing* 2. Decerebrate posturing 3. Flexion of the arms and legs 4. An expected position post-head injury *Rationale:* Decorticate posturing is an abnormal flexion of the upper extremities and an extension of the lower extremities with possible plantar flexion of the feet. Decerebrate posturing is an abnormal extension of the upper extremities with internal rotation of the upper arms and wrists, and an extension of the lower extremities with some internal rotation.

A nurse teaches a child with cystic fibrosis how to perform the "huff" maneuver and tells the child to take a:

*1. Deep breath then exhale, rapidly whispering the word "huff"* 2. Shallow breath then exhale, rapidly whispering the word "huff" 3. Deep breath, hold it for 15 seconds, then exhale slowly, whispering the word "huff" 4. Shallow breath, hold it for 10 seconds, then exhale rapidly, whispering the word "huff" *Rationale:* The "huff" maneuver (forced expiratory technique) is used to mobilize secretions. This technique reduces the likelihood of bronchial collapse. The child is taught to cough with an open glottis by taking a deep breath, then exhaling rapidly, whispering the word "huff."

A nurse is assisting in preparing a care plan for a client with diabetes mellitus who has hyperglycemia. The nurse focuses on which potential problem for this client?

*1. Dehydration* 2. The need for knowledge about the causes of hyperglycemia 3. Lack of knowledge about nutrition 4. Inability of family to cope with the client's diagnosis *rationale* Increased blood glucose will cause the kidneys to excrete the glucose in the urine. This glucose is accompanied by fluids and electrolytes, causing an osmotic diuresis that leads to dehydration. This fluid loss must be replaced when it becomes severe. Options 2, 3, and 4 may be concerns at some point but are not the priority with hyperglycemia.

A nurse is collecting data on a client with a diagnosis of hypothyroidism. Which of these behaviors, if present in the client's history, would the nurse determine as being likely related to the manifestations of this disorder?

*1. Depression* 2. Nervousness 3. Irritability 4. Anxiety *rationale* Hypothyroid clients experience a slow metabolic rate, and its manifestation includes apathy, fatigue, sleepiness, and depression. Options 2, 3, and 4 identify the clinical manifestations of hyperthyroidism.

A nurse is checking the capillary refill of a child with a cast applied to the left arm. The nurse compresses the nail bed of a finger and it returns to its original color in 2 seconds. Which action should be taken by the nurse? 1. Document the findings. 2. Notify the HCP. 3. Prepare the child for bivalving the cast. 4. Elevate the extremity and recheck the capillary refill immediately.

*1. Document the findings.* *Rationale:* When checking capillary refill, the nurse would expect to note that a compressed nail bed will return to its original color in less than 3 seconds. Options 2, 3, and 4 are unnecessary actions.

A nurse is assigned to care for a child after a myringotomy with the insertion of tympanostomy tubes. The nurse notes a small amount of reddish drainage from the child's ear after the surgery. On the basis of this finding, the nurse takes which action? 1. Documents the findings 2. Notifies the HCP immediately 3. Changes the ear tubes so that they do not become blocked 4. Checks the ear drainage for the presence of cerebrospinal fluid

*1. Documents the findings* *Rationale:* After a myringotomy with the insertion of tympanostomy tubes, the child is monitored for ear drainage. A small amount of reddish drainage is normal during the first few days after surgery. However, any heavy bleeding or bleeding that occurs after 3 days should be reported. The nurse would document the findings. Options 2, 3, and 4 are not necessary.

A school-age child with type 1 diabetes mellitus has soccer practice three afternoons a week. The nurse reinforces instructions regarding how to prevent hypoglycemia during practice. The nurse tells the child to: 1. Drink a half a cup of orange juice before soccer practice. 2. Eat twice the amount that is normally eaten at lunchtime. 3. Take half of the amount of prescribed insulin on practice days. 4. Take the prescribed insulin at noontime rather than in the morning.

*1. Drink a half a cup of orange juice before soccer practice.* *Rationale:* An extra snack of 10 g to 15 g of carbohydrates eaten before activities and for every 30 to 45 minutes of activity will prevent hypoglycemia. A half cup of orange juice will provide the needed carbohydrates. The child or parents should not be instructed to adjust the amount or time of insulin administration, and meal amounts should not be doubled.

A child with sickle cell anemia is being discharged after treatment for a crisis. Which instructions should the nurse provide to prevent another crisis from occurring? *Select all that apply.*

*1. Drink plenty of fluids.* 2. Avoid foods high in folic acid. 3. Use cold packs to relieve joint pain. 4. Restrict all activity to quiet board games. *5. Wash hands before meals and after playing.* *6. Report a sore throat immediately.* *Rationale:* Sickle cell crisis can be precipitated by cold, dehydration, stress, or infection. Increasing the amount of fluids will reduce the viscosity of blood, thus preventing vascular occlusion. A conscious effort to wash hands can improve the child's health by preventing infection. A sore throat is a sign of an infection and must be reported. It is important to avoid cold temperatures of any kind because this can cause vaso-occlusion. Folic acid avoidance is not necessary. Children need to be encouraged to set their own limits in play.

A nurse is reinforcing dietary instructions to a client newly diagnosed with diabetes mellitus. The nurse instructs the client that it is best to:

*1. Eat meals at approximately the same time each day.* 2. Adjust mealtimes depending on blood glucose levels. 3. Vary mealtimes if insulin is not administered at the same time every day. 4. Avoid being concerned about the time of meals as long as snacks are taken on time. *rationale* Mealtimes must be approximately the same time each day to maintain a stable blood glucose level. The client should not be instructed that mealtimes are varied, depending on blood glucose levels or insulin administration. Mealtimes should not be adjusted based on blood glucose levels or snacks.

A female adolescent with type 1 diabetes mellitus will become a member of the school's football cheerleader team. The adolescent excitedly reports to the school nurse to obtain information regarding adjustments needed in the treatment plan for the diabetes. The school nurse would instruct the adolescent to:

*1. Eat six graham crackers or drink a cup of orange juice before practice or game time.* 2. Eat half the amount of food normally eaten at lunchtime. 3. Take the prescribed insulin one half hour before practice or game time rather than in the morning. 4. Take two times the amount of prescribed insulin on practice and game days. *Rationale:* An extra snack of 15 to 30 g of carbohydrate eaten before activities such as cheerleader practice will prevent hypoglycemia. Six graham crackers or a cup of orange juice will provide 15 to 30 g of carbohydrate. The adolescent should not be instructed to adjust the amount or time of insulin administration. Meal amounts should not be decreased.

A client with a diagnosis of diabetic ketoacidosis (DKA) is being treated in the emergency department. Which of the following findings would the nurse expect to note as confirming this diagnosis?

*1. Elevated blood glucose and low plasma bicarbonate* 2. Decreased urine output 3. Increased respirations and an increase in pH 4. Coma *rationale* In DKA, the arterial pH is less than 7.35, plasma bicarbonate is less than 15 mEq/L, the blood glucose level is higher than 250 mg/dL, and ketones are present in the blood and urine. The client would be experiencing polyuria and Kussmaul's respirations. Coma may occur if DKA is not treated, but coma would not confirm the diagnosis.

A child with a brain tumor returns from the recovery room following "debulking" of the tumor. The nurse assigned to care for the child monitors the child for brainstem involvement. Which of the following signs would indicate that brainstem involvement occurred during the surgical procedure?

*1. Elevated temperature* 2. Orthostatic hypotension 3. Inability to swallow 4. Altered hearing ability *Rationale:* Vital signs and neurological status are checked frequently. Special attention is paid to the child's temperature, which may be elevated because of hypothalamus or brainstem involvement during surgery. A cooling blanket should be in place on the bed or be readily available if the child becomes hyperthermic. Options 3 and 4 are related to functional deficits following surgery. An elevated blood pressure and a widened pulse pressure may be associated with increasing intracranial pressure.

A diagnostic workup is performed on a 1-year-old child suspected of a diagnosis of neuroblastoma. Which finding specifically associated with this type of tumor should the nurse expect to find documented in the child's record?

*1. Elevated vanillylmandelic acid (VMA) levels in the urine* 2. The presence of blast cells in the bone marrow 3. Projectile vomiting occurring often in the morning 4. Positive Babinski's sign *Rationale:* Neuroblastoma is a solid tumor found only in children. It arises from neural crest cells that develop into the sympathetic nervous system and the adrenal medulla. Typically the tumor infringes on adjacent normal tissue and organs. Neuroblastoma cells may excrete catecholamines and their metabolites. Urine samples will indicate elevated VMA levels. The presence of blast cells in the bone marrow occurs in leukemia. Projectile vomiting occurring most often in the morning and a positive Babinski's sign are clinical manifestations of a brain tumor.

A nurse is assisting in developing a plan of care for a diagnosed with acute glomerulonephritis. The nurse includes which intervention in the plan of care?

*1. Encourage limited activity and provide safety measures.* 2. Force intake of oral fluids to prevent hypovolemic shock. 3. Catheterize the child to strictly monitor intake and output. 4. Encourage classmates to visit and to keep the child informed of school events. *Rationale:* Activity is limited and most children, because of fatigue, voluntarily restrict their activities during the active phase of the disease. Catheterization may cause a risk of infection. Fluids should not be forced. Visitors should be limited to allow for adequate rest.

A nurse is reviewing a plan of care for a client with Addison's disease. The nurse notes that the client is at risk for dehydration and suggests nursing interventions that will prevent this occurrence. Which nursing intervention is an appropriate component of the plan of care? *Select all that apply.*

*1. Encouraging fluid intake of at least 3000 mL/day* 2. Encouraging an intake of low-protein foods *3. Monitoring for changes in mental status* *4. Monitoring intake and output* 5. Maintaining a low-sodium diet *rationale* The client at risk for deficient fluid volume should be encouraged to eat regular meals and snacks and to increase the intake of sodium, protein, and complex carbohydrates. Oral replacement of sodium losses is necessary, and maintenance of adequate blood glucose levels is required.

A 1-year-old child with hypospadias is scheduled for surgery to correct this condition. A nurse is asked to assist in preparing a plan of care for this child and makes suggestions, knowing that this surgery is taking place at a time when:

*1. Fears of separation and mutilation are present* 2. Sibling rivalry will cause regression to occur 3. Embarrassment of voiding irregularities is common 4. Concern over size and function of the penis is present *Rationale:* At the age of 1 year, a child's fears of separation and mutilation are present because the child is facing the developmental task of trusting others. As the child gets older, fears about virility and reproductive ability may surface. The question does not provide enough data to determine that siblings exist. Options 3 and 4 may be issues if the child were older.

A nurse is caring for a 3-year-old child with suspected bacterial meningitis. Which signs and symptoms would the nurse expect to find during the initial data collection? *Select all that apply.*

*1. Fever* 2. Cough *3. Irritability* 4. Hypothermia *5. Nuchal rigidity* 6. Closed anterior fontanel *Rationale:* The initial signs and symptoms of bacterial meningitis include fever, nuchal rigidity, and irritability. The anterior fontanel closes by 12 to 18 months of age. Cough usually is not associated with bacterial meningitis.

A mother of a 3-year-old child tells the nurse that the child has been continuously scratching the skin and has developed a rash. On data collection, which finding indicates that the child may have scabies? 1. Fine, grayish-red lines 2. Purple-colored lesions 3. Thick, honey-colored crusts 4. Clusters of fluid-filled vesicles

*1. Fine, grayish-red lines* *Rationale:* Scabies appears as burrows or fine, grayish-red lines. They may be difficult to see if they are obscured by excoriation and inflammation. Purple-colored lesions may be indicative of various disorders, including systemic conditions. Thick, honey-colored crusts are characteristic of impetigo. Clusters of fluid-filled vesicles are seen in clients with herpesvirus.

Choose the home care instructions that the nurse would provide to the mother of a child with acquired immunodeficiency syndrome (AIDS). *Select all that apply.* 1. Frequent handwashing is important. 2. The child should avoid exposure to other illnesses. 3. The child's immunization schedule will need revision. 4. Kissing the child on the mouth will never transmit the virus. 5. Clean up body fluid spills with bleach solution (10:1 ratio of water to bleach). 6. Fever, malaise, fatigue, weight loss, vomiting, and diarrhea are expected to occur and do not require special intervention.

*1. Frequent handwashing is important.* *2. The child should avoid exposure to other illnesses.* *5. Clean up body fluid spills with bleach solution (10:1 ratio of water to bleach).* *Rationale:* AIDS is a disorder that is caused by the human immunodeficiency virus (HIV) and is characterized by a generalized dysfunction of the immune system. Both cellular and humoral immunity are compromised. The horizontal transmission of HIV occurs through intimate sexual contact or parenteral exposure to blood or body fluids that contain visible blood. Vertical (perinatal) transmission occurs when an HIV-infected pregnant woman passes the infection to her infant. Home care instructions include the following: frequent handwashing; monitoring for fever, malaise, fatigue, weight loss, vomiting, diarrhea, altered activity level, and oral lesions and notifying the health care provider if these occur; monitoring for signs and symptoms of opportunistic infections; administering antiretroviral medications, as prescribed; avoiding exposure to other illnesses; keeping immunizations up to date; avoiding kissing the child on the mouth; monitoring the weight and providing a high-calorie, high-protein diet; washing eating utensils in the dishwasher; and avoiding the sharing of eating utensils. Gloves are worn for care, especially when in contact with body fluids or changing diapers. Diapers are changed frequently and away from food areas, and soiled disposable diapers are folded inward, closed with their tabs, and disposed of in a tightly covered plastic-lined container. Any body fluid spills are cleaned with a bleach solution made up of a 10:1 ratio of water to bleach.

A nurse is admitting a child with a diagnosis of lactose intolerance. Which finding does the nurse expect to assess?

*1. Frothy stools* 2. Foul-smelling ribbon stools 3. Profuse, watery diarrhea and vomiting 4. Diffuse abdominal pain unrelated to meals or activity *Rationale:* Lactose intolerance causes frothy stools. Abdominal distention, crampy abdominal pain, and excessive flatus may also occur. Option 2 is a clinical manifestation of Hirschsprung's disease. Option 3 is a clinical manifestation of celiac disease. Option 4 is a clinical manifestation of irritable bowel syndrome.

A newborn infant is diagnosed with gastroesophageal reflux (GER). The mother of the infant asks the nurse to explain the diagnosis. The nurse plans to base the response on which description of this disorder?

*1. Gastric contents regurgitate back into the esophagus.* 2. The esophagus terminates before it reaches the stomach. 3. Abdominal contents herniate through an opening of the diaphragm 4. A portion of the stomach protrudes through the esophageal hiatus of the diaphragm. *Rationale:* GER is regurgitation of gastric contents back into the esophagus. Option 2 describes esophageal atresia. Option 3 describes a congenital diaphragmatic hernia. Option 4 describes a hiatal hernia.

A breast-feeding mother of an infant with lactose intolerance asks the nurse about dietary measures. Which of the following foods would the nurse instruct the mother to avoid?

*1. Hard cheeses* 2. Green, leafy vegetables 3. Dried beans 4. Egg yolk *Rationale:* Breast-feeding mothers of an infant with lactose intolerance need to be encouraged to limit dairy products. Cheese is a dairy product. Alternative calcium sources include egg yolk; green, leafy vegetables; dried beans; cauliflower; and molasses.

When checking a child's trochlear nerve function, the nurse would perform which data collection technique?

*1. Have the child look down and in.* 2. Have the child look toward the temporal side. 3. Have the child bite down hard and open the jaw. 4. Have the child show the teeth to note symmetry of expression. *Rationale:* Having the child look down and in will assess the function of the trochlear nerve. Option 2 is the technique for checking the abducens nerve. Option 3 is the technique for checking the trigeminal nerve. Option 4 is the data collection technique for checking the facial nerve.

A child has epistaxis. The nurse understands that an appropriate treatment for epistaxis is which of the following?

*1. Have the child sit up and lean forward.* 2. Have the child assume a supine position. 3. Have the child sit up and tilt the head backward. 4. Apply continuous pressure to the nose for at least 3 minutes. *Rationale:* Correct treatment for epistaxis (a nosebleed) involves having a client sit up and lean forward. Therefore options 2 and 3 are incorrect. Continuous pressure should be applied to the nose for at least 10 minutes.

A client with Addison's disease asks the nurse how a newly prescribed medication, fludrocortisone acetate (Florinef), will improve the condition. When formulating a response, the nurse should incorporate that a key action of this medication is to:

*1. Help restore electrolyte balance.* 2. Make the body produce more cortisol. 3. Replace insufficient circulating estrogens. 4. Alter the body's immune system functioning. *rationale* Fludrocortisone acetate is a long-acting oral medication with mineralocorticoid and moderate glucocorticoid activity. It is prescribed for the long-term management of Addison's disease. Mineralocorticoids cause renal reabsorption of sodium and chloride ions and the excretion of potassium and hydrogen ions. These actions help restore electrolyte balance in the body. The other options are incorrect.

In planning nutrition for the client with hypoparathyroidism, which diet would be appropriate?

*1. High in calcium and low phosphorous* 2. Low in vitamins A, D, E, and K 3. High in sodium with no fluid restriction 4. Low in water and insoluble fiber *rationale* Hypocalcemia is the end result of hypoparathyroidism resulting from either a lack of parathyroid hormone (PTH) secretion or ineffective PTH influence on tissue. Calcium is the major controlling factor of PTH secretion. Because of this, the diet needs to be high in calcium but low in phosphorus because these two electrolytes must exist in inverse proportions in the body. The other options are not dietary interventions with hypoparathyroidism.

A client is admitted to the hospital with a diagnosis of diabetic ketoacidosis (DKA). The initial serum glucose level was 950 mg/dL. Intravenous (IV) insulin was started along with rehydration with IV normal saline. The serum glucose level is now 240 mg/dL. The nurse who is assisting in caring for the client obtains which of the following items, anticipating a health care provider's prescription?

*1. IV infusion containing 5% dextrose* 2. NPH insulin and a syringe for subcutaneous injection 3. An ampule of 50% dextrose 4. Phenytoin (Dilantin) for prevention of seizures *rationale* During management of DKA, when the blood glucose level falls to 300 mg/dL, the infusion rate is reduced and 5% dextrose is added to maintain a blood glucose level of about 250 mg/dL, or until the client recovers from ketosis. NPH insulin is not used to treat DKA; 50% dextrose is used to treat hypoglycemia. Phenytoin is not a normal treatment measure in DKA.

A nurse is reviewing a health care provider's prescription for a child who was just admitted to the hospital with a diagnosis of Kawasaki disease. Which prescription should the nurse anticipate being part of the treatment plan?

*1. Immune globulin* 2. Heparin infusion 3. Morphine sulfate 4. Digoxin (Lanoxin) *Rationale:* Intravenous immune globulin (IVIG) is administered to the child with Kawasaki disease to decrease the incidence of coronary artery lesions and aneurysms and to decrease fever and inflammation. Options 2, 3, and 4 are not components of the treatment plan for this disease.

To ensure a safe environment for a child admitted to the hospital for a craniotomy to remove a brain tumor, the nurse should include which of the following in the plan of care? 1. Initiating seizure precautions 2. Using a wheelchair for out-of-bed activities 3. Assisting the child with ambulation at all times 4. Avoiding contact with other children on the nursing unit

*1. Initiating seizure precautions* *Rationale:* Safety of the child is the nursing priority. Seizure precautions should be implemented for any child with a brain tumor, both preoperatively and postoperatively. A thorough neurological assessment should be performed on the child, and the child's safety should be assessed before allowing the child to get out of bed without help. Assessment of the child's gait should be assessed daily. However, options 2 and 3 are not required unless functional deficits exist. Isolating the child, option 4, is not necessary.

A nursing student is asked to discuss the topic of clubfoot at a clinical conference. The student plans to tell the group that clubfoot: 1. Is a congenital anomaly 2. Always occurs bilaterally 3. Affects girls more often than boys 4. Is a rare deformity of the skeletal system

*1. Is a congenital anomaly* *Rationale:* Clubfoot, one of the most common deformities of the skeletal system, is a congenital anomaly characterized by a foot that has been twisted inward or outward. The condition generally affects both feet, and boys are affected twice as often as girls.

A mother of an infant diagnosed with Hirschsprung's disease asks the nurse about the disorder. The nurse plans to base the response on which of the following?

*1. It is a congenital aganglionosis or megacolon.* 2. It is a complete small intestinal obstruction. 3. It is a condition that causes the pyloric valve to remain open. 4. It is a severe inflammation of the gastrointestinal tract. *Rationale:* Hirschsprung's disease, also known as "congenital aganglionosis" or "megacolon," is the result of an absence of ganglion cells in the rectum and to varying degrees upward in the colon. Options 2, 3, and 4 are incorrect.

A nursing student is preparing a clinical conference, and the topic of the discussion is caring for the child with cystic fibrosis (CF). The student prepares a handout for the group and lists which of the following on the handout? *Select all that apply.*

*1. It is a disease that causes mucus formation to be abnormally thick.* *2. It is a chronic multisystem disorder affecting the exocrine glands.* *3. It is transmitted as an autosomal recessive trait.* 4. It is a disease that causes dilation of the passageways of all organs. 5. It is a disease that affects males only. 6. It is a disease that affects the lungs only. *Rationale:* CF is a chronic multisystem disorder affecting the exocrine glands. The mucus produced by these glands (particularly those of the bronchioles, small intestine, and pancreatic and bile ducts) is abnormally thick, causing obstruction of the small passageways of these organs. It is transmitted as an autosomal recessive trait and can affect both males and females.

A child seen in the clinic is found to have rubeola (measles), and the mother asks the nurse how to care for the child. The nurse tells the mother that she should: 1. Keep the child in a room with dim lights. 2. Give the child warm baths to help prevent itching. 3. Allow the child to play outdoors, because sunlight will help the rash. 4. Take the child's temperature every 4 hours and administer 1 baby aspirin for fever.

*1. Keep the child in a room with dim lights.* *Rationale:* A nursing consideration in rubeola is eye care. The child usually has *photophobia*, so the nurse should suggest that the parent keep the child out of brightly lit areas. Children with viral infections are not to be given aspirin because of the risk of Reye's syndrome. Warm baths and the sun will aggravate itching. In addition, the child needs to rest.

A cooling blanket is prescribed for a child with a fever. The nurse prepares to use the cooling blanket and avoids which of the following?

*1. Keeping the child uncovered to assist in reducing the fever* 2. Placing the cooling blanket on the bed and covering it with a sheet 3. Keeping the child dry while on the cooling blanket to prevent the risk of frostbite 4. Checking the skin condition of the child before, during, and after the use of the cooling blanket *Rationale:* While on a cooling blanket, the child should be covered lightly to maintain privacy and reduce shivering. Options 2, 3, and 4 are important interventions to prevent shivering, frostbite, and skin breakdown.

A nurse is caring for a child who was burned in a house fire. The nurse assists in developing a plan of care for monitoring the child during the treatment for burn shock. The nurse identifies which of the following assessments as providing the most accurate guide to determine the adequacy of fluid resuscitation?

*1. Level of consciousness* 2. Amount of edema at the site of the burn injury 3. Heart rate 4. Lung sounds *Rationale:* The sensorium, or level of consciousness, is an important guide to the adequacy of fluid resuscitation. The burn injury itself does not affect the sensorium, so the child should be alert and oriented. Any alteration in sensorium should be evaluated further. A neurological assessment would determine the level of consciousness in the child. Options 2, 3, and 4, although important in the assessment of the child with a burn injury, would not provide an accurate assessment of the adequacy of fluid resuscitation.

A nurse is reviewing the laboratory results of a child with aplastic anemia and notes that the white blood cell (WBC) count is 2000 cells/mm3 and the platelet count is 150,000 cells/mm3. Which of the following nursing interventions will the nurse incorporate into the plan of care?

*1. Maintain strict isolation precautions.* 2. Encourage the child to use a soft toothbrush. 3. Avoid unnecessary injections. 4. Encourage quiet play activities. *Rationale:* The normal WBC ranges from 5000 to 10,000 cells/mm3 and the normal platelet count ranges from 150,000 to 400,000/mm3. Strict isolation procedures would be required if the WBC count were low to protect the child from infection. Precautionary measures to prevent bleeding should be taken when a child has a low platelet count. These include no injections, no rectal temperatures, use of a soft toothbrush, and abstinence from contact sports or activities that could cause an injury.

A client with hypoparathyroidism has hypocalcemia. The nurse avoids giving the client the prescribed vitamin and calcium supplement with which of the following liquids?

*1. Milk* 2. Water 3. Iced tea 4. Fruit juice *rationale* Milk products are high in phosphates, which should be avoided by a client with hypoparathyroidism. Otherwise, calcium products are best absorbed with milk because the vitamin D in the milk promotes calcium absorption.

A nurse notes in the medical record that a client with Cushing's syndrome is experiencing fluid overload. Which interventions should be included in the plan of care? *Select all that apply.*

*1. Monitoring daily weight* *2. Monitoring intake and output* 3. Maintaining a low-potassium diet *4. Monitoring extremities for edema* *5. Maintaining a low-sodium diet* *rationale* The client with Cushing's syndrome experiencing fluid overload should be maintained on a high-potassium and low-sodium diet. Decreased sodium intake decreases renal retention of sodium and water. Monitoring weight, intake, output, and extremities for edema are all appropriate interventions for such a nursing diagnosis.

A nurse is checking the status of jaundice in a child with hepatitis. The nurse checks which of the following that will provide the best data regarding the presence of jaundice?

*1. Nailbeds* 2. Skin in the abdominal area 3. Skin in the sacral area 4. Membranes in the ear canal *Rationale:* Jaundice, if present, is best checked in the sclera, nailbeds, and mucous membranes. Generalized jaundice will appear in the skin throughout the body. Option 4 is not an appropriate assessment area for the presence of jaundice.

A nurse is monitoring a 7-year-old child who sustained a head injury in a motor vehicle accident for signs of increased intracranial pressure (ICP). The nurse assesses the child frequently for which early sign of increased ICP? 1. Nausea 2. Papilledema 3. Decerebrate posturing 4. Alterations in pupil size

*1. Nausea* *Rationale:* Nausea is an early sign of increased ICP. Late signs of increased ICP include a significant decrease in level of consciousness, Cushing's triad (increased systolic blood pressure and widened pulse pressure, bradycardia, and irregular respirations), and fixed and dilated pupils. Other late signs include decreased motor response to command, decreased sensory response to painful stimuli, posturing, Cheyne-Stokes respirations, and papilledema.

A clinic nurse reads the results of a Mantoux test performed on a 5-year-old child. The results indicate an area of induration measuring 8 mm. The nurse should interpret these results as: 1. Negative 2. Positive 3. Inconclusive 4. Definitive and requiring a repeat test

*1. Negative* *Rationale:* *Induration measuring 15 mm or greater is considered a positive result in a child 4 years or older* who has no associated risk factors. Since this child's results show an area of induration measuring 8 mm, the finding is negative. Options 2, 3, and 4 are incorrect interpretations.

A nurse is caring for a child following surgical removal of a brain tumor. The nurse is monitoring the child and notes that the pulse rate has increased and the blood pressure has dropped significantly. Bloody drainage also is noted on the posterior dressing. The initial nursing action is to:

*1. Notify the registered nurse (RN).* 2. Document the findings. 3. Recheck the vital signs in 1 hour. 4. Place the child in Trendelenburg's position. *Rationale:* In the event of bleeding and suspected shock, the health care provider is notified immediately. The nurse would contact the RN, who would then contact the health care provider. The child is never placed in Trendelenburg's position because it increases intracranial pressure (ICP) and the risk of bleeding. Rechecking the vital signs in 1 hour will delay necessary treatment. The nurse would document the findings, but the initial action would be to notify the RN to avoid any delays in treating this life-threatening situation.

A licensed practical nurse (LPN) is bathing a neonate and notices small dark tufts of fine hair on the neonate's lower back. The LPN should take which best action? 1. Notify the registered nurse of the finding. 2. Assess for other associated anomalies and document carefully. 3. Tell the mother and father that this may indicate spina bifida. 4. Recognize that this is normal in the neonate and continue the bath.

*1. Notify the registered nurse of the finding.* *Rationale:* The legal role of the LPN is to practice under the supervision of the registered nurse. In this instance, the tuft of hair may be indicative of a spinal anomaly, and the registered nurse should be notified of the finding. It is inappropriate to discuss abnormal findings with the parents because this is the responsibility of the health care provider, if an anomaly is suspected or diagnosed. The LPN should take the priority intervention of notifying the registered nurse before documenting in the chart.

A mother arrives at the emergency department with her child and a diagnosis of epiglottitis is documented. Which of the health care provider's prescriptions would be important for the nurse to question?

*1. Obtain a throat culture.* 2. Obtain axillary temperatures. 3. Administer humidified oxygen. 4. Administer antipyretics for fever. *Rationale:* The throat of a child with suspected epiglottitis should not be examined or cultured, because any stimulation with a tongue depressor or culture swab could cause laryngospasm and complete airway obstruction. Humidified oxygen and antipyretics are components of the treatment. Axillary rather than oral temperatures should be taken.

A nurse is checking a child for dehydration and documents that the child is moderately dehydrated. Which of the following symptoms would be noted in determining this finding?

*1. Oliguria* 2. Pale skin color 3. Severely depressed fontanels 4. Slightly dry, mucous membranes *Rationale:* In moderate dehydration, the fontanels would be slightly sunken, the mucous membranes would be very dry, the skin color would be dusky, and oliguria would be present. Options 2 and 4 describe mild dehydration. In mild dehydration, urine output would be decreased, but oliguria would not be present. Option 3 describes severe dehydration.

A nurse is assisting in preparing a plan of care for a child who is being admitted to the pediatric unit with a diagnosis of seizures. Which of the following would be a component of the plan of care? *Select all that apply.*

*1. Pad the side rails of the bed with blankets.* *2. Maintain the bed in a low position.* 3. Restrain the child if a seizure occurs. *4. Place the child in a side-lying lateral position if a seizure occurs.* *5. Protect the child's head, body, and extremities if a seizure occurs.* 6. Place a padded tongue blade in the child's mouth if a seizure occurs. *Rationale:* Restraints are not to be applied to a child with a seizure because they could cause injury to the child. The side rails of the bed are padded with blankets, and the bed is maintained in a low position to provide safety if the child has a seizure. The child's head and the rest of the body are protected from injury if a seizure occurs. Positioning the child on his or her side will prevent aspiration as the saliva drains out of the child's mouth during the seizure. Neither a padded tongue blade nor any other object is placed in the child's mouth once a seizure has started.

A child is admitted to the hospital with sickle cell crisis. The nurse checks this child for which frequent symptom of the disorder?

*1. Pain* 2. Diarrhea 3. Bradycardia 4. Blurred vision *Rationale:* Sickling crisis often causes pain in the bones and joints, accompanied by joint swelling. Pain is a classic symptom of the disease and may require large doses of opioid analgesics when it is severe. The symptoms listed in the other options are not part of the clinical picture.

A child is admitted to the hospital with a probable diagnosis of nephrotic syndrome. Which findings would the nurse expect to observe? *Select all that apply.* 1. Pallor 2. Edema 3. Anorexia 4. Proteinuria 5. Weight loss 6. Decreased serum lipids

*1. Pallor* *2. Edema* *3. Anorexia* *4. Proteinuria* *Rationale:* Nephrotic syndrome is a kidney disorder that is characterized by massive proteinuria, hypoalbuminemia, edema, elevated serum lipids, anorexia, and pallor. The urine volume is decreased, and the urine is dark and frothy in appearance. The child with this condition gains weight.

A 4-year-old child is hospitalized with a suspected diagnosis of Wilms' tumor. The nurse assists with developing a plan of care. The nurse questions which intervention that is written in the plan of care? 1. Palpating the abdomen for a mass 2. Checking the urine for the presence of hematuria 3. Monitoring the blood pressure for the presence of hypertension 4. Monitoring the temperature for the presence of a kidney infection

*1. Palpating the abdomen for a mass* *Rationale:* Wilms' tumor is an intra-abdominal and kidney tumor. If Wilms' tumor is suspected, the mass should not be palpated. Excessive manipulation can cause seeding of the tumor and thus cause the spread of the cancerous cells. Hematuria, hypertension, and fever are clinical manifestations that are associated with Wilms' tumor.

A child is diagnosed with scarlet fever. A nurse collects data regarding the child. Which of the following is a clinical manifestation of scarlet fever? 1. Pastia's sign 2. Abdominal pain and flaccid paralysis 3. Dense pseudoformation membrane in the throat 4. Foul-smelling and mucopurulent nasal drainage

*1. Pastia's sign* *Rationale:* Pastia's sign is a rash seen among children with scarlet fever that will blanch with pressure, except in areas of deep creases and in the folds of joints. The tongue is initially coated with a white furry covering with red projecting papillae (white strawberry tongue). By the fourth to fifth day, the white strawberry tongue sloughs off and leaves a red, swollen tongue (strawberry tongue). The pharynx is edematous and beefy red in color. Option 2 is associated with poliomyelitis. Options 3 and 4 are characteristics of diphtheria.

A 4-year-old child with acute lymphocytic leukemia has been admitted to the hospital in relapse. The priority concern is infection due to immunosuppression. Which of the following interventions would the nurse include in the plan of care?

*1. Perform oral hygiene four times a day.* 2. Monitor vital signs once a shift. 3. Inspect the child's mouth daily for mouth ulcers. 4. Administer acetaminophen (Tylenol) suppositories for increased temperature. *Rationale:* The child who is immunosuppressed is at risk for infection, and interventions must be performed frequently to prevent infection. The nursing interventions as stated in options 2 and 3 are incorrect because of the time frames. Suppositories are never administered when a child is immunocompromised because of the risk of perineal fissures.

A child has been diagnosed with Reye's syndrome. The nurse understands that a major symptom associated with Reye's syndrome is: 1. Persistent vomiting 2. Protein in the urine 3. Symptoms of hyperglycemia 4. A history of a Staphylococcus infection

*1. Persistent vomiting* *Rationale:* Persistent vomiting is a major symptom that is associated with increased intracranial pressure (ICP). Options 2, 3, and 4 are incorrect. Protein is not present in the urine. Reye's syndrome is related to a history of viral infections, and hypoglycemia is a symptom of this disease.

A nurse is caring for a hospitalized child with a diagnosis of rubella (German measles). The nurse reviews the health care provider's progress notes and reads that the child has developed Forchheimer sign. Based on this documentation, which of the following should the nurse expect to note in the child?

*1. Petechiae spots located on the palate* 2. Small blue-white spots noted on the buccal mucosa 3. A fiery red edematous rash on the cheeks 4. Swelling of the parotid gland *Rationale:* Forchheimer sign refers to petechiae spots, which are reddish and pinpoint and located on the soft palate. Small blue-white spots noted on the buccal mucosa are known as Koplik's spots seen in rubeola. A fiery red edematous rash on the cheeks, also called "slapped cheeks" is seen in erythema infectiosum. Swelling of the parotid gland is seen in mumps.

A child with a diagnosis of pertussis (whooping cough) is being admitted to the pediatric unit. As soon as the child arrives in the unit, the nurse would first:

*1. Place the child on a pulse oximeter.* 2. Weigh the child. 3. Take the child's temperature. 4. Ask the parents about the child. *Rationale:* To adequately determine whether the child is getting enough oxygen, the child is placed on a pulse oximeter. The pulse oximeter will then provide ongoing information on the child's oxygen level. The child is also immediately placed on a cardiorespiratory monitor to provide early identification of periods of apnea and bradycardia. The nurse would then gather data including taking the child's temperature and weight and asking the parents about the child.

A nurse is preparing for the admission of an infant with a diagnosis of bronchiolitis caused by the respiratory syncytial virus (RSV). Choose the interventions that would be included in the plan of care. *Select all that apply.* 1. Place the infant in a private room. 2. Place the infant in a room near the nurses' station. 3. Ensure that the infant's head is in a flexed position. 4. Wear a mask at all times when in contact with the infant. 5. Place the child in a tent that delivers warm, humidified air. 6. Position the infant side-lying, with the head lower than the chest.

*1. Place the infant in a private room.* *2. Place the infant in a room near the nurses' station.* *Rationale:* The infant with RSV should be isolated in a private room or in a room with another child with RSV. The infant should be placed in a room near the nurses' station for close observation. The infant should be positioned with the head and chest at a 30- to 40-degree angle and the neck slightly extended to maintain an open airway and to decrease pressure on the diaphragm. Cool, humidified oxygen is delivered to relieve dyspnea, hypoxemia, and insensible water loss from tachypnea. Contact precautions (wearing gloves and a gown) reduce the nosocomial transmission of RSV.

When a nurse notes that a client with type 1 diabetes mellitus has lipodystrophy on both upper thighs, what information should the nurse obtain from the client?

*1. Plan of injection rotation* 2. Consistency of aspiration 3. Preparation of the injection site 4. Angle at which the medication is administered *rationale* Lipodystrophy (i.e., the hypertrophy of subcutaneous tissue at the injection site) occurs in some diabetic clients when the same injection sites are used for prolonged periods of time. Thus, clients are instructed to adhere to a rotating injection site plan to avoid tissue changes. Preparation of the site, aspiration, and the angle of insulin administration do not produce tissue damage.

A nurse is monitoring a client who has been newly diagnosed with diabetes mellitus for signs of complications. Which of the following, if exhibited by the client, would indicate hyperglycemia and thus warrant health care provider notification?

*1. Polyuria* 2. Bradycardia 3. Diaphoresis 4. Hypertension *rationale* The classic symptoms of hyperglycemia include polydipsia, polyuria, and polyphagia. Options 2, 3, and 4 are not signs of hyperglycemia.

A nurse is caring for a client with a diagnosis of hypoparathyroidism. The nurse reviews the laboratory results drawn on the client and notes that the calcium level is extremely low. The nurse would expect to note which of the following on data collection of the client?

*1. Positive Trousseau's sign* 2. Negative Chvostek's sign 3. Unresponsive pupils 4. Hyperactive bowel sounds *rationale* Hypoparathyroidism is related to a lack of parathyroid hormone secretion or to a decreased effectiveness of parathyroid hormone on target tissues. The end result of this disorder is hypocalcemia. When serum calcium levels are critically low, the client may exhibit positive Chvostek's and Trousseau's signs, which indicate potential tetany. Options 2, 3, and 4 are not related to the presence of hypocalcemia.

A nurse reviews the results of a Mantoux test performed on a 3-year-old child. The results indicate an area of induration that measures 10 mm. The nurse would interpret these results as: 1. Positive 2. Negative 3. Inconclusive 4. Definitive, requiring a repeat test

*1. Positive* *Rationale:* An induration that measures 10 mm or more is considered to be a positive result for children who are younger than 4 years old and for those with chronic illness or with a high risk for environmental exposure to tuberculosis. A reaction of 5 mm or more is considered to be a positive result for those in the highest-risk groups. Repeat tests are not done, especially when a positive reaction occurs.

A nurse is monitoring the results of periodic serum laboratory studies drawn on a client with diabetic ketoacidosis (DKA) receiving an insulin infusion. The nurse determines that which of the following values needs to be reported?

*1. Potassium 3.1 mEq/L* 2. Calcium 9.2 mg/dL 3. Sodium 137 mEq/L 4. Serum osmolality 288 mOsm/kg H2O *rationale* The client with DKA initially becomes hyperkalemic as potassium leaves the cells in response to lowered pH. Once fluid replacement and insulin therapy are started, the potassium level drops quickly. This occurs because potassium is carried into the cells along with glucose and insulin and because potassium is excreted in the urine once rehydration has occurred. Thus the nurse carefully monitors the results of serum potassium levels and reports hypokalemia (option 1) promptly. The other laboratory values are within the normal ranges.

A client with diabetes mellitus visits the health care clinic. The client previously had been well controlled with glyburide (Diabeta), but recently, the fasting blood glucose has been running 180 to 200 mg/dL. Which of the following medications, if added to the client's regimen, may be contributing to the hyperglycemia?

*1. Prednisone* 2. Atenolol (Tenormin) 3. Phenelzine (Nardil) 4. Allopurinol (Zyloprim) *rationale* Prednisone may decrease the effect of oral hypoglycemics, insulin, diuretics, and potassium supplements. Options 2, a β-blocker, and 3, a monoamine oxidase inhibitor, have their own intrinsic hypoglycemic activity. Option 4 decreases urinary excretion of sulfonylurea agents, causing increased levels of the oral medications, which can lead to hypoglycemia.

A nurse is assigned to care for a child with hypertrophic pyloric stenosis who is scheduled for pyloromyotomy. Which of the following positions would the nurse place the child in during the preoperative period?

*1. Prone with the head of the bed elevated* 2. Prone with the head of the bed lowered to promote drainage 3. Supine with the head of the bed at a 30-degree angle 4. Supine with the head of the bed at a 45-degree angle *Rationale:* In the preoperative period, the infant is positioned prone with the head of the bed elevated to reduce the risk of aspiration. Options 2, 3, and 4 are inappropriate positions to prevent this risk.

After a tonsillectomy, a child is brought to the pediatric unit. The nurse places the child in which appropriate position? 1. Prone 2. Supine 3. Trendelenburg's 4. High Fowler's

*1. Prone* *Rationale:* The child should be placed in a prone or side-lying position after tonsillectomy to facilitate drainage. Options 2, 3, and 4 will not achieve this goal.

A nurse is reviewing the record of a child admitted to the hospital with nephrotic syndrome. Which finding would the nurse expect to note documented in the record?

*1. Proteinuria* 2. Weight loss 3. Increased appetite 4. Hyperalbuminemia *Rationale:* The term "nephrotic syndrome" refers to a kidney disorder characterized by proteinuria, hypoalbuminemia, and edema. The child experiences fatigue, anorexia, increased weight, abdominal pain, and a normal blood pressure.

Choose the interventions that a nurse would include when writing a care plan for a child with hepatitis? *Select all that apply.*

*1. Providing a low-fat, well-balanced diet* 2. Notifying the health care provider if jaundice is present *3. Teaching the child effective hand washing techniques* 4. Scheduling play time in the playroom with other children *5. Instructing the parents about the risks associated with taking medications* 6. Arranging for indefinite home schooling because the child will not be able to return to school *Rationale:* Because hepatitis can be viral, standard precautions should be instituted in the hospital. The child should be discouraged from sharing toys, so playtime in the playroom with other children is not part of the plan of care. The child will be allowed to return to school 1 week after the onset of jaundice, so indefinite home schooling would not need to be arranged. Jaundice is an expected finding with hepatitis and would not warrant notification of the health care provider. Provision of a low-fat, well-balanced diet is recommended. Parents are cautioned about administering any medication to the child because normal doses of many medications may become dangerous because of the liver's inability to detoxify and excrete them. Handwashing is the single most effective measure in control of hepatitis in any setting, and effective handwashing can prevent the compromised child from picking up an opportunistic type of infection.

A nurse is monitoring for signs of dehydration in a 1-year-old child who has been hospitalized for diarrhea and prepares to take the child's temperature. Which method of temperature measurement should be avoided? 1. Rectal 2. Axillary 3. Electronic 4. Tympanic

*1. Rectal* *Rationale:* Rectal temperature measurements should be avoided if diarrhea is present. The use of a rectal thermometer can stimulate peristalsis and cause more diarrhea. Axillary or tympanic measurements of temperature would be acceptable. Most measurements are performed via electronic devices.

A nurse is monitoring a child following a tonsillectomy. Which finding may indicate that the child is bleeding?

*1. Restlessness* 2. A decreased pulse rate 3. Complaints of discomfort 4. An elevation in blood pressure (BP) *Rationale:* Frequent swallowing, restlessness, a fast and thready pulse, and vomiting bright red blood are signs of bleeding. An elevated BP is not an indication of bleeding. Complaint of discomfort is an expected finding following a tonsillectomy.

A nurse is reviewing a health care provider's prescription for a child with sickle cell anemia who was admitted to the hospital for the treatment of vaso-occlusive crisis. Which prescriptions documented in the child's record should the nurse question? *Select all that apply.* 1. Restrict fluid intake. 2. Position for comfort. 3. Avoid strain on painful joints. 4. Apply nasal oxygen at 2 L per minute. 5. Provide a high-calorie, high-protein diet. 6. Administer meperidine (Demerol) 25 mg for pain.

*1. Restrict fluid intake.* *6. Administer meperidine (Demerol) 25 mg for pain.* *Rationale:* Sickle cell anemia is one of a group of diseases called hemoglobinopathies in which hemoglobin A is partly or completely replaced by abnormal sickle hemoglobin S. It is caused by the inheritance of a gene for a structurally abnormal portion of the hemoglobin chain. Hemoglobin S is sensitive to changes in the oxygen content of the red blood cell, and insufficient oxygen causes the cells to assume a sickle shape; the cells become rigid and clumped together, thus obstructing capillary blood flow. Oral and intravenous fluids are important parts of treatment. Meperidine (Demerol) is not recommended for the child with sickle cell disease because of the risk for normeperidine-induced seizures. Normeperidine, which is a metabolite of meperidine, is a central nervous system stimulant that produces anxiety, tremors, myoclonus, and generalized seizures when it accumulates with repetitive dosing. Therefore, the nurse would question the prescriptions for restricted fluids and meperidine for pain control. Positioning for comfort, avoiding strain in painful joints, oxygen, and a high-calorie, high-protein diet are important parts of the treatment plan.

A nurse is planning care for a child with hemolytic-uremic syndrome (HUS). The child has been anuric and will be receiving peritoneal dialysis treatment. The nurse plans to: 1. Restrict fluids, as prescribed. 2. Administer analgesics, as prescribed. 3. Care for the arteriovenous (AV) fistula. 4. Encourage the intake of foods that are high in potassium.

*1. Restrict fluids, as prescribed.* *Rationale:* HUS is thought to be associated with bacterial toxins, chemicals, and viruses that cause acute renal failure in children. Clinical features of the disease include acquired hemolytic anemia, thrombocytopenia, renal injury, and central nervous system symptoms. A child with HUS who is undergoing peritoneal dialysis for the treatment of anuria will be on fluid restrictions. Pain is not associated with HUS, and potassium would be restricted rather than encouraged if the child was anuric. Peritoneal dialysis does not require an AV fistula (only hemodialysis does).

A nurse reinforces home-care instructions to the parents of a child with celiac disease. Which of the following food items would the nurse advise the parents to include in the child's diet? 1. Rice 2. Oatmeal 3. Rye toast 4. Wheat bread

*1. Rice* *Rationale:* Dietary management is the mainstay of treatment for celiac disease. All wheat, rye, barley, and oats should be eliminated from the diet and replaced with corn and rice. Vitamin supplements, especially fat-soluble vitamins and folate, may be required during the early period of treatment to correct deficiencies. These restrictions are likely to be lifelong, although small amounts of grains may be tolerated after the gastrointestinal ulcerations have healed.

A nurse is assisting in developing a plan of care for a child who will be returning from the operating room following a tonsillectomy. The nurse plans to place the child in which position on return from the operating room?

*1. Side-lying* 2. Trendelenburg's and on the right side 3. Supine 4. High Fowler's and on the left side *Rationale:* The child should be placed in a prone or side-lying position following tonsillectomy to facilitate drainage. Options 2, 3, and 4 will not facilitate drainage.

A health care provider has prescribed propylthiouracil (PTU) for a client with hyperthyroidism, and the nurse assists in developing a plan of care for the client. A priority nursing measure to be included in the plan regarding this medication is to monitor the client for:

*1. Signs and symptoms of hypothyroidism* 2. Signs and symptoms of hyperglycemia 3. Relief of pain 4. Signs of renal toxicity *rationale* Excessive dosing with propylthiouracil may convert the client from a hyperthyroid state to a hypothyroid state. If this occurs, the dosage should be reduced. Temporary administration of thyroid hormone may be required. Propylthiouracil is not used for pain and does not cause hyperglycemia or renal toxicity.

A nurse reviews the plan of care for a child with Reye's syndrome. The nurse prioritizes the nursing interventions included in the plan and prepares to monitor for:

*1. Signs of increased intracranial pressure* 2. The presence of protein in the urine 3. Signs of a bacterial infection 4. Signs of hyperglycemia *Rationale:* Intracranial pressure and encephalopathy are major complications of Reye's syndrome. Protein is not present in the urine. Reye's syndrome is related to a history of viral infections, and hypoglycemia is a symptom of this disease.

A nurse is preparing to discharge a client who has had a parathyroidectomy. When teaching the client about the prescribed oral calcium supplement, what information should the nurse include?

*1. Take the calcium 30 to 60 minutes following a meal.* 2. Avoid sunlight because it can cause skin color change. 3. Store the calcium in the refrigerator to maintain potency. 4. Check the pulse daily and hold the dosage if it is below 60 beats per minute. *rationale* Oral calcium supplements can be taken 30 to 60 minutes after meals to enhance their absorption and decrease gastrointestinal irritation. All the other options are unrelated to oral calcium therapy.

A licensed practical nurse is providing care for a child with hydrocephalus who has had a ventriculoperitoneal shunt revision. Which data collection finding should be reported to the registered nurse immediately? 1. Temperature 100.9° F 2. Pulse 78 beats per minute 3. Blood pressure 110/70 mm Hg 4. Respirations 22 breaths per minute

*1. Temperature 100.9° F* *Rationale:* Fever may be an indication of an infection of the shunt, which is the primary concern in the postoperative period, related to a shunt insertion. All of the other vital signs are normal findings for this child.

An 8-year-old boy is being treated with percussion treatments for cystic fibrosis. How would the nurse determine whether the treatment is effective?

*1. The child has a productive cough of thick sputum.* 2. The child no longer has a fever. 3. The child's skin is no longer high in sodium. 4. The child's bowel movements are firmer. *Rationale:* Percussion treatments are intended to produce sputum. Thick sputum is characteristic of cystic fibrosis. Being afebrile is not necessarily reflective of effectiveness of percussion treatments. Although a high sodium content in the skin is a sign associated with cystic fibrosis, percussion treatments will not help this characteristic. The percussion treatments will not help bowel movements.

A child with a tracheal obstruction is brought to the emergency department by emergency medical services. The child aspirated a grape, and the foreign body was removed by direct laryngoscopy. Following the procedure, the nurse plans to inform the mother of the child that:

*1. The child will need to be hospitalized for observation.* 2. The child may go home with a prescription for antibiotics. 3. The child will need to return to the hospital for a chest x-ray in 1 week. 4. The child will require a bronchoscopy for follow-up evaluation in 1 month. *Rationale:* Removal of foreign bodies from the respiratory tract may need to be performed by direct laryngoscopy or bronchoscopy. After the procedure the child should remain hospitalized for observation for laryngeal edema and respiratory distress. Cool mist is provided, and antibiotic therapy is prescribed if appropriate. Options 2, 3, and 4 are incorrect.

A nurse is collecting data from a client newly diagnosed with diabetes mellitus regarding the client's learning readiness. Which client behavior indicates to the nurse that the client is not ready to learn?

*1. The client complains of fatigue whenever the nurse plans a teaching session.* 2. The client asks if the spouse can attend the classes also. 3. The client asks for written materials about diabetes before class. 4. The client asks appropriate questions about what will be taught. *rationale* Physical symptoms can interfere with an individual's ability to learn and can indicate to the teacher that the learner lacks motivation to learn if the symptoms repeatedly recur when teaching is initiated. Options 2, 3, and 4 identify the client as actively seeking information.

A client with diabetes mellitus is being discharged following treatment for hyperglycemic hyperosmolar nonketotic syndrome (HHNS) precipitated by acute illness. The client states to the nurse, "I will call the doctor next time I can't eat for more than a day or so." The nurse plans care, understanding that which of the following accurately reflects this client's level of knowledge?

*1. The client needs immediate education before discharge.* 2. The client's statement is accurate, but knowledge should be evaluated further. 3. The client's statement is inaccurate, and the client should be scheduled for outpatient diabetic counseling. 4. The client requires follow-up teaching regarding the administration of insulin. *rationale* If the client becomes ill and cannot retain fluids or food for a period of 4 hours, the health care provider should be notified. The client's statement in this question indicates a need for immediate education to prevent HHNS, a life-threatening emergency situation.

A nurse is caring for a 2-year-old child diagnosed with croup. The nurse collects data on the child, knowing that which of the following are characteristic of this illness? *Select all that apply.*

*1. The cough is harsh and metallic.* *2. Inspiratory stridor may be present.* *3. Symptoms usually worsen at night and are better during the day.* 4. Symptoms usually worsen during the day and are relieved during sleep. *5. It is usually preceded by several days of upper respiratory infection symptoms.* *Rationale:* Croup often begins at night and may be preceded by several days of upper respiratory infection symptoms. It is characterized by a sudden onset of a harsh, metallic cough, sore throat, and inspiratory stridor. Symptoms usually worsen at night and are better in the day.

An older client with a history of hyperparathyroidism and severe osteoporosis is hospitalized. The nurse caring for the client plans first to address which problem?

*1. The possibility of injury* 2. Constipation 3. Urinary retention 4. Need for teaching about the disorder *rationale* The client with severe osteoporosis as a result of hyperparathyroidism is at risk for injury as a result of pathological fractures that can occur from bone demineralization. The client may also have a risk for constipation from the disease process but this is a lesser priority than client safety. The client may or may not have urinary elimination problems, depending on other factors in the client's history. There is no information in the question to support whether the client needs teaching.

A nurse is developing a plan of care for a child who is at risk for seizures. Which interventions apply if the child has a seizure? *Select all that apply.* 1. Time the seizure. 2. Restrain the child. 3. Stay with the child. 4. Place the child in a prone position. 5. Move furniture away from the child. 6. Insert a padded tongue blade into the child's mouth.

*1. Time the seizure.* *3. Stay with the child.* *5. Move furniture away from the child.* *Rationale:* During a seizure, the child is placed on his or her side in a lateral position. This type of positioning will prevent aspiration, because saliva will drain out of the corner of the child's mouth. The child is not restrained, because this could cause injury. The nurse would loosen clothing around the child's neck and ensure a patent airway. Nothing is placed into the child's mouth during a seizure, because this action may cause injury to the child's mouth, gums, or teeth. The nurse would stay with the child to reduce the risk of injury and allow for the observation and timing of the seizure.

The nurse should implement which of the following in the care of a child who is having a seizure? *Select all that apply.* 1. Time the seizure. 2. Restrain the child. 3. Stay with the child. 4. Insert an oral airway. 5. Place the child in a supine position. 6. Loosen clothing around the child's neck.

*1. Time the seizure.* *3. Stay with the child.* *6. Loosen clothing around the child's neck.* *Rationale:* During a seizure, the child is placed on his or her side in a lateral position. Positioning on the side will prevent aspiration because saliva will drain out of the corner of the child's mouth. The child is not restrained because this could cause injury to the child. The nurse would loosen clothing around the child's neck and ensure a patent airway. Nothing is placed into the child's mouth during a seizure because this action may cause injury to the child's mouth, gums, or teeth. The nurse would stay with the child to reduce the risk of injury and allow for observation and timing of the seizure.

A nurse is monitoring a client following a thyroidectomy for signs of hypocalcemia. Which of the following signs, if noted in the client, likely indicates the presence of hypocalcemia?

*1. Tingling around the mouth* 2. Negative Chvostek's sign 3. Flaccid paralysis 4. Bradycardia *rationale* Following a thyroidectomy, the nurse assesses the client for signs of hypocalcemia and tetany. Early signs include tingling around the mouth and fingertips, muscle twitching or spasms, palpitations or dysrhythmias, and positive Chvostek's and Trousseau's signs. Options 2, 3, and 4 are not signs of hypocalcemia.

Which client complaint would alert the nurse to a possible hypoglycemic reaction?

*1. Tremors* 2. Anorexia 3. Hot, dry skin 4. Muscle cramps *rationale* Decreased blood glucose levels produce automatic nervous system symptoms, which are classically manifested as nervousness, irritability, and tremors. Option 3 is more likely to occur with hyperglycemia. Options 2 and 4 are unrelated to the signs of hypoglycemia.

A nurse is providing teaching regarding the prevention of Lyme disease to a group of teenagers going on a hike in a wooded area. Which of the following points should the nurse include in the session? *Select all that apply.*

*1. Tuck pant legs into socks.* *2. Wear closed shoes when hiking.* *3. Apply insect repellent containing DEET.* *4. Cover the ground with a blanket when sitting.* 5. Remove attached ticks by grasping with thumb and forefinger. 6. Wear long sleeves and long pants in dark colors when in high-risk areas. *Rationale:* Measures to prevent tick bites focus on covering the body as completely as possible and spraying insect repellent containing DEET on the skin and clothing. Long sleeves and pants tucked into the socks along with closed shoes will offer some protection. Light-colored clothing should be worn so that ticks would be easily visible. Hikers should not sit directly on the ground and should cover the ground with an item such as a blanket. Ticks should be removed with tweezers.

After a tonsillectomy, the child begins to vomit bright red blood. The initial nursing action would be to: 1. Turn the child to the side. 2. Notify the health care provider (HCP). 3. Administer the prescribed antiemetic. 4. Maintain nothing-by-mouth (NPO) status.

*1. Turn the child to the side.* *Rationale:* After a tonsillectomy, if bleeding occurs, the child is turned to the side, and the HCP is notified. An NPO status would be maintained, and an antiemetic may be prescribed; however, the initial nursing action would be to turn the child to the side.

A nurse is collecting data from a child with a diagnosis of diabetes insipidus. Which clinical finding is consistent with this diagnosis?

*1. Urinary output is increased.* 2. Urinary output is decreased. 3. Serum sodium is decreased. 4. Urine specific gravity is increased. *Rationale:* A child with a diagnosis of diabetes insipidus experiences increased urinary output, increased serum sodium, and decreased urine specific gravity. Decreased urinary output, decreased serum sodium, and increased urine specific gravity are consistent with a diagnosis of syndrome of inappropriate antidiuretic hormone (SIADH).

During routine postoperative assessment of a client who has undergone hypophysectomy, the client complains of thirst and frequent urination. Knowing the expected complications of this surgery, the nurse would next check the:

*1. Urine specific gravity* 2. Serum glucose 3. Respiratory rate 4. Blood pressure *rationale* Following hypophysectomy, diabetes insipidus can occur temporarily because of antidiuretic hormone (ADH) deficiency. This deficiency is related to surgical manipulation. The nurse should assess specific gravity and notify the registered nurse if the results are less than 1.005. Although options 2, 3, and 4 may be components of the assessment, the nurse would next check urine specific gravity.

A nurse is caring for a client with pheochromocytoma. The client is scheduled for an adrenalectomy. During the preoperative period, the priority nursing action would be to monitor the:

*1. Vital signs* 2. Intake and output 3. Blood urea nitrogen (BUN) level 4. Urine for glucose and acetone *rationale* Hypertension is the hallmark of pheochromocytoma. Severe hypertension can precipitate a brain attack (stroke) or sudden blindness. Although all of the options are accurate nursing interventions for the client with pheochromocytoma, the priority nursing action is to monitor the vital signs, particularly the blood pressure.

A client with diabetes mellitus is scheduled to have a fasting blood glucose level determined in the morning. The nurse tells the client not to eat or drink after midnight. When the client asks for further information, the nurse clarifies by stating that which of the following would be acceptable to take before the test?

*1. Water* 2. Coffee without any milk 3. Tea without any sugar 4. Clear liquids such as apple juice *rationale* When a client is scheduled for a fasting blood glucose level, the client should not eat or drink anything except water after midnight. This is needed to ensure accurate test results, which form the basis for adjustments or continuance of treatment. Options 2, 3, and 4 are inaccurate, and the client should not consume these items before the test.

A nurse is caring for a 1-year-old child following a cleft palate repair. Which solution should the nurse use after feedings to cleanse the child's mouth?

*1. Water* 2. Diluted hydrogen peroxide 3. A soft lemon glycerin swab 4. Half-strength povidone-iodine (Betadine) solution *Rationale:* Following a cleft palate repair, the mouth is rinsed with water after feedings to clean the palate repair site. Rinsing food and residual sugars from the suture line reduces the risk of infection. Options 2, 3, and 4 are incorrect procedures, and the solutions identified in these options should not be used.

An infant with congestive heart failure (CHF) is receiving diuretic therapy, and the nurse is closely monitoring the intake and output (I&O). Which is the best method for the nurse to use to monitor the urine output?

*1. Weighing the diapers* 2. Inserting a Foley catheter 3. Comparing intake with output 4. Measuring the amount of water added to formula *Rationale:* The best method to monitor urine output in an infant on diuretic therapy is to weigh the diapers. The weight of dry diapers is subtracted from the weight of wet diapers to determine the amount of urine excreted: 1 g is equivalent to 1 mL of urine. Comparing intake with output would not provide an accurate measure of urine output. Measuring the amount of water added to formula is unrelated to the amount of output. Although Foley catheter drainage is most accurate in determining output, it is not the best method and places the infant at risk for infection.

A nurse is preparing to administer digoxin (Lanoxin) to an infant with congestive heart failure (CHF). Before administering the medication, the nurse double-checks the dose, counts the apical heart rate for 1 full minute, and obtains a rate of 88 beats per minute. Based on this finding, which of the following is the appropriate nursing action? 1. Withhold the medication. 2. Administer the medication. 3. Double-check the apical heart rate and administer the medication. 4. Check the blood pressure and respirations and administer the medication.

*1. Withhold the medication.* *Rationale:* Digoxin is effective within a narrow therapeutic range (0.5 to 2 ng/mL). Safety in dosing is achieved by double-checking the dose and counting the apical heart rate for 1 full minute. *If the heart rate is less than 100 beats per minute in an infant, the nurse would withhold the dose and notify the health care provider*. Options 2, 3, and 4 are incorrect actions.

After a tonsillectomy, which of the following fluid or food items would be appropriate to offer to the child? 1. Yellow Jell-O 2. Cold ginger ale 3. Vanilla pudding 4. Cherry Popsicle

*1. Yellow Jell-O* *Rationale:* After a tonsillectomy, clear, cool liquids should be administered. Citrus, carbonated, and extremely hot or cold liquids need to be avoided, because they may irritate the throat. Milk and milk products (pudding) are avoided, because they coat the throat and cause the child to clear the throat, thus increasing the risk of bleeding. Red liquids need to be avoided, because they give the appearance of blood if the child vomits.

A nurse has reinforced home care instructions to the mother of a child who is being discharged after cardiac surgery. Which statement, if made by the mother, indicates the need for further instructions? 1. "A balance of rest and exercise is important." 2. "I can apply lotion or powder to the incision if it is itchy." 3. "Activities during which the child could fall need to be avoided for 2 to 4 weeks." 4. "Large crowds of people need to be avoided for at least 2 weeks after this surgery."

*2. "I can apply lotion or powder to the incision if it is itchy."* *Rationale:* The mother should be instructed that lotions and powders should not be applied to the incision site because these items can affect the skin integrity and the healing process. Options 1, 3, and 4 are accurate instructions regarding home care after cardiac surgery.

A 4-year-old child sustains a fall at home and is brought to the emergency department by the mother. After an x-ray, it is determined that the child has a fractured arm, and a plaster cast is applied. The nurse provides instructions to the mother regarding cast care for the child. Which statement by the mother indicates the need for further instructions? 1. "The cast may feel warm as it dries." 2. "I can use lotion or powder around the cast edges to relieve itching." 3. "A small amount of white shoe polish can touch up a soiled white cast." 4. "If the cast becomes wet, a blow-dryer set on the cool setting may be used to dry it."

*2. "I can use lotion or powder around the cast edges to relieve itching."* *Rationale:* The mother needs to be instructed to not use lotion or powders on the skin around the cast edges or inside the cast, because they can become sticky or caked and cause skin irritation. Options 1, 3, and 4 are appropriate instructions.

A nurse reinforces home-care instructions to the parents of a child with hepatitis regarding the care of the child and the prevention of the transmission of the virus. Which statement by a parent indicates a need for further instruction? 1. "Frequent handwashing is important." 2. "I need to provide a well-balanced, high-fat diet to my child." 3. "I need to clean contaminated household surfaces with bleach." 4. "Diapers should not be changed near any surfaces that are used to prepare food."

*2. "I need to provide a well-balanced, high-fat diet to my child."* *Rationale:* The child with hepatitis should consume a well-balanced, low-fat diet to allow the liver to rest. Options 1, 3, and 4 are components of the home-care instructions to the family of a child with hepatitis.

A nurse provides instruction to the parents of a child with leukemia regarding measures related to monitoring for infection. Which statement by the parents indicates the need for further instructions? 1. "I need to use proper handwashing techniques." 2. "I need to take my child's rectal temperature daily." 3. "I need to inspect my child's skin daily for redness." 4. "I need to inspect my child's mouth daily for lesions."

*2. "I need to take my child's rectal temperature daily."* *Rationale:* The risk of injury to the fragile mucous membranes is so great in the child with leukemia that only oral, axillary, or temporal or tympanic temperatures should be taken. Rectal abscesses can easily occur in damaged rectal tissue, so no rectal temperatures should be taken. In addition, oral temperatures should be avoided if the child has oral ulcers. Options 1, 3, and 4 are appropriate teaching measures.

A nurse is providing instructions to the parents of a child with scoliosis regarding the use of a brace. Which statement by a parent indicates the need for further instruction? 1. "I need to have my child wear a soft fabric under the brace." 2. "I will apply lotion under the brace to prevent skin breakdown." 3. "I need to encourage my child to perform the prescribed exercises." 4. "I need to avoid applying powder under the brace, because it will cake."

*2. "I will apply lotion under the brace to prevent skin breakdown."* *Rationale:* The use of either lotions or powders should be avoided, because they can become sticky or cake under the brace, thus causing irritation. Options 1, 3, and 4 are appropriate statements regarding the care of a child with a brace.

A nurse is preparing a 2-year-old child with suspected nephrotic syndrome for a renal biopsy to confirm the diagnosis. The mother asks the nurse, "Will my child ever look thin again?" The nurse appropriately responds by saying: 1. "Do you feel guilty because about your child's weight gain?" 2. "In most cases, medication and diet will control fluid retention." 3. "Wearing loose-fitting clothing should help conceal the extra weight." 4. "When children are little, it's expected that they'll look a little chubby."

*2. "In most cases, medication and diet will control fluid retention."* *Rationale:* It is important to give the mother information that addresses the issue that is the parent's concern. Most children experience remission with treatment. Options 1 and 3 are nontherapeutic and may add to the mother's guilt. Option 4 does not acknowledge the concern and is a stereotypical response.

A nursing instructor asks a student nurse to describe osteogenic sarcoma. Which statement by the student indicates the need to further research the disease? 1. "The femur is the most common site of this sarcoma." 2. "The child does not experience pain at the primary tumor site." 3. "If a weight-bearing limb is affected, then limping is a clinical manifestation." 4. "The symptoms of the disease during the early stage are almost always attributed to normal growing pains."

*2. "The child does not experience pain at the primary tumor site."* *Rationale:* Osteogenic sarcoma is the most common bone tumor in children. A clinical manifestation of osteogenic sarcoma is progressive, insidious, intermittent pain at the tumor site. By the time these children receive medical attention, they may be in considerable pain from the tumor. Options 1, 3, and 4 are accurate regarding osteogenic sarcoma.

Isoniazid (INH) is prescribed for a 2-year-old child with a positive Mantoux test. The mother of the child asks the nurse how long the child will need to take the medication. The appropriate response is: 1. 6 months 2. 9 months 3. 15 months 4. 18 months

*2. 9 months* *Rationale:* Isoniazid is given to prevent tuberculosis (TB) infection from progressing to active disease. A chest x-ray film is obtained before the initiation of preventive therapy. In infants and children, the recommended duration of INH therapy is 9 months. For children with human immunodeficiency virus infection, a minimum of 12 months is recommended.

A nurse is assigned to care for a child with a compound (open) fracture of the arm that occurred as a result of a fall. The nurse plans care, knowing that this type of fracture involves: 1. The entire bone fractured straight across 2. A greater risk of infection than a simple fracture 3. One side of the bone being broken and the other side being bent 4. The bone being fractured but not producing a break in the skin

*2. A greater risk of infection than a simple fracture* *Rationale:* In a compound (open) fracture, a wound in the skin leads to the broken bone, and there is an added danger of infection. Option 1 describes a transverse fracture. Option 3 describes a greenstick fracture. Option 4 describes a closed or simple fracture.

A nurse is caring for an 18-month-old child who has been vomiting. The appropriate position in which to place the child during naps and sleep time is: 1. A supine position 2. A side-lying position 3. Prone, with the head elevated 4. Prone, with the face turned to the side

*2. A side-lying position* *Rationale:* The vomiting child should be placed in an upright or side-lying position to prevent aspiration. Options 1, 3, and 4 will place the child at risk for aspiration if vomiting occurs.

A nurse is initiating seizure precautions for a child being admitted to the nursing unit. Which of the following items should the nurse place at the bedside? 1. Oxygen and a tongue depressor 2. A suction apparatus and oxygen 3. An airway and a tracheotomy set 4. An emergency cart and an oxygen mask

*2. A suction apparatus and oxygen* *Rationale:* Seizures cause a tightening of all body muscles that is followed by tremors. An obstructed airway and increased oral secretions are the major complications during and after the seizure. Suctioning and oxygen are helpful to prevent choking and cyanosis. Option 1 is incorrect; a tongue depressor is not needed and nothing is placed into the client's mouth during a seizure because of the risk for injury. Option 3 is incorrect, because inserting a tracheostomy is not done. Option 4 is incorrect, because an emergency cart would not be left at the bedside; however, it would be available in the treatment room or on the nursing unit.

An 18-month-old child is being discharged after surgical repair of hypospadias. Which postoperative nursing care measure should the nurse stress to the parents as they prepare to take this child home? 1. Leave diapers off to allow the site to heal. 2. Avoid tub baths until the stent has been removed. 3. Encourage toilet training to ensure that the flow of urine is normal. 4. Restrict the fluid intake to reduce urinary output for the first few days.

*2. Avoid tub baths until the stent has been removed.* *Rationale:* After hypospadias repair, the parents are instructed to avoid giving the child a tub bath until the stent has been removed to prevent infection. Diapers are placed on the child to prevent the contamination of the surgical site. Toilet training should not be an issue during this stressful period. Fluids should be encouraged to maintain hydration.

A nurse collects a urine specimen preoperatively from a child with epispadias who is scheduled for surgical repair. The nurse reviews the child's record for the laboratory results of the urine test and would most likely expect to note which of the following? 1. Hematuria 2. Bacteriuria 3. Glucosuria 4. Proteinuria

*2. Bacteriuria* *Rationale:* Epispadias is a congenital defect that involves the abnormal placement of the urethral orifice of the penis. In clients with this condition, the urethral opening is located anywhere on the dorsum of the penis. This anatomical characteristic leads to the easy access of bacterial entry into the urine. Options 1, 3, and 4 are not characteristically noted with this condition.

A mother arrives at the emergency department with her 5-year-old child and states that the child fell off a bunk bed. A head injury is suspected, and a nurse is monitoring the child continuously for signs of increased intracranial pressure (ICP). Which of the following is a late sign of increased ICP in this child? 1. Nausea 2. Bradycardia 3. Bulging fontanel 4. Dilated scalp veins

*2. Bradycardia* *Rationale:* Late signs of increased ICP include a significant decrease in the level of consciousness, bradycardia, and fixed and dilated pupils. Nausea is an early sign of increased ICP. A bulging fontanel and dilated scalp veins are early signs of increased ICP and would be noted in an infant rather than in a 5-year-old child.

A child with leukemia is experiencing nausea related to medication therapy. The nurse, concerned about the child's nutritional status, should offer which of the following during this episode of nausea? 1. Low-calorie foods 2. Cool, clear liquids 3. Low-protein foods 4. The child's favorite foods

*2. Cool, clear liquids* *Rationale:* When the child is nauseated, it is best to offer frequent intake of cool, clear liquids in small amounts because small portions are usually better tolerated. Cool, clear fluids are also soothing and better tolerated when a client is nauseated. It is best not to offer favorite foods when the child is nauseated because foods eaten during times of nausea will be associated with being sick. It is best to offer small, frequent meals of high-protein and high-calorie content once the nausea has been controlled with medication or has subsided.

A nurse is assisting with performing admission data collection on a 2-year-old child who has been diagnosed with nephrotic syndrome. The nurse collects data knowing that a common characteristic associated with nephrotic syndrome is: 1. Hypotension 2. Generalized edema 3. Increased urinary output 4. Frank, bright red blood in the urine

*2. Generalized edema* *Rationale:* Nephrotic syndrome is defined as massive proteinuria, hypoalbuminemia, and edema. The urine is dark, foamy, and frothy, but microscopic hematuria may be present. Frank, bright red blood in the urine does not occur. Urine output is decreased, and the blood pressure is normal or slightly decreased.

A nurse is assigned to care for a child who is scheduled for an appendectomy. Which prescriptions does the nurse anticipate will be prescribed? *Select all that apply.* 1. Administer a Fleet enema. 2. Initiate an intravenous line. 3. Maintain nothing-by-mouth status. 4. Administer intravenous antibiotics. 5. Administer preoperative medications. 6. Place a heating pad on the abdomen to decrease pain.

*2. Initiate an intravenous line.* *3. Maintain nothing-by-mouth status.* *4. Administer intravenous antibiotics.* *5. Administer preoperative medications.* *Rationale:* During the preoperative period, enemas or laxatives should not be administered. In addition, heat should not be applied to the abdomen. Any of these interventions can cause the rupture of the appendix and resultant peritonitis. Intravenous fluids would be started, and the child should receive nothing by mouth while awaiting surgery. Antibiotics are usually administered because of the risk of perforation. Preoperative medications are administered as prescribed.

Which of the following are characteristics of scabies? *Select all that apply.* 1. It is caused by a fungal infection. 2. It appears as burrows or fine, grayish-red lines. 3. It is transmitted by close personal contact with an infected person. 4. It is endemic among schoolchildren and institutionalized populations. 5. Meticulous skin care and the application of antifungal cream are components of treatment. 6. Household members and contacts of the infected child need to be treated at the same time that the child is being treated.

*2. It appears as burrows or fine, grayish-red lines.* *3. It is transmitted by close personal contact with an infected person.* *4. It is endemic among schoolchildren and institutionalized populations.* *6. Household members and contacts of the infected child need to be treated at the same time that the child is being treated.* *Rationale:* Scabies usually appears as burrows or fine, grayish-red lines. It is not caused by a fungal infection, and it is treated with the application of a topical scabicide. It is transmitted by close personal contact with an infected person, and it is endemic among schoolchildren and institutionalized populations. Household members and contacts of the infected child need to be treated at the same time that the child is being treated.

A nurse prepares a list of home care instructions for the parents of a child who has a plaster cast applied to the left forearm. Choose the instructions that would be included on the list. *Select all that apply.* 1. Use the fingertips to lift the cast while it is drying. 2. Keep small toys and sharp objects away from the cast. 3. Use a padded ruler or another padded object to scratch the skin under the cast if it itches. 4. Place a heating pad on the lower end of the cast and over the fingers if the fingers feel cold. 5. Contact the health care provider if the child complains of numbness or tingling in the extremity. 6. Elevate the extremity on pillows for the first 24 to 48 hours after casting to prevent swelling.

*2. Keep small toys and sharp objects away from the cast.* *5. Contact the health care provider if the child complains of numbness or tingling in the extremity.* *6. Elevate the extremity on pillows for the first 24 to 48 hours after casting to prevent swelling.* *Rationale:* While the cast is drying, the palms of the hands are used to lift the cast. If the fingertips are used, indentations in the cast could occur and cause constant pressure on the underlying skin. Small toys and sharp objects are kept away from the cast, and no objects (including padded objects) are placed inside of the cast because of the risk of altered skin integrity. A heating pad is not applied to the cast or fingers. Cold fingers could indicate neurovascular impairment, and the HCP should be notified. The extremity is elevated to prevent swelling, and the HCP is notified immediately if any signs of neurovascular impairment develop.

A nurse is caring for an infant with a diagnosis of tetralogy of Fallot. The infant suddenly becomes cyanotic and the oxygen saturation reading drops to 60%. Choose the interventions that the nurse should perform. *Select all that apply.* 1. Call a code blue. 2. Notify the registered nurse. 3. Place the infant in a prone position. 4. Prepare to administer morphine sulfate. 5. Prepare to administer intravenous fluids. 6. Prepare to administer 100% oxygen by face mask.

*2. Notify the HCP.* *4. Prepare to administer morphine sulfate.* *5. Prepare to administer intravenous fluids.* *6. Prepare to administer 100% oxygen by face mask.* *Rationale:* The child who is cyanotic with oxygen saturations dropping to 60% is having a hypercyanotic episode. Hypercyanotic episodes often occur among infants with tetralogy of Fallot, and they may occur among infants whose heart defect includes the obstruction of pulmonary blood flow and communication between the ventricles. If a hypercyanotic episode occurs, the infant is placed in a knee-chest position immediately. The registered nurse is notified, who will then contact the health care provider. The knee-chest position improves systemic arterial oxygen saturation by decreasing venous return so that smaller amounts of highly saturated blood reach the heart. Toddlers and children squat to get into this position and relieve chronic hypoxia. There is no reason to call a code blue unless respirations cease. Additional interventions include administering 100% oxygen by face mask, morphine sulfate, and intravenous fluids, as prescribed.

A nurse is monitoring for bleeding in a child after surgery for the removal of a brain tumor. The nurse checks the head dressing for the presence of blood and notes a colorless drainage on the back of the dressing. Which nursing action is appropriate? 1. Reinforce the dressing. 2. Notify the HCP. 3. Document the findings and continue to monitor. 4. Circle the area of drainage and continue to monitor.

*2. Notify the HCP.* *Rationale:* Colorless drainage on the dressing would indicate the presence of cerebrospinal fluid and should be reported to the HCP immediately. The colorless drainage should also be checked for evidence of cerebrospinal fluid; one method is to check for the presence of glucose using a dipstick. Options 1, 3, and 4 are incorrect and delay required immediate interventions.

A nurse is providing home care instructions to the mother of an infant who has just been found to have hemophilia. The nurse tells the mother to: 1. Use aspirin for pain relief. 2. Pad crib rails and table corners. 3. Use a soft toothbrush for dental hygiene. 4. Use a generous amount of lubricant when taking a temperature rectally.

*2. Pad crib rails and table corners.* *Rationale:* Establishment of an age-appropriate safe environment is of paramount importance for hemophiliac clients. Providing a safe environment for an infant includes padding table corners and crib rails, providing extra "joint" padding on clothes, observing a mobile infant at all times, and keeping items that can be pulled down onto the infant out of reach. Use of a soft toothbrush is an appropriate measure for a child with hemophilia, but is not typically necessary for an infant. Rectal temperature measurements and the use of aspirin are contraindicated in hemophiliac individuals because of the risk of bleeding.

A nurse is reviewing the record of a child with a diagnosis of pyloric stenosis. Which data would the nurse expect to note as having been documented in the child's record? 1. Watery diarrhea 2. Projectile vomiting 3. Increased urine output 4. Vomiting large amounts of bile

*2. Projectile vomiting* *Rationale:* Clinical manifestations of pyloric stenosis include projectile, nonbilious vomiting; irritability; hunger and crying; constipation; and signs of dehydration, including a decrease in urine output.

A nurse is assigned to care for a child who is suspected of having glomerulonephritis. The nurse reviews the child's record and notes that which finding is associated with the diagnosis of glomerulonephritis? 1. Hypotension 2. Red-brown urine 3. Low urinary specific gravity 4. A low blood urea nitrogen (BUN) level

*2. Red-brown urine* *Rationale:* Gross hematuria resulting in dark, smoky, cola-colored or red-brown urine is a classic symptom of glomerulonephritis, and hypertension is also common. A mid to high urinary specific gravity is associated with glomerulonephritis. BUN levels may be elevated.

A nurse who is working in the emergency department is caring for a child who has been diagnosed with epiglottitis. Indications that the child may be experiencing airway obstruction include which of the following? 1. Nasal flaring and bradycardia 2. The child thrusts the chin forward and opens the mouth 3. A low-grade fever and complaints of a sore throat 4. The child leans backward, supporting himself or herself with the hands and arms

*2. The child thrusts the chin forward and opens the mouth* *Rationale:* Clinical manifestations that are suggestive of airway obstruction include tripod positioning (leaning forward supported by the hands and arms with the chin thrust out and the mouth open), nasal flaring, tachycardia, a high fever, and a sore throat.

The mother of a child with juvenile idiopathic arthritis calls the nurse because the child is experiencing a painful exacerbation of the disease. The mother asks the nurse if the child should perform range-of-motion (ROM) exercises at this time. The nurse makes which response to the mother? 1. "Avoid all exercise during painful periods." 2. "The ROM exercises must be performed every day." 3. "Have the child perform simple isometric exercises during this time." 4. "Administer additional pain medication before performing the ROM exercises."

*3. "Have the child perform simple isometric exercises during this time."* *Rationale:* During painful episodes, hot or cold packs, splinting, and positioning the affected joint in a neutral position help to reduce the pain. Although resting the extremity is appropriate, it is important to begin simple isometric or tensing exercises as soon as the child is able. These exercises do not involve joint movement.

Griseofulvin (Gris-PEG) is prescribed for a child with tinea capitis. The nurse provides instructions to the family regarding administration of the medication. Which statement by the mother indicates a need for further instructions? 1. "I need to keep my child out of the sun." 2. "I need to continue the therapy as long as it is prescribed." 3. "I need to administer the medication 2 hours before meals." 4. "I need to shake the oral suspension before preparing the dose."

*3. "I need to administer the medication 2 hours before meals."* *Rationale:* Gris-PEG is given with or after meals to avoid gastrointestinal (GI) irritation and to increase absorption. Oral suspensions should be shaken well. Parents are instructed to continue therapy as prescribed and not to miss a dose. Exposure to the sun is avoided during treatment.

A nurse provides information to the mother of a 2-week-old infant who was diagnosed with clubfoot at the time of birth. Which statement by the mother indicates the need for further instruction regarding this disorder? 1. "Treatment needs to be started as soon as possible." 2. "I realize my child will require follow-up care until full grown." 3. "I need to bring my child back to the clinic in 1 month for a new cast." 4. "I need to come to the clinic every week with my child for the casting."

*3. "I need to bring my child back to the clinic in 1 month for a new cast."* *Rationale:* The treatment for clubfoot is started as soon as possible after birth. Serial manipulation and casting are performed at least weekly. If sufficient correction is not achieved within 3 to 6 months, surgery is usually indicated. Because clubfoot can recur, all children with the condition require long-term interval follow-up until they reach skeletal maturity to ensure an optimal outcome.

A nurse is reinforcing discharge instructions to the mother of a 2-year-old child who has had an orchiopexy to correct cryptorchidism. Which of the following statements, if made by the mother of the child, indicates that further teaching is necessary? 1. "I'll check his temperature." 2. "I'll give him medication so he'll be comfortable." 3. "I'll let him decide when to return to his play activities." 4. "I'll check his voiding to be sure there are no problems."

*3. "I'll let him decide when to return to his play activities."* *Rationale:* All vigorous activities should be restricted for 2 weeks after surgery to promote healing and prevent injury. This will prevent dislodging of the suture, which is internal. Normally 2-year-old children will want to be very active. Therefore, allowing the child to decide when to return to his play activities may prevent healing and cause injury. The parents should be taught to monitor the child's temperature, provide analgesics, as needed, and monitor the urine output.

A nurse has provided instructions to the mother of a child who has been diagnosed with bacterial conjunctivitis. Which of the following, if stated by the mother, would indicate the need for further instructions? 1. "I need to wash my hands frequently." 2. "I need to clean the eye, as prescribed." 3. "It is OK to share towels and washcloths." 4. "I need to give the eyedrops, as prescribed."

*3. "It is OK to share towels and washcloths."* *Rationale:* Bacterial conjunctivitis is highly contagious, and infection control measures should be taught; these include frequent handwashing and not sharing towels and washcloths. Options 1, 2, and 4 are correct measures.

A nurse assists with providing an instructional session to parents regarding impetigo. Which statement by a parent indicates the need for further instruction? 1. "It is extremely contagious." 2. "It is most common during humid weather." 3. "Lesions are most often located on the arms and chest." 4. "It begins in an area of broken skin, such as an insect bite."

*3. "Lesions are most often located on the arms and chest."* *Rationale:* Impetigo is most common during the hot and humid summer months. It begins in an area of broken skin, such as an insect bite. It may be caused by Staphylococcus aureus, group A β-hemolytic streptococci, or a combination of these bacteria. It is extremely contagious. *Lesions are most often located around the mouth and nose*, but they may be present on the extremities.

A nurse provides instructions regarding the use of permethrin 1% (Nix) to the parents of a child who has been diagnosed with pediculosis capitis (head lice). Which statement by a parent indicates the need for further instruction? 1. "The hair should not be shampooed for 24 hours after treatment." 2. "The medication can be obtained over the counter in a local pharmacy." 3. "The medication is applied to the hair after shampooing and left on for 24 hours." 4. "The medication is applied to the hair after shampooing, left on for 5 to 10 minutes, and then rinsed out."

*3. "The medication is applied to the hair after shampooing and left on for 24 hours."* *Rationale:* Permethrin 1% is an over-the-counter anti-lice product that kills lice and eggs with one application and that has residual activity for 10 days. *It is applied to dried hair after shampooing and left for 5 to 10 minutes before it is rinsed (not shampooed) out*. The hair should not be shampooed for 24 hours after the treatment.

A nurse reinforces home care instructions to the parents of a child hospitalized with pertussis. The child is in the convalescent stage and is being prepared for discharge. Which statement by the parents indicates a need for further instructions? 1. "We need to encourage adequate fluid intake." 2. "Coughing spells may be triggered by dust or smoke." 3. "We need to maintain droplet precautions and a quiet environment for at least 2 weeks." 4. "Good hand-washing techniques need to be instituted to prevent spreading the disease to others."

*3. "We need to maintain droplet precautions and a quiet environment for at least 2 weeks."* *Rationale:* Pertussis is transmitted by direct contact or respiratory droplets from coughing. The communicable period occurs primarily during the catarrhal stage. Droplet precautions are not required during the convalescent phase. Options 1, 2, and 4 are components of home care instructions.

A child is scheduled to receive a measles, mumps, and rubella (MMR) vaccine. The nurse who is preparing to administer the vaccine reviews the child's record. Which finding should make the nurse question the health care provider's prescription? 1. Recent recovery from a cold 2. A history of frequent respiratory infections 3. A history of an anaphylactic reaction to neomycin 4. A local reaction at the site of a previous MMR vaccine injection

*3. A history of an anaphylactic reaction to neomycin* *Rationale:* The MMR vaccine contains minute amounts of neomycin. A history of an anaphylactic reaction to neomycin is considered a contraindication to the MMR vaccine. The general contraindication to all immunizations is a severe febrile illness. The presence of a minor illness such as the common cold is not a contraindication. In addition, a history of frequent respiratory infections is not a contraindication to receiving a vaccine. A local reaction to an immunization is treated with cool packs for the first 24 hours after injection, and this is followed by warm or cool compresses if the inflammation persists.

A 6-month-old infant receives a diphtheria, tetanus, and acellular pertussis (DTaP) immunization at the well-baby clinic. The mother returns home and calls the clinic to report that the infant has developed swelling and redness at the site of injection. Which instruction by the nurse is appropriate? 1. Monitor the infant for a fever. 2. Bring the infant back to the clinic. 3. Apply an ice pack to the injection site. 4. Leave the injection site alone, because this always occurs.

*3. Apply an ice pack to the injection site.* *Rationale:* Occasionally tenderness, redness, or swelling may occur at the site of the injection. This can be relieved with cool packs for the first 24 hours and followed by warm or cool compresses if the inflammation persists. It is not necessary to bring the infant back to the clinic. Option 1 may be an appropriate intervention, but it is not specific to the subject of the question.

Which of the following assessment findings may indicate that a child had a tonic-clonic seizure during the night? 1. High-pitched cry 2. Blanched toenails 3. Blood on the pillow 4. Migraine headaches

*3. Blood on the pillow* *Rationale:* The complications associated with seizures include airway compromise, extremity and teeth injuries, and tongue lacerations. Night seizures can cause the child to bite down on the tongue. Seizures do not cause a high-pitched cry unless a tumor or intracranial pressure is the cause of the seizure diagnosis. Cyanosis can occur during the tonic-clonic part of the seizure activity, but blanching does not occur. Migraine headaches are not common in children with seizures.

A 4-year-old child is admitted to the hospital with suspected acute lymphocytic leukemia (ALL). The nurse understands that which diagnostic study will confirm this diagnosis? 1. A platelet count 2. A lumbar puncture 3. Bone marrow biopsy 4. White blood cell (WBC) count

*3. Bone marrow biopsy* *Rationale:* The confirmatory test for leukemia is microscopic examination of bone marrow obtained by bone marrow aspirate and biopsy. The WBC count may be high or low in leukemia. A lumbar puncture may be done to look for blast cells in the spinal fluid that are indicative of central nervous system disease. An altered platelet count occurs as a result of chemotherapy.

Which of the following is the most appropriate location for assessing the pulse of an infant who is less than 1 year old? 1. Radial 2. Carotid 3. Brachial 4. Popliteal

*3. Brachial* *Rationale:* To assess a pulse in an infant (i.e., a child <1 year old), the pulse is checked at the brachial artery. The infant's relatively short, fat neck makes palpation of the carotid artery difficult. The popliteal and radial pulses are also difficult to palpate in an infant.

A nurse reinforces instructions to the mother about dietary measures for a 5-year-old child with lactose intolerance. The nurse tells the mother that which of the following supplements will be required as a result of the need to avoid lactose in the diet? 1. Fats 2. Zinc 3. Calcium 4. Thiamine

*3. Calcium* *Rationale:* Lactose intolerance is the inability to tolerate lactose, which is the sugar that is found in dairy products. Removing milk from the diet can provide relief from symptoms. Additional dietary changes may be required to provide adequate sources of calcium and, if the child is an infant, protein and calories.

A nurse reviews the record of an infant who is seen in the clinic. The nurse notes that a diagnosis of esophageal atresia with tracheoesophageal fistula (TEF) is suspected. The nurse expects to note which most likely clinical manifestation of this condition in the medical record? 1. Incessant crying 2. Coughing at nighttime 3. Choking with feedings 4. Severe projectile vomiting

*3. Choking with feedings* *Rationale:* Any child who exhibits the "3 Cs"—coughing and choking during feedings and unexplained cyanosis—should be suspected of having TEF. Options 1, 2, and 4 are not specifically associated with TEF.

A nurse assists with admitting a child with a diagnosis of acute-stage Kawasaki disease. When obtaining the child's medical history, which clinical manifestation is likely to be reported? 1. Cracked lips 2. A normal appearance 3. Conjunctival hyperemia 4. Desquamation of the skin

*3. Conjunctival hyperemia* *Rationale:* During the acute stage of Kawasaki disease, the child presents with fever, conjunctival hyperemia, a red throat, swollen hands, a rash, and enlargement of the cervical lymph nodes. During the subacute stage, cracking lips and fissures, desquamation of the skin on the tips of the fingers and toes, joint pain, cardiac manifestations, and thrombocytosis occur. During the convalescent stage, the child appears normal, but signs of inflammation may be present.

The nurse provides instructions regarding droplet precautions to the mother of a child with mumps. The mother asks the nurse about the length of time required for the respiratory precautions. Which response by the nurse is accurate? 1. Droplet isolation is not necessary. 2. Mumps is not transmitted by the respiratory system. 3. Droplet precautions are indicated during the period of communicability. 4. Droplet precautions are indicated for 18 days after the onset of parotid swelling.

*3. Droplet precautions are indicated during the period of communicability.* *Rationale:* Mumps is transmitted via direct contact or droplets spread from an infected person and possibly by contact with urine. Droplet precautions are indicated during the period of communicability. Options 1, 2, and 4 are incorrect.

A nurse who is caring for a child with aplastic anemia reviews the laboratory results and notes a white blood cell (WBC) count of 6000 cells/ mm3 and a platelet count of 27,000 cells/mm3. Which nursing intervention should be incorporated into the plan of care? 1. Encourage naps. 2. Encourage a diet high in iron. 3. Encourage quiet play activities. 4. Maintain strict isolation precautions.

*3. Encourage quiet play activities.* *Rationale:* Precautionary measures to prevent bleeding should be taken when a child has a low platelet count. These include no injections, no rectal temperatures, the use of a soft toothbrush, and abstinence from contact sports or activities that could cause an injury. Strict isolation would be required if the WBC count was low. Naps and a diet high in iron are unrelated to the risk of bleeding.

A mother of a 6-year-old child with type 1 diabetes mellitus calls the clinic nurse and tells the nurse that the child has been sick. The mother reports that she checked the child's urine and it showed positive ketones. Which of the following would the nurse instruct the mother to do? 1. Hold the next dose of insulin. 2. Come to the clinic immediately. 3. Encourage the child to drink liquids. 4. Administer an additional dose of regular insulin.

*3. Encourage the child to drink liquids.* *Rationale:* When the child is sick, the mother should test for urinary ketones with each voiding. If ketones are present, liquids are essential to help with clearing them. The child should be encouraged to drink liquids. It is not necessary to bring the child to the clinic immediately, and insulin doses should not be adjusted or changed.

A nurse reviews the record of a child who was just seen by a health care provider (HCP). The HCP has documented a diagnosis of suspected aortic stenosis. Which clinical manifestation that is specifically found in children with this disorder should the nurse anticipate? 1. Pallor 2. Hyperactivity 3. Exercise intolerance 4. Gastrointestinal disturbances

*3. Exercise intolerance* *Rationale:* The child with aortic stenosis shows signs of exercise intolerance, chest pain, and dizziness when standing for long periods. Pallor may be noted, but it is not specific to this type of disorder alone. Options 2 and 4 are not related to this disorder.

A nurse provides instructions to the mother of a child with impetigo regarding the application of antibiotic ointment. The mother asks the nurse when the child can return to school. Which response by the nurse is accurate? 1. Ten days after using the antibiotic ointment 2. One week after using the antibiotic ointment 3. Forty-eight hours after using the antibiotic ointment 4. Twenty-four hours after using the antibiotic ointment

*3. Forty-eight hours after using the antibiotic ointment* *Rationale:* The child should not attend school for 24 to 48 hours after the initiation of systemic antibiotics or for 48 hours after the use of the antibiotic ointment. The school should be notified of the diagnosis. Therefore options 1, 2, and 4 are incorrect.

A nursing instructor asks a nursing student about the cause of hemophilia. The student correctly responds by telling the instructor that: 1. Hemophilia is a Y-linked hereditary disorder. 2. A splenectomy resolves the bleeding disorders. 3. Hemophilia A results from deficiency of factor VIII. 4. A bone marrow transplant is the treatment of choice.

*3. Hemophilia A results from deficiency of factor VIII.* *Rationale:* The term "hemophilia" refers to a group of bleeding disorders. The identification of the specific factor deficiencies allows for definitive treatment with replacement agents. Hemophilia A results from a deficiency of factor VIII. Hemophilia B (Christmas disease) is a deficiency of factor IX. Hemophilia is inherited in a recessive manner via a genetic defect on the X chromosome, not the Y chromosome. Neither a bone marrow transplant nor splenectomy is used to treat this disorder.

A nurse is assigned to care for an infant with cryptorchidism. The nurse anticipates that diagnostic studies will be prescribed to evaluate: 1. DNA synthesis 2. Babinski reflex 3. Kidney function 4. Chromosomal analysis

*3. Kidney function* *Rationale:* Cryptorchidism [undescended testicle] may be the result of hormone deficiency, intrinsic abnormality of a testis, or a structural problem. *Diagnostic tests would assess kidney function, because the kidneys and testes arise from the same germ tissue*. Babinski's reflex tests neurological function and is unrelated to this diagnosis. DNA synthesis and a chromosomal analysis are also unrelated to this diagnosis.

Which of the following represents a primary characteristic of autism? 1. Normal social play 2. Consistent imitation of others' actions 3. Lack of social interaction and awareness 4. Normal verbal and nonverbal communication

*3. Lack of social interaction and awareness* *Rationale:* Autism is a severe form of an autism spectrum disorder. A primary characteristic is a lack of social interaction and awareness. Social behaviors in autism include a lack of or an abnormal imitation of others' actions and a lack of or abnormal social play. Additional characteristics include a lack of or impaired verbal communication and markedly abnormal nonverbal communication.

The nurse assists in planning care for a child who sustained a burn injury based on which of the following accurate statements? 1. Scarring is not as severe in a child as in an adult. 2. Children are at a lower risk of infection than adults because of their strong immune systems. 3. Lower burn temperatures and shorter exposure to heat can cause a more severe burn in a child than an adult because a child's skin is thinner. 4. Infants and children are at decreased risk for protein and calorie deficiency because they have smaller muscle mass and less body fat than adults.

*3. Lower burn temperatures and shorter exposure to heat can cause a more severe burn in a child than an adult because a child's skin is thinner.* *Rationale:* Lower burn temperatures and shorter exposure to heat can cause a more severe burn in a child than an adult because a child's skin is thinner. Scarring is more severe in a child; additionally, disturbed body image will be a distinct issue for a child or adolescent, especially as growth continues. An immature immune system presents an increased risk of infection for infants and young children. Infants and children are at increased risk for protein and calorie deficiency because they have smaller muscle mass and less body fat than adults.

A nurse is assigned to care for a child after a spinal fusion for the treatment of scoliosis. The child complains of abdominal discomfort and begins to have episodes of vomiting. On data collection, the nurse notes abdominal distention. Which action should the nurse take? 1. Administer an antiemetic. 2. Increase the intravenous fluids. 3. Notify the HCP. 4. Place the child in a side-lying Sims' position.

*3. Notify the HCP.* *Rationale:* A complication after the surgical treatment of scoliosis is superior mesenteric artery syndrome. This disorder is caused by mechanical changes in the position of the child's abdominal contents that result from the lengthening of the child's body. It results in a syndrome of emesis and abdominal distention that is similar to that which occurs with intestinal obstruction or paralytic ileus. Postoperative vomiting among children with body casts or among those who have undergone spinal fusion warrants attention because of the possibility of superior mesenteric artery syndrome. Therefore, the remaining options are incorrect.

A nurse is performing a neurovascular check on a child with a cast applied to the lower leg. The child complains of tingling in the toes distal to the fracture site. Which action should be taken by the nurse? 1. Elevate the extremity. 2. Document the findings. 3. Notify the health care provider (HCP). 4. Ambulate the child with crutches.

*3. Notify the health care provider (HCP).* *Rationale:* Reduced sensation to touch or complaints of numbness or tingling at a site distal to the fracture may indicate poor tissue perfusion. This finding should be reported to the HCP. Options 1, 2, and 4 are inappropriate and would delay the required and immediate interventions.

A nurse is monitoring an infant for signs of increased intracranial pressure (ICP) and notes that the anterior fontanel bulges when the infant is sleeping. Based on this finding, which of the following is the priority nursing action? 1. Increase oral fluids. 2. Document the finding. 3. Notify the HCP. 4. Place the infant supine in a side-lying position.

*3. Notify the registered nurse.* *Rationale:* The anterior fontanel is diamond-shaped and located on the top of the head. It should be soft and flat in a normal infant, and it normally closes by 12 to 18 months of age. A larger-than-normal fontanel may be a sign of increased ICP within the skull. Although the anterior fontanel may bulge slightly when the infant cries, *bulging at rest may indicate increased ICP*. Options 1 and 4 are inaccurate interventions. Although the nurse would document the finding, the first action is to report the finding to the health care provider.

An infant returns to the nursing unit after the surgical repair of a cleft lip located on the right side of the lip. The best position in which to place this infant at this time is: 1. A flat position 2. A prone position 3. On his or her left side 4. On his or her right side

*3. On his or her left side* *Rationale:* After the repair of a cleft lip, the infant should be positioned on the side opposite to the repair to prevent contact of the suture lines with the bed linens. In this case, it is best to place the infant on his or her left side. Additionally, the flat or prone position can result in aspiration if the infant vomits.

A nurse is assigned to care for a child who is in skeletal traction. The nurse avoids which of the following when caring for the child? 1. Keeping the weights hanging freely 2. Ensuring that the ropes are in the pulleys 3. Placing the bed linens on the traction ropes 4. Ensuring that the weights are out of the child's reach

*3. Placing the bed linens on the traction ropes* *Rationale:* Bed linens should not be placed on the traction ropes because of the risk of disrupting the traction apparatus. Options 1, 2, and 4 are appropriate measures when caring for a child who is in skeletal traction.

A nurse is caring for a hospitalized infant with bronchiolitis. Diagnostic tests have confirmed respiratory syncytial virus (RSV). On the basis of this finding, which of the following would be the appropriate nursing action? 1. Initiate strict enteric precautions. 2. Wear a mask when caring for the child. 3. Plan to move the infant to a room with another child with RSV. 4. Leave the infant in the present room, because RSV is not contagious.

*3. Plan to move the infant to a room with another child with RSV.* *Rationale:* RSV is a highly communicable disorder, but it is not transmitted via the airborne route. It is usually transferred by the hands, and meticulous handwashing is necessary to decrease the spread of organisms. The infant with RSV is isolated in a single room or placed in a room with another child with RSV. Enteric precautions are not necessary; however, the nurse should wear a gown when the soiling of clothing may occur.

The child with cryptorchidism is being discharged after orchiopexy, which was performed on an outpatient basis. The nurse informs the parents about which priority care measure? 1. Measuring intake and output 2. Administering anticholinergics 3. Preventing infection at the surgical site 4. Applying cold, wet compresses to the surgical site

*3. Preventing infection at the surgical site* *Rationale:* The most common complications associated with orchiopexy are bleeding and infection. The parents are instructed in postoperative home care measures, including the prevention of infection, pain control, and activity restrictions. The measurement of intake and output is not required. Anticholinergics are prescribed for the relief of bladder spasms; they are not necessary after orchiopexy. Cold, wet compresses are not prescribed. The moisture from a wet compress presents a potential for infection.

A nurse prepares a teaching plan regarding the administration of eardrops for the parents of a 2-year-old child. Which of the following would be included in the plan? 1. Wear gloves when administering the eardrops. 2. Pull the ear up and back before instilling the eardrops. 3. Pull the earlobe down and back before instilling the ear drops. 4. Hold the child in a sitting position when administering the ear drops.

*3. Pull the earlobe down and back before instilling the ear drops.* *Rationale:* When administering eardrops to a child who is less than 3 years old, the ear should be pulled down and back. For children who are more than 3 years old, the ear is pulled up and back. Gloves do not need to be worn by the parents, but handwashing needs to be performed before and after the procedure. The child should be in a side-lying position with the affected ear facing upward to facilitate the flow of medication down the ear canal by gravity.

A nurse caring for an infant with congenital heart disease is monitoring the infant closely for signs of congestive heart failure (CHF). The nurse looks for which early sign of CHF? 1. Pallor 2. Cough 3. Tachycardia 4. Slow and shallow breathing

*3. Tachycardia* *Rationale:* The early signs of CHF include tachycardia, tachypnea, profuse scalp sweating, fatigue, irritability, sudden weight gain, and respiratory distress. A cough may occur with CHF as a result of mucosal swelling and irritation, but it is not an early sign. Pallor may be noted in the infant with CHF, but it is also not an early sign.

A nurse is providing home care instructions to the mother of a child with bacterial conjunctivitis. The nurse should tell the mother: 1. That the child may attend school if antibiotics have been started 2. To save any unused eye medication in case a sibling gets the eye infection 3. That the child's towels and washcloths should not be used by other members of the household 4. To wipe any crusted material from the eye with a cotton ball soaked in warm water, starting at the outer aspect of the eye and moving toward the inner aspect

*3. That the child's towels and washcloths should not be used by other members of the household* *Rationale:* Bacterial conjunctivitis is highly contagious, and infection control measures should be taught. These include good handwashing and not sharing towels and washcloths with others. The child should be kept home from school until 24 hours after antibiotics are started. Bottles of eye medication should never be shared with others. Crusted material may be wiped from the eye with a cotton ball soaked in warm water, starting at the inner aspect of the eye and moving toward the outer aspect.

A day care nurse is observing a 2-year-old child and suspects that the child may have strabismus. Which of the following observations may be indicative of this condition? 1. The child has difficulty hearing. 2. The child does not respond when spoken to. 3. The child consistently tilts his or her head to see. 4. The child consistently turns his or her head to see.

*3. The child consistently tilts his or her head to see.* *Rationale:* The nurse may suspect strabismus in a child when the child complains of frequent headaches, squints, or tilts the head to see. Options 1, 2, and 4 are not indicative of this condition.

A nurse provides instructions to the parents of an infant with hip dysplasia regarding care of the Pavlik harness. Which instruction provided by the nurse is accurate? 1. The harness must be worn 8 hours a day. 2. The infant should never be moved when out of the harness. 3. The harness needs to be removed to check the skin and for bathing. 4. The harness must be removed for diaper changes and for feeding.

*3. The harness needs to be removed to check the skin and for bathing.* *Rationale:* The harness should be worn 23 hours a day and should be removed only to check the skin and for bathing. The hips and buttocks should be supported carefully when the infant is out of the harness. The harness does not need to be removed for diaper changes or feedings.

A child with a diagnosis of a hernia has been scheduled for a surgical repair in 2 weeks. The nurse reinforces instructions to the parents about the signs of possible hernial strangulation. The nurse tells the parents that which of the following signs would require health care provider (HCP) notification by the parents? 1. Fever 2. Diarrhea 3. Vomiting 4. Constipation

*3. Vomiting* *Rationale:* The parents of a child with a hernia need to be instructed about the signs of strangulation. These signs include vomiting, pain, and an irreducible mass. The parents should be instructed to contact the HCP immediately if strangulation is suspected. Fever, diarrhea, and constipation are not associated with strangulation of a hernia.

A nursing instructor asks a nursing student about phenylketonuria (PKU). Which statement, if made by the student, indicates an understanding of this disorder? 1. "PKU is an autosomal-dominant disorder." 2. "PKU primarily affects the gastrointestinal system." 3. "Treatment of PKU includes the dietary restriction of tyramine." 4. "All 50 states require routine screening of all newborns for PKU."

*4. "All 50 states require routine screening of all newborns for PKU."* *Rationale:* PKU is an autosomal-*recessive* disorder. Treatment includes the dietary restriction of phenylalanine intake (not tyramine intake). PKU is a genetic disorder that results in central nervous system (CNS) damage from toxic levels of phenylalanine in the blood.

A nurse is told that a child with rheumatic fever (RF) will be arriving to the nursing unit for admission. Which question should the nurse ask the family to elicit information specific to the development of RF? 1. "Has the child complained of back pain?" 2. "Has the child complained of headaches?" 3. "Has the child had any nausea or vomiting?" 4. "Did the child have a sore throat or an unexplained fever within the past 2 months?"

*4. "Did the child have a sore throat or an unexplained fever within the past 2 months?"* *Rationale:* Rheumatic fever (RF) characteristically presents 2 to 6 weeks after an untreated or partially treated group A β-hemolytic streptococcal infection of the upper respiratory tract. Initially, the nurse determines if the child has had a sore throat or an unexplained fever within the past 2 months. Options 1, 2, and 3 are unrelated to RF.

A nurse provides home care instructions to the mother of a child recovering from Reye's syndrome. Which statement by the mother indicates a need for further instruction? 1. "I need to check for jaundiced skin and eyes every day." 2. "I need to have my child nap during the day to provide rest." 3. "I need to decrease the stimuli at home to prevent intracranial pressure." 4. "I need to give frequent, small, nutritious meals if my child starts to vomit."

*4. "I need to give frequent, small, nutritious meals if my child starts to vomit."* *Rationale:* The vomiting that occurs in Reye's syndrome is caused by cerebral edema and is a symptom of increased intracranial pressure. Small, frequent meals will not affect the amount of vomiting, and the health care provider is notified if vomiting occurs. Options 1, 2, and 3 are all correct statements. Decreasing stimuli and providing rest decrease stress on the brain tissue. Checking for jaundice will assist in identifying the presence of liver complications, which are characteristic of Reye's syndrome.

A nurse reinforces home-care instructions to the parents of a 3-year-old child who has been hospitalized with hemophilia. Which statement by a parent indicates the need for further instructions? 1. "I will supervise my child closely." 2. "I will pad the corners of the furniture." 3. "I will remove household items that can easily fall over." 4. "I will avoid immunizations and dental hygiene treatments for my child."

*4. "I will avoid immunizations and dental hygiene treatments for my child."* *Rationale:* The nurse needs to stress the importance of immunizations, dental hygiene, and routine well-child care. Options 1, 2, and 3 are appropriate statements. The parents are also provided instructions regarding measures to take in the event of blunt trauma (especially trauma that involves the joints), and they are instructed to apply prolonged pressure to superficial wounds until the bleeding has stopped.

A nurse provides home care instructions to the parents of a child with congestive heart failure regarding the procedure for the administration of digoxin (Lanoxin). Which statement, if made by a parent, indicates the need for further instruction? 1. "I will not mix the medication with food." 2. "If more than one dose is missed, I will call the health care provider." 3. "I will take my child's pulse before administering the medication." 4. "If my child vomits after medication administration, I will repeat the dose."

*4. "If my child vomits after medication administration, I will repeat the dose."* *Rationale:* The parents need to be instructed that, if the child vomits after the digoxin is administered, they are not to repeat the dose. Options 1, 2, and 3 are accurate instructions regarding the administration of this medication. Additionally, the parents should be instructed that if a dose is missed and it is not noticed until 4 hours later, the dose should not be administered.

A mother with human immunodeficiency virus (HIV) infection brings her 10-month-old infant to the clinic for a routine checkup. A health care provider has documented that the infant is asymptomatic for HIV infection. After the checkup, the mother tells the nurse that she is so pleased that the infant will not get HIV. Which response by the nurse is appropriate? 1. "I am also so pleased that everything has turned out fine." 2. "Since symptoms have not developed, it is unlikely that the infant will develop HIV infection." 3. "Everything looks great, but be sure that you return with your infant next month for the scheduled visit." 4. "Most children infected with HIV develop symptoms within the first 9 months of life, and some become symptomatic at some point before the age of 3 years."

*4. "Most children infected with HIV develop symptoms within the first 9 months of life, and some become symptomatic at some point before the age of 3 years."* *Rationale:* Most children who are infected with HIV develop symptoms within the first 9 months of life. The remainder of these infected children become symptomatic sometime before the age of 3 years. Children, with their immature immune systems, have a much shorter incubation period than adults. Options 1, 2, and 3 are incorrect responses.

A nurse is teaching cardiopulmonary resuscitation to a group of nursing students. The nurse asks a student to describe the reason why blind finger sweeps are avoided in infants. The nurse determines that the student understands the reason if the student makes which statement? 1. "The object may have been swallowed." 2. "The infant may bite down on the finger" 3. "The mouth is too small to see the object." 4. "The object may be forced back further into the throat."

*4. "The object may be forced back further into the throat."* *Rationale:* Blind finger sweeps are not recommended for infants and children because of the risk of forcing the object further down into the airway. Options 1, 2, and 3 are not related directly to the subject of the question.

A nurse is caring for a child recently diagnosed with cerebral palsy. The parents of the child ask the nurse about the disorder. The nurse bases the response to the parents on the understanding that cerebral palsy is: 1. An infectious disease of the central nervous system 2. An inflammation of the brain as a result of a viral illness 3. A congenital condition that results in moderate to severe retardation 4. A chronic disability characterized by a difficulty in controlling the muscles

*4. A chronic disability characterized by a difficulty in controlling the muscles* *Rationale:* Cerebral palsy is a chronic disability characterized by difficulty in controlling the muscles as a result of an abnormality in the extrapyramidal or pyramidal motor system. Meningitis is an infectious process of the central nervous system. Encephalitis is an inflammation of the brain that occurs as a result of viral illness or central nervous system infection. Down syndrome is an example of a congenital condition that results in moderate to severe retardation.

A nurse is instructing the mother of a child with cystic fibrosis (CF) about the appropriate dietary measures. Which of the following meals best illustrates the most appropriate diet for a client with cystic fibrosis? 1. A veggie salad and a caramel apple 2. A strawberry jelly sandwich and pretzels 3. A plate of nachos and cheese and a cupcake 4. A piece of fried chicken and a loaded baked potato

*4. A piece of fried chicken and a loaded baked potato* *Rationale:* Children with CF are managed with a high-calorie, high-protein diet. Pancreatic enzyme replacement therapy is undertaken, and fat-soluble vitamin supplements are administered. Fats are not restricted unless steatorrhea cannot be controlled by increased levels of pancreatic enzymes. A piece of fried chicken and a loaded baked potato provides a high-calorie and high-protein meal that includes fat.

A nursing student is assigned to help administer immunizations to children in a clinic. The nursing instructor asks the student about the contraindications to receiving an immunization. Immunization is contraindicated in the presence of which condition? 1. A cold 2. Otitis media 3. Mild diarrhea 4. A severe febrile illness

*4. A severe febrile illness* *Rationale:* A severe febrile illness is a reason to delay immunization, but only until the child has recovered from the acute stage of the illness. Minor illnesses such as a cold, otitis media, or mild diarrhea are not contraindications to immunization.

Acetylsalicylic acid (aspirin) is prescribed for a child with rheumatic fever (RF). The nurse would question this prescription if the child had documented evidence of which condition? 1. Arthralgia 2. Joint pain 3. Facial edema 4. A viral infection

*4. A viral infection* *Rationale:* Anti-inflammatory agents, including aspirin, may be prescribed by the health care provider for the child with RF. Aspirin should not be given to a child who has chickenpox or other viral infections such as influenza because of the risk of Reye's syndrome. Options 1 and 2 are clinical manifestations of RF. Facial edema may be associated with the development of a cardiac complication.

A nurse is monitoring the daily weight of an infant with congestive heart failure (CHF). Which of the following alerts the nurse to suspect fluid accumulation and thus to the need to notify the registered nurse? 1. Bradypnea 2. Diaphoresis 3. Decreased blood pressure (BP) 4. A weight gain of 1 lb in 1 day

*4. A weight gain of 1 lb in 1 day* *Rationale:* A weight gain of 0.5 kg (1 lb) in 1 day is a result of the accumulation of fluid. The nurse should monitor the urine output, monitor for evidence of facial or peripheral edema, check the lung sounds, and report the weight gain. Tachypnea and an increased BP would occur with fluid accumulation. Diaphoresis is a sign of CHF, but it is not specific to fluid accumulation, and it usually occurs with exertional activities.

A child with rubeola (measles) is being admitted to the hospital. When preparing for the admission of the child, which precautions should be implemented? 1. Enteric 2. Contact 3. Protective 4. Airborne

*4. Airborne* *Rationale:* Rubeola is transmitted via airborne particles or direct contact with infectious droplets. Airborne precautions are required, and a mask is worn by those who come in contact with the child. Gowns and gloves are not indicated. Articles that are contaminated should be bagged and labeled. Options 1, 2, and 3 are not indicated for rubeola.

The parents of a newborn have been told that their child was born with bladder exstrophy, and the parents ask the nurse about this condition. The nurse bases the response on knowledge that this condition is: 1. A hereditary disorder that occurs in every other generation 2. Caused by the use of medications taken by the mother during pregnancy 3. A condition in which the urinary bladder is abnormally located in the pelvic cavity 4. An extrusion of the urinary bladder to the outside of the body through a defect in the lower abdominal wall

*4. An extrusion of the urinary bladder to the outside of the body through a defect in the lower abdominal wall* *Rationale:* Bladder exstrophy is a congenital anomaly that is characterized by the extrusion of the urinary bladder to the outside of the body through a defect in the lower abdominal wall. The cause is unknown, and there is a higher incidence among males. Options 1, 2, and 3 are not characteristics of this disorder.

A corticosteroid cream is prescribed by a health care provider for a child with atopic dermatitis (eczema). The nurse teaches the mother how to apply the cream. Which instruction is appropriate? 1. Apply the cream over the entire body. 2. Apply a thick layer of cream to affected areas only. 3. Avoid cleansing the area before applying the cream. 4. Apply a thin layer of cream, and rub it into the area thoroughly.

*4. Apply a thin layer of cream, and rub it into the area thoroughly.* *Rationale:* Corticosteroid cream should be applied sparingly and rubbed into the area thoroughly. The affected area should be cleansed gently before application. The cream should not be applied over extensive areas. Systemic absorption is more likely to occur with extensive application.

Permethrin 5% (Elimite) is prescribed for a 4-year-old child with a diagnosis of scabies. The nurse instructs the mother regarding the use of this treatment. Which instruction is appropriate? 1. Apply the lotion and leave it on for 4 hours. 2. Apply the lotion to the hair, the face, and the entire body. 3. The child should wear no clothing while the lotion is in place. 4. Apply the lotion to cool, dry skin at least half an hour after bathing.

*4. Apply the lotion to cool, dry skin at least half an hour after bathing.* *Rationale:* Permethrin is applied from the neck downward, with care taken to ensure that the soles of the feet, the areas behind the ears, and the areas under the toenails and fingernails are covered. The lotion should be kept on for 8 to 14 hours, and then the child should be given a bath. The lotion should be applied at least 30 minutes after bathing, and it should be applied only to cool, dry skin. The child should be clothed during treatment.

An infant is seen in a clinic and is diagnosed with unilateral hip dysplasia. Which finding is associated with this condition? 1. Limited range of motion in the unaffected hip 2. An apparent short femur on the unaffected side 3. Adduction of the affected hip when placed supine with the knees and hips flexed 4. Asymmetry of the gluteal skin folds when the infant is placed prone and the legs are extended against the examining table

*4. Asymmetry of the gluteal skin folds when the infant is placed prone and the legs are extended against the examining table* *Rationale:* Asymmetry of the gluteal folds when the infant is placed prone would be a finding in hip dysplasia in infants beyond the newborn period. Options 1, 2, and 3 are inaccurate assessment findings in this disorder.

A nurse is caring for a child with a diagnosis of intussusception. Which of the following symptoms would the nurse expect to note in this child? 1. Watery diarrhea 2. Ribbon-like stools 3. Profuse projectile vomiting 4. Blood and mucus in the stools

*4. Blood and mucus in the stools* *Rationale:* The child with intussusception classically presents with severe abdominal pain that is crampy and intermittent and that causes the child to draw in his or her knees to the chest. Vomiting may be present, but it is not projectile. Bright red blood and mucus are passed through the rectum and commonly described as currant jelly-like stools. Ribbon-like stools are not a manifestation of this disorder.

A nurse is preparing to perform a neurovascular check for tissue perfusion in the child with an arm cast. Which of the following is the priority when performing this procedure? 1. Taking the temperature 2. Taking the blood pressure 3. Checking the apical heart rate 4. Checking the peripheral pulse in the affected arm

*4. Checking the peripheral pulse in the affected arm* *Rationale:* The neurovascular check for tissue perfusion is performed on the toes or fingers distal to an injury or cast and includes checking peripheral pulse, color, capillary refill time, warmth, motion, and sensation. Options 1, 2, and 3 may be components of care, but they are not the priority in this situation.

A nurse is caring for a child diagnosed with Down syndrome. In describing the disorder to the parents, the nurse bases the explanation on the fact that Down syndrome is a: 1. Condition characterized by above-average intellectual functioning with deficits in adaptive behavior 2. Condition characterized by average intellectual functioning and the absence of deficits in adaptive behavior 3. Condition characterized by subaverage intellectual functioning with the absence of deficits in adaptive behavior 4. Congenital condition that results in moderate to severe retardation and has been linked to an extra chromosome 21 (group G)

*4. Congenital condition that results in moderate to severe retardation and has been linked to an extra chromosome 21 (group G)* *Rationale:* Down syndrome is a form of mental retardation. It is a congenital condition that results in moderate to severe mental retardation. The syndrome has been linked to an extra group G chromosome, chromosome 21 (trisomy 21). Options 1, 2, and 3 are incorrect descriptions.

A nurse instructs the mother of a child with sickle cell disease regarding the precipitating factors related to pain crisis. Which of the following, if identified by the mother as a precipitating factor, indicates the need for further instructions? 1. Stress 2. Trauma 3. Infection 4. Fluid overload

*4. Fluid overload* *Rationale:* Pain crisis may be precipitated by infection, dehydration, hypoxia, trauma, or general stress. The mother of a child with sickle cell disease should encourage a fluid intake of 1.5 to 2 times the daily requirement to prevent dehydration.

A nurse reviews the record of a 3-week-old infant and notes that the health care provider has documented a diagnosis of suspected Hirschsprung's disease. The nurse understands that which of the following symptoms led the mother to seek health care for the infant? 1. Diarrhea 2. Projectile vomiting 3. The regurgitation of feedings 4. Foul-smelling, ribbon-like stools

*4. Foul-smelling, ribbon-like stools* *Rationale:* Chronic constipation that begins during the first month of life and that results in foul-smelling, ribbon-like or pellet-like stools is a clinical manifestation of Hirschsprung's disease. The delayed passage or absence of meconium stool during the neonatal period is a characteristic sign. Bowel obstruction (especially during the neonatal period), abdominal pain and distention, and failure to thrive are also clinical manifestations. Options 1, 2, and 3 are incorrect.

The primary goal to be included in the plan of care for a child who has cerebral palsy is to: 1. Eliminate the cause of the disease. 2. Improve muscle control and coordination. 3. Prevent the occurrence of emotional disturbances. 4. Maximize the child's assets and minimize the limitations.

*4. Maximize the child's assets and minimize the limitations.* *Rationale:* The goal of managing the child with cerebral palsy is early recognition and intervention to maximize the child's abilities. The cause of the disease cannot be eliminated. It is best to minimize emotional disturbances, if possible, but not to prevent them because it is healthy for the child to express emotions. Improvement of muscle control and coordination is a component of the plan, but the primary goal is to maximize the child's assets and minimize the limitations caused by the disease.

A child is diagnosed with infectious mononucleosis. The nurse provides home-care instructions to the parents about the care of the child. Which information given by the nurse is accurate? 1. Maintain the child on bedrest for 2 weeks. 2. Maintain respiratory precautions for 1 week. 3. Notify the health care provider if the child develops a fever. 4. Notify the HCP if the child develops abdominal or left shoulder pain.

*4. Notify the HCP if the child develops abdominal or left shoulder pain.* *Rationale:* The parents need to be instructed to notify the HCP if abdominal pain (especially in the left upper quadrant) or it left shoulder pain occurs, because this may indicate splenic rupture. Children with enlarged spleens are also instructed to avoid contact sports until the splenomegaly resolves. Bedrest is not necessary, and children usually self-limit their activity. Respiratory precautions are not required, although transmission can occur via direct intimate contact or contact with infected blood. Fever is treated with acetaminophen (Tylenol).

A nurse is collecting data about a child who has been admitted to the hospital with a diagnosis of seizures. The nurse checks for causes of the seizure activity by: 1. Testing the child's urine for specific gravity 2. Asking the child what happens during a seizure 3. Obtaining a family history of psychiatric illness 4. Obtaining a history regarding factors that may occur before the seizure activity

*4. Obtaining a history regarding factors that may occur before the seizure activity* *Rationale:* Fever and infections increase the body's metabolic rate. This can cause seizure activity among children who are less than 5-years-old. Dehydration and electrolyte imbalance can also contribute to the occurrence of a seizure. Falls can cause head injuries, which would increase intracranial pressure or cerebral edema. Some medications could cause seizures. Specific gravity would not be a reliable test, because it varies, depending on the existing condition. Psychiatric illness has no impact on seizure occurrence or cause. Children do not remember what happened during the seizure itself.

A nurse is reviewing the health record of a 14-year-old child who is suspected of having Hodgkin's disease. Which of the following is the primary characteristic of this disease? 1. Fever and malaise 2. Anorexia and weight loss 3. Painful, enlarged inguinal lymph nodes 4. Painless, firm, and movable lymph nodes in the cervical area

*4. Painless, firm, and movable lymph nodes in the cervical area* *Rationale:* Clinical manifestations specifically associated with Hodgkin's disease include painless, firm, and movable adenopathy in the cervical and supraclavicular areas. Hepatosplenomegaly is also noted. Although anorexia, weight loss, fever, and malaise are associated with Hodgkin's disease, these manifestations are not the primary characteristics and are seen with many disorders.

A nurse is assisting with data collection from an infant who has been diagnosed with hydrocephalus. If the infant's level of consciousness diminishes, a priority intervention is: 1. Taking the apical pulse 2. Taking the blood pressure 3. Testing the urine for protein 4. Palpating the anterior fontanel

*4. Palpating the anterior fontanel* *Rationale:* A full or bulging anterior fontanel indicates an increase in cerebrospinal fluid collection in the cerebral ventricle. Apical pulse and blood pressure changes and proteinuria are not specifically associated with increasing cerebrospinal fluid in the brain tissue in an infant.

The nurse assists with preparing a nursing care plan for a child who has Reye's syndrome. Which of the following is the priority nursing intervention? 1. Monitoring the output 2. Checking for hearing loss 3. Changing the body position every 2 hours 4. Providing a quiet atmosphere with dimmed lights

*4. Providing a quiet atmosphere with dimmed lights* *Rationale:* The major elements of care for a child who has Reye's syndrome are to maintain effective cerebral perfusion and to control intracranial pressure. Decreasing stimuli in the environment would decrease the stress on the cerebral tissue and the neuron responses. Cerebral edema is a progressive part of this disease process. Hearing loss and output are not affected. Changing the body position every 2 hours would not directly affect the cerebral edema and intracranial pressure. The child should be in a head-elevated position to decrease the progression of the cerebral edema and to promote the drainage of cerebrospinal fluid.

A nurse is assisting a health care provider (HCP) during the examination of an infant with hip dysplasia. The HCP performs the Ortolani maneuver. Which of the following best describes the action/purpose of the Ortolani maneuver? 1. Determining the extent of range of motion 2. Checking for asymmetry on the affected side 3. Pushing the unstable femoral head out of the acetabulum 4. Reducing the dislocated femoral head back into the acetabulum

*4. Reducing the dislocated femoral head back into the acetabulum* *Rationale:* With the Ortolani maneuver, the examiner reduces the dislocated femoral head back into the acetabulum. A positive Ortolani maneuver is a palpable clunk as the femoral head moves over the acetabular ring. Options 1 and 2 are data collection techniques for the identification of the clinical manifestations of hip dysplasia, but they do not describe the Ortolani maneuver. When performing the Barlow maneuver, the examiner pushes the unstable femoral head out of the acetabulum.

A child has a basilar skull fracture. Which of the following health care provider's prescriptions should the nurse question? 1. Restrict fluid intake. 2. Insert an indwelling urinary catheter. 3. Keep an intravenous (IV) line patent. 4. Suction via the nasotracheal route as needed.

*4. Suction via the nasotracheal route as needed.* *Rationale:* Nasotracheal suctioning is contraindicated in a child with a basilar skull fracture. Because of the location of the injury, the suction catheter may be introduced into the brain. Fluids are restricted to prevent fluid overload. The child may require a urinary catheter for the accurate monitoring of intake and output. An IV line is maintained to administer fluids or medications, if necessary.

A nurse is reviewing the health record of a child who has been recently diagnosed with glomerulonephritis. Which finding noted in the child's record is associated with the diagnosis of glomerulonephritis? 1. The child fell off a bike and onto the handlebars. 2. The child has had nausea and vomiting for the last 24 hours. 3. The child had urticaria and itching for 1 week before diagnosis. 4. The child had a streptococcal throat infection 2 weeks before diagnosis.

*4. The child had a streptococcal throat infection 2 weeks before diagnosis.* *Rationale:* Group A β-hemolytic streptococcal infection is a cause of glomerulonephritis. The child often becomes ill with streptococcal infection of the upper respiratory tract and then develops symptoms of acute poststreptococcal glomerulonephritis after an interval of 1 to 2 weeks. The data presented in options 1, 2, and 3 are unrelated to a diagnosis of glomerulonephritis.

A nurse is providing information to the family of a child about a synthetic cast that has been applied to the child for the treatment of a clubfoot. Which information should the nurse provide to the mother? 1. The synthetic cast takes 24 hours to dry. 2. The synthetic cast is heavier than a plaster cast. 3. The synthetic cast is stronger than a plaster cast. 4. The synthetic cast allows for greater mobility than a plaster cast.

*4. The synthetic cast allows for greater mobility than a plaster cast.* *Rationale:* Synthetic casts dry quickly (in less than 30 minutes) and are lighter than plaster casts. *Synthetic casts allow for greater mobility than a plaster cast*. However, synthetic casts are not as strong as plaster casts and are more expensive.

A health care provider has prescribed oxygen as needed for a 10-year-old child with congestive heart failure (CHF). In which situation would the nurse administer the oxygen to the child? 1. When the child is sleeping 2. When changing the child's diapers 3. When the mother is holding the child 4. When drawing blood for the measurement of electrolyte levels

*4. When drawing blood for the measurement of electrolyte levels* *Rationale:* Oxygen administration may be prescribed for the infant with CHF for stressful periods, especially during bouts of crying or invasive procedures. Drawing blood is an invasive procedure that would likely cause the child to cry.

The health care provider prescribes laboratory studies for an infant of a woman positive for human immunodeficiency virus (HIV) to determine the presence of HIV antigen in the infant. The nurse anticipates that which laboratory study will be prescribed for the infant? 1. Chest x-ray 2. Western blot 3. CD4+ cell count 4. p24 antigen assay

*4. p24 antigen assay* *Rationale:* The detection of HIV in infants is confirmed by a p24 antigen assay, virus culture of HIV, or polymerase chain reaction. A chest x-ray evaluates the presence of other manifestations of HIV infection, such as pneumonia. A Western blot test confirms the presence of HIV antibodies. The CD4+ cell count indicates how well the immune system is working.

A nurse is preparing to perform nasotracheal suctioning on a client. The nurse places the client's bed in which position to effectively perform this procedure? Refer to figure.

1

A nurse is providing post-procedure teaching after a client underwent an upper gastrointestinal (GI) series. The nurse reminds the client that the stools will remain white for approximately

1-2 days

A client with a burn injury is scheduled for a heterograft. The nurse is preparing the client for the skin grafting, and the client asks the nurse what "heterograft" means. The appropriate response to the client is: 1. "It is skin from another species." 2. "It is skin from a cadaver." 3. "It is skin from the burned client." 4. "It is skin from a skin bank."

1. "It is skin from another species."

An adult client trapped in a burning house suffered burns to the back of the head, the upper half of the posterior trunk, and the back of both arms. Using the rule of nines, the nurse determines the extent of the burn injury to be which of the following? 1. 22.5% 2. 31.5% 3. 36% 4. 40.5%

1. 22.5%

Which of the following individuals is least likely at risk for the development of psoriasis? 1. A 32-year-old African American 2. A client with a systemic illness 3. An individual who has experienced a significant amount of emotional distress 4. A woman experiencing menopause

1. A 32-year-old African American

A nurse is reviewing the health care record of a client with a lesion that has been diagnosed as malignant melanoma. The nurse would expect which characteristic of this type of lesion to be documented in the client's record? 1. An irregularly shaped lesion 2. A small papule with a dry, rough scale 3. A firm nodular lesion topped with a crust 4. A pearly papule with a central crater and a waxy border

1. An irregularly shaped lesion

A nurse is preparing a client for skin grafting and notes that the health care provider has documented that the client is scheduled for a heterograft. The nurse understands that a heterograft used for the burn client is skin from: 1. Another species 2. A cadaver 3. The burned client 4. A skin bank

1. Another species

A client with a burn injury begins to cry and states to the nurse, "I don't want anyone seeing me. I look awful." The nurse determines that the client is experiencing a problem with which of the following? 1. Appearance 2. Fear 3. Self-Esteem 4. Ability to keep a job

1. Appearance

An older client is transferred to the nursing unit following a graft to a stage 4 pressure ulcer. Which combination of dietary items would the nurse encourage the client to eat to promote wound healing? 1. Chicken breast, broccoli, strawberries, milk 2. Salad, watermelon, tea 3. Baked potatoes, Jell-O, water 4. Spaghetti, bread, cola

1. Chicken breast, broccoli, strawberries, milk

A nurse is assigned to care for a client with herpes zoster. Which of the following characteristics would the nurse expect to note when assessing the lesions of this infection? 1. Clustered skin vesicles 2. A generalized body rash 3. Small blue-white spots with red bases 4. A fiery red edematous rash on the cheeks

1. Clustered skin vesicles

A nurse in a health care provider's office has scheduled a client with dermatitis to be seen in 1 week for a patch test. The nurse would tell the client to do which of the following before the procedure? 1. Discontinue the prescribed antihistamine 2 days before the test. 2. Refrain from eating solid food on the day of the test. 3. Do not eat or drink anything on the morning of the test. 4. Shower using povidone-iodine on the morning of the test.

1. Discontinue the prescribed antihistamine 2 days before the test.

A client has sustained partial-thickness burns on the posterior thorax and legs. The nurse who is assisting in caring for the client would monitor for which of the following during the first 24 hours after the burn injury? 1. Elevated hematocrit levels 2. Increased urinary output 3. Decreased heart rate 4. Decreased blood pressure

1. Elevated hematocrit levels Rationale: The emergent phase begins at the time of injury and ends with the restoration of capillary permeability, usually at 48 to 72 hours following the injury. During the emergent phase, the hematocrit rises above normal because of hemoconcentration from the large fluid shifts. Hematocrit levels of 50% to 55% are expected during the first 24 hours after injury but generally return to normal by 36 hours after injury. Initially, blood is shunted away from the kidneys, reducing renal perfusion and glomerular filtration. This leads to a decreased urine output. Pulse rates are typically higher than normal; the blood pressure is normal or slightly elevated unless hypovolemia is severe.

A nurse prepares to assist in instructing a client about prevention of Lyme disease. Which of the following would the nurse include in the instructions? 1. It is caused by a tick carried by deer. 2. It is caused by contamination from cat feces. 3. It is contagious by skin contact with an infected individual. 4. It is caused by the inhalation of spores from bird droppings.

1. It is caused by a tick carried by deer.

A client with jaundice is complaining of pruritus. Which of the following strategies should the nurse institute to help control the problem and prevent injury? 1. Pat the skin dry after bathing. 2. Maintain a warm environment. 3. Bathe the client with hot water only. 4. Avoid application of emollient creams.

1. Pat the skin dry after bathing.

A nurse reviews a client's chart and notes that the health care provider has documented a diagnosis of paronychia. Based on this diagnosis, which of the following would the nurse expect to note during data collection? 1. Red, shiny skin around the nail bed 2. White, taut skin in the popliteal area 3. White, silvery patches on the elbows 4. Swelling of the skin near the parotid gland

1. Red, shiny skin around the nail bed

A nurse is discussing skin biopsy with a client scheduled for the procedure. The nurse tells the client to expect how much discomfort during the procedure? 1. Slight because the local anesthetic may burn or sting 2. None because it is done under general anesthesia 3. None because it is painless 4. Somewhat painful but easily managed with opioids afterward

1. Slight because the local anesthetic may burn or sting

Which of the following would be the anticipated therapeutic outcome of an escharotomy procedure performed for a circumferential arm burn? 1. The return of distal pulses 2. Decreasing edema formation 3. Brisk bleeding from the injury site 4. The formation of granulation tissue

1. The return of distal pulses

The health education nurse provides instructions to a group of clients regarding measures that will assist in preventing skin cancer. Which instruction(s) should the nurse reinforce to the client? Select all that apply. 1. Use sunscreen when participating in outdoor activities. 2. Wear a hat, opaque clothing, and sunglasses when in the sun. 3. Avoid sun exposure in the late afternoon and early evening hours. 4. Examine your body monthly for any lesions that may be suspicious. 5. Sunscreen should be applied every 8 hours.

1. Use sunscreen when participating in outdoor activities. 2. Wear a hat, opaque clothing, and sunglasses when in the sun. 4. Examine your body monthly for any lesions that may be suspicious.

A nurse provides instructions to the mother of a child diagnosed with pediculosis (head lice). Permethrin (Nix) has been prescribed. Which statement by the mother regarding the use of the medication indicates a need for further instructions?

1. "After rinsing out the medication, I need to avoid washing my child's hair for 24 hours." 2. "I need to shampoo my child's hair, apply the medication, leave it on for 10 minutes, and then rinse it out." *3. "I need to shampoo my child's hair, apply the medication, and leave the medication on for 24 hours."* 4. "I need to purchase the medication from the pharmacy." *Rationale:* Permethrin is an over-the-counter antilice product that kills both lice and eggs with one application and has residual activity for 10 days. It is applied to the hair after shampooing and left for 10 minutes before rinsing out. The hair should not be shampooed for 24 hours after the rinsing treatment.

A client with diabetes mellitus who takes insulin is seen in the health care clinic. The client tells the nurse that after giving the injection, the insulin seems to leak through the skin. The nurse can appropriately determine the problem by asking the client which of the following?

1. "Are you placing an air bubble in the syringe before injection?" 2. "Are you using a 1-inch needle to give the injection?" 3. "Are you aspirating before you inject the insulin?" *4. "Are you rotating the injection site?"* *rationale* The client should be instructed that insulin injection sites should be rotated within one anatomical area before moving to another. This rotation process promotes uniform absorption of insulin and reduces the chances of irritation. Options 1, 2, and 3 are not associated with the condition (skin leakage of insulin) presented in the question.

A pediatric nursing instructor asks a nursing student to describe the cause of the clinical manifestations that occur in sickle cell disease. Which is the correct response by the nursing student?

1. "Bone marrow depression occurs because of the development of sickled cells." 2. "Sickled cells increase the blood flow through the body and cause a great deal of pain." 3. "The sickled cells mix with the unsickled cells and cause the immune system to become depressed." *4. "Sickled cells are unable to flow easily through the microvasculature, and their clumping obstructs blood flow."* *Rationale:* All the clinical manifestations of sickle cell disease are a result of the sickled cells being unable to flow easily through the microvasculature, and their clumping obstructs blood flow. With reoxygenation, most of the sickled red blood cells resume their normal shape. Options 1, 2, and 3 are inaccurate.

A nurse is providing instructions to the parents of an infant with a ventriculoperitoneal shunt. The nurse plans to include which of the following instructions?

1. "Call the health care provider if the infant is fussy." 2. "Position the infant on the side of the shunt when the infant is put to bed." 3. "Expect an increased urine output from the shunt." *4. "Call the health care provider if the infant has a high-pitched cry."* *Rationale:* If the shunt is broken or malfunctioning, the fluid from the ventricular part of the brain will not be diverted to the peritoneal cavity. The cerebrospinal fluid will build up in the cranial area. The result is increased intracranial pressure, which then causes a high-pitched cry in the infant. The infant should not be positioned on the side of the shunt because this will cause pressure on the shunt and skin breakdown. This type of shunt affects the gastrointestinal system, not the genitourinary system, and an increased urinary output is not expected. Option 1 is a concern only if other signs indicative of a complication are occurring.

A nursing instructor asks a student to describe the pathophysiology that occurs in Cushing's disease. Which statement by the student indicates an accurate understanding of this disorder?

1. "Cushing's disease is characterized by an oversecretion of insulin." *2. "Cushing's disease is characterized by an oversecretion of glucocorticoid hormones."* 3. "Cushing's disease is characterized by an undersecretion of corticotropic hormones." 4. "Cushing's disease is characterized by an undersecretion of glucocorticoid hormones." *rationale* Cushing's syndrome is characterized by an oversecretion of glucocorticoid hormones. Addison's disease is characterized by the failure of the adrenal cortex to produce and secrete adrenocortical hormones. Options 1 and 4 are inaccurate regarding Cushing's syndrome.

A nurse is collecting data on a child recently diagnosed with glomerulonephritis. Which of the following questions to the mother would elicit information about the cause of this disease?

1. "Did your child sustain any injuries to the kidney area?" *2. "Did your child recently complain of a sore throat?"* 3. "Has your child had any diarrhea?" 4. "Have you noticed any rashes on your child?" *Rationale:* Group A beta hemolytic streptococcal infection is a cause of glomerulonephritis. Often the child becomes ill with streptococcal infection of the upper respiratory tract and then develops symptoms of acute poststreptococcal glomerulonephritis after an interval of 1 to 2 weeks. The questions to the mother in options 1, 3, and 4 are unrelated to a diagnosis of glomerulonephritis.

A nurse is collecting data on a client with hyperparathyroidism. Which of the following questions would elicit the accurate information about this condition from the client?

1. "Do you have tremors in your hands?" *2. "Are you experiencing pain in your joints?"* 3. "Have you had problems with diarrhea lately?" 4. "Do you notice swelling in your legs at night?" *rationale* Hyperparathyroidism causes an oversecretion of parathyroid hormone (PTH), which causes excessive osteoblast growth and activity within the bones. When bone reabsorption is increased, calcium is released from the bones into the blood, causing hypercalcemia. The bones suffer demineralization as a result of calcium loss, leading to bone and joint pain, and pathological fractures.

A client with Cushing's syndrome verbalizes concern to the nurse regarding the appearance of the buffalo hump that has developed. Which response by the nurse is appropriate?

1. "Don't be concerned, this problem can be covered with clothing." 2. "This is permanent, but looks are deceiving and not that important." *3. "Usually, these physical changes slowly improve following treatment."* 4. "Try not to worry about it. There are other things to be concerned about." *rationale* The client with Cushing's syndrome should be reassured that most physical changes resolve with treatment. Options 1, 2, and 4 are not therapeutic responses.

A nurse is providing information to the mother of a child with nephrotic syndrome regarding the edematous appearance of the child. Which of the following statements should the nurse make to the mother?

1. "Dress the child in loose-fitting clothing to hide the extra weight." 2. "Children always look a little bit fat, so don't be concerned." *3. "The fluid retention should be controlled by medication and diet."* 4. "The child will always have this appearance, and preparing the child for the body image change is important." *Rationale:* Most children experience remission with treatment and corticosteroids. Diuretics also may be a component of the treatment plan, and a restricted sodium diet is recommended. It is important to give the parent information in a matter-of-fact manner and address the issue that is the parent's concern. Options 1, 2, and 4 are inaccurate and inappropriate statements to the mother.

A nurse is collecting data on a child with a diagnosis of rheumatic fever. Which of the following questions would the nurse initially ask the mother of the child?

1. "Has the child had any diarrhea?" 2. "Has the child been vomiting?" 3. "Does the child complain of chest pain?" *4. "Has the child complained of a sore throat within the past few months?"* *Rationale:* Rheumatic fever characteristically presents 2 to 6 weeks following an untreated or partially treated group A β-hemolytic streptococcal infection of the upper respiratory tract. Initially, the nurse determines whether any family members have had a sore throat or unexplained fever within the past 2 months. Although options 1, 2, and 3 may be asked during data collection, they would not be the initial concerns for a child with rheumatic fever.

A nurse is providing home care instructions to the mother of a child diagnosed with pneumonia. Which statement by the mother indicates the need for further instructions?

1. "I can administer acetaminophen [Tylenol] for a fever." *2. "I can use a warm mist humidifier to keep the secretions loose."* 3. "I should administer the antibiotics until the prescribed amount is completed." 4. "I can give my child warm liquids to loosen secretions." *Rationale:* A cool mist humidifier rather than a warm mist should be used for the child with pneumonia. In addition, vaporizers that produce steam pose a danger of burns. Options 1, 3, and 4 are appropriate home care instructions regarding care of the child with pneumonia.

A nurse is providing dietary instructions to the mother of a child with celiac disease. Which statement by the mother indicates a need for further instructions?

1. "I can give my child rice." *2. "I am so pleased that I won't have to eliminate oatmeal from my child's diet."* 3. "My child loves corn. I will be sure to include corn in the diet." 4. "I will be sure to give my child vitamin supplements every day." *Rationale:* Dietary management is the mainstay of treatment for the child with celiac disease. All wheat, rye, barley, and oats should be eliminated from the diet and replaced with corn and rice. Vitamin supplements, especially fat-soluble vitamins and folate, may be needed in the early period of treatment to correct deficiencies.

A nurse has reinforced instructions to the client with hyperparathyroidism regarding home care measures related to exercise. Which statement by the client indicates a need for further instruction? *Select all that apply.*

1. "I enjoy exercising but I need to be careful." 2. "I need to pace my activities throughout the day." *3. "I need to limit playing football to only the weekends."* 4. "I should gauge my activity level by my energy level." *5. "I should exercise in the evening to encourage a good sleep pattern."* *rationale* The client should be instructed to avoid high-impact activity or contact sports such as football. Exercising late in the evening may interfere with restful sleep. The client with hyperparathyroidism should pace activities throughout the day and plan for periods of uninterrupted rest. The client should plan for at least 30 minutes of walking each day to support calcium movement into the bones. The client should be instructed to use energy level as a guide to activity.

The mother of an infant newly diagnosed with cystic fibrosis is being taught proper nutritional needs for the infant. The nurse determines that the mother understands nutritional needs when the mother replies:

1. "I know that my infant needs to drink predigested formula until she has her stool pattern developed." 2. "When I begin feeding my infant cereal, I will make sure to warm the cereal and administer the pancreatic enzyme mixed in." 3. "I will make sure that I give my infant fat-free milk as a supplement to her predigested formula, because she is not able to digest fat." *4. "I know I need to monitor my infant's stools and if there are more than four stools a day, I will increase the pancreatic enzyme."* *Rationale:* Cystic fibrosis requires a high-calorie, high-protein diet with pancreatic enzyme replacement therapy. The infant needs to remain on the predigested formula until 1 year of age, when formula can be discontinued and then fat-free milk consumed. The pancreatic enzyme should not be mixed with warmed foods because this inactivates the enzyme. Stools must be monitored, and pancreatic enzymes are administered based on the stool pattern.

An adolescent is seen in the health care clinic with complaints of chronic fatigue. On physical examination, the nurse notes that the adolescent has swollen lymph nodes. A laboratory test is performed, and the results indicate the presence of Epstein-Barr virus (mononucleosis). The nurse calls the mother of the adolescent to inform the mother of the test results and provides instructions regarding the care of the adolescent. Which statement by the mother indicates an understanding of the care measures?

1. "I need to keep my child on bedrest for 3 weeks." 2. "I will call the health care provider if my child is still feeling tired in 1 week." *3. "I need to call the health care provider if my child complains of abdominal pain or left shoulder pain."* 4. "I need to isolate my child so that the respiratory infection is not spread to others." *Rationale:* The mother needs to be instructed to notify the health care provider if abdominal pain, especially in the left upper quadrant, or left shoulder pain occurs because this may indicate splenic rupture. Children with enlarged spleens are also instructed to avoid contact sports until splenomegaly resolves. Bedrest is not necessary, and children usually self-limit their activity. No isolation precautions are required, although transmission can occur via saliva, close intimate contact, or contact with infected blood. The child may still feel tired in 1 week as a result of the virus.

A nurse provides instructions to the parents of an infant with hip dysplasia regarding care of the Pavlik harness. Which statement, if made by one of the parents, indicates an understanding of the use of the harness?

1. "I need to remove the harness to feed my infant." 2. "I need to remove the harness to change the diaper." 3. "My infant needs to remain in the harness at all times." *4. "I can remove the harness to bathe my infant."* *Rationale:* The harness should be worn 23 hours a day and should be removed only to check the skin and for bathing. The hips and buttocks should be supported carefully when the infant is out of the harness. The harness does not need to be removed for diaper changes or feedings. Option 3 is incorrect.

A client has been prescribed valproic acid (Depakene) for the treatment of generalized seizures, and the nurse teaches the child about the potential side effects of the medication. Which statement by the client would indicate that further teaching is required?

1. "I need to take the pills whole and not crush them." 2. "I need to take the medication with food so that I won't get an upset stomach." *3. "I am so glad that I won't lose any of my hair. I was worried what my friends would think."* 4. "I know that I might gain weight with the medication so I need to be careful to not eat a lot of sweets and to eat more fruits and vegetables." *Rationale:* Side effects of valproic acid include nausea and vomiting, tremors, weight gain, and hair loss. It is important to take the medication whole and not crush or cut the medication.

The nurse is reinforcing instructions to an adolescent with type 1 diabetes mellitus regarding insulin administration and rotation sites. Which statement, if made by the adolescent, would indicate an understanding of the instructions?

1. "I need to use a different site for each insulin injection." 2. "I should use only my stomach and my thighs for injections." 3. "I need to use the same site for 1 month before rotating to another site." *4. "I need to use one major site for the morning injection and another major site for the evening injection for 2 to 3 weeks before changing major sites."* *Rationale:* To help decrease variations in absorption from day to day, the child should use one location within a major site for the morning injection. The child should then rotate to another site for the evening injection, and a third site for the bedtime injection. The child should follow this pattern for a period of 2 to 3 weeks before changing major sites.

A nurse provides home care instructions to the parents of an infant following surgical intervention for imperforate anus and tells the parents about the procedure for anal dilation. Which statement by the parents indicates the need for further instructions?

1. "I need to use only dilators supplied by the health care provider." 2. "I need to use a water-soluble lubricant." 3. "I will insert the dilator no more than 1 to 2 cm into the anus." *4. "I will insert a glycerin suppository before the dilation."* *Rationale:* Following this surgery, anal dilation at home by the parents is necessary to achieve and maintain bowel patency. Inserting a glycerin suppository before dilation is not a component of this procedure. Options 1, 2, and 3 are accurate instructions and will prevent damage to the rectal mucosa.

A nurse has reinforced instructions about measuring blood glucose levels to a client newly diagnosed with diabetes mellitus. The nurse determines that the client understands the procedure when making which most accurate statement?

1. "I should check my blood glucose level before eating a big meal." *2. "I should check my blood glucose level before eating each meal, regardless of how much I eat."* 3. "I should check my blood glucose level 2 hours after each meal." 4. "I should check my blood glucose level once a day." *rationale* The most effective and accurate measure for testing blood glucose is to test the level before each meal regardless of the amount of food to be eaten. The client should also check the blood glucose level at bedtime. Checking the level after the meal will provide an inaccurate assessment of diabetic control. Checking the level once daily will not provide enough data related to controlling the diabetes mellitus.

A client with type 1 diabetes mellitus calls the nurse to report recurrent episodes of hypoglycemia. Which statement by the client indicates a correct understanding of NPH insulin and exercise?

1. "I should not exercise after lunch." 2. "I should not exercise after breakfast." 3. "I should not exercise in the late evening." *4. "I should not exercise in the late afternoon."* *rationale* A hypoglycemic reaction may occur in response to increased exercise. Clients should avoid exercise during the peak time of insulin. NPH insulin peaks at 12 to 14 hours; therefore, late afternoon exercise would occur during the peak of the medication.

A nurse is collecting data from a client with type 2 diabetes mellitus. Which statement by the client indicates an understanding of the medication regimen?

1. "I should take my metformin (Glucophage) only if my blood glucose is elevated." 2. "By taking these medications, I am able to eat more." 3. "When I become ill, I need to increase the number of pills I take." *4. "The medication that I am taking helps release the insulin I already make."* *rationale* Clients with type 2 diabetes mellitus have decreased or impaired insulin secretion. Oral hypoglycemic agents are given to these clients to facilitate glucose use and need to be taken on a regular schedule as prescribed. To maintain normal blood glucose levels throughout the day, oral hypoglycemic agents such as metformin are not taken on an as-needed basis depending on the blood glucose levels. Insulin injections may be given during times of stress-induced hyperglycemia. Oral insulin is not available or effective because of the breakdown of the insulin by digestion.

A client with pheochromocytoma is scheduled for surgery and says to the nurse, "I'm not sure that surgery is the best thing to do." What response by the nurse is appropriate?

1. "I think you are making the right decision to have the surgery." 2. "You are very ill. Your health care provider has made the correct decision." 3. "There is no reason to worry. Your health care provider is a wonderful surgeon." *4. "You have concerns about the surgical treatment for your condition."* *rationale* Paraphrasing is restating the client's message in the nurse's own words. Option 4 addresses the therapeutic communication technique of paraphrasing. The client is reaching out for understanding. In option 3, the nurse is offering a false reassurance, and this type of response will block communication. Option 2 also represents a communication block because it reflects a lack of the client's right to an opinion. In option 1, the nurse is expressing approval, which can be harmful to a nurse-client relationship.

When instructing the caregiver of a child about cast care, the nurse anticipates the need for further teaching when the caregiver states:

1. "I will encourage my child to avoid standing for too long." 2. "I will instruct my child to not put anything inside the cast." *3. "I will allow my child to put cotton balls inside the cast to relieve pressure."* 4. "I will encourage my child to keep the injured extremity elevated while resting." *Rationale:* Cast care includes keeping the casted extremity elevated on pillows or similar support for the first day, or as directed by the health professional; elevating a lower limb when sitting and avoid standing for too long; encouraging frequent rest for a few days; keeping the injured extremity elevated while resting; and not allowing the child to put anything inside the cast.

A nurse provides discharge instructions to the mother of a child following a myringotomy with insertion of tympanostomy tubes. Which statement by the mother indicates a need for further instructions?

1. "I will not allow my child to swim in lake water." 2. "I will not allow my child to swim in deep water." 3. "I will put earplugs in my child's ears during bathing." *4. "I need to be sure my child uses soft tissues to blow his nose."* *Rationale:* Parents need to be instructed that the child should not blow the nose for 7 to 10 days. Bath and lake water are potential sources of bacterial contamination. Diving and swimming deeply under water are prohibited. The child's ears need to be kept dry. Options 1, 2, and 3 are appropriate statements.

The school nurse is visiting a kindergarten classroom to teach the students the importance of handwashing. During the teaching session the nurse notes that one girl is scratching her head. On inspection the nurse determines that the child has pediculosis capitis. When teaching the mother about care of this condition, which statement by the mother indicates that she needs further teaching regarding this condition?

1. "I will put all the stuffed animals in a sealed plastic bag for 14 days." *2. "I will call a carpet cleaning service to clean all my carpets in the house."* 3. "My two daughters should not share their hairbrushes or hair ribbons." 4. "I will machine wash all the washable clothing, towels, and bed linens in hot water." *Rationale:* Teaching about measures to prevent the spread of pediculosis capitis includes washing items in hot water, vacuuming carpets, discouraging sharing of personal items, and sealing items in plastic bags that cannot be vacuumed. Option 2 is too costly for many families and is unnecessary. Option 2 indicates the mother does not understand the measures that will prevent the spread of the parasite.

A health care provider prescribes a 24-hour urine collection for vanillylmandelic acid (VMA). The nurse instructs the client in the procedure for the collection of the urine. Which statement by the client would indicate a need for further instruction?

1. "I will start the collection in 2 days. I cannot eat or drink any tea, chocolate, vanilla, or fruit until the test is completed." 2. "When I start the collection, I will urinate and discard that specimen." 3. "I will pour the urine into the collection bottle each time I urinate and refrigerate the urine." *4. "I can take any medications if I need to before the collection."* *rationale* Because a 24-hour urine collection is a timed quantitative determination, it is essential that the client start the test with an empty bladder. Therefore the client is instructed to void and discard the first urine and note the time and start the test. The 24-hour urine specimen collection bottle must be kept on ice or refrigerated. In a VMA collection, the client is instructed to avoid tea, chocolate, vanilla, and all fruits for 2 days before urine collection begins. Also clients are reminded not to take certain medications for 2 to 3 days before the test.

When the nurse is teaching a client who has been newly diagnosed with type 1 diabetes mellitus, which statement by the client would indicate that teaching has been effective?

1. "I will stop taking my insulin if I'm too sick to eat." 2. "I will decrease my insulin dose during times of illness." 3. "I will adjust my insulin dose according to the level of glucose in my urine." *4. "I will notify my health care provider if my blood glucose level is greater than 250 mg/dL."* *rationale* During illness, the client should monitor the blood glucose level, and he or she should notify the health care provider (HCP) if the level is greater than 250 mg/dL. Insulin should never be stopped. In fact, insulin may need to be increased during times of illness. Doses should not be adjusted without the HCP's advice.

A nurse has reinforced prior teaching of a school-age child who was given a brace to wear for the treatment of scoliosis. The nurse interprets that the client has not fully understood the information presented if the child makes which statement?

1. "I will wear my brace under my clothes." 2. "I will do back exercises at least five times a week." 3. "I will wear my brace whenever I am not sleeping." *4. "This brace will correct my curve."* *Rationale:* Bracing can halt the progression of most curvatures, although it is not curative for scoliosis. The statements in options 1, 2, and 3 represent correct understanding on the part of the child.

A nurse provides dietary instructions to a client with diabetes mellitus regarding the prescribed diabetic diet. Which statement, if made by the client, indicates the need for further teaching?

1. "I'll eat a balanced meal plan." 2. "I need to drink diet soft drinks." *3. "I need to buy special dietetic foods."* 4. "I will snack on fruit instead of cake." *rationale* It is important to emphasize to the client and family that they are not eating a diabetic diet but rather following a balanced meal plan. Adherence to nutrition principles is an important component of diabetic management, and an individualized meal plan should be developed for the client. It is not necessary for the client to purchase special dietetic foods.

A nurse provides home care instructions to a client with sickle cell anemia. Which statement by the client indicates a need for further instruction?

1. "I'm going to take a painting class." 2. "I've learned to knit and sew my own clothes." *3. "When I'm feeling better, I'm returning to the soccer team."* 4. "I'm using a schedule to maintain my increased fluid intake." *Rationale:* Clients with sickle cell anemia are advised to avoid strenuous activities. Quiet activities as tolerated are recommended when the client is feeling well. Increasing fluid intake is encouraged to assist in preventing sickle cell crisis.

A 6-year-old child has just been diagnosed with localized Hodgkin's disease, and chemotherapy is planned to begin immediately. The mother of the child asks the nurse about radiation therapy because it was not prescribed as a part of treatment. The appropriate response to the mother is:

1. "I'm not sure. I'll discuss it with the health care provider." *2. "The child is too young to have radiation therapy."* 3. "It's very costly, and chemotherapy works just as well." 4. "The health care provider would prefer that you discuss treatment options with the oncologist." *Rationale:* Radiotherapy is usually delayed until a child is 8 years of age whenever possible to prevent retardation of bone growth and soft tissue development. Low doses of radiation may also be recommended. Options 1 and 4 are nontherapeutic and place the mother's inquiry on hold. Option 3 is a blunt and uncaring response.

A nurse in an outpatient diabetes clinic is assisting in caring for a client on insulin pump therapy. Which statement by the client indicates that a knowledge deficit exists regarding insulin pump therapy?

1. "If my blood sugars are elevated, I can bolus myself with additional insulin as prescribed." 2. "I'll need to check my blood sugars before meals in case I need a pre-meal insulin bolus." *3. "Now that I have this pump, I don't have to worry about insulin reactions or ketoacidosis occurring again."* 4. "I still need to follow an appropriate diet and exercise plan even though I don't have to inject myself daily anymore." *rationale* All of the statements are correct in regard to insulin pump therapy, except the one that mentions insulin reactions and ketoacidosis. Hypoglycemic reactions can occur if there is an error in calculating the insulin dose or if the pump malfunctions. Ketoacidosis can occur if too little insulin is used or if there is an increase in metabolic need. The pump does not have a built-in blood glucose monitoring feedback system, so the client is subject to the usual complications associated with insulin administration without the use of a pump.

A nurse is reinforcing instructions to the mother of an 8-year-old child who had a tonsillectomy. The mother tells the nurse that the child loves tacos and asks when the child can safely eat one. The nurse makes which response to the mother?

1. "In 1 week". *2. "In 3 weeks".* 3. "Two days following surgery". 4. "When the health care provider says it's OK". *Rationale:* Rough, scratchy foods or spicy foods are to be avoided for 3 weeks. Citrus juices, which irritate the throat, need to be avoided for 10 days. Red liquids are avoided because they will give the appearance of blood if the child vomits. A full liquid diet is allowed on the second postoperative day, and soft foods are allowed as the child tolerates them.

A child is diagnosed with lactose intolerance. The child's mother asks the nurse about the disease. The appropriate nursing response is which of the following?

1. "It is the inability to fully digest the protein part of wheat, barley, rye, and oats." *2. "It is the inability to tolerate sugar found in dairy products."* 3. "It results from the absence of ganglion cells in the rectum." 4. "It results from increased bowel motility that leads to spasm and pain." *Rationale:* Lactose intolerance is the inability to tolerate lactose, the sugar found in dairy products. It results from absence or deficiency of lactase, an enzyme found in the secretions of the small intestine required for the digestion of lactose. Option 1 describes celiac disease. Option 3 describes Hirschsprung's disease. Option 4 describes irritable bowel syndrome.

A student nurse examines an Asian-American infant's eyes and notes that the infant's eyes are crossed. Which statement by the student to the nurse indicates an understanding of this finding?

1. "It probably is strabismus because the baby's mother has abused tranquilizers." *2. "It probably isn't strabismus but appears that way because of the child's ethnic background."* 3. "You will want to call the pediatrician immediately because this could lead to a detached retina." 4. "Strabismus isn't life threatening, but it requires surgery in the first two months to prevent the crossed eyes from being a lifelong condition." *Rationale:* Asian-American, American-Indian, and Alaskan-Native infants often have a pseudostrabismus because of a flattened nasal bridge. It needs to be distinguished from a true strabismus in the assessment. Options 1, 3, and 4 are inaccurate statements.

A nurse determines that an adolescent client with diabetes mellitus needs further information about glycosylated hemoglobin levels and their purpose if the client made which statement when told that a level will be drawn?

1. "Most of my recent blood glucose levels were close to 170 mg/dL, so this result will probably be a little high." 2. "Last time this test was taken the result was 13. I hope it will be lower this time." *3. "I already had a complete blood cell [CBC] count drawn an hour ago, so this test is not necessary."* 4. "I have followed my diet these past 3 months, so hopefully the test result will be OK." *Rationale:* Glycosylated hemoglobin reflects the average blood glucose levels during the previous 3 to 4 months. It assesses glucose control in the client with diabetes mellitus. Glucose molecules attach to the hemoglobin A molecules found in red blood cells (RBCs) and remain there for the lifetime of the RBCs, approximately 120 days.

A client with type 1 diabetes mellitus takes NPH insulin every morning and checks the blood glucose level four times per day. The client tells the nurse that yesterday the late afternoon blood glucose was 60 mg/dL and that she "felt funny." Which statement by the client would indicate an understanding of this occurrence?

1. "My blood glucose levels are running low because I'm tired." *2. "I forgot to take my usual afternoon snack yesterday."* 3. "I took less insulin this morning so I won't feel funny today." 4. "I don't know why I have to check my blood glucose four times a day. That seems too much." *rationale* Hypoglycemia is a blood glucose level of 60 mg/dL or less. The causes are multiple, but, in this case, omitting the afternoon snack is the cause. Fatigue and self-adjustment of dose are incorrect options. Recommended blood glucose testing for the client with type 1 diabetes mellitus is at least four times a day.

Which of the following statements made by the nursing student demonstrates an understanding of the hormone oxytocin?

1. "Production of oxytocin occurs in the ovaries." 2. "It is produced by the anterior pituitary gland." *3. "It causes contractions of the uterus during birth."* 4. "Release of oxytocin stimulates the pancreas to produce insulin." *rationale* Oxytocin is produced by the posterior pituitary, not the anterior pituitary gland, and stimulates the uterus to produce contractions during birth. The ovaries are the endocrine glands that produce estrogen and progesterone. The pancreas produces insulin and other enzymes that aid digestion. Oxytocin does not stimulate the pancreas to produce insulin.

The mother of a child who had a myringotomy with insertion of tympanostomy tubes calls the nurse and tells the nurse that the "tubes" fell out. The nurse makes which response to the mother?

1. "Replace the tubes immediately so that the created opening does not close." 2. "This is an emergency and requires immediate intervention. Bring the child to the emergency department." *3. "This is not an emergency. I will speak to the health care provider and call you right back."* 4. "Soak the tubes in alcohol for 1 hour before replacing them in the child's ears." *Rationale:* The size and appearance of the tympanostomy tubes should be described to the parents following surgery. They should be reassured that if the tubes fall out, it is not an emergency but that the health care provider should be notified. Options 1, 2, and 4 are incorrect.

Several children have contracted measles (rubeola) in a local school, and the nurse provides information to the mothers of the children about this communicable disease. Which statement by a mother indicates a need for further information?

1. "Respiratory symptoms such as a profuse runny nose, cough, and fever occur before the development of a rash." 2. "Small blue-white spots with a red base may appear in the mouth." 3. "The rash usually begins behind the ears at the hairline." *4. "The infectious period ranges from 10 days before symptoms start to 15 days after the rash appears."* *Rationale:* The infectious period for rubeola ranges from 1 to 2 days before the onset of symptoms to 4 days after the rash appears. Options 1, 2, and 3 are accurate descriptions of rubeola. Option 4 describes the infectious period for rubella (German measles).

Several children have contracted rubeola (measles) in a local school and the school nurse conducts a teaching session for the parents of the school-children. Which statement, if made by a mother, indicates a need for further teaching regarding this communicable disease?

1. "Small blue-white spots with a red base may appear in the mouth." 2. "The rash usually begins centrally and spreads downward to the limbs." *3. "The communicable period ranges from 10 days before the onset of symptoms to 15 days after the rash appears."* 4. "Respiratory symptoms such as a very runny nose, cough, and fever occur before the development of a rash." *Rationale:* The communicable period for rubeola ranges from 4 days before to 5 days after the rash appears, mainly during the prodromal (catarrhal) stage. Options 1, 2, and 4 are accurate descriptions of rubeola. The small blue-white spots found in this communicable disease are called Koplik spots. Option 3 describes the incubation period for rubella, not rubeola.

A nurse is reinforcing home care instructions to a client with a diagnosis of Cushing's syndrome. Which statement reflects a need for further client education?

1. "Taking my medications exactly as prescribed is essential." *2. "I need to read the labels on any over-the-counter medications I purchase."* 3. "My family needs to be familiar with the signs and symptoms of hypoadrenalism." 4. "I could experience the signs and symptoms of hyperadrenalism because of Cushing's." *rationale* The client with Cushing's syndrome should be instructed to take the medications exactly as prescribed. The nurse should emphasize the importance of continuing medications, consulting with the health care provider before purchasing any over-the-counter medications, and maintaining regular follow-up care. The nurse should also instruct the client in the signs and symptoms of both hypoadrenalism and hyperadrenalism.

A nursing student is caring for a child with increased intracranial pressure. On review of the chart, the student nurse notes that a transtentorial herniation has occurred. A nursing instructor asks the student about this type of herniation. Which statement by the student indicates a need for further research about this condition?

1. "The brain herniates downward and around the tentorium cerebelli." 2. "The herniation can be either unilateral or bilateral in nature." *3. "It involves only the anterior portions of the client's brain."* 4. "It can cause death if large amounts of tissue are involved." *Rationale:* Transtentorial herniation occurs when part of the brain herniates downward and around the tentorium cerebelli. It can be unilateral or bilateral and may involve anterior or posterior portions of the brain. If a large amount of tissue is involved, it can cause death because vital brain structures are compressed and become unable to perform their functions.

A child is brought to the emergency department, and a fracture of the left lower arm is suspected. The mother states that the child was rollerblading and attempted to break a fall with an outstretched arm. The child receives diagnostic x-rays, from which it has been determined that a fracture is present. A plaster of Paris cast is applied to the arm, and the nurse provides instructions to the mother regarding cast care at home. Which of the following instructions would the nurse provide to the mother?

1. "The cast should be dry in about 6 hours." 2. "The cast is water resistant, so the child is able to take a bath or a shower." 3. "The cast will not mold to the body and should heal the fracture in no time at all." *4. "The cast needs to be kept dry because, when wet, it will begin to disintegrate."* *Rationale:* Plaster of Paris is a heavier material than that used in a synthetic cast. It molds easily to the extremity and is less expensive than a synthetic cast. It takes about 24 hours to dry, and drying time could be longer depending on the size of the cast. Plaster of Paris is not water resistant, and when wet, it will begin to disintegrate.

A nursing student is asked to discuss juvenile idiopathic arthritis (JIA) at a clinical conference scheduled for the end of the clinical day. Which statement by the nursing student indicates the need to further research this disorder?

1. "The cause of this disease is unknown." 2. "JIA most often occurs by age of 10 years." *3. "This disease is twice as likely to occur in boys rather than girls."* 4. "Clinical manifestations include morning stiffness and painful, stiff, swollen joints." *Rationale:* JIA is twice as likely to occur in girls as in boys. The cause of JIA is unknown. JIA has two peak ages of onset: between 1 and 3 years of age and between 8 and 10 years of age. This autoimmune inflammatory disease causes painful inflammation of joints.

A hospitalized 2-year-old child with croup is receiving corticosteroid therapy. The mother asks the nurse why the health care provider did not prescribe antibiotics. The nurse makes which response to the mother?

1. "The child may be allergic to antibiotics." 2. "The child is too young to receive antibiotics." *3. "Antibiotics are not indicated unless a bacterial infection is present."* 4. "The child still has the maternal antibodies from birth and does not need antibiotics." *Rationale:* Antibiotics are not indicated in the treatment of croup unless a bacterial infection is present. Options 1, 2, and 4 are incorrect. In addition, the question does not include any supporting data to indicate that the child may be allergic to antibiotics.

A child is scheduled for a tonsillectomy in the day-stay surgical unit. On the day following surgery, the mother calls the surgical unit and expresses concern because the child has a very bad mouth odor. The nurse makes which response to the mother?

1. "The child probably has an infection." 2. "You need to contact the health care provider immediately." *3. "Bad mouth odor is normal and may be relieved by drinking more liquids."* 4. "Have the child gargle with mouthwash." *Rationale:* Bad mouth odor is normal following tonsillectomy and may be relieved by drinking more liquids. Options 1, 2, and 4 are incorrect. Additionally, mouthwash gargles will irritate the throat.

A nurse is providing instructions to a mother of a child with cystic fibrosis regarding the immunization schedule for the child. Which statement should the nurse make to the mother?

1. "The immunization schedule will need to be altered." 2. "The child should not receive any hepatitis vaccines." 3. "The child will receive all of the immunizations except for the polio series." *4. "The child will receive the recommended basic series of immunizations along with a yearly influenza vaccination."* *Rationale:* It is essential that children with cystic fibrosis be adequately protected from communicable diseases by immunization. It is recommended that in addition to the basic series of immunizations, children with cystic fibrosis also should receive yearly influenza vaccines.

A nurse is reinforcing instructions to the parents of a child with a hernia regarding measures that will promote reducing the hernia. The nurse determines that the parents understand these measures if they state which of the following?

1. "We will be sure to give our child a Fleet enema every day to prevent constipation." 2. "We will make sure that our child participates in physical activity every day." *3. "We will provide comfort measures to reduce any crying periods by our child."* 4. "We will encourage our child to cough every few hours on a daily basis." *Rationale:* A warm bath and comfort measures to reduce crying periods are all simple measures to promote reducing a hernia. Coughing and crying increase the strain on the hernia. Likewise, physical activities and enemas of any type would increase the strain on the hernia.

A client with myxedema has changes in intellectual function such as impaired memory, decreased attention span, and lethargy. The client's husband is upset and shares his concerns with the nurse. Which statement by the nurse is helpful to the client's husband?

1. "Would you like me to ask the health care provider for a prescription for a stimulant?" 2. "Give it time. I've seen dozens of clients with this problem that fully recover." 3. "I don't blame you for being frustrated, because the symptoms will only get worse." *4. "It's obvious that you are concerned about your wife's condition, but the symptoms may improve with continued therapy."* *rationale* Using therapeutic communication techniques, the nurse acknowledges the husband's concerns and conveys that the client's symptoms are common with myxedema. With thyroid hormone therapy, these symptoms should decrease, and cognitive function often returns to normal. Option 1 is not helpful, and it blocks further communication. Option 3 is pessimistic and untrue. Option 2 is not appropriate and offers false reassurance.

A nurse is caring for a client newly diagnosed with diabetes mellitus. The client asks the nurse whether eating at a restaurant will affect the diabetic control and whether this is allowed. Which nursing response is appropriate?

1. "You really should not eat in restaurants." 2. "If you plan to eat in a restaurant, you need to avoid carbohydrates." *3. "You should order a half-portion meal and have fresh fruit for dessert."* 4. "You should increase your daily dose of insulin by half on the day you plan to eat out." *rationale* Clients with diabetes mellitus are instructed to make adjustments in their total daily intake to plan for meals at restaurants or parties. Some useful strategies include ordering half portions, salads with dressing on the side, fresh fruit for dessert, and baked or steamed entrées. Clients are not instructed to avoid any food group or to increase their prescribed insulin dosage.

A mother of a 5-year-old child brings the child to the emergency department and tells the nurse that the child fell. A fracture is suspected and an x-ray is taken. The results indicate that the child has a comminuted fracture of the right humerus. The mother asks the nurse to describe this type of fracture, and the nurse draws a picture for the mother. Which picture identifies this type of a fracture? Refer to figure.

1. 1 *2. 2* 3. 3 4. 4 *Rationale:* When small fragments of bone are broken from the fracture shaft and lie in the surrounding tissues, the fracture is called comminuted. An open or compound fracture (option 1) is a fracture with an open wound from which the bone is or has protruded. In an oblique fracture (option 3), a diagonal line across the bone is noted. In a greenstick fracture (option 4), the bone is partially bent and partially broken.

A licensed practical nurse (LPN) is assisting a high school nurse in conducting a session with female adolescents regarding the menstrual cycle. The LPN tells the adolescents that the normal duration of the menstrual cycle is about:

1. 14 days *2. 28 days* 3. 30 days 4. 45 days *rationale* The normal duration of the menstrual cycle is about 28 days, although it may range from 20 to 45 days. The first day of the menstrual period is counted as day 1 of the woman's cycle. Options 1, 3, and 4 are incorrect.

A nurse has just supervised a newly diagnosed diabetes mellitus client self-inject NPH insulin at 7:30 ᴀᴍ. The nurse reviews the time frames for peak insulin action with the client, telling the client to be especially watchful for a hypoglycemic reaction between:

1. 7:30 ᴀᴍ and 9:30 ᴀᴍ *2. 1:30 ᴘᴍ and 7:30 ᴘᴍ* 3. 8:30 ᴘᴍ and 12:00 ᴀᴍ 4. 2:30 ᴀᴍ and 4:30 ᴀᴍ *rationale* NPH is an intermediate-acting insulin. It begins to work in 1 to 2 hours (onset), peaks in 6 to 12 hours, and lasts for 18 to 24 hours (duration). Hypoglycemic reactions most likely occur during peak time, which in this case is option 2.

A mother of a child with cystic fibrosis asks the nurse when the postural drainage should be performed. The mother states that the child eats meals at 8:00 AM, 12 noon, and at 6:00 PM The nurse tells the mother that the postural drainage should be performed at:

1. 8:00 AM, 2:00 PM, and 6:00 PM 2. 9:00 AM, 1:00 PM and 6:00 PM 3. 8:00 AM, 12:00 noon, and 6:00 PM *4. 10:00 AM, 2:00 PM and 8:00 PM* *Rationale:* Respiratory treatments should be performed at least 1 hour before meals or 2 hours after meals to prevent vomiting. In some children with cystic fibrosis, treatments are prescribed every 2 hours, particularly if infection is present. It is also important to perform these treatments before bedtime to clear airways and facilitate rest.

A nurse is preparing to feed a 1-year-old hospitalized child. The nurse prepares the amount of formula to be given to this child, knowing that generally a 1-year-old consumes approximately:

1. 90 mL per feeding 2. 100 mL per feeding *3. 175 mL per feeding* 4. 380 mL per feeding *Rationale:* A 1-year-old child consumes approximately 175 mL (6 ounces) of formula per feeding. Options 1, 2, and 4 are incorrect.

A nurse provides instructions to the parents of an infant with gastroesophageal reflux (GER) regarding proper positioning to manage reflux. The nurse tells the parents that the infant should be maintained in:

1. A 30-degree angle when supine 2. A 60-degree angle when prone *3. An upright angle 24 hours a day* 4. A 20-degree angle when side-lying *Rationale:* Proper positioning is an important component of reflux management. Ideally the goal is to maintain the infant in an upright angle 24 hours a day, at a 60-degree angle when supine, and at a 30-degree angle when prone. This position is maintained until the infant remains asymptomatic for 6 weeks.

A nurse in a newborn nursery is told that a newborn with spina bifida (myelomeningocele type) will be transported from the delivery room. The nurse is asked to prepare for the arrival of the newborn. The nurse places which of the following priority items at the newborn's bedside?

1. A blood pressure cuff 2. A rectal thermometer 3. A specific gravity urinometer *4. A bottle of sterile normal saline* *Rationale:* The newborn with spina bifida is at risk for infection before the closure of the gibbus. A sterile normal-saline dressing is placed over the gibbus to maintain moisture of the gibbus and its contents. This prevents tearing or breakdown of the skin integrity at the site. Blood pressure is difficult to assess during the newborn period and is not the best indicator of infection. Urine concentration is not well developed in the newborn stage of development. A thermometer will be needed to assess the temperature, but in this newborn the priority is to maintain sterile normal-saline dressings over the gibbus.

A nursing student is presenting a clinical conference and discusses the causative factors related to beta-thalassemia. Which group is at greatest risk of developing this disorder?

1. A child of Mexican descent *2. A child of Mediterranean descent* 3. A child whose intake of iron is extremely poor 4. A child breast-fed by a mother with chronic anemia *Rationale:* Beta-thalassemia is an autosomal recessive disorder. This disorder is found primarily in individuals of Mediterranean descent. The disease also has been reported in Asian and African populations. Options 1, 3, and 4 are not risk factors for this disorder.

Which of the following clients is at risk for developing thyrotoxicosis?

1. A client with hypothyroidism *2. A client with Graves' disease who is having surgery* 3. A client with diabetes mellitus scheduled for debridement of a foot ulcer 4. A client with diabetes insipidus scheduled for an invasive diagnostic test *rationale* Thyrotoxicosis is usually seen in clients with Graves' disease with the symptoms precipitated by a major stressor. This complication typically occurs during periods of severe physiological or psychological stress such as trauma, sepsis, the birth process, or major surgery. It also must be recognized as a potential complication following a thyroidectomy.

A nurse is caring for a client with Addison's disease. The nurse checks the vital signs and determines that the client has orthostatic hypotension. The nurse determines that this finding relates to which of the following?

1. A decrease in cortisol release *2. A decreased secretion of aldosterone* 3. An increase in epinephrine secretion 4. Increased levels of androgens *rationale* A decreased secretion of aldosterone results in a limited reabsorption of sodium and water; therefore the client experiences fluid volume deficit. A decrease in cortisol, an increase in epinephrine, and an increase in androgen secretion do not result in orthostatic hypotension.

A nurse is monitoring a client receiving glipizide (Glucotrol). Which outcome indicates an ineffective response from the medication?

1. A decrease in polyuria 2. A decrease in polyphagia 3. A fasting plasma glucose of 100 mg/dL *4. A glycosylated hemoglobin level of 12%* *rationale* Glipizide (Glucotrol) is an oral hypoglycemic agent administered to decrease the serum glucose level and the signs and symptoms of hyperglycemia. Therefore, a decrease in both polyuria and polyphagia would indicate a therapeutic response. Laboratory values are also used to monitor a client's response to treatment. A fasting blood glucose level of 100 mg/dL is within normal limits. However, glycosylated hemoglobin of 12% indicates poor glycemic control.

A nurse is reviewing the prescriptions of a client diagnosed with diabetes mellitus who was admitted because of an infected foot ulcer. Which health care provider's prescription supports the treatment of this condition?

1. A decreased amount of NPH daily insulin *2. An increased amount of NPH daily insulin* 3. An increased-calorie diet 4. A decreased-calorie diet *rationale* Infection is a physiological stressor that can cause an increase in the level of epinephrine in the body. An increase in epinephrine causes an increase in blood glucose levels. When the client is under stress, such as when an infection exists, the client will require an increase in the dose of insulin to facilitate the transport of excess glucose into the cells. The client does not necessarily need an adjustment in the daily diet.

A preliminary diagnosis is made for a child with acute lymphoblastic leukemia (ALL). In reviewing the complete blood cell count (CBC) of the child, the nurse would expect to find:

1. A hematocrit (Hct) count of 36 cells in 1 mL of peripheral blood 2. A hemoglobin (Hgb) count of 12 cells in 1 mL of peripheral blood *3. An erythrocyte (red blood cell [RBC]) count of 2 cells in 1 mL of peripheral blood* 4. A white blood cell (WBC) count of 15,000 cells in 1 mL of peripheral blood *Rationale:* ALL is diagnosed based on the history and the signs and symptoms a child presents during a health care provider's visit. The diagnosis also can be made during a routine physical exam or unrelated injury in which the child is brought to a medical clinic. A CBC is done initially to assist with a diagnosis. The CBC would indicate leukopenia (lower than normal WBCs), anemia, and thrombocytopenia. The values in options 1 and 2 are normal results for this age group and do not indicate anemia. Option 4 does not indicate leukopenia. Option 3 is the only laboratory value that is a lower result than a normal RBC count for this age group and is an indicator of anemia or possible leukemia.

A nurse assigned to care for a child with mumps is monitoring the child for the signs and symptoms associated with the common complication of mumps. The nurse monitors for which of the following that is indicative of this common complication?

1. A red, swollen testicle *2. Nuchal rigidity* 3. Pain 4. Deafness *Rationale:* The most common complication of mumps is aseptic meningitis, with the virus being identified in the cerebrospinal fluid. Common signs include nuchal rigidity, lethargy, and vomiting. A red, swollen testicle may be indicative of orchitis. Although this complication appears to cause most concern among parents, it is not the most common complication. Although mumps is one of the leading causes of unilateral nerve deafness, it does not occur frequently. Muscular pain, parotid pain, or testicular pain may occur, but pain does not indicate a sign of a common complication.

A nurse is assigned to care for an infant with a diagnosis of tricuspid atresia. The nurse plans care, knowing that in this disorder:

1. A single vessel overrides both ventricles. 2. Frequent episodes of hypercyanotic spells occur. *3. There is no communication from the right atrium to the right ventricle.* 4. There is no communication from the systemic and pulmonary circulations. *Rationale:* In tricuspid atresia, there is no communication from the right atrium to the right ventricle. Option 1 describes truncus arteriosus. Option 4 describes transposition of the great arteries. Frequent episodes of hypercyanotic spells occur in tetralogy of Fallot.

A nurse is assisting in preparing to care for a child with a brain tumor who will be returning from the recovery room following debulking of the tumor. Which of the following items will the nurse place at the bedside in preparation for the child's return from surgery?

1. A suction machine *2. A cooling blanket* 3. Protective isolation equipment 4. Skeletal traction equipment *Rationale:* Special attention is paid to the child's temperature postoperatively, which may be elevated because of hypothalamus or brainstem involvement during surgery. A cooling blanket should either be in place on the bed or readily available if the child becomes hyperthermic. Suctioning is avoided because it can cause increased intracranial pressure. Protective isolation is unnecessary, and there is no need for skeletal traction equipment.

A nurse is caring for a client with pheochromocytoma. Which data would indicate a potential complication associated with this disorder?

1. A urinary output of 50 mL/hr 2. A coagulation time of 5 minutes *3. Congestion heard on auscultation of the lungs* 4. A blood urea nitrogen (BUN) level of 20 mg/dL *rationale* The complications associated with pheochromocytoma include hypertensive retinopathy and nephropathy, myocarditis, congestive heart failure (CHF), increased platelet aggregation, and stroke. Death can occur from shock, stroke, renal failure, dysrhythmias, or dissecting aortic aneurysm. Congestion heard on auscultation of the lungs is indicative of CHF. A urinary output of 50 mL/hr is an appropriate output; the nurse would become concerned if the output were less than 30 mL/hr. A coagulation time of 5 minutes is normal. A BUN level of 20 mg/dL is a normal finding.

A client with diabetes mellitus demonstrates acute anxiety when admitted to the hospital for the treatment of hyperglycemia. The appropriate intervention to decrease the client's anxiety would be to:

1. Administer a sedative. *2. Convey empathy, trust, and respect toward the client.* 3. Ignore the signs and symptoms of anxiety so that they will soon disappear. 4. Make sure the client knows all the correct medical terms so that he or she can understand what is happening. *rationale* The appropriate intervention is to address the client's feelings related to the anxiety and to convey empathy, trust, and respect toward the client. Administering a sedative is not the most appropriate intervention. The nurse should not ignore the client's anxious feelings. A client will not relate to medical terms, particularly when anxiety exists.

Choose the interventions for a child older than 2 years of age with type 1 diabetes mellitus who has a blood glucose level of 60 mg/dL. *Select all that apply.*

1. Administer regular insulin. 2. Encourage the child to ambulate. *3. Give the child a teaspoon of honey.* 4. Provide electrolyte replacement therapy intravenously. 5. Wait 30 minutes and confirm the blood glucose reading. *6. Prepare to administer glucagon subcutaneously if unconsciousness occurs.* *Rationale:* Hypoglycemia is defined as a blood glucose level less than 70 mg/dL. Hypoglycemia occurs as a result of too much insulin, not enough food, or excessive activity. If able, the nurse should confirm hypoglycemia with a blood glucose reading. Glucose is administered orally immediately; the rapid-releasing sugar (such as honey) is followed by a complex carbohydrate and protein, such as a slice of bread or a peanut butter cracker. An extra snack is given if the next meal is not planned for more than 30 minutes or if activity is planned. If the child becomes unconscious, cake frosting or glucose paste can be squeezed onto the gums, and the blood glucose level is retested. If the child does not improve within 15 minutes, administration of glucagon may be necessary, and the nurse should be prepared for this intervention. In the hospital setting the nurse should be prepared to administer dextrose intravenously. Encouraging the child to ambulate and administering regular insulin will result in a lowered blood glucose level. Providing electrolyte replacement therapy intravenously is an intervention to treat diabetic ketoacidosis. Waiting 30 minutes to confirm the blood glucose level delays necessary intervention.

A nurse is reviewing the health care provider's prescriptions for a child with rheumatic fever who is suspected of having a viral infection. The nurse notes that acetylsalicylic acid (aspirin) is prescribed for the child. Which of the following nursing actions is appropriate?

1. Administer the aspirin if the child's temperature is elevated. 2. Administer the aspirin if the child experiences any joint pain. *3. Consult with the registered nurse to verify the prescription.* 4. Administer acetaminophen (Tylenol) instead of the aspirin for temperature elevation. *Rationale:* Anti-inflammatory agents, including aspirin, may be prescribed for the child with rheumatic fever. Aspirin should not be given to a child who has chickenpox or other viral infections. The nurse would not administer acetaminophen without specific health care provider prescriptions. Options 1 and 2 are not appropriate actions.

A client with Graves' disease has exophthalmos and is experiencing photophobia. Which intervention would best assist the client with this problem?

1. Administering methimazole (Tapazole) every 8 hours 2. Lubricating the eyes with tap water every 2 to 4 hours 3. Instructing the client to avoid straining or heavy lifting *4. Obtaining dark glasses for the client* *rationale* Because photophobia (light intolerance) accompanies this disorder, dark glasses are helpful in alleviating the symptom. Medical therapy for Graves' disease does not help alleviate the clinical manifestation of exophthalmos. Other interventions may be used to relieve the drying that occurs from not being able to completely close the eyes; however, the question is asking what the nurse can do for photophobia. Tap water, which is hypotonic, could actually cause more swelling to the eye because it could pull fluid into the interstitial space. In addition, the client is at risk for developing an eye infection because the solution is not sterile. There is no need to prevent straining with exophthalmos.

A nurse is assigned to care for a child with a diagnosis of Wilms' tumor. In planning care for the child, the nurse understands that this tumor is:

1. An abdominal tumor *2. A renal tumor* 3. A brain tumor 4. A bone tumor *Rationale:* Wilms' tumor, or nephroblastoma, is the most common renal tumor in children. Arising from the renal parenchyma of the kidney, this tumor grows very rapidly. It may be present unilaterally and localized, or bilaterally, and sometimes with metastasis to other organs.

A 1-year-old child is diagnosed with intussusception. The mother of the child asks the nurse to describe the disorder. The nurse tells the mother that this disorder is:

1. An acute bowel obstruction *2. A condition in which a proximal segment of the bowel prolapses into a distal segment of the bowel* 3. A condition in which a distal segment of the bowel prolapses into a proximal segment of the bowel 4. A condition that causes an acute inflammatory process in the bowel *Rationale:* Intussusception occurs when a proximal segment of the bowel prolapses into a distal segment of the bowel. It is a common cause of acute bowel obstruction in infants and young children. It is not an inflammatory process.

A nurse is caring for an infant who has been diagnosed with primary hypothyroidism. The nurse is reviewing the results of the laboratory tests and would expect to note which of the following?

1. An elevated T4 level *2. An elevated thyroid-stimulating hormone (TSH) level* 3. A decreased TSH level 4. A normal T4 level *Rationale:* Diagnostic findings in primary hypothyroidism include a low T4 level and a high TSH level. Options 1, 3, and 4 are not diagnostic findings in this condition.

Laboratory studies are performed on a child suspected of iron deficiency anemia. The nurse reviews the laboratory results, knowing that which of the following would indicate this type of anemia?

1. An elevated hemoglobin level with a low hematocrit level 2. A decreased reticulocyte count 3. An elevated red blood cell (RBC) count *4. RBCs that are microcytic and hypo chromic* *Rationale:* The results of a complete blood cell count in children with iron deficiency anemia will show low hemoglobin levels and microcytic and hypochromic RBCs. The reticulocyte count is usually normal or slightly elevated.

A nurse is caring for a child with a fracture who is placed in skeletal traction. The nurse monitors for the most serious complication associated with this type of traction by checking for:

1. An increase in the blood pressure 2. A decrease in the urinary output 3. A lack of appetite *4. An elevated temperature* *Rationale:* The most serious complication associated with skeletal traction is osteomyelitis, an infection involving the bone. Organisms gain access to the bone systemically or through the opening created by the metal pins or wires used with the traction. Osteomyelitis may occur with any open fracture. Clinical manifestations include complaints of localized pain, swelling, warmth, tenderness, an unusual odor from the fracture site, and an elevated temperature.

A nurse is preparing for the administration of ribavirin (Virazole) to a child with respiratory syncytial virus. Which of the following supplies will the nurse obtain for the administration of this medication?

1. An intravenous (IV) pole 2. An intramuscular (IM) syringe *3. A pair of goggles* 4. A protective isolation gown *Rationale:* Some caregivers experience headaches, burning nasal passages and eyes, and crystallization of soft contact lenses as a result of administration of ribavirin. Specific to this medication is the use of goggles. A mask may be worn. Handwashing is to be performed before and after any child contact. A gown is not necessary. The medication is administered via hood, face mask, or oxygen tent, not by the IM or IV route.

A nurse reviews the record of a 1-year-old child seen in the clinic and notes that the health care provider has documented a diagnosis of celiac crisis. Which of the following symptoms would the nurse expect to note in this condition?

1. Anorexia 2. Joint pain *3. Profuse, watery diarrhea* 4. Constipation *Rationale:* Clinical manifestations associated with celiac crisis include profuse, watery diarrhea and vomiting that quickly lead to severe dehydration and metabolic acidosis. The cause of the crisis is usually infection or hidden sources of gluten. The child may require intravenous fluids to correct fluid and acid-base imbalances, albumin to treat shock, and corticosteroids to decrease severe mucosal inflammation.

A mother of a 9-year-old child calls the emergency department and tells the nurse that her child received a minor burn on the hand after accidentally touching a grill during a family cookout. The mother asks the nurse for advice on how to treat the burn. The nurse tells the mother to immediately:

1. Apply a tepid compress to the child's hand. 2. Apply an ice pack to the child's hand. *3. Place the child's hand under cool running water.* 4. Apply a sterile bandage tightly over the burn area to prevent swelling. *Rationale:* Most minor burns can be handled at home by the parents. For minor burns, exposure to cool running water is the best treatment. This stops the burning process and helps alleviate pain. Ice is contraindicated because it may add more damage to already injured skin. Option 4 is an incorrect measure. In addition, the mother may not have a sterile dressing available.

A nursing student is assigned to care for an infant with a diagnosis of congestive heart failure (CHF). The student develops a plan of care for the child that is focused on monitoring for fluid overload. The student plans to best assess the urine output of the infant by:

1. Asking the health care provider for permission to insert a Foley catheter 2. Monitoring the intake closely 3. Comparing the intake with the output *4. Weighing the diapers* *Rationale:* The best method to assess urine output in an infant is to weigh the diapers. Comparing intake with output would not provide an accurate measure of urine output. Measuring the intake is not directly related to the subject of the question. Although Foley catheter drainage is most accurate in determining output, it is not the most appropriate method in an infant. In addition, insertion of a Foley catheter places the infant at risk for infection.

A mother brings her 5-month-old daughter into the pediatrician's office with complaints that the child has been vomiting during feedings. The mother also states that the child is sometimes very fussy. The nurse's initial action would be to:

1. Assess the child's growth status. *2. Obtain a complete history of the child's feeding habits.* 3. Assess whether any other children in the family have had the same problem. 4. Explain to the mother that the health care provider will prescribe a barium swallow and upper gastrointestinal (GI) series. *Rationale:* In most situations, a complete history and physical examination of the child is the initial step in diagnosing gastroesophageal reflux disease. The child's feeding habits will give the nurse an indicator of the growth status. The child is weighed and measured after the initial interview is completed with the parent. Hereditary factors are not the priority. Further diagnostic studies may be ordered but only after a complete history is obtained.

A nurse is reviewing the health care record of an infant suspected of having unilateral hip dysplasia. Which assessment finding would the nurse expect to note documented in the infant's record regarding this condition?

1. Asymmetric adduction of the affected hip when placed supine with the knees and hips flexed *2. Asymmetry of the gluteal skin folds when the infant is placed prone and the legs are extended against the examining table * 3. An apparent short femur on the unaffected side 4. Full range of motion in the affected hip *Rationale:* Asymmetry of the gluteal skin folds when the infant is placed prone and the legs are extended against the examining table is noted in hip dysplasia. Asymmetrical abduction of the affected hip, when an infant is placed supine with the knees and hips flexed, also would be an assessment finding in hip dysplasia in infants beyond the newborn period. An apparent short femur on the affected side is noted, as well as limited range of motion.

The nurse in the newborn nursery is preparing to feed a newborn the first feeding of sterile water. During the feeding, the newborn suddenly begins to cough, choke, and become cyanotic. Based on these symptoms, the nurse might suspect that the newborn has which of the following conditions?

1. Atrial septal defect *2. Tracheoesophageal fistula* 3. Bronchopulmonary dysplasia 4. Respiratory distress syndrome *Rationale:* The first feeding a newborn receives is sterile water to assess whether the newborn might have one of the tracheoesophageal (TE) conditions. Although sterile water is more easily absorbed and causes less aspiration than formula, the newborn with a suspected TE fistula condition will cough and choke during feedings. These symptoms are not associated with the conditions noted in options 1, 3, or 4.

A nurse is collecting data on a client admitted to the hospital with a diagnosis of myxedema. Which data collection technique will provide data necessary to support the admitting diagnosis?

1. Auscultation of lung sounds *2. Inspection of facial features* 3. Percussion of the thyroid gland 4. Palpation of the adrenal glands *rationale* Inspection of facial features will reveal the characteristic coarse features, presence of edema around the eyes and face, and a blank expression that are characteristic of myxedema. The techniques in the remaining options will not reveal any data that would support the diagnosis of myxedema.

A hospitalized client is newly diagnosed with diabetes mellitus. The client must take both NPH and Regular insulin for glucose control. The nurse develops a teaching plan to help the client meet which outcome as a first step in managing the disease?

1. Avoid all strenuous exercise. 2. Maintain health at an optimum level. 3. Lose 40 pounds to achieve ideal body weight. *4. Adjust insulin according to capillary blood glucose levels.* *rationale* There are many learning goals for the client who is newly diagnosed with diabetes mellitus. The client must learn dietary control, medication management, and proper exercise in order to control the disease. As a first step, the client learns to adjust medication (insulin) according to blood glucose results as prescribed by the health care provider. The client should then focus on long-term dietary control and weight loss, which will often lead to a decreased need for insulin. At the same time that diet is being controlled, the client should begin a regular exercise program to aid in weight loss.

A client is diagnosed with hyperparathyroidism. The nurse teaching the client about dietary alterations to manage the disorder tells the client to limit which of the following foods in the diet?

1. Bananas 2. Oatmeal *3. Ice cream* 4. Chicken breast *rationale* The client with hyperparathyroidism is likely to have elevated calcium levels. This client should reduce intake of dairy products such as milk, cheese, ice cream, or yogurt. Apples, bananas, chicken, oatmeal, and pasta are low-calcium foods.

A nurse assists in monitoring for early signs of meningitis in a child and assists with attempting to elicit Kernig's sign. The appropriate procedure to elicit Kernig's sign is to:

1. Bend the head toward the knees and hips and check for pain. 2. Tap the facial nerve and check for spasm. 3. Compress the upper arm and check for tetany. *4. Extend the leg and knee and check for pain.* *Rationale:* Kernig's sign is pain that occurs with extension of the leg and knee. Brudzinski's sign occurs when flexion of the head causes flexion of the hips and knees. Chvostek's sign, seen in tetany, is a spasm of the facial muscles elicited by tapping the facial nerve in the region of the parotid gland. Trousseau's sign is a sign for tetany in which carpal spasm can be elicited by compressing the upper arm and causing ischemia to the nerves distally.

A nurse is preparing to care for a child who received an allogenic bone marrow transplant (BMT). The nurse understands that which of the following is the priority concern?

1. Bleeding *2. Infection* 3. Sensory alterations 4. Social isolation *Rationale:* Once the marrow is infused, nursing care focuses on preventing immunocompromised children from developing a life-threatening infection until they engraft and produce their own white blood cells to fight infections. Although options 1, 3, and 4 are considerations in the plan of care, the potential for infection is the priority for a child following a BMT.

A nurse is assessing a pediatric client with a diagnosis of retinoblastoma. The nurse assesses for which most common clinical finding for a child with this diagnosis?

1. Blindness 2. Strabismus *3. Cat's-eye reflex* 4. Red, painful eye *Rationale:* Clinical manifestations of retinoblastoma include cat's-eye reflex (most common sign); strabismus (second most common sign); red, painful eye; and blindness (late signs). Cat's-eye reflex is commonly observed by the parent and is described as a whitish "glow" in the pupil. This represents visualization of the tumor as the light momentarily falls on the mass, and is the most common sign.

A nurse is caring for a newborn with spina bifida (myelomeningocele type) who is scheduled for the removal of the gibbus (sac on the back filled with cerebrospinal fluid, meninges, and some of the spinal cord). In the preoperative period, the priority nursing action is to monitor:

1. Blood pressure *2. Moisture of the normal saline dressing on the gibbous area* 3. Specific gravity of the urine 4. Anterior fontanel for depression *Rationale:* The newborn is at risk for infection before closure of the gibbus. A sterile normal saline dressing is placed over the gibbus to maintain moisture of the gibbus and its contents. This prevents tearing or breakdown of the skin integrity at the site. Blood pressure is difficult to determine during the newborn period and is not the best indicator of infection. Urine concentration is not well developed in the newborn stage of development. Depression of the anterior fontanel is a sign of dehydration. With spina bifida, an increase in intracranial pressure is more of a priority. A complication of spina bifida would demonstrate a bulging or taut anterior fontanel.

A 4-year-old child is diagnosed with otitis media, and the mother asks the nurse about the causes of this illness. The nurse responds, knowing that which of the following is an unassociated risk factor related to otitis media?

1. Bottle-feeding 2. Household smoking 3. A history of urinary tract infections 4. Exposure to illness in other children *Rationale:* Factors that increase the risk of otitis media include exposure to illness, household smoking, bottle-feeding, and congenital conditions such as Down syndrome and cleft palate. The use of a pacifier beyond age 6 months has also been identified as a risk factor. Allergies are also thought to precipitate otitis media. Urinary tract infections are not with a risk factor for otitis media.

A nurse is reviewing the record of a child scheduled for a health care provider's visit. Before data collection, the nurse notes documentation that the child has enuresis. Based on this diagnosis, the nurse plans to focus on which of the following when collecting data?

1. Bowel function *2. Bladder function* 3. Motor development 4. Nutritional status and weight gain *Rationale:* Enuresis refers to a condition in which the child is unable to control bladder function, although he or she has reached an age at which control of voiding is expected. Nocturnal enuresis, or bed-wetting, is common in children.

A nurse is monitoring a client with Graves' disease for signs of thyrotoxic crisis (thyroid storm). Which of the following signs and symptoms, if noted in the client, will alert the nurse to the presence of this crisis? *Select all that apply.*

1. Bradycardia *2. Fever* *3. Sweating* *4. Agitation* 5. Pallor *rationale* Thyrotoxic crisis (thyroid storm) is an acute, potentially life-threatening state of extreme thyroid activity that represents a breakdown in the body's tolerance to a chronic excess of thyroid hormones. The clinical manifestations include fever greater than 100° F, severe tachycardia, flushing and sweating, and marked agitation and restlessness. Delirium and coma can occur.

A nurse is collecting data on a 12-month-old child with iron deficiency anemia. Which of the following findings would the nurse expect to note in this child?

1. Bradycardia *2. Tachycardia* 3. Hyperactivity 4. A reddened appearance to the cheeks *Rationale:* Clinical manifestations of iron deficiency anemia will vary with the degree of anemia but usually include extreme pallor with porcelain-like skin, tachycardia, lethargy, and irritability.

A client with a pituitary tumor will undergo transsphenoidal hypophysectomy. The nurse includes which priority item in the preoperative teaching plan for the client?

1. Brushing the teeth vigorously and frequently is important to minimize bacteria in the mouth. *2. Blowing the nose following surgery is prohibited.* 3. A small area will be shaved at the base of the neck. 4. It will be necessary to cough and deep breathe following the surgery. *rationale* The approach used for this surgery is the oronasal route, specifically where the upper lip meets the gum. The surgeon then uses a route through the sphenoid sinus to get to the pituitary gland. The client is not allowed to blow the nose, sneeze, or cough vigorously because these activities could raise intracranial pressure. The client also is not allowed to brush the teeth, to avoid disrupting the surgical site. Alternate methods for performing mouth care are used.

A nurse is assigned to care for an infant with tetralogy of Fallot. The mother of the infant calls the nurse to the room because the infant suddenly seems to be having difficulty breathing. The nurse enters the room and notes that the infant is experiencing a hypercyanotic episode. The initial nursing action is to:

1. Call a code. 2. Contact the respiratory therapy department. 3. Place the infant in a prone position. *4. Place the infant in a knee-chest position.* *Rationale:* If a hypercyanotic episode occurs, the infant is placed in a knee-chest position. The nurse would contact the registered nurse, who would then contact the health care provider. The knee-chest position is thought to increase pulmonary blood flow by increasing systemic vascular resistance. This position also improves systemic arterial oxygen saturation by decreasing venous return so that smaller amounts of highly saturated blood reach the heart. Toddlers and children squat to obtain this position and relieve chronic hypoxia.

The wife of a client with diabetes mellitus who takes insulin calls the nurse in a health care provider's office about her husband. She states that her husband is sleepy and that his skin is warm and flushed. She adds that his breathing is faster than normal and his pulse rate seems fast. Which of the following should the nurse tell the woman to do first?

1. Call an ambulance. 2. Take his temperature. *3. Check his blood glucose level.* 4. Drive him to the health care provider's office. *rationale* The client's signs and symptoms are consistent with hyperglycemia. The wife should first obtain a blood glucose reading, which the nurse would then report to the health care provider. Option 1 or 4 may be done at a later time if required. Option 2 is unrelated to the client's immediate problem.

An adolescent with diabetes mellitus is attending gym class and suddenly becomes flushed and complains of dizziness and a headache. The gym teacher quickly takes the adolescent to the school nurse's office. The nurse obtains a blood glucose level, and the results indicate a level of 65 mg/dL. The appropriate initial nursing intervention is to:

1. Call the child's mother for permission to treat the child. 2. Call the school health care provider immediately. 3. Let the child rest until the blood glucose has an opportunity to rise. *4. Give the child 6 oz of a regular cola drink.* *Rationale:* A blood glucose level below 70 mg/dL indicates hypoglycemia. The child is participating in an activity that requires more energy than that of the normal routine at school. Insulin and food requirements change with situations that require more energy. When signs of hypoglycemia occur, the child needs an immediate source of glucose. Options 1, 2, and 3 do not address the hypoglycemic state immediately and delay required treatment.

A nurse is caring for a client with a diagnosis of myasthenia gravis. The health care provider plans to perform an Enlon test on the client to determine the presence of cholinergic crisis. In addition to planning care for the client during this testing, which of the following will the nurse ensure is at the bedside?

1. Cardiac monitor *2. Oxygen equipment* 3. Vial of protamine sulfate and a syringe 4. Potassium injection and a liter of normal saline solution *rationale* An Enlon test is performed to distinguish between myasthenic and cholinergic crisis. Following administration of Enlon, if symptoms intensify, the crisis is cholinergic. Because the symptoms of cholinergic crisis will worsen with the administration of Enlon, atropine sulfate and oxygen should be immediately available whenever Enlon is used.

A nurse is caring for an infant. A urinalysis has been prescribed, and the nurse plans to collect the specimen. The nurse implements which appropriate method to collect the specimen?

1. Catheterizes the infant, using a No. 5 French Foley *2. Attaches a urinary collection device to the infant's perineum* 3. Obtains the specimen from the diaper, using a syringe, after the infant voids 4. Monitors the urinary patterns and prepares to collect the specimen into a cup when the infant voids *Rationale:* Although many methods have been used to collect urine from an infant, the most reliable method is the urine collection device. This device is a plastic bag that has an opening lined with adhesive so that it may be attached to the perineum. Urine for certain tests, such as specific gravity, may be obtained from a diaper. Urinary catheterization is not to be done unless specifically prescribed because of the risk of infection. It is not reasonable to monitor urinary patterns and attempt to collect the specimen in a cup when the infant voids.

A nurse is collecting data regarding a client after a thyroidectomy and notes that the client has developed hoarseness and a weak voice. Which nursing action is appropriate?

1. Check for signs of bleeding. 2. Administer calcium gluconate. 3. Notify the registered nurse immediately. *4. Reassure the client that this is usually a temporary condition.* *rationale* Weakness and hoarseness of the voice can occur as a result of trauma of the laryngeal nerve. If this develops, the client should be reassured that the problem will subside in a few days. Unnecessary talking should be discouraged. It is not necessary to notify the registered nurse immediately. These signs do not indicate bleeding or the need to administer calcium gluconate.

A nurse is monitoring a child with a cast on the forearm for signs of compartment syndrome. The nurse understands that which data collection technique is unlikely to provide information about this complication?

1. Checking the quality of the radial pulse 2. Checking the child's ability to extend the fingers 3. Checking for effectiveness of analgesics administered for pain *4. Checking the child's ability to perform range of motion to the shoulder area of the affected extremity* *Rationale:* Compartment syndrome occurs when swelling causes pressure to rise within a compartment (sheath of inelastic fascia). The increased pressure compromises circulation to the muscles and nerves within the compartment and can result in paralysis and necrosis of tissues. Signs of compartment syndrome include severe pain, often unrelieved by analgesics, and signs of neurovascular impairment. Compartment syndrome is not uncommon in fractures of the forearms; therefore the quality of the radial pulse and the ability to extend the fingers should be assessed. If extension of the fingers produces pain, the health care provider should be notified. Option 4 is unlikely to provide information about compartment syndrome.

A nurse is caring for a newborn with a diagnosis of spina bifida (myelomeningocele). To monitor for a major symptom associated with this disorder, the nurse:

1. Checks the capillary refill on the nail beds of the upper extremities 2. Tests the urine for blood 3. Palpates the abdomen for masses *4. Checks for responses to painful stimuli from the torso downward* *Rationale:* Newborns with spina bifida (myelomeningocele type) demonstrate lack of nerve innervation from below the site of the gibbus (sac containing the meninges and spinal cord). They therefore show diminished or no responses to painful stimuli in the areas below the gibbus. Options 1, 2, and 3 are incorrect because the area above the gibbus is not affected. The capillary refill would be normal. The urine would not have blood present. If the kidneys are affected, proteinuria could be present but this is not generally noted in the newborn period. No masses are present besides the gibbus on the back area, externally protruding from the vertebral deformity.

A nursing instructor asks a nursing student about the use of bacillus Calmette-Guerin vaccine (BCG). The nursing student responds correctly, knowing that the BCG vaccine is used for:

1. Children with a positive Mantoux test 2. All children to prevent tuberculosis (TB) 3. Children with both a positive Mantoux test and positive chest x-ray *4. Asymptomatic human immunodeficiency virus (HIV)-infected children who are at increased risk for developing TB* *Rationale:* The BCG vaccine is used mainly for children with a negative chest x-ray and skin test results who have had repeated exposures to TB and for asymptomatic HIV-infected children who are at increased risk for developing TB.

A sweat test is performed on a child with a suspected diagnosis of cystic fibrosis (CF). Which test result is suggestive of cystic fibrosis and will require further assessment and investigation?

1. Chloride level of 5 mEq/L 2. Chloride level of 10 mEq/L 3. Chloride level of 20 mEq/L *4. Chloride level of 40 mEq/L* *Rationale:* In a sweat test, sweating is stimulated on the child's forearm with pilocarpine, the sample is collected on absorbent material, and the amount of sodium and chloride is measured. A sample of at least 50 mg of sweat is required for accurate results. A chloride level greater than 60 mEq/L is considered to be a positive test result. A chloride level of 40 mEq/L is suggestive of CF and requires a repeat test. Options 1, 2, and 3 do not identify results that are positive for CF.

A nurse working on an endocrine nursing unit understands that which correct concept is used in planning care?

1. Clients with Cushing's syndrome are likely to experience episodic hypotension. 2. Clients with hyperthyroidism must be monitored for weight gain. 3. Clients who have diabetes insipidus should be assessed for fluid excess. *4. Clients who have hyperparathyroidism should be protected against falls.* *rationale* Hyperparathyroidism is a disease that involves excess secretion of parathyroid hormone (PTH). Elevation of PTH causes excess calcium to be removed from the bones. There is a decline in bone mass, which may cause a fracture if a fall occurs. Cushing's syndrome is likely to cause hypertension. Clients with hypothyroidism must be monitored for weight gain and clients with hyperthyroidism must be monitored for weight loss. Clients who have diabetes insipidus should be assessed for fluid deficit.

A nurse is planning to instruct a client with diabetes mellitus who has hypertension about "sick day management." Which of the following does the nurse avoid putting on a list of easily consumed carbohydrate-containing beverages for use when the client cannot tolerate food orally?

1. Cola 2. Ginger ale 3. Apple juice *4. Mineral water* *rationale* Diabetic clients should take in approximately 15 g of carbohydrate every 1 to 2 hours when unable to tolerate food because of illness. Each of the beverages listed in options 1, 2, and 3 provides approximately 13 to 15 g of carbohydrate in a half-cup serving. Mineral water is incorrect for two reasons. First, it contains sodium and should not be used by the client with hypertension. Second, it is not a source of carbohydrates.

A nurse is caring for a 9-year-old child with leukemia who is hospitalized for the administration of chemotherapy. The nurse monitors the child for central nervous system (CNS) involvement by checking which of the following?

1. Color, motion, and sensation of the extremities 2. Pupillary reaction *3. Level of consciousness (LOC)* 4. The presence of petechiae in the sclera *Rationale:* The CNS is monitored because of the risk of infiltration of blast cells into the CNS. The child's level of consciousness is assessed, and the child is monitored for signs of irritability, vomiting, and lethargy. Color, motion, and sensation of the extremities are neurovascular assessments. Changes in pupillary reaction are most often noted in conditions related to increased intracranial pressure. The presence of petechiae in the sclera is an objective sign that may occur in leukemia.

A client who returned to the nursing unit 8 hours ago after hypophysectomy has clear drainage saturating the nasal dressing. The nurse should take which action first?

1. Continue to observe for further drainage. *2. Test the drainage for glucose.* 3. Put the head of the bed flat. 4. Test the drainage for occult blood. *rationale* Following hypophysectomy the client should be monitored for rhinorrhea (clear nasal drainage), which could indicate a cerebrospinal fluid (CSF) leak. If this occurs, the drainage should be collected and tested for the presence of CSF by testing it for glucose. CSF tests positive for glucose, whereas true nasal secretions would not. It is not necessary to test drainage that is clear for occult blood. The head of the bed should not be lowered, to prevent a rise in intracranial pressure. Continuing to observe the drainage without taking action could put the client at risk for developing a serious complication.

A nursing instructor is observing a nursing student caring for an infant with a diagnosis of bladder exstrophy. The nursing student provides appropriate care to the infant by:

1. Covering the bladder with a dry sterile dressing 2. Covering the bladder with a wet-to-dry dressing 3. Applying sterile water soaks to the bladder mucosa *4. Covering the bladder with a nonadhering plastic wrap* *Rationale:* Care should be taken to protect the exposed bladder tissue from drying while allowing drainage of urine. This is best accomplished by covering the bladder with a nonadhering plastic wrap. The use of wet-to-dry dressings should be avoided because this type of dressing adheres to the mucosa and may damage the delicate tissue when removed. Sterile dressings and dressings soaked in solutions can also dry out and damage the mucosa when removed.

A nurse is caring for a client with pheochromocytoma. The client asks for a snack and something warm to drink. The appropriate choice for this client to meet nutritional needs would be which of the following?

1. Crackers with cheese and tea *2. Graham crackers and warm milk* 3. Toast with peanut butter and cocoa 4. Vanilla wafers and coffee with cream and sugar *rationale* The client with pheochromocytoma needs to be provided with a diet that is high in vitamins, minerals, and calories. Of particular importance is that food or beverages that contain caffeine (e.g., chocolate, coffee, tea, and cola) are prohibited.

A nurse is reviewing the laboratory results of a child scheduled for tonsillectomy. Which laboratory value would be significant to review?

1. Creatinine 2. Urinalysis *3. Platelet count* 4. Blood urea nitrogen (BUN) *Rationale:* Before the surgical procedure, the child is assessed for signs of active infection and for redness and exudate of the throat. Because the tonsillar area is so vascular, postoperative bleeding is a concern. The prothrombin (PT), partial thromboplastin time (PTT), platelet count, hemoglobin and hematocrit (H&H), white blood cell (WBC) count, and urinalysis are performed preoperatively. The platelet count result would identify a potential for bleeding. The BUN and creatinine would not determine the potential for bleeding but rather evaluate renal function.

A mother brings her 4-month-old infant to the well-baby clinic for immunizations. Which immunizations should be administered to this infant?

1. DTaP (diphtheria, tetanus, acellular pertussis), MMR (measles, mumps, rubella), IPV (inactivated poliovirus vaccine) 2. MMR, Hib (Haemophilus influenzae type b), DTaP *3. DTaP, Hib, IPV, pneumococcal vaccine (PCV)* 4. Varicella and hepatitis B vaccines *Rationale:* DTaP, Hib, IPV, and PCV are administered at 4 months of age. DTaP is administered at 2 months, 4 months, 6 months, between 12 and 18 months, and between 4 and 6 years of age. Hib is administered at 2 months, 4 months, 6 months, and between 12 and 15 months of age. IPV is administered at 2 months, 4 months, 6 months, and between 4 and 6 years of age. The first dose of MMR is administered between 12 and 15 months of age; the second dose is administered at 4 to 6 years of age (if the second dose was not given by 4 to 6 years of age, it should be given at the next visit). The first dose of hepatitis B vaccine is administered between birth and 2 months, the second dose is administered between 1 and 4 months, and the third dose is administered between 6 and 18 months of age. Varicella zoster vaccine is administered between 12 and 18 months of age. PCV is administered at 2, 4, and 6 months of age and between 12 and 15 months of age.

A nurse is preparing to care for a child with a head injury. On review of the records, the nurse notes that the health care provider has documented decorticate posturing. The nurse plans care, knowing that this type of posturing indicates which of the following?

1. Damage to the midbrain 2. Dysfunction of the pons 3. Damage to the diencephalon *4. Dysfunction in the cerebral hemisphere* *Rationale:* Decorticate posturing indicates a lesion in the cerebral hemisphere or disruption of the corticospinal tracts. Decerebrate posturing indicates damage in the diencephalon, midbrain, or pons.

A nurse is providing instructions to a client newly diagnosed with diabetes mellitus. The nurse gives the client a list of the signs of hyperglycemia. Which of the following specific signs of this complication should be included on the list?

1. Decreased urine output 2. Profuse sweating *3. Increased thirst* 4. Shakiness *rationale* The classic signs of hyperglycemia include polydipsia, polyuria, and polyphagia. Profuse sweating and shakiness would be noted in a hypoglycemic condition.

The nurse caring for a client who has had a subtotal thyroidectomy reviews the plan of care and determines which problem is the priority for this client in the immediate postoperative period?

1. Dehydration 2. Infection 3. Urinary retention *4. Bleeding* *rationale* Hemorrhage is one of the most severe complications that can occur following thyroidectomy. The nurse must frequently check the neck dressing for bleeding and monitor vital signs to detect early signs of hemorrhage, which could lead to shock. T3 and T4 do not regulate fluid volumes in the body. Infection is a concern for any postoperative client but is not the priority in the immediate postoperative period. Urinary retention can occur in postoperative clients as a result of medication and anesthesia but is not the priority from the options provided.

A nurse is developing goals for a school-age child with a knowledge deficit related to the use of inhalers and peak flowmeters. The nurse identifies which of the following as an appropriate goal for this child?

1. Denies shortness of breath or difficulty breathing 2. Has regular respirations at a rate of 18 to 22 breaths per minute *3. Expresses feelings of mastery and competence with breathing devices* 4. Watches the educational video and reads printed information provided *Rationale:* School-age children strive for mastery and competence to achieve the developmental task of industry and accomplishment. Options 1 and 2 do not relate to the knowledge deficit, which is the subject of the question. Option 4 is an intervention rather than a goal.

A client with type 2 diabetes mellitus has a blood glucose of more than 600 mg/dL and is complaining of polydipsia, polyuria, weight loss, and weakness. The nurse reviews the health care provider's documentation and would expect to note which of the following diagnoses?

1. Diabetic ketoacidosis (DKA) 2. Hypoglycemia *3. Hyperglycemic hyperosmolar nonketotic syndrome (HHNS)* 4. Pheochromocytoma *rationale* HHNS is seen primarily in individuals with type 2 diabetes who experience a relative deficiency of insulin. The onset of symptoms may be gradual. The symptoms may include polyuria, polydipsia, dehydration, mental status alterations, weight loss, and weakness. DKA normally occurs in type 1 diabetes mellitus. The clinical manifestations noted in the question are not signs of hypoglycemia. Pheochromocytoma is not related to these clinical manifestations.

An emergency department nurse is caring for a child brought to the emergency department following the ingestion of approximately one half bottle of acetylsalicylic acid (aspirin). The nurse anticipates that the likely initial treatment will be:

1. Dialysis 2. The administration of vitamin K *3. The administration of activated charcoal* 4. The administration of sodium bicarbonate *Rationale:* Initial treatment of salicylate overdose includes administration of activated charcoal to decrease absorption of the aspirin. Intravenous (IV) fluids and inducing emesis may be prescribed to enhance excretion but would not be the initial treatment. Dialysis is used in extreme cases if the child is unresponsive to therapy. Vitamin K is the antidote for warfarin (Coumadin) overdose.

After several diagnostic tests, a client is diagnosed with diabetes insipidus. The nurse understands that which symptom is indicative of this disorder?

1. Diarrhea *2. Polydipsia* 3. Weight gain 4. Blurred vision *rationale* Polydipsia and polyuria are classic symptoms of diabetes insipidus. The urine is pale in color, and its specific gravity is low. Anorexia and weight loss occur. Diarrhea, weight loss, and blurred vision are not manifestations of the disorder.

A nurse is collecting data from a client who is being admitted to the hospital for a diagnostic workup for primary hyperparathyroidism. The nurse understands that which client complaint would be characteristic of this disorder?

1. Diarrhea *2. Polyuria* 3. Polyphagia 4. Weight gain *rationale* Hypercalcemia is the hallmark of hyperparathyroidism. Elevated serum calcium levels produce osmotic diuresis (polyuria). This diuresis leads to dehydration and the client would lose weight. Options 1, 3, and 4 are gastrointestinal (GI) symptoms but are not associated with the common GI symptoms typical of hyperparathyroidism (nausea, vomiting, anorexia, constipation).

A child is scheduled for a tonsillectomy. Which of the following would present the highest risk of aspiration during surgery?

1. Difficulty swallowing 2. Bleeding during surgery 3. Exudate in the throat area *4. The presence of loose teeth* *Rationale:* In the preoperative period, the child should be observed for the presence of loose teeth to decrease the risk of aspiration during surgery. Options 1 and 3 are incorrect. Bleeding during surgery will be controlled via packing and suction as needed.

A nurse collecting data on a child suspects physical abuse. The nurse understands that which of the following is a primary and legal nursing responsibility?

1. Document the child's physical assessment findings accurately and thoroughly. *2. Report the case in which the abuse is suspected.* 3. Refer the family to the appropriate support groups. 4. Assist the family in identifying resources and support systems. *Rationale:* The primary legal nursing responsibility when child abuse is suspected is to report the case. All 50 states require health care professionals to report all cases of suspected abuse. Although documentation of findings, assisting the family, and referring the family to appropriate resources and support groups are important, the primary legal responsibility is to report the case.

Following hypophysectomy, a client complains of being very thirsty and having to urinate frequently. The initial nursing action is to:

1. Document the complaints. 2. Increase fluid intake. *3. Check the urine specific gravity.* 4. Check for urinary glucose. *rationale* Following hypophysectomy, diabetes insipidus can occur temporarily because of antidiuretic hormone deficiency. This deficiency is related to surgical manipulation. The nurse should check the urine for specific gravity and report the results if they are less than 1.005. Urinary glucose and diabetes mellitus is not a concern here. In this situation, increasing fluid intake would require a health care provider's prescription. The client's complaint would be documented but not as an initial action.

A child is seen in the clinic, and the primary health care provider documents a diagnosis of primary nocturnal enuresis. The mother asks the nurse about the diagnosis. The nurse bases the response on the fact that primary nocturnal enuresis:

1. Does not respond to treatment 2. Is caused by a psychiatric problem 3. Requires surgical intervention to improve the problem *4. Is common and most children will outgrow bed-wetting without therapeutic intervention* *Rationale:* Primary nocturnal enuresis is bedwetting and is described as occurring in a child that has never been dry at night for extended periods. It is common in children, most of whom will outgrow bedwetting without therapeutic intervention. The child is not able to sense a full bladder and does not awaken to void. The child may have delayed maturation of the central nervous system (CNS). It is not caused by a psychiatric problem. Behavioral conditioning with use of alarms has been used for treatment in the older child with nocturnal enuresis. A device that contains a moisture-sensitive alarm is worn on the child's pajamas. As the child starts to void, the alarm goes off, awakening the child. The alarm system may need to be used consistently over 15 weeks for resolution.

A nurse is caring for a client following an adrenalectomy and is monitoring for signs of adrenal insufficiency. Which of the following, if noted in the client, indicates signs and symptoms related to adrenal insufficiency? *Select all that apply.*

1. Double vision *2. Hypotension* *3. Mental status changes* *4. Weakness* *5. Fever* *rationale* The nurse should be alert to signs and symptoms of adrenal insufficiency in a client following adrenalectomy. These signs and symptoms include weakness, hypotension, fever, and mental status changes. Double vision is generally not associated with this condition.

A nurse is monitoring for fluid volume deficit in an infant who is vomiting and having diarrhea. The nurse weighs the infant's diaper after each voiding and stool and carefully calculates fluid volume, knowing that:

1. Each gram of diaper weight is equivalent to 0.5 mL of urine. *2. Each gram of diaper weight is equivalent to 1 mL of urine.* 3. Each gram of diaper weight is equivalent to 2 mL of urine. 4. Each gram of diaper weight is equivalent to 2.5 mL of urine. *Rationale:* When monitoring for fluid volume deficit, the nurse should weigh the infant's diaper after each voiding and stool. Each gram of diaper weight is equivalent to 1 mL of urine. Therefore options 1, 3, and 4 are incorrect.

A nurse is reinforcing instructions with a client with diabetes mellitus who is recovering from diabetic ketoacidosis (DKA) regarding measures to prevent a recurrence. Which instruction is important for the nurse to emphasize?

1. Eat six small meals daily. 2. Test the urine ketone levels. *3. Monitor blood glucose levels frequently.* 4. Receive appropriate follow-up health care. *rationale* Client education after DKA should emphasize the need for home glucose monitoring four to five times per day. It is also important to instruct the client to notify the health care provider when illness occurs. The presence of urinary ketones indicates that DKA has already occurred. The client should eat well-balanced meals with snacks, as prescribed.

Which clinical manifestation should the nurse expect to note when assessing a client with Addison's disease?

1. Edema 2. Obesity 3. Hirsutism *4. Hypotension* *rationale* Common manifestations of Addison's disease include postural hypotension from fluid loss, syncope, muscle weakness, anorexia, nausea, vomiting, abdominal cramps, weight loss, depression, and irritability. The manifestations in options 1, 2, and 3 are not associated with Addison's disease.

A nurse is assisting in preparing a plan of care for a child who will be returning from surgery following the application of a hip spica cast. Which of the following would be the priority in the plan of care for this child on return from the procedure?

1. Elevate the head of the bed. 2. Turn the child onto the right side. *3. Check circulation in the feet.* 4. Abduct the hips using pillows. *Rationale:* During the first few hours after a cast is applied, the primary concern is swelling that may cause the cast to produce a tourniquet-like effect and restrict circulation. Therefore circulatory assessment is a priority. Elevating the head of the bed of a child in a hip spica would cause discomfort. Using pillows to abduct the hips is not necessary because a hip spica immobilizes the hip and the knee. Turning the child side to side at least every 2 hours is important because it allows the body cast to dry evenly and prevents complications related to immobility; however, it is not as important as checking circulation.

A nurse is administering medications to a 6-year-old child with nephrotic syndrome. To reduce proteinuria, the nurse should expect that which medication would be prescribed?

1. Enalapril (Vasotec) *2. Furosemide (Lasix)* 3. Prednisone 4. Cyclophosphamide *Rationale:* The child is usually placed on diuretic therapy with furosemide (Lasix) until protein loss is controlled. Enalapril is most commonly used to control hypertension. Corticosteroids, such as prednisone, may be prescribed to decrease inflammation. Corticosteroids also suppress the autoimmune response and stimulate vascular reabsorption of edema. Cyclophosphamide is an alkylating agent and may be used in maintaining remission.

A nursing student notes in the medical record that a client with Cushing's syndrome is experiencing body image disturbances. The need for additional education regarding this problem is identified when the nursing student suggests which nursing intervention?

1. Encouraging the client's expression of feelings 2. Evaluating the client's understanding of the disease process 3. Encouraging family members to share their feelings about the disease process *4. Evaluating the client's understanding that the body changes need to be dealt with* *rationale* Evaluating the client's understanding that the body changes that occur in this disorder need to be dealt with is an inappropriate nursing intervention. This option does not address the client's feelings. Options 1, 2, and 3 are appropriate because they address the client and family feelings regarding the disorder.

A nurse is reviewing the health record of an infant with a diagnosis of gastroesophageal reflux. Which clinical manifestation of this disorder would the nurse expect to note documented in the record?

1. Excessive oral secretions 2. Bowel sounds heard over the chest *3. Hiccupping and spitting up after a meal* 4. Coughing, wheezing, and short periods of apnea *Rationale:* Clinical manifestations of all types of gastroesophageal reflux include vomiting (spitting up) after a meal, hiccupping, and recurrent otitis media related to pooled secretions in the nasopharynx during sleep. Option 1 is a clinical manifestation of esophageal atresia and tracheoesophageal fistula. Option 2 is a clinical manifestation of congenital diaphragmatic hernia. Option 4 is a clinical manifestation of hiatal hernia.

A nurse is assisting in developing a plan of care for a child admitted with a diagnosis of Kawasaki disease. In developing the initial plan of care, the nurse suggests that the child should be monitored for signs of:

1. Failure to thrive 2. Bleeding *3. Congestive heart failure (CHF)* 4. Decreased tolerance to stimulation *Rationale:* Nursing care for Kawasaki disease initially centers around observing for signs of CHF. The nurse monitors for increased respiratory rate, increased heart rate, dyspnea, lung congestion, and abdominal distention. Options 1, 2, and 4 are not findings directly associated with this disorder.

A mother of a 6-year-old-child calls a nurse who lives in the neighborhood and tells the nurse that her child accidentally rubbed waterproof sunscreen in his eyes. The nurse should tell the mother to immediately:

1. Flush the child's eyes for 15 minutes with water. 2. Have the child wipe the eyes with a wet towel. 3. Tell the child to blink continuously to get the sunscreen out of the eye. *4. Call the poison control center.* *Rationale:* Waterproof sunscreen should never be placed near the eyes. Waterproof sunscreen causes severe pain and a chemical burn that can damage the child's vision. Flushing the eyes with water does not stop the burning. The mother should be instructed to call the poison control center and to take the child to the emergency department. Special chemicals will be needed to flush the sunscreen out of the eyes and preserve vision. Wiping the eyes will increase the pain and burning. Blinking will not alleviate the pain or remove the sunscreen from the eyes.

A nurse is assigned to care for a 2-year-old child who has been admitted to the hospital for surgical correction of cryptorchidism. The highest priority in the postoperative plan of care for this child is to:

1. Force oral fluids. *2. Prevent tension on the suture.* 3. Test the urine for glucose. 4. Encourage coughing. *Rationale:* When a child returns from surgery, the testicle is held in position by an internal suture that passes through the testes and scrotum and is attached to the thigh. It is important not to dislodge this suture. Depending on the type of anesthesia used, option 4 may be appropriate but is not the priority. Although adequate hydration is important to maintain, fluids should not be forced. Testing urine for glucose is not related to this type of surgery.

A nurse is reviewing the health record of a child with a diagnosis of celiac disease. Which clinical manifestation should the nurse expect to note documented in the health record?

1. Frothy diarrhea *2. Profuse watery diarrhea and vomiting* 3. Foul-smelling ribbon stools 4. Diffuse abdominal pain unrelated to meals or activity *Rationale:* Celiac disease causes profuse watery diarrhea and vomiting. Option 1 is a clinical manifestation of lactose intolerance. Option 3 is a clinical manifestation of Hirschsprung's disease. Option 4 is a clinical manifestation of irritable bowel syndrome.

A mother of a child who underwent a myringotomy with insertion of tympanostomy tubes calls the nurse and reports that the child is complaining of discomfort. The nurse tells the mother to:

1. Give the child children's aspirin for the discomfort. 2. Be sure that the child is resuming normal activities. *3. Give the child acetaminophen (Tylenol) for the discomfort.* 4. Speak to the health care provider because the child should not be having any discomfort. *Rationale:* Following myringotomy with insertion of tympanostomy tubes, the child may experience some discomfort. Acetaminophen can be given to relieve the discomfort. Aspirin should not be administered to the child. The child should rest if discomfort is present.

A nurse is caring for a hospitalized child newly diagnosed with type 1 diabetes mellitus. At 11:00 AM, the child suddenly complains of weakness, headache, and blurred vision. The nurse should immediately:

1. Give the child ½ cup of orange juice to drink. *2. Obtain a blood glucose reading.* 3. Call the dietary department and ask that the lunch tray be delivered early. 4. Contact the health care provider. *Rationale:* The signs of hypoglycemia and hyperglycemia may be difficult to distinguish. Weakness, headache, and blurred vision can occur in either blood glucose alteration. A blood glucose reading will assist in confirming the diagnosis so that the appropriate action can be taken. Option 1 would be implemented if the child had hypoglycemia. Option 3 is inappropriate because the child should eat meals at basically the same time each day to achieve the best diabetic control. Contacting the health care provider would not be the immediate action; however, the nurse should inform the registered nurse of the situation.

A nurse caring for a client scheduled for a transsphenoidal hypophysectomy to remove a tumor in the pituitary gland assists to develop a plan of care for the client. The nurse suggests including which specific information in the preoperative teaching plan?

1. Hair will need to be shaved. 2. Deep breathing and coughing will be needed after surgery. *3. Toothbrushing will not be permitted for at least 2 weeks following surgery.* 4. Spinal anesthesia is used. *rationale* Based on the location of the surgical procedure, spinal anesthesia would not be used. In addition, the hair would not be shaved. Although coughing and deep breathing are important, specific to this procedure is avoiding toothbrushing to prevent disruption of the surgical site. Also, coughing may disrupt the surgical site.

When checking a child's glossopharyngeal nerve function, the nurse would perform which data collection technique?

1. Have child shrug the shoulders while applying mild pressure. 2. Have child follow a light in the six cardinal positions of gaze. *3. Test sense of sour or bitter taste on the posterior segment of the tongue.* 4. Test sense of sweet or salty taste on the anterior section of the tongue. *Rationale:* To test glossopharyngeal nerve function, the nurse would test the sense of sour or bitter taste on the posterior segment of the tongue. Option 1 is the data collection technique for checking the accessory nerve. Option 2 is the technique for checking the oculomotor nerve. Option 4 is the data collection technique for checking the facial nerve.

A nursing student is assigned to care for a child with hemophilia. The nursing instructor reviews the plan of care with the student and asks the student to describe the characteristics of this disorder. Which statement by the student indicates a need for further research?

1. Hemophilia is inherited in a recessive manner via a genetic defect on the X chromosome. *2. Males inherit hemophilia from their fathers.* 3. Females inherit the carrier status from their fathers. 4. Hemophilia A results from deficiency of factor VIII. *Rationale:* Males inherit hemophilia from their mothers, and females inherit the carrier status from their fathers. Some females who are carriers have an increased tendency to bleed, and, although it is rare, females can have hemophilia if their fathers have the disorder and their mothers are carriers of the genetic disorder. Hemophilia is inherited in a recessive manner via a genetic defect on the X chromosome. Hemophilia A results from a deficiency of factor VIII. Hemophilia B (Christmas disease) is a deficiency of factor IX.

A client is brought to the emergency department with suspected diabetic ketoacidosis (DKA). Which of the following findings would the nurse note as being consistent with this diagnosis?

1. High serum glucose level and an increase in pH 2. Low serum potassium and high serum bicarbonate level *3. High serum glucose level and low serum bicarbonate level* 4. Decreased urine output and Kussmaul's respirations *rationale* In DKA the blood glucose level is higher than 250 mg/dL, and ketones are present in the blood and urine. The arterial pH is low, less than 7.35. The plasma bicarbonate is also low. The client would exhibit polyuria and Kussmaul's respirations. The potassium level usually is elevated as a result of dehydration.

A comatose client with an admitting diagnosis of diabetic ketoacidosis (DKA) has a blood glucose of 368 mg/dL, arterial pH of 7.2, arterial bicarbonate of 14 mEq/L, and a positive for serum ketones. The diagnosis is supported by which noted data?

1. Hypertension *2. Fruity breath odor* 3. Slow regular breathing 4. Moist mucous membranes *rationale* Diabetic ketoacidotic coma is usually identified with a fruity breath odor, dry cracked mucous membranes, hypotension, and rapid deep breathing.

A health care provider has told the mother of a newborn diagnosed with strabismus that surgery will be necessary to realign the weakened eye muscles. The mother asks the nurse when the surgery might be performed. The nurse responds by telling the mother that surgery will probably be performed:

1. Immediately *2. Before the child is 3 years old* 3. Shortly before the child starts school 4. Just before the child begins to learn to read *Rationale:* In a child diagnosed with strabismus, surgery may be indicated to realign the weakened muscles. It is most often indicated when amblyopia (decreased vision in the deviated eye) is present. The surgery should be performed before the child is 3 years old.

A nurse is caring for a child with a suspected diagnosis of rheumatic fever (RF). The nurse reviews the laboratory results. Which laboratory study would assist in confirming the diagnosis of RF?

1. Immunoglobulin 2. Red blood cell count *3. Antistreptolysin O titer* 4. White blood cell count *Rationale:* A diagnosis of RF is confirmed by the presence of two major manifestations or one major and two minor manifestations from the Jones criteria. In addition, evidence of a recent streptococcal infection is confirmed by a positive antistreptolysin O titer, streptozyme, or an anti-DNase B assay. Options 1, 2, and 4 will not assist in confirming the diagnosis of RF.

A nurse is caring for a client experiencing thyroid storm. Which of the following would be a priority concern for this client?

1. Inability to cope with the treatment plan 2. Lack of sexual drive 3. Self-consciousness about body appearance *4. Potential for cardiac disturbances* *rationale* Clients in thyroid storm are experiencing a life-threatening event, which is associated with uncontrolled hyperthyroidism. It is characterized by high fever, severe tachycardia, delirium, dehydration, and extreme irritability. The signs and symptoms of the disorder develop quickly, and therefore emergency measures must be taken to prevent death. These measures include maintaining hemodynamic status and patency of airway as well as providing adequate ventilation. Options 1, 2, and 3 are not a priority in the care of the client in thyroid storm.

A nurse is developing a plan of care for a child with autism. The nurse identifies which of the following as the priority problem for this child?

1. Inability to interact socially *2. Risk for injury* 3. Troubling thought processes 4. Inability to verbally communicate *Rationale:* Risk for injury related to an inability to anticipate danger, a tendency for self-mutilation, and sensory perceptual deficits is the priority concern. Inability to interact socially, troubling thought processes, and inability to verbally communicate are also appropriate problems for the child with autism, but the priority is the risk for injury.

A client scheduled for a thyroidectomy says to the nurse, "I am so scared to get cut in my neck." Based on the client's statement, the nurse determines that the client is experiencing which problem?

1. Inadequate knowledge about the surgical procedure *2. Fear about impending surgery* 3. Embarrassment about the changes in personal appearance 4. Lack of support related to the surgical procedure *rationale* The client is having a difficult time coping with the scheduled surgery. The client is able to express fears but is scared. No data in the question support options 1, 3, and 4.

A nurse is caring for a postoperative parathyroidectomy client. Which of the following would require the nurse's immediate attention?

1. Incisional pain *2. Laryngeal stridor* 3. Difficulty voiding 4. Abdominal cramps *rationale* During the postoperative period, the nurse carefully observes the client for signs of hemorrhage, which cause swelling and the compression of adjacent tissue. Laryngeal stridor is a harsh, high-pitched sound heard on inspiration and expiration that is caused by the compression of the trachea and that leads to respiratory distress. It is an acute emergency situation that requires immediate attention to avoid the complete obstruction of the airway.

A 2-year-old child is diagnosed with constipation. Which of the following describes a characteristic of this disorder?

1. Incomplete development of the anus 2. Invagination of a section of the intestine into the distal bowel *3. The infrequent and difficult passage of dry stools* 4. The presence of fecal incontinence *Rationale:* Constipation can affect any child at any time, although its incidence peaks at ages 2 to 3 years. Option 4 describes encopresis, which can develop as a result of constipation and is one of the major concerns regarding constipation. Encopresis generally affects preschool and school-age children. Option 1 describes imperforate anus, which is diagnosed in the neonatal period. Option 2 describes intussusception, which is the most common cause of bowel obstruction in children ages 3 months to 6 years.

A nurse is caring for a client diagnosed with hyperparathyroidism who is prescribed furosemide (Lasix). The nurse reinforces dietary instructions to the client. Which of the following is an appropriate instruction?

1. Increase dietary intake of calcium. *2. Drink at least 2 to 3 L of fluid daily.* 3. Eat sparely when experiencing nausea. 4. Decrease dietary intake of potassium. *rationale* The aim of treatment in the client with hyperparathyroidism is to increase the renal excretion of calcium and decrease gastrointestinal absorption and bone resorption. This is aided by the sufficient intake of fluids. Dietary restriction of calcium may be used as a component of therapy. The parathyroid is responsible for calcium production, and the term, "hyperparathyroidism" can be indicative of an increase in calcium. The client should eat foods high in potassium, especially if the client is taking furosemide. Limiting nutrients is not advisable.

A mother of a child being treated for right lower lobe pneumonia at home calls the clinic nurse. The mother tells the nurse that the child has discomfort on the right side and that the acetaminophen (Tylenol) is not very effective. The appropriate suggestion by the nurse would be to:

1. Increase the dose of the acetaminophen. 2. Encourage the child to lie on the left side. *3. Encourage the child to lie on the right side.* 4. Increase the frequency of the acetaminophen. *Rationale:* Splinting of the affected side by lying on that side may decrease discomfort. It is inappropriate to advise the mother to increase the dose or frequency of the acetaminophen. Lying on the left side will not be helpful in alleviating discomfort.

A child suspected of sickle cell disease is seen in the clinic, and laboratory studies are performed. The nurse reviews the results of the laboratory studies and expects to note which of the following that is a characteristic of this disease?

1. Increased hematocrit count 2. Increased platelet count *3. Increased reticulocyte count* 4. Increased hemoglobin count *Rationale:* A laboratory diagnosis is established on the basis of a complete blood cell count, examination for sickled red blood cells (RBCs) on the peripheral smear, and hemoglobin electrophoresis. Laboratory studies will show decreased hemoglobin, hematocrit, and platelet count, increased reticulocyte count, and the presence of nucleated red blood cells. Elevated reticulocyte counts occur in children with sickle cell disease because the life span of their sickled RBCs is shortened.

A child is admitted to the pediatric unit with a diagnosis of coarctation of the aorta (COA). The health care provider prescribes that the child's blood pressure be taken every 4 hours in the legs and arms. The nurse would expect the blood pressure in the child's legs and arms to be:

1. Increased in the arms and the legs 2. Decreased in the arms and the legs *3. Decreased in the legs and increased in the arms* 4. Increased in the legs and decreased in the arms *Rationale:* Coarctation indicates a narrowing in the aorta. This would indicate an increased pressure proximal to the defect and a decreased pressure distal to the defect. This would result in a lower blood pressure in the legs and a higher blood pressure in the arms, which is indicated in option 3. Therefore options 1, 2, and 4 are incorrect.

A nurse is caring for a client following a thyroidectomy. The client tells the nurse that she is concerned because of voice hoarseness. The client asks the nurse whether the hoarseness will subside. The nurse appropriately tells the client that the hoarseness:

1. Indicates nerve damage 2. Is harmless but permanent 3. Will worsen before it subsides *4. Is normal and will gradually subside* *rationale* Hoarseness that develops in the postoperative period is usually the result of laryngeal pressure or edema and will resolve within a few days. The client should be reassured that the effects are transitory. Options 1, 2, and 3 are incorrect.

A nurse assists in preparing a plan of care for the infant with bladder exstrophy. The nurse identifies which of the following immediate problems as the priority for the infant?

1. Infection 2. Elimination *3. Skin disruption* 4. Lack of parental understanding *Rationale:* In bladder exstrophy, the bladder is exposed and external to the body. The highest priority is skin disruption related to the exposed bladder mucosa. Although the infant needs to be monitored for elimination patterns and kidney function, this is not the priority concern for this condition. Lack of parenteral understanding related to the diagnosis and treatment of the condition will need to be addressed, but again is not the priority. Although infection related to the anatomically located defect can be a problem, it is not the immediate one.

A male child who had surgery to correct hypospadias is seen in a health care provider's office for a well-baby check-up. The nurse provides instructions to the mother, knowing that which long-term complication is associated with hypospadias?

1. Infertility *2. Renal anomalies* 3. Erectile dysfunction 4. Decreased urinary output *Rationale:* The nurse should ask the child's parents about the child's kidney function because hypospadias may be associated with renal anomalies. The incorrect options are not associated with a long-term effect of hypospadias.

A nurse is reinforcing instructions to a client newly diagnosed with diabetes mellitus regarding insulin administration. The health care provider has prescribed a mixture of NPH and regular insulin. The nurse should stress that the first step is to:

1. Inject air equal to the amount of regular insulin prescribed into the vial of regular insulin. *2. Inject air equal to the amount of NPH insulin prescribed into the vial of NPH insulin.* 3. Draw up the correct dosage of regular insulin into the syringe. 4. Draw up the correct dosage of NPH insulin into the syringe. *rationale* The initial step in preparing an injection of insulin that is a mixture of NPH and regular is to inject air into the NPH bottle equal to the amount of insulin prescribed. The client is instructed to next inject an amount of air equal to the amount of prescribed insulin into the regular insulin bottle. The regular insulin should then be withdrawn followed by the NPH insulin. Contamination of regular insulin with NPH insulin will convert part of the regular insulin into a longer-acting form.

A 4-year-old child is being transported to the trauma center from a local community hospital for treatment of a burn injury that is estimated as covering over 40% of the body. The burns are partial- and full-thickness burns. The nurse is asked to prepare for the arrival of the child and gathers supplies anticipating that which of the following will be prescribed initially?

1. Insertion of a nasogastric tube *2. Insertion of a Foley catheter* 3. Administration of an anesthetic agent for sedation 4. Application of an antimicrobial agent to the burns *Rationale:* A Foley catheter is inserted into the child's bladder so that urine output can be measured accurately on an hourly basis. Although pain medication may be required, the child would not receive an anesthetic agent and should not be sedated. The burn wounds would be cleansed and treated after assessment, but this would not be the initial action. Intravenous fluids are administered at a rate sufficient to keep the child's urine output at 1 mL/kg of body weight per hour, thus reflecting adequate tissue perfusion. A nasogastric tube may or may not be required but would not be the priority intervention.

A nurse is reviewing the postoperative prescriptions for a client who had a transsphenoidal hypophysectomy. Which health care provider's prescription, if noted on the record, indicates the need for clarification?

1. Instruct the client about the need for a Medic-Alert bracelet. *2. Apply a loose dressing if any clear drainage is noted.* 3. Monitor vital signs and neurological status. 4. Instruct the client to avoid blowing the nose. *rationale* The nurse should observe for clear nasal drainage, constant swallowing, and a severe, persistent, generalized, or frontal headache. These signs and symptoms indicate cerebrospinal fluid leak into the sinuses. If clear drainage is noted following this procedure, the health care provider needs to be notified immediately. Options 1, 3, and 4 indicate appropriate postoperative interventions.

A nurse caring for an infant with bronchiolitis is monitoring for signs of dehydration. The nurse monitors which of the following as the reliable method of determining fluid loss?

1. Intake 2. Output 3. Skin turgor *4. Body weight* *Rationale:* Body weight is the most reliable method of measurement of body fluid loss or gain. One kilogram of weight change represents 1 L of fluid loss or gain. Although options 1, 2, and 3 may be used to determine fluid status, they are not the most reliable determinants.

A preoperative client is scheduled for adrenalectomy to remove a pheochromocytoma. The nurse would most closely monitor which of the following items in the preoperative period?

1. Intake and output 2. Blood urea nitrogen (BUN) *3. Vital signs* 4. Urine glucose and ketones *rationale* Hypertension is the hallmark of pheochromocytoma. Severe hypertension can precipitate a stroke or sudden blindness. Although all the items are appropriate nursing assessments for the client with pheochromocytoma, the priority is to monitor the vital signs, especially the blood pressure.

A nurse receives a call from the mother whose child has a foreign body in the eye. The object is clearly visible and not embedded. When the mother asks for the most effective way to get it out, the nurse responds:

1. Irrigate the eye with natural tears. 2. Irrigate the eye with running tap water. 3. Let the object just "work its way out" of the eye. *4. Touch the object gently with a cotton swab, and lift it out.* *Rationale:* The most effective method that would cause the least amount of trauma would be to lift the foreign body from the eye. It should not be allowed to remain and "work its way out." Irrigating the eye may cause the foreign body to move and cause trauma in another area of the eye.

A child with croup is being discharged from the hospital. The nurse provides home care instructions to the mother and advises the mother to bring the child to the emergency department if the child:

1. Is irritable 2. Appears tired *3. Develops stridor* 4. Takes fluids poorly *Rationale:* The mother should be instructed that if the child develops stridor at rest, cyanosis, severe agitation or fatigue, moderate to severe retractions, or is unable to take oral fluids, to bring the child to the emergency department.

A mother brings her child to the clinic because the child has developed a rash on the trunk and scalp. The child is diagnosed with varicella. The mother inquires about the infectious period associated with varicella, and the nurse tells the mother that the infectious period:

1. Is unknown *2. Is 1 to 2 days before the onset of the rash to 5 days after the onset of lesions and the crusting of lesions* 3. Is 10 days before the onset of symptoms to 15 days after the rash appears 4. Ranges from 2 weeks or less up to several months *Rationale:* Varicella is known as chickenpox. The infectious period for varicella is 1 to 2 days before the onset of the rash to 5 days after the onset of lesions and the crusting of lesions. In roseola, the infectious period is unknown. Option 3 describes rubella. Option 4 describes diphtheria.

A nurse is caring for a child with a diagnosis of Kawasaki disease. The mother of the child asks the nurse about the disorder. Which statement most accurately describes Kawasaki disease?

1. It is an acquired cell-mediated immunodeficiency disorder. *2. It is also called mucocutaneous lymph node syndrome and is a febrile generalized vasculitis of unknown cause.* 3. It is a chronic multi-system autoimmune disease characterized by the inflammation of connective tissue. 4. It is an inflammatory autoimmune disease that affects the connective tissue of the heart, joints, and subcutaneous tissues. *Rationale:* Kawasaki disease, also called mucocutaneous lymph node syndrome, is a febrile generalized vasculitis of unknown etiology. Option 1 describes human immunodeficiency virus (HIV) infection. Option 3 describes systemic lupus erythematosus. Option 4 describes rheumatic fever.

Oral iron is prescribed for a child with an iron deficiency anemia, and the nurse provides instructions to the mother regarding the administration of the iron. The nurse instructs the mother to administer the iron:

1. Just before a meal 2. Just after a meal *3. Between meals* 4. With a fruit low in vitamin C *Rationale:* The mother should be instructed to administer oral iron supplements between meals. The iron should be given with a citrus fruit or juice high in vitamin C because vitamin C increases the absorption of iron by the body.

A 1-year-old child is admitted to the hospital for control of tonic-clonic seizures. The nurse would do which of the following in order to protect the child from injury? *Select all that apply.*

1. Keep a padded tongue blade at the bedside for use during a seizure. *2. Remove toys that have bright, blinking lights on them.* *3. Keep side rails and other hard objects padded.* *4. Turn the client to the side during a seizure.* 5. Restrict the client's fluid intake. *Rationale:* Attempting to place something in a child's mouth during a seizure is not helpful even if it is padded. The mouth is usually clenched, and one would have to use force to open the mouth. One must attempt to keep the airway clear and can do that by positioning (option 4). Option 2 may be helpful in preventing a seizure, and option 3 safeguards the client's physical safety. Option 5 is not necessary.

An adolescent with diabetes mellitus becomes flushed and complains of hunger and dizziness. A blood glucose level is drawn and the results indicate a glucose level of 60 mg/dL. The appropriate intervention is to:

1. Keep the child NPO. 2. Contact the health care provider. *3. Give the child a glass of fruit juice.* 4. Let the child rest until the dizziness subsides. *Rationale:* A blood glucose less than 70 mg/dL indicates hypoglycemia. When signs of hypoglycemia occur, the child needs an immediate source of glucose. Options 1, 2, and 4 do not address the hypoglycemic condition.

A child with cerebral palsy (CP) is working to achieve maximum potential for locomotion, self-care, and socialization in school. The nurse would work with the child to meet these goals by:

1. Keeping the child in a special education classroom with other children with similar disabilities 2. Laying the child in the supine position with a 30-degree elevation of the head to facilitate feeding 3. Removing ankle-foot orthoses and braces once the child arrives at school *4. Placing the child on a wheeled scooter board* *Rationale:* Option 4 provides the child with maximum potential in locomotion, self-care, and socialization. The child can move around independently on the abdomen anywhere the child wants to go and can interact with others as desired. Orthoses must be used all the time to aid locomotion (option 3). Option 1 does not provide for maximum socialization and normalization; rather, children with CP need to be mainstreamed as much as cognitively able. Not all children with CP are intellectually challenged. Option 2 does not provide for normalization in self-care. Just as children without CP sit up and use assistive devices when eating, so should children with CP.

A nurse is observing a student preparing to suction a pediatric client through a tracheostomy. The nurse intervenes if the student verbalizes to:

1. Limit insertion and suctioning time to 5 seconds. 2. Reoxygenate the child between suction catheter passes. *3. Apply continuous suction when inserting the catheter.* 4. Use a twisting motion on the catheter when withdrawing the catheter. *Rationale:* The nurse would not use continuous suction on the catheter during insertion; suction is applied only when withdrawing the catheter. Options 1, 2, and 4 represent correct interventions regarding this procedure.

A client who is managing diabetes mellitus with insulin injections asks the nurse for information about any necessary changes in her diet to avoid hyperinsulinism. Which of the following diets would be appropriate for the client?

1. Low-fiber, high-fat diet 2. Limit carbohydrate intake to three meals per day 3. Large amounts of carbohydrates between low protein meals *4. Small frequent meals with protein, fat, and carbohydrates at each meal* *rationale* The definition of hyperinsulinism is an excessive insulin secretion in response to carbohydrate-rich foods leading to hypoglycemia. It is often treated with a diet that provides for limited stimulation of the pancreas. Carbohydrates can produce a rapid rise in blood glucose levels. However, carbohydrates are necessary in the diet. Proteins do not stimulate insulin secretion. Fats are needed in the diet to provide calories. The best diet for hyperinsulinism will contain proteins and fats whenever carbohydrates are consumed and delivered in frequent but portion-controlled meals. Diets high in soluble fiber may be beneficial.

A nurse is preparing to provide instructions to a client with Addison's disease regarding diet therapy. The nurse understands that which of the following diets would likely be prescribed for this client?

1. Low-protein diet 2. Low-sodium diet *3. High-sodium diet* 4. Low-carbohydrate diet *rationale* A high-sodium, high-complex carbohydrate, and high-protein diet will be prescribed for the client with Addison's disease. To prevent excess fluid and sodium loss, the client is instructed to maintain an adequate salt intake of up to 8 g of sodium daily and to increase salt intake during hot weather, before strenuous exercise, and in response to fever, vomiting, or diarrhea.

A nurse is caring for a hospitalized infant and is monitoring for increased intracranial pressure (ICP). The nurse notes that the anterior fontanel bulges when the infant cries. Based on this finding, which of the following actions would the nurse take?

1. Lower the head of the bed. *2. Document the findings.* 3. Place the infant on nothing-by-mouth (NPO) status. 4. Ask the registered nurse to notify the health care provider immediately. *Rationale:* The anterior fontanel is diamond shaped and located on the top of the head. It should be soft and flat in a normal infant, and it normally closes by 18 to 24 months of age. The posterior fontanel closes by 2 to 3 months of age. A bulging or tense fontanel may result from crying or increased ICP. If the nurse notes a bulging fontanel when the infant cries, this is a normal finding that should be documented and monitored. It is not necessary to notify the health care provider for this finding. Options 1 and 3 are inappropriate actions.

A nurse is reinforcing instructions to the mother of a preschool child who was recently diagnosed with pediculosis capitis (head lice). Which item should be included in discussions to prevent a reinfestation?

1. Machine wash all of the child's clothing, towels, and bed linens, and place in a warm dryer for at least 20 minutes. 2. Shave the child's hair if pediculicide and nit-removal combs prove ineffective. 3. Spray the home's furniture and beds with insecticide. *4. Seal nonwashable items in a plastic bag for 2 to 3 weeks in a warm place if they cannot be vacuumed or dry cleaned.* *Rationale:* The adult louse can survive up to 48 hours away from a host, although nits can hatch in 7 to 10 days if they are shed into the environment. Thus, 2 weeks represents a safe interval of time that prevents reinfestation from occurring. Hot water and hot air should be used in the washer and dryer. Shaving the hair is unnecessary with proper treatment and would have an adverse psychological effect on the child. Insecticides can endanger children and animals and should not be sprayed on furniture and beds.

A nurse reviews a plan of care for a postoperative client following a thyroidectomy and notes that the client is at risk for breathing difficulty. Which of the following nursing interventions will the nurse suggest to include in the plan of care?

1. Maintain a supine position. 2. Encourage coughing and deep breathing exercises. *3. Monitor neck circumference frequently.* 4. Maintain a pressure dressing on the operative site. *rationale* Following a thyroidectomy, the client should be placed in an upright position to facilitate air exchange. The nurse should assist the client with deep breathing exercises, but coughing is minimized to prevent tissue damage and stress to the incision. A pressure dressing is not placed on the operative site because it could affect breathing. The nurse should monitor the dressing closely and should loosen the dressing if necessary. Neck circumference is monitored at least every 4 hours to assess for postoperative edema.

A nurse is reviewing the laboratory results of an infant suspected of having hypertrophic pyloric stenosis. Which of the following would the nurse expect to note in this infant?

1. Metabolic acidosis *2. Metabolic alkalosis* 3. Respiratory acidosis 4. Respiratory alkalosis *Rationale:* Laboratory findings in an infant with hypertrophic pyloric stenosis include metabolic alkalosis as a result of the vomiting (depletes acid) that occurs in this disorder. Additional findings include decreased serum potassium and sodium levels, increased pH and bicarbonate, and decreased chloride level.

A 3-year-old child is brought to the emergency department. The mother states that the child has had flulike symptoms with vomiting and diarrhea for the past 2 days. On data collection the nurse finds that the child's heart rate is slightly elevated and the blood pressure is normal. The child is irritable and crying only a few tears. The mother states that the child's weight before the illness was 33 pounds. The nurse finds the current weight to be 31 pounds. The nurse correctly interprets this as what level of dehydration?

1. Mild dehydration 2. Severe dehydration 3. Very mild dehydration *4. Moderate dehydration* *Rationale:* Moderate dehydration demonstrates itself with a weight loss in children of 6% to 8% of weight. Mild dehydration would not present with these symptoms. In severe dehydration, additional findings would include lethargy and listlessness. The symptoms listed are all characteristics of moderate dehydration. Very mild dehydration is not a term used to describe dehydration.

Following tonsillectomy, which of the health care provider's prescriptions would the nurse question?

1. Monitor vital signs. 2. Monitor for bleeding. *3. Allow ice cream when awake.* 4. Offer clear, cool liquids when awake. *Rationale:* Clear, cool liquids are encouraged. Milk and milk products are avoided initially because they coat the throat, which causes the child to clear the throat, increasing the risk of bleeding. Options 1 and 2 are important nursing interventions following any type of surgery.

A nursing student is asked to discuss human immunodeficiency virus (HIV) during clinical conference. The nursing student includes which correct item in the discussion?

1. Most newborns of HIV-positive women test positive for HIV virus. 2. HIV primarily attacks the hematological system. 3. In HIV, the B cells are depleted and cannot signal T4 cells to form protective antibodies. *4. HIV virus attacks the immune system by destroying T lymphocytes.* *Rationale:* Children born to HIV-positive women test positive for HIV antibody, not HIV virus. This is actually a measure of maternal antibody and not indicative of true infection. T4 cells are depleted in number and cannot signal B cells to form protective antibodies to fight off the invading virus. The virus attacks the immune system by destroying T lymphocytes.

A client is brought to the emergency department in an unresponsive state, and a diagnosis of hyperglycemic hyperosmolar nonketotic syndrome (HHNS) is made. The nurse who is assisting to care for the client obtains which of the following immediately in preparation for the treatment of this syndrome?

1. NPH insulin 2. A nasal cannula 3. Intravenous (IV) infusion of sodium bicarbonate *4. IV infusion of normal saline* *rationale* The primary goal of treatment is to rehydrate the client to restore fluid volume and to correct electrolyte deficiency. IV fluid replacement is similar to that administered in diabetic ketoacidosis (DKA) and begins with IV infusion of normal saline. Regular insulin, not NPH insulin would be administered. The use of sodium bicarbonate to correct acidosis is avoided because it can precipitate a further drop in serum potassium levels. A nasal cannula for oxygen administration is not necessarily required to treat HHNS.

An adult client just admitted to the hospital with heart failure also has a history of diabetes mellitus. The nurse calls the health care provider to verify a prescription for which medication that the client was taking before admission?

1. NPH insulin 2. Regular insulin 3. Acarbose (Precose) *4. Chlorpropamide* *rationale* Chlorpropamide is an oral hypoglycemic agent that exerts an antidiuretic effect and should be administered cautiously or avoided in the client with cardiac impairment or fluid retention. It is a first-generation sulfonylurea. Insulin does not cause or aggravate fluid retention. Acarbose is a miscellaneous oral hypoglycemic agent.

A nurse is reinforcing information to parents regarding the signs of meningitis. The nurse informs the parents that the primary signs of meningitis include:

1. Nausea and delirium 2. Anorexia and back pain 3. Night blindness and confusion *4. Severe headache and neck stiffness* *Rationale:* The primary signs of meningitis include severe headache, fever, stiff neck, and a change in the level of consciousness. Although nausea, confusion, delirium, and back pain may occur in meningitis, these are not the classic signs. Night blindness is not related to meningitis.

A 3-year-old child is seen in the health care clinic, and a diagnosis of encopresis is made. The nurse reviews the record, expecting to note which of the following that is a sign of this disorder?

1. Nausea and vomiting 2. Diarrhea *3. Evidence of soiled clothing* 4. Malaise anorexia *Rationale:* Encopresis is defined as fecal incontinence and is a major concern if the child is constipated. Signs include evidence of soiling clothing, scratching, or rubbing the anal area because of irritation, fecal odor without apparent awareness by the child, and social withdrawal.

A client newly diagnosed with diabetes mellitus takes NPH insulin every day at 7:00 ᴀᴍ. The nurse has taught the client how to recognize the signs of hypoglycemia. The nurse determines that the client understands the information presented if the client watches for which of the following signs in the late afternoon?

1. Nausea and vomiting, and abdominal pain *2. Hunger; shakiness; and cool, clammy skin* 3. Drowsiness; red, dry skin; and fruity breath odor 4. Increased urination; thirst; and rapid, deep breathing *rationale* The client taking NPH insulin obtains peak medication effects approximately 6 to 12 hours after administration. At the time that the medication peaks, the client is at risk of hypoglycemia if food intake is insufficient. The nurse should teach the client to watch for signs and symptoms of hypoglycemia including anxiety, confusion, difficulty concentrating, blurred vision, cold sweating, headache, increased pulse, shakiness, and hunger. The other options list various signs and symptoms of hyperglycemia.

A client with Cushing's disease is being admitted to the hospital after a stab wound to the abdomen. The nurse plans care and places highest priority on which potential problem?

1. Nervousness *2. Infection* 3. Concern about appearance 4. Inability to care for self *rationale* The client with a stab wound has a break in the body's first line of defense against infection. The client with Cushing's disease is at great risk for infection because of excess cortisol secretion and subsequent impaired antibody function and decreased proliferation of lymphocytes. The client may also have a potential for the problems listed in the other options but these are not the highest priority at this time.

A client has a blood glucose level drawn for suspected hyperglycemia. After interviewing the client, the nurse determines that the client ate lunch approximately 2 hours before the blood specimen was drawn. The laboratory reports that the blood glucose to be 180 mg/dL, and the nurse analyzes this result to be:

1. Normal 2. Lower than the normal value *3. Elevated from the normal value* 4. A dangerously high value requiring immediate health care provider notification *rationale* Normal fasting blood glucose values range from 70 to 120 mg/dL. A 2-hour postprandial blood glucose level should be less than 140 mg/dL. In this situation, the blood glucose value was 180 mg/dL 2 hours after the client ate, which is an elevated value as compared to normal. Although the result may be reported to the health care provider, it is not a dangerously high one.

A client with diabetes mellitus calls the clinic nurse to report that the blood glucose level is 150 mg/dL. After obtaining further data from the client, the nurse determines that the client ate lunch approximately 2 hours ago. The nurse would interpret these results to be:

1. Normal 2. Lower than the normal value *3. Slightly higher than the normal value* 4. A value that indicates immediate health care provider notification *rationale* Normal fasting blood glucose values range from 70 to 120 mg/dL. The 2-hour postprandial blood glucose level should be less than 140 mg/dL. In this situation, the blood glucose value was 150 mg/dL 2 hours after the client ate, which is slightly elevated above normal. This value does not require health care provider notification.

A nursing student is asked to discuss the pathophysiology related to childhood leukemia during a clinical conference and reviews the planned presentation with the nursing instructor. The nursing instructor advises the student to review the disorder before the clinical conference if the student states that which of the following is associated with this type of cancer?

1. Normal bone marrow is replaced by blast cells. 2. Red blood cells (RBCs) and platelet production become affected. 3. The reticuloendothelial system is affected. *4. Reed-Sternberg cells are found on biopsy.* *Rationale:* In leukemia, normal bone marrow is replaced by malignant blast cells. As the blast cells take over the bone marrow, eventually RBC and platelet production is affected and the child becomes anemic and thrombocytopenic. The reticuloendothelial system is affected, thus disturbing the body's defense system and rendering these children unable to fight infections normally. The Reed-Sternberg cell is found in Hodgkin's disease.

A nurse is assigned to assist in caring for a newborn with a colostomy that was created during surgical intervention for imperforate anus. When the newborn returns from surgery, the nurse checks the stoma and notes that it is red and edematous. Which of the following is the appropriate nursing intervention?

1. Notify the registered nurse immediately. *2. Document the findings.* 3. Apply ice immediately. 4. Elevate the buttocks. *Rationale:* A fresh colostomy stoma will be red and edematous but this will decrease with time. The colostomy site will then be pink without evidence of abnormal drainage, swelling, or skin breakdown. The nurse would document these findings because this is a normal expectation. Options 1, 3, and 4 are inappropriate interventions.

A nurse checks the vital signs of an infant with a respiratory infection and notes that the respiratory rate is 50 breaths per minute. Which action is appropriate?

1. Notify the registered nurse. 2. Administer oxygen. 3. Recheck the respiratory rate in 15 minutes. *4. Document the findings.* *Rationale:* The normal respiratory rate in an infant is 30 to 60 breaths per minute. The nurse would document the findings.

A nurse is assigned to care for a child with a spica cast. Which action should be avoided when caring for the child?

1. Observing for nonverbal signs of pain *2. Using pillows to elevate the head and shoulders* 3. Checking neurovascular status of the extremities 4. Placing the child on a stretcher and bringing the child to the playroom *Rationale:* Pillows should not be used to elevate the head or shoulders of a child in a body cast because the pillows will thrust the child's chest against the cast and cause discomfort and respiratory difficulty. Neurovascular checks are a critical component of care to ensure that the cast is not causing circulatory compromise. The nurse should observe for nonverbal signs of pain and ask the older child if pain is experienced. A ride on a stretcher to the playroom or around the hospital provides changes of position and scenery.

A male client recently diagnosed with diabetes mellitus requiring insulin tells the clinic nurse that he is traveling by air throughout the next week. The client asks the nurse for any suggestions about managing the disorder while traveling. The nurse tells the client to:

1. Obtain referrals to health care providers in the destination cities. 2. Check the blood glucose every 2 hours during the flight. *3. Keep snacks in carry-on luggage to prevent hypoglycemia during the flight.* 4. Pad the insulin and syringes against breakage and place in a suitcase to be stowed. *rationale* A frequent concern of diabetics during air travel is the availability of food at times that correspond with the timing and peak action of the client's insulin. For this reason, the nurse may suggest that the client have carbohydrate snacks on hand. Insulin equipment and supplies should always be placed in carry-on luggage (not stowed). This provides ready access to treat hyperglycemia, if needed, and prevents loss of equipment if luggage is lost. Options 1 and 2 are unnecessary.

A nurse determines that a child with type 1 diabetes mellitus is having a hypoglycemic reaction. The nurse should give the child which of the following to treat the reaction?

1. One sugar cube 2. 1 teaspoon of sugar 3. ½ cup of diet cola *4. ½ cup of fruit juice* *Rationale:* Hypoglycemia is immediately treated with 10 to 15 g of carbohydrate. Glucose tablets or glucose gel may be administered. Other items used to treat hypoglycemia include ½ cup of fruit juice, ½ cup of regular (nondiet) soft drink, 8 ounces of skim milk, 6 to 10 hard candies, 4 cubes of sugar or 4 teaspoons of sugar, 6 saltines, 3 graham crackers, or 1 tablespoon of honey or syrup. The items in options 1, 2, and 3 would not adequately treat hypoglycemia.

A mother arrives at the clinic with her child. The mother tells the nurse that the child has had a fever and a cough for the past 2 days, and this morning the child began to wheeze. Viral pneumonia is diagnosed. Which of the following would the nurse anticipate to be a component of the treatment plan?

1. Oral antibiotics *2. Supportive treatment* 3. Hospitalization and antibiotics 4. Intravenous (IV) fluid administration *Rationale:* With viral pneumonia, treatment is supportive. More severely ill children may be hospitalized and given oxygen, chest physiotherapy, and IV fluids. Antibiotics are not given. Bacterial pneumonia, however, is treated with antibiotic therapy.

A nursing instructor asks the nursing student to plan and conduct a clinical conference on phenylketonuria (PKU). The student researches the topic and plans to include which of the following in the conference?

1. PKU is an autosomal dominant disorder. *2. PKU results in central nervous system (CNS) damage.* 3. Some state laws require routine screening of all newborn infants for PKU. 4. Treatment includes dietary restriction of sodium. *Rationale:* PKU is an autosomal recessive disorder. Treatment includes dietary restriction of phenylalanine intake (not sodium). PKU is a genetic disorder that results in CNS damage from toxic levels of phenylalanine in the blood. All 50 states require routine screening of all newborn infants for PKU.

A nurse reviews the nursing care plan of an older client with diabetic neuropathy of the lower extremities as a result of type 2 diabetes mellitus. The nurse plans care, knowing that which problem has the highest priority for this client?

1. Pain as a result of intermittent claudication 2. Lack of self-confidence as a result of impaired ability to walk 3. Lack of self-esteem as a result of perceived loss of abilities *4. The possibility of injury as a result of decreased sensation in the legs and feet* *rationale* The client with diabetic neuropathy of the lower extremities has diminished ability to feel sensations in the legs and feet. This client is at risk for tissue injury and for falls as a result of this nervous system impairment. Thus the highest priority problem is option 4, which can be determined using Maslow's Hierarchy of Needs theory. Options 2 and 3 represent problems that are more psychosocial in nature, and as such are secondary needs using Maslow's theory. Option 1 is incorrect because intermittent claudication is not directly associated with diabetic neuropathy.

A nurse is assisting in planning discharge instructions to the mother of a child following orchiopexy, which was performed on an outpatient basis. Which of the following is the priority in the plan of care?

1. Pain control measures 2. Measurement of intake *3. Wound care* 4. Cold and heat applications *Rationale:* The most common complications associated with orchiopexy are bleeding and infection. Discharge instruction should include demonstration of proper wound cleansing and dressing and teaching parents to identify signs of infection such as redness, warmth, swelling, or discharge. Testicles will be held in a position to prevent movement, and great care should be taken to prevent contamination of the suture line. Analgesics may be prescribed but are not the priority, considering the options presented. Option 2 is not necessary. Option 4 is not a prescribed treatment measure.

A child is to be admitted to the orthopedic unit following a Harrington rod insertion for the treatment of scoliosis. The nurse is assisting in preparing a plan of care for the child. The nurse plans to monitor which priority item in the immediate postoperative period?

1. Pain level *2. Capillary refill, sensation, and motion in all extremities* 3. Ability to turn using the logroll technique 4. Ability to flex and extend the lower extremities *Rationale:* When the spinal column is manipulated during surgery, altered neurovascular status is a possible complication; therefore neurovascular assessments including circulation, sensation, and motion should be done every 2 hours. Level of pain and ability to flex and extend the lower extremities are important postoperative assessments but not the priorities of the options provided. Logrolling would be performed by nurses.

A nurse is asked to prepare for the admission of a child to the pediatric unit with a diagnosis of Wilms' tumor. The nurse assists in developing a plan of care for the child and suggests including which of the following in the plan of care?

1. Palpate the abdomen for an increase in the size of the tumor every 8 hours. *2. Inspect the urine for the presence of hematuria at each voiding.* 3. Monitor the temperature for hypothermia. 4. Monitor the blood pressure for hypotension. *Rationale:* If Wilms' tumor is suspected, the tumor mass should not be palpated. Excessive manipulation can cause seeding of the tumor and cause spread of the cancerous cells. Fever (not hypothermia), hematuria, and hypertension (not hypotension) are clinical manifestations associated with Wilms' tumor.

Which test would the nurse anticipate for a teenage client who has been treated for vaginal candida repeatedly in the last 6 months to assist in the identification of the underlying chronic pathology?

1. Pap smear 2. Blood culture 3. Throat culture *4. Blood glucose level* *Rationale:* A blood glucose level is an indicator of diabetes mellitus. In females, monilial infections of the genitourinary tract are a common manifestation of diabetes mellitus. Pap smears are specific for detecting cancer of the cervix. A throat culture may show a candidal infection, but this test is unrelated to an undiagnosed underlying chronic disease. An infection of the blood (diagnosed by a blood culture) is indicative of an acute systemic disease.

A nurse is caring for a client with hypothyroidism who is overweight. Which food items would the nurse suggest to include in the plan?

1. Peanut butter, avocado, and red meat *2. Skim milk, apples, whole-grain bread, and cereal* 3. Organ meat, carrots, and skim milk 4. Seafood, spinach, and cream cheese *rationale* Clients with hypothyroidism may have a problem with being over-weight because of their decreased metabolic need. They should consume foods from all food groups, which will provide them with the necessary nutrients; however, the foods should be low in calories. Option 2 is the only option that identifies food items that are low in calories.

A nurse is caring for a postoperative adrenalectomy client. Which of the following does the nurse specifically monitor for in this client?

1. Peripheral edema 2. Bilateral exophthalmos 3. Signs and symptoms of hypocalcemia *4. Signs and symptoms of hypovolemia* *rationale* Following adrenalectomy, the client is at risk for hypovolemia. Aldosterone, secreted by the adrenal cortex, plays a major role in fluid volume balance by retaining sodium and water. A deficiency of adrenocortical hormones does not cause the clinical manifestations noted in options 1, 2, and 3.

A preschool child who was admitted to the hospital for a minor surgery develops a rash on the second day after hospitalization and is diagnosed with chickenpox (varicella). The nurse should take which of the following actions to provide safety for all children on the unit?

1. Place only the infected child in isolation. 2. Keep siblings from visiting the infected child. 3. Place the child and any other child who were exposed in isolation. *4. Place the infected child and any immunocompromised children in isolation.* *Rationale:* The period of communicability for chickenpox is 1 day before the eruption of vesicles to about 1 week when crusts are formed. The infected child should be isolated until vesicles have dried, and other high-risk children (immunocompromised) should be isolated from the infected client.

A health care provider prescribes "eye patching" for a child with strabismus of the right eye. The nurse instructs the mother regarding this procedure and tells the mother to:

1. Place the patch on both eyes. *2. Place the patch on the left eye.* 3. Place the patch on the right eye. 4. Alternate the patch from the right to left eye hourly. *Rationale:* Eye patching may be used in the treatment of strabismus to strengthen the weak eye. In this treatment, the "good" eye is patched. This encourages the child to use the weaker eye. It is most successful when done during the preschool years. The schedule for patching is individualized and is prescribed by the ophthalmologist.

An infant returns to the nursing unit following surgery for an esophageal atresia with tracheoesophageal fistula (TEF). The infant is receiving intravenous (IV) fluids, and a gastrostomy tube is in place. The nurse who is assisting in caring for the infant will ensure that the gastrostomy tube is:

1. Placed to gravity 2. Attached to low suction 3. Taped to the bed linens *4. Elevated* *Rationale:* In the immediate postoperative period, the gastrostomy tube is elevated, allowing gastric contents to pass to the small intestine and air to escape. This promotes comfort and decreases the risk of leakage at the anastomosis. Options 1, 2, and 3 are incorrect.

A child suspected of having sickle cell disease (SCD) is seen in a clinic, and laboratory studies are performed. Which laboratory value is likely to be increased in sickle cell disease?

1. Platelet count 2. Hematocrit level 3. Hemoglobin level *4. Reticulocyte count* *Rationale:* A diagnosis is established on the basis of a complete blood count, examination for sickled red blood cells (RBCs) in the peripheral smear, and hemoglobin electrophoresis. Laboratory studies will show decreased hemoglobin and hematocrit levels and a decreased platelet count, an increased reticulocyte count, and the presence of nucleated red blood cells. Increased reticulocyte counts occur in children with SCD because the life span of their sickled RBCs is shortened.

A nurse reinforces teaching with a client with diabetes mellitus regarding differentiating between hypoglycemia and ketoacidosis. The client demonstrates an understanding of the teaching by stating that glucose will be taken if which symptom develops?

1. Polyuria *2. Shakiness* 3. Blurred vision 4. Fruity breath odor *rationale* Shakiness is a sign of hypoglycemia, and it would indicate the need for food or glucose. Fruity breath odor, blurred vision, and polyuria are signs of hyperglycemia.

A nurse employed in the emergency department is collecting data on a 7-year-old child with a fractured arm. The child is hesitant to answer questions that the nurse is asking and consistently looks at the parents in a fearful manner. The nurse suspects physical abuse and continues with the data collection procedures. Which of the following findings would most likely assist in verifying the suspicion?

1. Poor hygiene *2. Bald spots on the scalp* 3. Lacerations in the anal area 4. Swelling of the genitals *Rationale:* Bald spots on the scalp are most likely to be associated with physical abuse. The most likely findings in sexual abuse include difficulty walking or sitting; torn, stained, or bloody underclothing; pain; swelling or itching of the genitals; and bruises, bleeding, or lacerations in the genital or anal area. Poor hygiene may be indicative of physical neglect.

The mother of a child arrives at the clinic because the child has been experiencing scratchy, red, and swollen eyes. The nurse notes a discharge from the eyes and a culture is sent to the laboratory for analysis. Chlamydial conjunctivitis is diagnosed. Based on this diagnosis, which of the following would require further investigation?

1. Possible trauma *2. Possible sexual abuse* 3. The presence of an allergy 4. The presence of a respiratory infection *Rationale:* A diagnosis of chlamydial conjunctivitis in a non-sexually active child should signal the health care provider to assess the child for possible sexual abuse. Allergy, infection, and trauma can cause conjunctivitis but not chlamydial conjunctivitis.

A client is in metabolic acidosis caused by diabetic ketoacidosis (DKA). The nurse prepares for the administration of which of the following medications as a primary treatment for this problem?

1. Potassium *2. Regular insulin* 3. Sodium bicarbonate 4. Calcium gluconate *rationale* The primary treatment for any acid-base imbalance is treatment of the underlying disorder that caused the problem. In this case, the underlying cause of the metabolic acidosis is anaerobic metabolism as a result of the lack of ability to use circulating glucose. Administration of regular insulin corrects this problem.

A child with diabetes mellitus is brought to the emergency department by her mother, who states that her daughter has been complaining of abdominal pain and has a fruity odor on the breath. Diabetic ketoacidosis (DKA) is diagnosed. The nurse assisting to care for the child checks the intravenous (IV) and medication supply area for which of the following?

1. Potassium 2. NPH insulin 3. 5% dextrose IV infusion *4. 0.9% normal saline IV infusion* *Rationale:* Rehydration is the initial step in resolving DKA. Normal saline is the initial IV rehydration fluid. NPH insulin is never administered by the IV route. Dextrose solutions are added to the treatment when the blood glucose levels reach an acceptable level. IV potassium may be required depending on the potassium level, but would not be part of the initial treatment.

A nurse enters the room of a client with type 1 diabetes mellitus and finds the client difficult to arouse. The client's skin is warm and flushed, and the pulse and respiratory rate are elevated from the client's baseline. The nurse would immediately:

1. Prepare for the administration of an insulin drip. 2. Give the client a glass of orange juice. 3. Prepare for the administration of a bolus dose of 50% dextrose. *4. Check the client's capillary blood glucose.* *rationale* The nurse must first obtain a blood glucose reading to determine the client's problem. Options 2 and 3 would be implemented as needed in the treatment of hypoglycemia. Insulin therapy is guided by blood glucose measurement.

When caring for a client diagnosed with pheochromocytoma, what information should the nurse know when assisting with planning care?

1. Profound hypotension may occur. *2. Excessive catecholamines are released.* 3. The condition is not curable and is treated symptomatically. 4. Hypoglycemia is the primary presenting symptom. *rationale* Pheochromocytoma is a catecholamine-producing tumor of the adrenal gland and causes secretion of excessive amounts of epinephrine and norepinephrine. Hypertension is the principal manifestation, and the client has episodes of a high blood pressure accompanied by pounding headaches. The excessive release of catecholamine also results in excessive conversion of glycogen into glucose in the liver. Consequently, hyperglycemia and glucosuria occur during attacks. Pheochromocytoma is curable. The primary treatment is surgical removal of one or both of the adrenal glands, depending on whether the tumor is unilateral or bilateral.

A nurse is assisting in admitting to the hospital a 4-month-old infant with a diagnosis of vomiting and dehydration. The nurse assists in developing a plan of care for the infant and suggests including in the plan to position the infant in a(n):

1. Prone position *2. Side-lying position* 3. Modified Trendelenburg's position 4. Infant car seat with the head of the seat in a flat position *Rationale:* The vomiting infant or child should be placed in an upright or side-lying position to prevent aspiration. The positions identified in options 1, 3, and 4 will increase the risk of aspiration if vomiting occurs.

A nurse assists the health care provider in performing a lumbar puncture on a 3-year-old child with leukemia suspected of central nervous system (CNS) disease. In which position will the nurse place the child during this procedure?

1. Prone with knees flexed to the abdomen and head bent with chin resting on the chest 2. Modified Sims' position *3. Lateral recumbent with the knees flexed to the abdomen and head bent with the chin resting on the chest* 4. Lithotomy position *Rationale:* A lateral recumbent with the knees flexed to the abdomen and the head bent with the chin resting on the chest is assumed for a lumbar puncture. This position separates the spinal processes and facilitates needle insertion into the subarachnoid space. Options 1, 2, and 4 are incorrect positions.

A nurse would expect to note which interventions in the plan of care for a client with hypothyroidism? *Select all that apply.*

1. Provide a cool environment for the client. 2. Instruct the client to consume a high-fat diet. *3. Instruct the client about thyroid replacement therapy.* *4. Encourage the client to consume fluids and high-fiber foods in the diet.* *5. Instruct the client to contact the health care provider if episodes of chest pain occur.* 6. Inform the client that iodine preparations will be prescribed to treat the disorder. *rationale* The clinical manifestations of hypothyroidism are the result of decreased metabolism from low levels of thyroid hormone. Interventions are aimed at replacement of the hormones and providing measures to support the signs and symptoms related to a decreased metabolism. The nurse encourages the client to consume a well-balanced diet that is low in fat for weight reduction and high in fluids and high-fiber foods to prevent constipation. The client often has cold intolerance and requires a warm environment. The client would notify the health care provider if chest pain occurs since it could be an indication of overreplacement of thyroid hormone. Iodine preparations are used to treat hyperthyroidism. These medications decrease blood flow through the thyroid gland and reduce the production and release of thyroid hormone.

What would the nurse anticipate being included in the plan of care for a client who has been diagnosed with Graves' disease?

1. Provide a high-fiber diet. *2. Provide a restful environment.* 3. Provide three small meals per day. 4. Provide the client with extra blankets. *rationale* Because of the hypermetabolic state, the client with Graves' disease needs to be provided with an environment that is restful both physically and mentally. Six full meals a day that are well balanced and high in calories are required, because of the accelerated metabolic rate. Foods that increase peristalsis (e.g., high-fiber foods) need to be avoided. These clients suffer from heat intolerance and require a cool environment.

A nurse is assisting in preparing a plan of care for a 4-year-old child hospitalized with nephrotic syndrome. Which dietary intervention is most appropriate for this child?

1. Provide a high-salt diet. 2. Provide a high-protein diet. 3. Discourage visitors at mealtimes. *4. Encourage the child to eat in the playroom.* *Rationale:* Mealtimes should center on pleasurable socialization. The child should be encouraged to eat meals with other children on the unit. A diet that is normal in protein with a sodium restriction is normally prescribed for a child with nephrotic syndrome. Parents or other family members should be encouraged to be present at mealtimes with a hospitalized child.

A nurse has collected data on a client with diabetes mellitus. Findings include a fasting blood glucose of 130 mg/dL, temperature 101° F, pulse of 88 beats per minute, respirations of 22 breaths per minute, and a blood pressure of 118/78 mm Hg. Which finding would be of concern to the nurse?

1. Pulse and respirations 2. Blood pressure 3. Blood glucose *4. Temperature* *rationale* Elevated temperature may be indicative of infection, which is a leading cause of hyperglycemic hyperosmolar nonketotic syndrome (HHNS) or diabetic ketoacidosis (DKA). Options 1, 2, and 3 are findings that are within a normal range.

A 3-year-old child has returned to his room following a tonsillectomy. Which assessment finding needs immediate notification of the registered nurse?

1. Pulse rate 90, respirations 24 per minute *2. Nasal flaring and rib retractions* 3. Drooling slightly blood-tinged saliva 4. Refusal to take sips of his favorite soda *Rationale:* Nasal flaring and rib retractions are signs of respiratory distress, a major concern following a tonsillectomy. These signs require immediate notification. The vital signs are normal for a 3-year-old child. Drooling slightly blood-tinged saliva and refusal to take sips of liquids are common after a tonsillectomy.

A 4-year-old child is hospitalized for severe gastroenteritis. The child is crying and clinging to the mother. The mother becomes very upset and is afraid to leave the child. Which of the following nursing interventions would be most appropriate to alleviate the child's fears and the mother's anxiety?

1. Reassure the mother that the child will be fine after she leaves. 2. Give the mother the telephone number of the pediatric unit, and tell the mother to call at any time. *3. Ask the mother if she would like to stay overnight with the child.* 4. Tell the mother to bring the child's favorite toys the next time she comes to the hospital to visit. *Rationale:* Although a 4-year-old may already be spending some time away from his or her parents at a day care center or preschool, illness adds a stressor that makes separation more difficult. The only option that addresses the mother's anxiety, while at the same time alleviating the fears of the child is option 3. Options 1, 2, and 4 do not address the fears and anxieties of the mother and child.

A nurse caring for a child with nephrotic syndrome reviews the medication record. The nurse notes that prazosin hydrochloride (Minipress) is prescribed for the child. The nurse determines that this medication has been prescribed to:

1. Reduce proteinuria. 2. Decrease inflammation. 3. Suppress the autoimmune response. *4. Control hypertension.* *Rationale:* Prazosin hydrochloride (Minipress) may be used to control hypertension. The child also may be placed on diuretic therapy until protein loss is controlled. Corticosteroids, such as prednisone, may be prescribed to decrease inflammation. Corticosteroids also suppress the autoimmune response and stimulate vascular reabsorption of edema. Cyclophosphamide is an alkylating agent and may be used in maintaining remission.

A nurse assists in developing a plan of care for a client with hyperparathyroidism receiving calcitonin-human (Cibacalcin). Which outcome has the highest priority regarding this medication?

1. Relief of pain 2. Absence of side effects *3. Reaching normal serum calcium levels* 4. Verbalization of appropriate medication knowledge *rationale* Hypercalcemia can occur in clients with hyperparathyroidism, and calcitonin is used to lower plasma calcium level. The highest priority outcome in this client situation would be a reduction in serum calcium level. Option 1 is unrelated to this medication. Although options 2 and 4 are expected outcomes, they are not the highest priority for administering this medication.

A nurse is caring for a child with osteosarcoma following amputation of the left lower limb. The child is continually complaining of aching and cramping in the missing limb. The initial nursing action is which of the following?

1. Request a referral for a psychiatric consultation. 2. Ask the health care provider for a prescription for a placebo. *3. Reassure the child that this is a temporary condition.* 4. Tell the child that the prosthesis will relieve this sensation. *Rationale:* Following amputation, phantom limb pain is a temporary condition that some children may experience. This sensation of burning, aching, or cramping in the missing limb is most distressing to the child. The child needs to be reassured that the condition is normal and only temporary. Options 1 and 2 are inappropriate. Although the sensation of phantom pain is temporary, the prosthesis will not necessarily relieve this sensation.

A child with a fractured femur is placed in Buck's skin traction and the nurse is planning care for the client. Which information about this type of traction is correct?

1. Requires frequent pin care 2. Places the child at risk for infection 3. Uses skeletal traction and weights to provide a counterforce *4. Is a type of skin traction that pulls the hip and leg into extension* *Rationale:* Buck's skin traction is a type of skin traction used in fractures of the femur and in hip and knee contractures. It pulls the hip and leg into extension. Countertraction is applied by the child's body. Options 1, 2, and 3 describe skeletal traction.

A 6-year-old child with leukemia is hospitalized and is receiving combination chemotherapy. Laboratory results indicate that the child is neutropenic, and the nurse prepares to implement protective isolation procedures. Which interventions would the nurse initiate? *Select all that apply.*

1. Restrict all visitors. *2. Place the child on a low-bacteria diet.* *3. Change dressings using sterile technique.* 4. Encourage the consumption of fresh fruits and vegetables. *5. Perform meticulous handwashing before caring for the child.* 6. Allow fresh-cut flowers in the room as long as they are kept in a vase with fresh water. *Rationale:* For the hospitalized neutropenic child, flowers or plants should not be kept in the room because standing water and damp soil harbor Aspergillus and Pseudomonas, to which these children are very susceptible. Fruits and vegetables not peeled before being eaten harbor molds and should be avoided until the white blood cell count rises. The child is placed on a low-bacteria diet. Dressings are always changed with sterile technique. Not all visitors need to be restricted, but anyone who is ill should not be allowed in the child's room. Meticulous handwashing is required before caring for the child. In addition, gloves, a mask, and a gown are worn (per agency policy).

Which nursing measure would be effective in preventing complications in a client with Addison's disease?

1. Restricting fluid intake 2. Offering foods high in potassium 3. Checking family support systems *4. Monitoring the blood glucose* *rationale* The decrease in cortisol secretion that characterizes Addison's disease can result in hypoglycemia. Therefore monitoring the blood glucose would detect the presence of hypoglycemia so that it can be treated early to prevent complications. Fluid intake should be encouraged to compensate for dehydration. Potassium intake should be restricted because of hyperkalemia. Option 3 is not a priority for this client.

A nurse is assisting in developing a plan of care for a 10-year-old child diagnosed with acute glomerulonephritis. Following review of the plan of care, the nurse determines that which of the following is the priority for the child?

1. Restricting oral fluids 2. Allowing the child to play with the other children in the playroom *3. Promoting bedrest* 4. Encouraging visits from friends *Rationale:* Bedrest is required during the acute phase, and activity is gradually increased as the condition improves. Providing for quiet play according to the developmental stage of the child is important. Fluids should not be forced or restricted. Visitors should be limited to allow for adequate rest.

A nurse has provided dietary instructions to the mother of a child with celiac disease. The nurse determines that further instructions are needed if the mother states that she will include which of the following in the child's nutritional plan?

1. Rice 2. Corn *3. Oatmeal* 4. Vitamin supplements *Rationale:* Dietary management is the mainstay of treatment for the child with celiac disease. All wheat, rye, barley, and oats should be eliminated from the diet and replaced with corn and rice. Vitamin supplements, especially fat-soluble vitamins and folate, may be needed in the early period of treatment to correct deficiencies.

A nurse is assisting a health care provider with an assessment of a child with a diagnosis of suspected appendicitis. In assessing the intensity and progression of the pain, the health care provider palpates the child at McBurney's point. The nurse understands that McBurney's point is located midway between the:

1. Right anterior inferior iliac crest and the umbilicus 2. Left anterior superior iliac crest and the umbilicus *3. Right anterior superior iliac crest and the umbilicus* 4. Left anterior superior iliac crest and the umbilicus *Rationale:* McBurney's point is midway between the right anterior superior iliac crest and the umbilicus. It is usually the location of greatest pain in the child with appendicitis. Options 1, 2, and 4 are incorrect.

The school nurse notes that the child has a rash and suspects that it is caused by erythema infectiosum (fifth disease). The nurse bases this determination on the observation that the rash results in:

1. Rose-pink maculopapulars 2. Pruritic macule-to-papules 3. Pinkish red maculopapulars *4. A "slapped-face" appearance* *Rationale:* The classic rash of erythema infectiosum or fifth disease is the erythema on the face. The discrete rose-pink maculopapular rash is the rash of exanthema subitum (roseola). The highly pruritic profuse macule-to-papule rash is the rash of varicella (chickenpox). The discrete pinkish red maculopapular rash is the rash of rubella (German measles).

A mother brings her 15-month-old child to the health care provider's office with complaints that the child has suddenly developed a bright red rash on her cheeks. She has no other symptoms and has been playing and eating as usual. Based on the appearance of the child, the nurse might suspect that the child has:

1. Rubella 2. Roseola *3. Fifth disease* 4. Chickenpox *Rationale:* Fifth disease has the general appearance of "slapped cheeks." Many children do not have any symptoms prior to the appearance of the reddened cheeks. This characteristic is not associated with the communicable diseases identified in options 1, 2, or 4.

A nursing student is asked to discuss sudden infant death syndrome (SIDS) at the clinical conference being held at the end of the clinical day. The student plans to include which of the following in the discussion during the conference?

1. SIDS usually occurs during sleep and is more common in girls. *2. SIDS usually occurs during sleep and is more common in premature infants.* 3. SIDS usually occurs during sleep and is more common in high-birth-weight infants. 4. SIDS usually occurs during sleep and most frequently occurs between 8 and 10 months of age. *Rationale:* SIDS usually occurs during sleep. It most frequently occurs between the second and fourth months of life. It is more common in boys, low-birth-weight infants, and premature infants.

A nurse participating in a free health screening at the local mall obtains a random blood glucose level of 200 mg/dL on an otherwise healthy client. The nurse tells the client to do which of the following as a next step?

1. Seek treatment for diabetes mellitus. 2. Ask the pharmacist about starting insulin therapy. 3. Begin blood glucose monitoring three times a day. *4. Call the health care provider to have the value rechecked as soon as possible.* *rationale* Adult diabetes mellitus can be diagnosed either by symptoms (polydipsia, polyuria, polyphagia) or by laboratory values. Diabetes is also diagnosed by an abnormal glucose tolerance test, when random plasma glucose levels are greater than 200 mg/dL, or fasting plasma glucose levels are greater than 140 mg/dL on two separate occasions. Further confirmation of this result is needed to ensure appropriate diagnosis and therapy.

A client is admitted with a diagnosis of pheochromocytoma. The nurse would monitor which of the following to detect the most common sign of pheochromocytoma?

1. Skin temperature *2. Blood pressure* 3. Urine ketones 4. Weight *rationale* Hypertension is the major symptom associated with pheochromocytoma and is monitored by taking the client's blood pressure. Glycosuria, weight loss, and diaphoresis are other clinical manifestations of pheochromocytoma; however, hypertension is the most common sign.

A nurse is assisting in preparing a plan of care for the client with diabetes mellitus and plans to reinforce the client's understanding regarding the symptoms of hypoglycemia. Which symptoms will the nurse review?

1. Slow pulse; lethargy; and warm, dry skin 2. Elevated pulse; lethargy; and warm, dry skin *3. Elevated pulse; shakiness; and cool, clammy skin* 4. Slow pulse, confusion, and increased urine output *rationale* Symptoms of mild hypoglycemia include tachycardia; shakiness; and cool, clammy skin. Options 1, 2, and 4 are not symptoms of hypoglycemia.

A nurse is assisting with preparing a teaching plan for the client with diabetes mellitus regarding proper foot care. Which instruction should be included in the plan of care?

1. Soak the feet in hot water. 2. Avoid using soap to wash the feet. *3. Apply a moisturizing lotion to dry feet, but not between the toes.* 4. Always have a podiatrist cut your toenails; never cut them yourself. *rationale* The client should use a moisturizing lotion on his or her feet, but should avoid applying the lotion between the toes. The client should also be instructed to not soak the feet and to avoid hot water to prevent burns. The client may cut the toenails straight across and even with the toe itself, but he or she should consult a podiatrist if the toenails are thick or hard to cut or if his or her vision is poor. The client should be instructed to wash the feet daily with a mild soap.

A nurse is reinforcing home care instructions to the mother of a child with hemophilia. Which activity should the nurse suggest that the child can safely participate in with peers?

1. Soccer 2. Basketball *3. Swimming* 4. Field hockey *Rationale:* Children with hemophilia need to avoid contact sports and need to take precautions, such as wearing elbow and knee pads and helmets, when participating in other sports. The safest activity that will prevent injury is swimming.

A nurse working in the day care center is told that a child with autism will be attending the center. The nurse collaborates with the staff of the day care center and assists in planning activities that will meet the child's needs. The nurse understands that the priority consideration in planning activities for the child is to ensure:

1. Social interactions with other children in the same age group *2. Safety with activities* 3. Familiarity with all activities and providing orientation throughout the activities 4. Activities providing verbal stimulation *Rationale:* Safety with all activities is a priority in planning activities with the child. The child with autism is unable to anticipate danger, has a tendency for self-mutilation, and has sensory-perceptual deficits. Although providing social interactions, verbal communications, and familiarity and orientation are also appropriate interventions, the priority is safety.

A nursing student is asked to administer a tepid bath to a child with a fever. The student avoids which of the following when performing this procedure?

1. Squeezes water over the child's body, using a washcloth *2. Applies alcohol-soaked cloths over the child's body* 3. Uses a water toy to distract the child during the bath 4. Places lightweight pajamas on the child after the bath *Rationale:* Alcohol should never be used for bathing the child with a fever because it can cause rapid cooling, peripheral vasoconstriction, and chilling, thus elevating the temperature further. Washcloths can be used to squeeze water over the child's body. Towels are used to dry the child. Toys, especially water toys, can be used to provide distraction during the bath. Lightweight clothing should be placed on the child after the child is dried.

The anticipated intended effect of fludrocortisone acetate (Florinef) for the treatment of Addison's disease is to:

1. Stimulate the immune response. *2. Promote electrolyte balance.* 3. Stimulate thyroid production. 4. Stimulate thyrotropin production. *rationale* Florinef is a long-acting oral medication with mineralocorticoid and moderate glucocorticoid activity used for long-term management of Addison's disease. Mineralocorticoids act on the renal distal tubules to enhance the reabsorption of sodium and chloride ions and the excretion of potassium and hydrogen ions. In small doses, fludrocortisone acetate causes sodium retention and increased urinary potassium excretion. The client rapidly can develop hypotension and fluid and electrolyte imbalance if the medication is discontinued abruptly. Options 1, 3, and 4 are not associated with the effects of this medication.

A nurse is caring for an infant with congenital heart disease. Which of the following signs, if noted in the infant, would alert the nurse to the early development of congestive heart failure (CHF)?

1. Strong sucking reflex 2. Slow and shallow breathing 3. Pallor *4. Diaphoresis during feeding* *Rationale:* The early symptoms of CHF include tachypnea, poor feeding, and diaphoresis during feeding. Tachycardia would occur during feeding. Pallor may be noted in the infant with CHF, but it is not an early symptom. A strong sucking reflex is unrelated to the development of CHF.

A nursing instructor assigns a student nurse to present a clinical conference to the student group about brain tumors in children. The student prepares for the conference and plans to include which of the following in the presentation?

1. Surgery is not normally performed because of the risk of functional deficits occurring as a result of the surgery. 2. Head shaving is no longer required before removal of the brain tumor. 3. The common site of metastasis is the kidneys. *4. The significant symptoms are headaches and morning vomiting.* *Rationale:* The hallmark symptoms of children with brain tumors are headache and morning vomiting related to the child getting out of bed. Initial intervention is "debulking" or operating to remove as much of the tumor as possible while minimally disturbing the surrounding brain tissue, so that the child's neurological functioning is preserved as much as possible. Before surgery, the child's head will be shaved, although every effort is made to shave as much hair as is necessary only. Depending on the type of tumor, a myelogram may be done to determine metastatic disease in the spinal cord.

A nurse is caring for a child who sustained a head injury in an automobile accident and is monitoring the child for signs of increased intracranial pressure (ICP). The nurse plans to monitor for the earliest sign of increased ICP by assessing for:

1. Tachycardia *2. Changes in level of consciousness (LOC)* 3. Posturing 4. Apnea *Rationale:* An altered level of consciousness is an early sign of increased ICP. Late signs of increased ICP include tachycardia leading to bradycardia, apnea, systolic hypertension, widening pulse pressure, and posturing.

Following a cleft lip repair, the nurse provides instructions to the parents regarding cleaning of the lip repair site. Which of the following solutions would the nurse use in demonstrating this procedure to the parents?

1. Tap water *2. Sterile water* 3. Full-strength hydrogen peroxide 4. Half-strength hydrogen peroxide *Rationale:* The lip repair site is cleansed with sterile water using a cotton swab after feeding and as prescribed. The parents should be instructed to use a rolling motion from the suture line out. The parents should also demonstrate performance of the correct procedure to the nurse.

A nurse prepares to administer a pancreatic enzyme powder to the child with cystic fibrosis (CF). Which of the following food items will the nurse mix with the medication?

1. Tapioca *2. Applesauce* 3. Hot oatmeal 4. Mashed potatoes *Rationale:* Pancreatic enzyme powders are not to be mixed with hot foods or foods containing tapioca or other starches. Enzyme powder should be mixed with non-fat, non-protein foods such as applesauce. Pancreatic enzymes are inactivated by heat and are partially degraded by gastric acids.

A child with croup is placed in a cool-mist tent. The mother becomes concerned because the child is frightened, consistently crying, and tries to climb out of the tent. The appropriate nursing action would be to:

1. Tell the mother that the child must stay in the tent 2. Call the health care provider and obtain a prescription for a mild sedative 3. Place a toy in the tent to make the child feel more comfortable *4. Let the mother hold the child and direct a cool mist over the child's face* *Rationale:* Crying aggravates laryngospasm and increases hypoxia, which may cause airway obstruction. If the use of a tent or hood is causing distress, treatment may be more effective if the child is held by the parent and a cool mist is directed toward the child's face. A mild sedative would not be administered to the child. Options 1 and 3 will not alleviate the child's fear.

A licensed practical nurse (LPN) is assigned to assist in caring for a hospitalized child who is receiving a continuous infusion of intravenous (IV) potassium for the treatment of dehydration. The LPN monitors the child closely and notifies the registered nurse if which of the following is noted?

1. Temperature of 100.8° F rectally 2. Weight increase of 0.5 kg *3. A decrease in urine output to 0.5 mL/kg/hr* 4. Blood pressure (BP) unchanged from baseline *Rationale:* The priority assessment is to monitor the status of urine output. Potassium should never be administered in the presence of oliguria or anuria. If urine output is less than 1 to 2 mL/kg/hr, potassium should not be administered. A slight elevation in temperature would be expected in a child with dehydration. A weight increase of 0.5 kg is relatively insignificant. A BP that is unchanged is a positive indicator unless the baseline was abnormal. However, there is no information in the question to support such data.

A 1-year-old child is seen in the health care provider's office with complaints of an elevated temperature that began the previous evening. When gathering subjective data from the mother, the nurse notices that which of the following would most likely indicate the child has acute otitis media?

1. The child is crying and irritable. 2. The temperature is 40° C (104° F). 3. The child is pulling at her ear and rolling her head from side to side. *4. The mother states the child had purulent discharge from the ear last night.* *Rationale:* Subjective data are what the mother tells the nurse. Therefore option 4 is correct because the mother is describing the child's ear drainage that occurred last night. The other options are considered objective data, which are observations that the nurse makes.

A child is brought to a clinic after developing a rash on the trunk and on the scalp. The parents report that the child has had a low-grade fever, has not felt like eating, and has been generally tired. The child is diagnosed with chickenpox. Which statement by the nurse is accurate regarding chickenpox?

1. The communicable period is unknown. 2. The communicable period ranges from 2 weeks or less up to several months. 3. The communicable period is 10 days before the onset of symptoms to 15 days after the rash appears. *4. The communicable period is 1 to 2 days before the onset of the rash to 6 days after the onset and crusting of lesions.* *Rationale:* The communicable period for chickenpox is 1 to 2 days before the onset of the rash to 6 days after the onset and crusting of lesions. In roseola the communicable period is unknown. Option 2 describes diphtheria. Option 3 describes rubella.

A nurse is reviewing the immunization schedule for a child with human immunodeficiency virus (HIV) infection with the mother. Which of the following will be a component of the instructions that the nurse provides to the mother?

1. The immunization schedule must be altered because of the HIV infection. *2. No live virus vaccines should be administered to the child.* 3. Immunizations will not be given to the child with HIV infection. 4. Immunizations will be given to the child with HIV infection but will not be initiated until the child is 3 years old. *Rationale:* The mother should be instructed that the child with HIV should keep immunizations up to date. No live virus vaccines should be administered because the child with HIV is immunocompromised. The immunization schedule would not be altered in any other way, and it is important for the mother to understand the immunization schedule clearly.

A nurse is caring for a client with type 1 diabetes mellitus who is hyperglycemic. Which problem would the nurse consider first, when planning care for this client?

1. The need for knowledge about the diagnosis 2. Insomnia 3. Lack of appetite *4. Signs of dehydration* *rationale* Hyperglycemia can develop into ketoacidosis in the client with type 1 diabetes mellitus. Polyuria develops as the body attempts to get rid of the excess glucose, and the client will lose large amounts of fluid. Because glucose is hyperosmotic, fluid is pulled from the tissue. Nausea and vomiting can occur as a result of hyperglycemia and can lead to a loss of sodium and water. Water also is lost from the lungs in an attempt to get rid of excess carbon dioxide. The severe dehydration that occurs can lead to hypovolemic shock. Of the problems listed, dehydration is considered first.

A child is admitted to the hospital, and a diagnosis of bacterial meningitis is suspected. A lumbar puncture is performed, and the results reveal cloudy cerebrospinal fluid (CSF) with high protein and low glucose levels. The nurse determines that these results are indicative of:

1. The need to repeat the test 2. Possible contamination of the specimen *3. Confirmation of the diagnosis* 4. A negative test *Rationale:* A diagnosis of meningitis is made by testing CSF obtained by lumbar puncture. In the case of bacterial meningitis, findings usually include increased pressure, cloudy CSF, and high protein and low glucose levels. Options 2 and 4 are incorrect. Option 1 is an unnecessary measure.

An infant is suspected to be human immunodeficiency virus (HIV) positive, and the nurse provides information to the parents about the care of their infant. Which of the following indicates to the nurse that the parents need further information about the care of their HIV-positive infant?

1. The parents state they will not allow anyone with a cold to hold and kiss the baby. 2. The parents are able to verbalize signs and symptoms of failure to thrive. 3. The parents ask about a prescription for an antiretroviral medication. *4. The parents plan to use rice cereal to help with watery stools when they occur.* *Rationale:* If an infant is having diarrhea, the parents need to seek medical attention because this could be the beginning of an opportunistic infection. Self-treatment is not encouraged. Asking for antiretroviral therapy, understanding signs and symptoms of failure to thrive, and being protective of an immunocompromised infant are evidence of understanding the needs of the infant.

A 12-year-old child is seen in the clinic, and a diagnosis of Hodgkin's disease is suspected. Several diagnostic studies are performed to determine the presence of this disease. When evaluating the diagnostic results, the nurse would expect to note which of the following if this child had Hodgkin's disease?

1. The presence of blast cells in the bone marrow *2. The presence of Reed-Sternberg cells* 3. The presence of Epstein-Barr virus 4. Elevated creatinine level *Rationale:* Hodgkin's disease is a neoplasm of lymphatic tissue. The presence of giant, multinucleated cells (Reed-Sternberg cells) is the hallmark of this disease. The presence of blast cells in the bone marrow is indicative of leukemia. Infectious mononucleosis and the Epstein-Barr virus have been associated with Hodgkin's disease but would not determine the presence of Hodgkin's. An elevated creatinine level is indicative of a renal system disorder.

A nurse employed in an emergency department is instructed to monitor a child diagnosed with epiglottitis. The nurse notes that the child is leaning forward with the chin thrust out. The nurse interprets this finding as indicating:

1. The presence of dehydration 2. The presence of pain 3. Extreme fatigue *4. An airway obstruction* *Rationale:* Clinical manifestations suggestive of airway obstruction include tripod positioning (leaning forward supported by arms, chin thrust out, mouth open), nasal flaring, tachycardia, a high fever, and sore throat. The data in the question do not relate to options 1, 2, or 3.

A child is diagnosed with intussusception. The nurse collects data on the child, knowing that which of the following is a characteristic of this disorder?

1. The presence of fecal incontinence 2. Incomplete development of the anus 3. The infrequent and difficult passage of dry stools *4. Invagination of a section of the intestine into the distal bowel* *Rationale:* Intussusception is an invagination of a section of the intestine into the distal bowel. It is the most common cause of bowel obstruction in children age 3 months to 6 years. Option 1 describes encopresis. Option 2 describes imperforate anus, and this disorder is diagnosed in the neonatal period. Option 3 describes constipation. Constipation can affect any child at any time, although it peaks at ages 2 to 3 years. Encopresis generally affects preschool and school-age children.

A nurse is monitoring a newborn with a suspected diagnosis of imperforate anus. The nurse understands that which of the following is unassociated with this disorder?

1. The presence of stool in the urine 2. Failure to pass a rectal thermometer 3. Failure to pass meconium in the first 24 hours after birth *4. The passage of currant jelly-like stools* *Rationale:* During the newborn assessment, imperforate anus should be easily identified visually. However, a rectal thermometer or tube may be necessary to determine patency if meconium is not passed in the first 24 hours after birth. The presence of stool in the urine or vagina should be reported immediately as an indication of abnormal anorectal development. Currant jelly-like stool is not a clinical manifestation of this disorder.

A 9-year-old child is diagnosed with chlamydial conjunctivitis. The nurse consults with the primary health care provider regarding necessary follow-up because this infection can be associated with:

1. The presence of systemic allergies 2. The cleanliness of the home environment 3. The presence of otitis media *4. Possible sexual abuse* *Rationale:* A diagnosis of chlamydial conjunctivitis in a child who is not sexually active should signal the health care provider to assess the child for possible sexual abuse. Allergy and infection can cause conjunctivitis, but the infecting organism would not be chlamydial. Although the infection can be transmitted, it is not directly associated with cleanliness in the home. Chlamydial conjunctivitis also may be suspected in a sexually active adolescent with chronic infection that is unresponsive to other treatment.

A nurse is providing discharge instructions to a client who had a unilateral adrenalectomy. Which of the following will be a component of the instructions?

1. The reason for maintaining a diabetic diet *2. Instructions about early signs of a wound infection* 3. Teaching regarding proper application of an ostomy pouch 4. The need for lifelong replacement of all adrenal hormones *rationale* A client who is undergoing a unilateral adrenalectomy will be placed on corticosteroids temporarily to avoid a cortisol deficiency. These medications will be gradually weaned in the postoperative period until they are discontinued. Because of the anti-inflammatory properties of corticosteroids, clients who undergo an adrenalectomy are at increased risk for developing wound infections. Because of this increased risk for infection, it is important for the client to know measures to prevent infection, early signs of infection, and what to do if an infection is present. Options 1, 3, and 4 are incorrect instructions.

A nurse is discussing foot care with a diabetic client and spouse. The nurse includes which of the following during this informational session?

1. There is decreased risk of infection when feet are soaked in hot water. 2. Lanolin should be applied to dry feet, especially the heels and between the toes. *3. The toenails should be cut straight across.* 4. Strong soap should be used to decrease skin bacteria. *rationale* The client should be instructed to cut the toenails straight across. The client should not soak the feet in hot water, to prevent burns. The client should be instructed to wash the feet daily using a mild soap. Moisturizing lotion can be applied to the feet but should not be placed between the toes.

A nurse provides feeding instructions to a mother of an infant diagnosed with gastroesophageal reflux (GER). To assist in reducing the episodes of emesis, the nurse tells the mother to:

1. Thin the feedings by adding water to the formula. *2. Thicken the feedings by adding rice cereal to the formula.* 3. Provide less frequent, larger feedings. 4. Burp less frequently during feedings. *Rationale:* Small, more frequent feedings with frequent burping are often tried as the first line of treatment in GER. Feedings thickened with rice cereal may reduce episodes of emesis. Thickened feedings do not affect reflux time, however. If thickened formula is prescribed, 1 to 3 teaspoons of rice cereal per ounce of formula is most commonly used and may require cross-cutting the nipple. Options 1, 3, and 4 are incorrect.

A client with newly diagnosed Cushing's syndrome expresses concern about personal appearance, specifically about the "buffalo hump" that has developed at the base of the neck. When counseling the client about this manifestation, the nurse should incorporate the knowledge that:

1. This is a permanent feature. 2. It can be minimized by wearing tight clothing. *3. It may slowly improve with treatment of the disorder.* 4. It will quickly disappear once medication therapy is started. *rationale* The client with Cushing's syndrome should be reassured that most physical changes resolve over time with treatment. The other options are incorrect.

Glucagon hydrochloride injection would most likely be prescribed for which disorder?

1. Thyroid crisis *2. Type 1 diabetes mellitus* 3. Hypoadrenalism 4. Excess growth hormone secretion *rationale* Glucagon hydrochloride is a medication that can be administered subcutaneously or intramuscularly. It is prescribed to stimulate the liver to release glucose when a client is experiencing hypoglycemia and unable to take oral glucose replacement. It is important to teach a person other than the client how to administer the medication because the client's symptoms may prevent self-injection. Therefore options 1, 3, and 4 are incorrect.

A child is seen in the health care clinic and received an immunization of DPT (diphtheria, pertussis, tetanus vaccine). One hour later, the mother calls the clinic and tells the nurse that the injection site is painful and red. Which of the following instructions would the nurse provide to the mother?

1. To return to the health care clinic immediately 2. To call the health care provider 3. To apply warm compresses on the site *4. To apply cold compresses for 24 hours following the injection* *Rationale:* For painful or red injection sites, the nurse should instruct the mother to apply cold compresses for the first 24 hours, then to use warm or cold compresses as long as needed. Options 1, 2, and 3 are incorrect. It is not necessary for the mother to bring the child to the clinic immediately, and it is not necessary for the mother to contact the health care provider.

A mother of a child with cystic fibrosis asks the clinic nurse about the disease. The nurse tells the mother that it is:

1. Transmitted as an autosomal dominant trait *2. A chronic multisystem disorder affecting the exocrine glands* 3. A disease that causes the formation of multiple cysts in the lungs 4. A disease that causes dilation of the passageways of many organs *Rationale:* Cystic fibrosis is a chronic multisystem disorder affecting the exocrine glands. The mucus produced by these glands (particularly those of the bronchioles, small intestine, and the pancreatic and bile ducts) is abnormally thick, causing obstruction of the small passageways of these organs. It is transmitted as an autosomal recessive trait.

A client with type 1 diabetes mellitus is to begin an exercise program, and the nurse is providing instructions to the client regarding the program. Which of the following should the nurse include in the teaching plan?

1. Try to exercise before mealtime. 2. Administer insulin after exercising. *3. Take a blood glucose test before exercising.* 4. Exercise should be performed during peak times of insulin. *rationale* A blood glucose test performed before exercising provides information to the client regarding the need to eat a snack first. Exercising during the peak times of insulin effect or before mealtime places the client at risk for hypoglycemia. Insulin should be administered as prescribed.

A lethargic, pale child is brought to the health care provider's office with symptoms of periorbital edema and reduced quantity of urine output. The urine is cloudy and smoky in color. The nurse asks the mother if the child has had any recent infections, to which the mother responds that the child had a very sore throat a few weeks ago. The health care provider suspects that the child might have acute poststreptococcal glomerulonephritis. Which of the following laboratory tests would rule out a past streptococcal infection in the child?

1. Urinalysis 2. Throat culture *3. Antistreptolysin titer* 4. Creatinine clearance *Rationale:* Option 3 is the only laboratory test that will determine if a streptococcal infection was present. The other options do not relate to a past streptococcal infection. Option 1 will determine if protein is present in the urine, which is present in glomerulonephritis. Option 2 will determine if a current throat infection is present. Option 4 will determine glomerular filtration rate.

An adolescent is admitted to the hospital with complaints of lower right abdominal pain. The health care provider prescribes laboratory tests to rule out ectopic pregnancy rather than appendicitis. Which of the following is most significant in ruling out an ectopic pregnancy?

1. Urinalysis 2. White blood count 3. C-reactive protein *4. Serum human chorionic gonadotropin* *Rationale:* The test to rule out an ectopic pregnancy is the serum human chorionic gonadotropin. The other tests may be prescribed to rule out appendicitis, but because the client is an adolescent it would be necessary to rule out an ectopic pregnancy as well. Urinalysis will rule out a urinary tract infection, and the white blood count and the C-reactive protein will rule out some other types of infection.

A nursing student is preparing to conduct a clinical conference, and the topic is hepatitis in children. The nursing instructor advises the student to further research the topic if the student plans to include which of the following in the discussion?

1. Vaccines are available to prevent hepatitis A (HAV) and hepatitis B (HBV). 2. Cases of hepatitis should be promptly reported to health care officials. *3. Enteric precautions are necessary for HBV but not for HAV.* 4. The child's stools will be pale and clay-colored. *Rationale:* Prevention of the spread of infection is an essential intervention for HAV. This should include enteric precautions for at least 1 week after the onset of jaundice and strict handwashing. Options 1, 2, and 4 are accurate regarding hepatitis.

The nurse is caring for a pediatric client in skin traction. To prevent skin breakdown, the best nursing intervention for this child is to:

1. Vigorously massage bony prominences every 4 hours. 2. Replace the elastic bandage on skin traction every 8 hours. *3. Stimulate circulation with gentle massage over pressure areas.* 4. Change the child's position at least every 4 hours to relieve pressure. *Rationale:* Nonadhesive straps and/or elastic bandage on skin traction are replaced when permitted and/or when absolutely necessary. Circulation should be stimulated with gentle, not vigorous, massage over pressure areas. The child's position should be changed at least every 2 hours to relieve pressure.

A nurse is preparing to administer an injection of regular insulin. The vial of the regular insulin has been refrigerated. On inspection of the vial, the nurse finds that the medication is frozen. The nurse should:

1. Wait for the insulin to thaw at room temperature. 2. Check the temperature settings of the refrigerator. *3. Discard the insulin and obtain another vial.* 4. Rotate the vial between the hands until the medication becomes liquid. *rationale* Insulin preparations are stable at room temperature for up to 1 month without significant loss of activity. Insulin should not be frozen. If the insulin is frozen, it should be discarded and the nurse should obtain another vial. Options 1, 2, and 4 are incorrect.

A 10-year-old child with asthma is treated for acute exacerbation. Which finding would indicate that the condition is worsening?

1. Warm, dry skin 2. Increased wheezing *3. Decreased wheezing* 4. A pulse rate of 90 beats per minute *Rationale:* Decreased wheezing in a child who is not improving clinically may be interpreted incorrectly as a positive sign, when in fact it may signal an inability to move air. A "silent chest" is an ominous sign during an asthma episode. With treatment, increased wheezing may actually signal that the child's condition is improving. Warm, dry skin indicates an improvement in the condition because the child is normally diaphoretic during exacerbation. The normal pulse rate in a 10-year-old is 70 to 110 beats per minute.

A client newly diagnosed with diabetes mellitus is having difficulty learning the technique of blood glucose measurement. The nurse should teach the client to do which of the following to perform the procedure properly?

1. Wash the hands first using cold water. 2. Puncture the center of the finger pad. 3. Puncture the finger as deeply as possible. *4. Let the arm hang dependently and milk the digit.* *rationale* Before doing a fingerstick for blood glucose measurement, the client should first wash the hands. Warm water should be used to stimulate the circulation to the area. The finger is punctured near the side, not the center, because there are fewer nerve endings along the side of the finger. The puncture is only deep enough to obtain an adequately sized drop of blood; excessively deep punctures may lead to pain and bruising. The arm should be allowed to hang dependently, and the finger may be milked to promote obtaining a good-size blood drop.

A 2-year-old child is admitted to the hospital with a diagnosis of nephrotic syndrome. In planning care for this child, which of the following nursing interventions would be of highest priority?

1. Weigh morning and afternoon. 2. Maintain a strict intake and output. *3. Dipstick the urine for protein every 4 hours.* 4. Take vital signs with blood pressure every 4 hours. *Rationale:* Continuous monitoring of fluid retention and excretion is an important nursing intervention in the care of the child with nephrotic syndrome. Although it is important to maintain a strict intake and output in monitoring fluid retention and excretion, the goal of treatment with this child is to decrease the amount of protein lost in the urine. Because this is the goal, option 3 has the highest priority. Although weight is monitored, it is not necessary to check the weight morning and evening. Taking vital signs with blood pressure is important but is not the priority in this situation.

A health care provider prescribes intravenous potassium for a child with hypertonic dehydration. The nurse assigned to assist in caring for the child would check which highest-priority item before administration of the potassium?

1. Weight *2. Urine output* 3. Temperature 4. Blood pressure *Rationale:* The priority assessment would be to check the status of urine output. Potassium should never be administered in the presence of oliguria or anuria. If urine output is less than 1 to 2 mL/kg/hr, it should not be administered. Although options 1, 3, and 4 may be a component of the data collected, they are not specifically related to the administration of this medication.

A nurse is caring for a hospitalized infant with a diagnosis of bronchiolitis. The nurse positions the infant:

1. With the head at a 60-degree angle with the neck slightly flexed 2. In a supine, side-lying position *3. With the head and chest at a 30-degree angle, with the neck slightly extended* 4. Prone, with the head of the bed elevated 15 degrees *Rationale:* The nurse should position the infant with the head and the chest at a 30- to 40-degree angle with the neck slightly extended to maintain an open airway and to decrease pressure on the diaphragm. Options 1, 2, and 4 do not achieve these goals.

While collecting data on a client being prepared for an adrenalectomy, the nurse obtains a temperature reading of 100.8° F. The nurse analyzes this temperature reading as:

1. Within normal limits *2. A finding that needs to be reported immediately* 3. An expected finding caused by the operative stress response 4. Slightly abnormal but an insignificant finding *rationale* An adrenalectomy is performed because of excess adrenal gland function. Excess cortisol production impairs the immune response, which puts the client at risk for infection. Because of this, the client needs to be protected from infection, and minor variations in normal vital sign values must be reported so that infections are detected early, before they become overwhelming. In addition, the surgeon may elect to postpone surgery in the event of a fever because it can be indicative of infection. Options 1, 3, and 4 are not correct interpretations.

The nurse documents that the urine collected from a client diagnosed with early stage polycystic kidney disease is dilute with a low-specific gravity. Based on this documentation, which specific gravity result was likely present?

1.000

A nurse is suctioning a client through a tracheostomy tube. The nurse plans to apply suction during the withdrawal of the catheter for a period of time no greater than:

10 seconds

Tonometry is performed on the client with a suspected diagnosis of glaucoma. The nurse reviews the test results as documented in the client's chart and understands that normal intraocular pressure is approximately:

10 to 21 mm Hg

The nurse reviews home care instructions with a client diagnosed with impetigo. Which statement indicates that the client does not understand the measures that will prevent the spread of infection? 1. "I need to take the full course of the antibiotics." 2. "My clothes can be laundered with other household members' clothes." 3. "I must wash my hands thoroughly and frequently throughout the day." 4. "I need to wash my dishes and eating utensils separate from other household members."

2. "My clothes can be laundered with other household members' clothes."

A client asks the nurse about the causes of acne. The nurse most appropriately responds by telling the client: 1. "It is caused by oily skin." 2. "The exact cause of acne is not known." 3. "It is caused as a result of exposure to heat and humidity." 4. "Acne is caused by eating chocolate, nuts, and fatty foods."

2. "The exact cause of acne is not known."

A client scheduled for a skin biopsy asks the nurse how painful the procedure is. The nurse makes which response to the client? 1. "There is no pain associated with this procedure." 2. "The local anesthetic may cause a burning or stinging sensation." 3. "There is some pain, but the health care provider will prescribe an analgesic following the procedure." 4. "A preoperative medication will be given so you will be sleeping and will not feel any pain."

2. "The local anesthetic may cause a burning or stinging sensation."

A nurse is caring for a client who sustained burns on the entire right leg and anterior thorax. Using the rule of nines, the extent of the burn injury would be which of the following? 1. 31.5% 2. 36% 3. 42% 4. 45%

2. 36%

A client is undergoing radiation therapy to treat lung cancer. Following the treatment, the nurse notes that the chest and neck are red, and the client is complaining of pain at the radiation site. The nurse interprets this data as: 1. An allergic reaction to the radiation 2. A superficial injury to tissue from the radiation 3. An ischemic injury, much like decubitus formation 4. A cutaneous reaction to products formed by lysis of the neoplastic cells

2. A superficial injury to tissue from the radiation

A nurse is preparing a poster for a health fair about prevention and early detection of skin cancer. The nurse would include on the poster instructions to avoid which of the following activities? 1. Wearing a hat, opaque clothing, and sunglasses when in the sun 2. Being in the sun for prolonged periods between 10:00 ᴀᴍ and 3:00 ᴘᴍ 3. Using sunscreen when spending time outdoors 4. Examining the skin monthly for any lesions that might be cancerous

2. Being in the sun for prolonged periods between 10:00 ᴀᴍ and 3:00 ᴘᴍ Rationale: The client should be instructed to avoid sun exposure between the hours of 10:00 ᴀᴍ and 3:00 ᴘᴍ. Sunscreen, a hat, opaque clothing, and sunglasses should be worn when spending time outdoors. The client should examine the body monthly for the appearance of any possible cancerous or precancerous lesions.

A client comes to a health care provider's office complaining of a bite on the arm. The client reports that he recently removed a tick from the same location. Which characteristic is a classic sign of Lyme disease that can result from an infected tick? 1. Painful rash around a necrotic lesion 2. Bullseye rash 3. Line of papules and vesicles that appear 1 to 3 days after exposure 4. Patch of oval lesions

2. Bullseye rash

A nurse is assisting in caring for a client with a severe burn who has just received an autograft to the knee area of the right leg. The nurse plans to keep the right leg positioned in which manner? 1. Elevated only when out of bed to the chair 2. Elevated and immobilized 3. Flat at all times 4. Dependent and covered with a blanket

2. Elevated and immobilized

A nurse notes the presence of a partial-thickness skin loss of the upper layer of the skin in the sacral area of a client on bedrest. The nurse documents these findings as a: 1. Stage 1 pressure ulcer 2. Stage 2 pressure ulcer 3. Stage 3 pressure ulcer 4. Stage 4 pressure ulcer

2. Stage 2 pressure ulcer

During the inspection of a client's skin, the nurse notes redness and an abrasion type wound on the sacrum area. The nurse determines that this finding is indicative of a: 1. Stage 1 pressure ulcer 2. Stage 2 pressure ulcer 3. Stage 3 pressure ulcer 4. Stage 4 pressure ulcer

2. Stage 2 pressure ulcer Rationale: In a stage 1 pressure ulcer, the skin is intact; the area is red and does not blanch with external pressure. In a stage 2 pressure ulcer, the skin is not intact; the ulcer is superficial and may characterize as an abrasion, blister, or shallow crater. In stage 3, skin loss is full thickness and the skin has a deep crater-like appearance. In stage 4, skin loss is full thickness with extensive destruction, tissue necrosis, or damage to muscle, bone, or supporting structures.

A nurse is assisting with caring for a client who is receiving intravenous fluids and who has sustained full-thickness burn injuries of the back and legs. The nurse understands that which of the following would provide the most reliable indicator for determining the adequacy of the fluid resuscitation? 1. Vital signs 2. Urine output 3. Mental status 4. Peripheral pulses

2. Urine output

A nurse prepares to care for a client with acute cellulitis of the lower leg. Which of the following would the nurse anticipate being prescribed for the client? 1. Cold compresses to the affected area 2. Warm compresses to the affected area 3. Alternating hot and cold compresses continuously 4. Intermittent heat-lamp treatments four times per day

2. Warm compresses to the affected area

A client has a fiberglass (nonplaster) cast applied to the lower leg. The client asks the nurse when he will be able to walk on the cast. The nurse replies that the client will be able to bear weight on the cast:

20 to 30 minutes

A nurse is preparing to administer a soapsuds enema to a client. Into which position does the nurse place the client to administer the enema? Refer to figure.

3

A nurse is reviewing the health care provider's prescriptions written for a client admitted with a diagnosis of acute cellulitis of the lower leg. The nurse should question which prescription? 1. Obtain blood cultures. 2. Administer antibiotics. 3. Apply cold compresses to the affected area. 4. Administer acetaminophen (Tylenol) for fever.

3. Apply cold compresses to the affected area.

A client is receiving topical corticosteroid therapy in the treatment of psoriasis. The nurse expects the health care provider to prescribe which measure to maximize the effectiveness of this therapy? 1. Rubbing the application into the skin 2. Applying a dry sterile dressing over the affected area 3. Covering the application with a warm, moist dressing and an occlusive outer wrap 4. Placing the area under a heat lamp for 20 minutes

3. Covering the application with a warm, moist dressing and an occlusive outer wrap

A nurse notes that the health care provider has documented a diagnosis of herpes zoster in the client's chart. On the basis of an understanding of the cause of this disorder, the nurse would determine that this diagnosis was made after the use of which diagnostic test? 1. Patch test 2. Skin biopsy 3. Culture of the lesion 4. Wood's light examination

3. Culture of the lesion

A nurse prepares to help a health care provider examine the client's skin with a Wood's light. Which of the following would be included in the plan for this procedure? 1. Prepare a local anesthetic. 2. Obtain an informed consent. 3. Darken the room for the examination. 4. Shave the skin and scrub it with a povidone-iodine (Betadine) solution.

3. Darken the room for the examination.

A client who sustained an inhalation injury arrives in the emergency department. On data collection of the client the nurse notes that the client is very confused and combative. The nurse determines that the client is experiencing: 1. Anxiety 2. Fear 3. Hypoxia 4. Pain

3. Hypoxia

A nurse inspects the oral cavity of a client with cancer and notes white patches on the mucous membranes. The nurse determines that this occurrence: 1. Is common 2. Suggests that the client is anemic 3. Is characteristic of a thrush infection 4. Is indicative that oral hygiene needs to be improved

3. Is characteristic of a thrush infection

A client returns to the clinic for follow-up treatment after a skin biopsy of a suspicious lesion that was performed 1 week ago. The biopsy report indicates that the lesion is a melanoma. The nurse understands that which of the following describes a characteristic of this type of a lesion? Submit 1. Metastasis is rare. 2. It is encapsulated. 3. It is highly metastatic. 4. It is characterized by local invasion.

3. It is highly metastatic.

A nurse inspects the skin of a client who is suspected of having scabies. Which of the following findings would the nurse note if this disorder was present? 1. Patchy hair loss and round, red macules with scales 2. The presence of wheal patches scattered about the trunk 3. Multiple straight or wavy threadlike lines beneath the skin 4. The appearance of vesicles or pustules with a thick, honey-colored crust

3. Multiple straight or wavy threadlike lines beneath the skin

A nurse is assisting in caring for a victim of a burn injury during the emergent/resuscitative phase. On data collection of the client the nurse notes that the urine output has decreased and the blood pressure is dropping. The nurse should immediately: 1. Increase the intravenous (IV) flow rate. 2. Cover the client with a warm blanket. 3. Notify the registered nurse. 4. Check the client in 30 minutes.

3. Notify the registered nurse.

A client with acquired immunodeficiency syndrome (AIDS) is diagnosed with cutaneous Kaposi's sarcoma. Based on this diagnosis, the nurse understands that this has been determined by which of the following? 1. Swelling in the genital area 2. Swelling in the lower extremities 3. Punch biopsy of the cutaneous lesions 4. Appearance of reddish-blue lesions on the skin

3. Punch biopsy of the cutaneous lesions

A nurse is assisting in caring for a client who suffered an inhalation injury from a wood stove. The carbon monoxide level reveals a level of 45%. Based on this level, the nurse would anticipate which of the following signs in the client? 1. Flushing 2. Dizziness 3. Tachycardia 4. Coma

3. Tachycardia

A nurse is caring for a client who has just been admitted to the nursing unit after receiving flame burns to the face and chest. The nurse notes a hoarse cough and that the client is expectorating sputum with black flecks. The client's eyelashes and eyebrows are singed, and the eyelids are swollen. The client suddenly becomes restless, and his color becomes dusky. The nurse interprets this data as indicating which of the following? 1. The client is hypotensive. 2. Pain is present from the burn injury. 3. The burn has probably caused laryngeal edema, which has occluded the airway. 4. The client is afraid and is having a panic attack as a result of the unfamiliar surroundings.

3. The burn has probably caused laryngeal edema, which has occluded the airway.

A client enters the ambulatory clinic, stating she has just been stung by a bee. Her vital signs are stable, and she has no previously known allergy to bee stings. The "stinger" is still visible in her arm. What should the nurse's first action be? 1. Use tweezers to remove the insect stinger. 2. Apply a warm compress to relieve the discomfort. 3. Use the edge of a sterile surgical tool to scrape out the stinger. 4. Apply an occlusive dressing over the stinger.

3. Use the edge of a sterile surgical tool to scrape out the stinger.

A client had a radical neck dissection with a musculocutaneous flap. Twenty-four hours following the procedure, the nurse notes that the flap has a slightly blue hue. The nurse concludes: 1. This is a normal expectation. 2. Heat should be applied to the area. 3. Venous circulation is being impaired. 4. The client is exhibiting generalized hypoxia.

3. Venous circulation is being impaired.

A client is newly admitted to the hospital with cellulitis of the lower leg. The nurse checks the health care provider's prescription sheet to see if which of the following therapies has been prescribed for site care? 1. Intermittent heat lamp treatments 2. Alternating hot and cold compresses 3. Warm compresses 4. Cold compresses

3. Warm compresses Rationale: Warm compresses may be used to decrease the discomfort, erythema, and edema that accompany cellulitis. Definitive treatment includes antibiotic therapy after appropriate cultures have been done. Other supportive measures also are used to manage symptoms such as fatigue, fever, chills, headache, or myalgia. Heat lamps are not used because of the risk of burns and because moist heat is most useful in treating this disorder.

A nurse is told that an assigned client is suspected of having scabies. Which of the following precautions will the nurse institute during the care of the client? 1. Wear gloves only. 2. Wear a mask and gloves. 3. Wear a gown and gloves. 4. Avoid touching the client's clothes.

3. Wear a gown and gloves.

A client, admitted to the emergency department, is suspected of having frostbite of the hands. Which finding should the nurse note in this condition? 1. A pink edematous hand 2. Red skin with edema in the nail beds 3. White skin that is insensitive to touch 4. Black fingertips surrounded by an erythematous rash

3. White skin that is insensitive to touch

An adult client is admitted to the emergency department following a burn injury. The burn initially affected the client's upper half of the anterior torso, and there were circumferential burns to the lower half of both of the arms. The client's clothes caught on fire, and the client ran, causing subsequent burn injuries to the entire face (anterior half of the head) and the upper half of the posterior torso. Using the rule of nines, what would be the percent of the burn injury? Refer to the figure.

31.5%

An adult client was burned as a result of an explosion. The burn initially affected the client's entire face (the anterior half of the head) and the upper half of the anterior torso, and there were circumferential burns to the lower half of both of the arms. The client's clothes caught on fire, and the client ran, which caused subsequent burn injuries of the posterior surface of the head and the upper half of the posterior torso. According to the rule of nines, what is the extent of this client's burn injury?

36 %

A client with chronic dermatitis has decided to receive testing to determine the cause of the condition. A patch test will be performed at the scheduled clinic visit in 2 weeks. The nurse provides instructions to the client regarding preparation of the test. Which statement by the client indicates an understanding regarding the preparation for this procedure? 1. "I need to have clear fluids only on the morning of the test." 2. "I need to take my prednisone on the morning of the test." 3. "I need to shower on the morning of the test using povidone-iodine (Betadine)." 4. "I need to stop taking my antihistamine 2 days before I come to the clinic for the test."

4. "I need to stop taking my antihistamine 2 days before I come to the clinic for the test."

A nurse has reinforced discharge instructions to a client who had a skin biopsy. Which statement by the client indicates the need for further instruction? 1. "I will use the antibiotic ointment, as prescribed." 2. "I will return in 7 days to have the sutures removed." 3. "I will call the health care provider (HCP) if I see any drainage from the wound." 4. "I will remove the dressing when I get home and wash the site with tap water."

4. "I will remove the dressing when I get home and wash the site with tap water."

The client arrives at the emergency department after a burn injury that occurred in the basement at home, and an inhalation injury is suspected. Which of the following should the nurse anticipate as being prescribed for the client? 1. Oxygen via nasal cannula at 10 L 2. Oxygen via nasal cannula at 15 L 3. 100% oxygen via an aerosol mask 4. 100% oxygen via a tight-fitting, nonrebreather face mask

4. 100% oxygen via a tight-fitting, nonrebreather face mask

A client sustains a burn injury to the entire right and left arms, right leg, and anterior thorax. According to the rule of nines, the nurse would determine that this injury constitutes which of the following body percentages? 1. 27% 2. 36% 3. 45% 4. 54%

4. 54%

A client is being admitted to the hospital for the treatment of acute cellulitis of the lower left leg. The client asks the nurse to explain what cellulitis means. The nurse bases the response on the understanding that the characteristics of cellulitis include: 1. An acute superficial infection 2. An inflammation of the lymphatics 3. A superficial infection caused by Staphylococcus 4. A skin infection into the deep dermis and subcutaneous fat

4. A skin infection into the deep dermis and subcutaneous fat

After 7 days of wound care, a client who has a well-granulated pressure ulcer reports to the nurse, "Feeling better overall." Which nursing intervention most likely contributed to the client's feelings? 1. Pain management program 2. Frequent comfort measures 3. Dressing change twice daily 4. Ambulation three times daily

4. Ambulation three times daily Rationale: The effects of exercise include client reports of feeling better generally, because the benefits of exercise are wide ranging. Ambulation can enhance tissue oxygenation and other cardiovascular, pulmonary, metabolic, integumentary, neuromuscular, and conditioning benefits. Thus the wide-ranging benefits of exercise are more likely to promote an overall sense of feeling better versus benefiting from pain control, less discomfort, or a well-granulated wound. The benefits of pain management, comfort measures, and dressing changes are more limited (options 1, 2, and 3).

Which of the following individuals is least likely at risk for the development of Kaposi's sarcoma? 1. A renal transplant client 2. A male with a history of same-sex partners 3. A client receiving antineoplastic medications 4. An individual working in an environment in which exposure to asbestos is possible

4. An individual working in an environment in which exposure to asbestos is possible

Which of the following individuals would be at the greatest risk for development of an integumentary disorder? 1. An adolescent 2. An older female 3. A physical education teacher 4. An outdoor construction worker

4. An outdoor construction worker

A nurse is caring for a client with a diagnosis of pemphigus vulgaris. The nurse understands that a characteristic of this condition is: 1. Dry skin 2. Hard skin 3. Leathery skin 4. Blistering skin

4. Blistering skin

A nurse is caring for a client with circumferential burns of both legs. Which of the following leg positions is appropriate for this type of a burn? 1. A dependent position 2. Elevation of the knees 3. Flat, without elevation 4. Elevation above the level of the heart

4. Elevation above the level of the heart

A client suffered smoke inhalation and burns to the anterior trunk during a house fire. The nurse reviews the plan of care and notes that the client has an airway problem. Which action is contraindicated as the nurse delivers care to this client? 1. Suctioning the airway on an as-needed basis 2. Encouraging regular use of an incentive spirometer 3. Repositioning side to side every 2 hours 4. Keeping the client in a supine position

4. Keeping the client in a supine position

Following diagnostic evaluation, it has been determined that the client has Lyme disease, stage 2. The nurse understands that which of the following is most indicative of this stage? 1. Arthralgias 2. Joint enlargement 3. Erythematous rash 4. Neurological deficits

4. Neurological deficits

A client has a non-infected pressure ulcer on the left heel. The nurse should use which of the following sterile solutions to cleanse the wound as part of a dressing change procedure? 1. Hydrogen peroxide 2. Povidone-iodine 3. Warm water 4. Normal saline

4. Normal saline

A nurse is checking for the presence of cyanosis in a dark-skinned client. Which body area would provide the best information? 1. Sacrum 2. Earlobes 3. Back of the hands 4. Palms of the hands

4. Palms of the hands

An evening nurse reviews the nursing documentation in the client's chart and notes that the day nurse has documented that the client has a stage 2 pressure ulcer in the sacral area. Which of the following would the nurse expect to note when checking the client's sacral area? 1. Intact skin 2. The presence of tunneling 3. A deep, crater-like appearance 4. Partial-thickness skin loss of the epidermis

4. Partial-thickness skin loss of the epidermis

A nurse is assigned to assist in caring for a client with frostbite of the toes. Which of the following would the nurse anticipate to be prescribed for this condition? 1. Rapid and continual rewarming of the toes when flushing occurs 2. Rapid and continual rewarming of the toes in cold water for 45 minutes 3. Rapid and continual rewarming of the toes in hot water for 15 to 20 minutes 4. Rapid and continual rewarming of the toes in a warm-water bath until flushing of the skin occurs

4. Rapid and continual rewarming of the toes in a warm-water bath until flushing of the skin occurs

An older client is complaining of chronic dry skin and occasional pruritus. The nurse tells the client to avoid which of the following that will aggravate the condition? 1. Applying emollient to the skin after a shower 2. Using a humidifier, especially during the winter months 3. Drinking 8 to 10 glasses of water a day 4. Using astringents to clean the skin

4. Using astringents to clean the skin Rationale: The client should avoid the use of rubbing alcohol, astringents, or other drying agents on the skin. The client should take one 15- to 20-minute warm bath or one shower per day. The client should then apply an emollient to prevent water evaporation from the hydrated epidermis. A room humidifier is useful during the winter months or whenever a furnace is in use. The client should maintain a daily fluid intake of 3000 mL, unless contraindicated, and should avoid ingestion of alcohol and caffeine.

A client with suspected Guillain-Barré syndrome has a lumbar puncture performed. The cerebrospinal fluid (CSF) is analyzed for protein. The nurse reviews the protein values and notes that the value that supports the diagnosis of Guillain-Barré syndrome is:

75 mg/dL

A client is positively diagnosed with stage 1 Lyme disease. The client asks the nurse about the treatment for the disease. The nurse responds to the client, anticipating that which treatment will be included in the care plan?

A 3- to 4- week course of oral antibiotic therapy

fat free

A client is scheduled for an oral cholecystography. The nurse would plan to obtain what type of diet for the evening meal before the test?

A nurse is providing client education regarding symptoms of testicular cancer. The nurse encourages the client to report which symptoms as being associated with testicular cancer? Select all that apply.

A grainy mass palpated in a testicle, An enlargement of one of the testes

A nurse is inspecting the stoma of a client after creation of a ureterostomy. Which of the following would the nurse expect to note?

A red and moist stoma

A health care provider asks the nurse to obtain a Salem Sump tube for gastric intubation. The nurse would correctly select which of the following tubes from the unit storage area?

A tube with a larger lumen and an air vent

A client is complaining of pain underneath a cast in the area of a bony prominence. The nurse interprets that this client may need to have:

A window cut in the cast

A 36 weeks gestation pregnant woman is complaining of urinary urgency and frequency. The nurse explained that the enlarging fetus is pressing the bladder which causes frequent urination. This is normally occuring during the first and third trimesters of pregnancy. The nurse advices the patient to do the following measures to prevent urinary frequency. Select all the necessary measures that the nurse can provide to the patient. A. Drink 2 quarts of fluid during the day B. Engaging in a regular exercise C. Performing Kegel exercises D. Soaking in a warm sitz bath E. Limiting fluid intake during the evening

A, C, E

A client has undergone fluoroscopy-assisted aspiration biopsy of a chest lesion. The nurse determines that the client is experiencing complications from the procedure if the nurse notes which of the following?

Absence of breath sounds in the right upper lobe

The nurse is assisting in administering immunizations at a health care clinic. The nurse understands that immunization provides which?

Acquired immunity from disease

The client is receiving an eyedrop and an eye ointment to the right eye. The nurse should:

Administer the eyedrop first, followed by the eye ointment

The nurse is caring for a client who had a renal biopsy. Which interventions should the nurse include in the plan of care for the client after this procedure? Select all that apply.

Administering pain medication as prescribed Monitoring vital signs and the puncture site frequently Testing serial urine samples with dipsticks for occult blood

A nurse is caring for the client who is going to have an arthrogram using a contrast medium. Which of the following data collected by the nurse would be of highest priority?

Allergy to iodine or shellfish

A nurse is preparing a client who is scheduled to have cerebral angiography performed. The nurse should check the client for:

Allergy to iodine or shellfish

Which individual is least at risk for the development of Kaposi's sarcoma?

An individual working in an environment where exposure to asbestos exists

Which of the following information will the nurse reinforce to the client scheduled for a lumbar puncture?

An informed consent will be required.

A nurse is caring for a client who has had spinal fusion with insertion of hardware. The nurse should be especially concerned with which of the following findings?

An oral temperature of 101° F orally

A caloric test is prescribed for a client suspected of having a disease of the labyrinth. The nurse obtains what essential item in preparation for this test?

An otoscope

A client undergoing computed tomography (CT) scan develops chest pain, wheezing, and stridor after injection of contrast media. Which type of shock is this client most likely exhibiting?

Anaphylactic

A client is treated in the health care provider's office after a fall, which sprained an ankle. Radiography has ruled out fracture. Before sending the client home, the nurse would plan to teach the client about which item that is to be avoided in the next 24 hours?

Application of a heating pad

A client who was hit in the eye with a baseball bat sustains a contusion of the eyeball. The emergency department nurse implements which immediate action?

Applies ice to the affected eye

A nurse is providing instructions to a client with a diagnosis of hordeolum regarding the treatment plan. Which of the following will the nurse include in the instructions to the client?

Apply a warm compress for 15 minutes four times daily

A client calls the emergency department and tells the nurse that he received a bee sting to the arm while weeding a garden. The client states that he has received bee stings in the past and is not allergic to bees. The client states that the site is painful and asks the nurse for advice to alleviate the pain. Which action should the nurse tell the client to take?

Apply ice and elevate the site

A nurse is preparing to suction an adult client through the client's tracheostomy tube. Which intervention(s) would the nurse perform for this procedure? Select all that apply.

Apply suction for up to 10 to 15 seconds. Hyperoxygenate the client before suctioning. Apply intermittent suction while rotating and withdrawing the catheter. Advance the catheter until resistance is met and then pull the catheter back 1 cm.

A nurse is teaching the client who is about to begin external radiation therapy how to maintain optimal skin integrity during therapy. The nurse determines that further teaching is needed if the client states that he will do which of the following?

Apply tight dressings over the area to prevent bleeding.

A nurse is planning to use an external cardiac defibrillator on a client. Which one of the following actions should the nurse perform to check the cardiac rhythm?

Applying the adhesive patch electrodes to the skin and moving away from the client

A client arrives at the health care clinic requesting to be tested for Lyme disease. The client tells the nurse that he removed the tick and flushed it down the toilet. Which nursing action is appropriate?

Arrange for the client to return in 4 to 6 weeks to be tested

A nurse is reviewing the record of a client with acute respiratory distress syndrome (ARDS). The nurse determines that which finding documented in the client's record is consistent with the most expected characteristic of this disorder?

Arterial PaO2 of 48

A client who has had a right total knee replacement asks the nurse how long the right leg must be kept in the continuous passive motion (CPM) machine. The nurse's response is based on the understanding that the device should be used:

As much as the client can tolerate

A client has a prescription for continuous monitoring of oxygen saturation by pulse oximetry. The nurse performs which of the following as the best action to ensure accurate readings on the oximeter?

Ask the client to limit motion in the hand attached to the pulse oximeter.

A nurse is assisting in planning care for a client with Hodgkin's disease who is neutropenic as a result of radiation and chemotherapy. Which should be included in the client's plan of care to decrease the risk of infection? Select all that apply.

Asking visitors with respiratory infection symptoms to not visit the client, Ensuring meticulous handwashing before caring for the client, Monitoring white blood cell counts daily

The nurse has a prescription to collect a 24-hour urine specimen from a client. The unlicensed assistive personnel (UAP) has been instructed on the collection technique. Which action by the UAP demonstrates the UAP needs further teaching?

Asks the client to void, save the specimen, and note the start time

A nurse gathers data from a client admitted to the hospital with gastroesophageal reflux disease (GERD) who is scheduled for a Nissen fundoplication. Based on an understanding of this disease, the nurse determines that the client may be at risk for which complication

Aspiration

A nurse is caring for a client with a tracheostomy tube who is receiving mechanical ventilation. The nurse is monitoring for complications related to the tracheostomy and suspects tracheoesophageal fistula when:

Aspiration of gastric contents occurs when suctioning

A cardiac monitor alarm sounds, and a nurse notes a straight line on the monitor screen. The immediate nursing action is to:

Assess the client.

A hospitalized client with a history of angina pectoris is ambulating in the corridor. The client suddenly complains of severe substernal chest pain. The nurse should take which action first?

Assist the client to sit or lie down.

A male client has a history of urinary tract infections due to urinary retention. Which intervention should the nurse implement to decrease the risk of infection?

Assist the client to stand for voiding

A nurse is preparing a plan of care for a client with a brain attack (stroke) who has global aphasia. The nurse incorporates communication strategies in the plan of care, knowing that the client's speech will be:

Associated with poor comprehension

A nurse is teaching breast self-examination (BSE) to a client who has had a hysterectomy. The nurse tells the client to perform the BSE:

At a specific day of the month and on that same day every month thereafter

A clinic nurse is obtaining cardiovascular data on a client. The nurse prepares to check the client's apical pulse and places the stethoscope in which of the following positions?

At the midclavicular line at the fifth left intercostal space

Pilocarpine hydrochloride (Isopto Carpine) is prescribed for the client with glaucoma. Which medication does the nurse plan to have available in the event of systemic toxicity?

Atropine sulfate

A nurse is caring for a client with cancer of the prostate after a prostatectomy. The nurse provides discharge instructions and plans to include which of the following?

Avoid lifting objects heavier than 20 pounds for at least 6 weeks

A nurse is caring for a client diagnosed with Paget's disease. The nurse understands that this condition usually affects which bones?

Axial skeleton including vertebrae

Which of the following would the nurse identify as a presumptive sign of pregnancy? A. Hegar sign B. Nausea and vomiting C. skin pigmentation changes D. positive serum pregnancy test

B Explanation resumptive signs of pregnancy are subjective signs. Of the signs listed, only nausea and vomiting are presumptive signs. Hegar sign, skin pigmentation changes, and a positive serum pregnancy test are considered probably signs, which are strongly suggestive of pregnancy.

During admission data collection, a nurse asks the client to run the heel of one foot down the lower anterior surface of the other leg. The nurse notices rhythmic tremors of the leg being tested and concludes that the client has interference in the area of:

Balance and coordination

A nurse is caring for a client dying of ovarian cancer. During care, the client states, "If I can just live long enough to attend my daughter's graduation, I'll be ready to die." Which phase of coping is this client experiencing?

Bargaining

The nurse is caring for the client with epididymitis. Which treatment modalities should be implemented? Select all that apply.

Bed rest Sitz bath Antibiotics Scrotal elevation

A client is diagnosed with thrombophlebitis. The nurse tells the client that which of the following will likely be prescribed?

Bedrest, with elevation of the affected extremity

A client who has a history of chronic ulcerative colitis is anemic. The nurse interprets that which factor is likely responsible for this laboratory finding?

Blood loss

A nurse is monitoring a client with a blunt head injury sustained from a motor vehicle crash. Which of the following would indicate a basal skull fracture as a result of the injury?

Bloody or clear drainage from the auditory canal

A nurse is performing tracheal suctioning on an assigned client. The nurse uses which parameter as the accurate indicator of the effectiveness of suctioning?

Breath sounds are now clear.

A client who underwent a bronchoscopy was returned to the nursing unit 1 hour ago. The nurse determines that the client is experiencing complications of the procedure if the nurse notes:

Breath sounds greater on the right than the left

A nurse is caring for a client following enucleation. Which post-surgical observation requires immediate attention by the nurse?

Bright red drainage on the dressin

A nurse is collecting data from a client with pneumonia. Chest auscultation over areas of consolidation reveals this breath sound. (Click on the sound icon.) The nurse would interpret this breath sound to be which of the following?

Bronchial breath sounds

A nurse in the emergency department is listening to the breath sounds of a client with respiratory distress and hears this sound. (Click on the sound icon.) The nurse determines that this finding is characteristic of which disorder?

Bronchitis

The nurse observes that a client with a nasogastric tube connected to continuous gastric suction is mouth breathing, has dry mucous membranes, and has a foul breath odor. In planning care, which nursing intervention would be best to maintain the integrity of this client's oral mucosa?

Brush the client's teeth frequently. Use diluted mouthwash and water to rinse the mouth

A client with Bell's palsy exhibits facial asymmetry and cannot close the eye completely on one side. The client is also drooling and has loss of tearing in one eye. The nurse documents that the client displays symptoms of involvement of which of the following cranial nerves (CNs)?

CN VII

A nurse evaluates the client following treatment for carbon monoxide poisoning. The nurse would document that the treatment has been successful when the:

Carboxyhemoglobin levels are less than 5%

A client's serum calcium level is 7.9 mg/dL. The nurse is immediately concerned, knowing that this level could lead to:

Cardiac arrest

A client is diagnosed with glaucoma. Which data gathered by the nurse indicate a risk factor associated with glaucoma?

Cardiovascular disease

A client experiences an episode of Bell's palsy and complains about increasing clumsiness. The nurse should prepare the client for which diagnostic study (studies) to determine the cause of the complaints? Select all that apply.

Cerebral angiography Lumbar puncture (LP) Computed tomography

A client is wearing a continuous cardiac monitor, which begins to alarm at the nurse's station. The nurse sees no electrocardiographic complexes on the screen. The nurse would first:

Check the client status and lead placement.

A nurse is caring for the client who has had skeletal traction applied to the left leg. The client is complaining of severe left leg pain. Which action should the nurse take first?

Check the client's alignment in bed.

The nurse monitoring a client receiving peritoneal dialysis notes that the client's outflow is less than the inflow. The nurse should take which actions? Select all that apply.

Check the level of the drainage bag Reposition the client to his or her side Place the client in good body alignment Check the peritoneal dialysis sytem for kinks

A client with a fractured femur is placed in skeletal traction. The nurse should do which to monitor for nerve injury?

Check the neurovascular status of the affected extremity.

A nurse is caring for a client following the application of a plaster cast because of a fractured left radius. The nurse monitors the neurovascular status of the client's casted extremity because:

Check the neurovascular status of the toes on the casted leg.

A nurse enters a client's room and finds the client slumped down in the chair. Breathing is shallow and a pulse is present. Based on these data, the nurse determines that the priority would be to:

Check the vital signs and level of consciousness.

A client has undergone esophagogastroduodenoscopy (EGD). The nurse places highest priority on which of the following items as part of the client's care plan?

Checking for return of a gag reflex

A nurse is teaching a client about foods in the diet that could minimize the risk of osteoporosis. The nurse would encourage the client to increase intake of which of these foods?

Cheese

The nurse is assisting a client with cystitis to select foods that are appropriate for an acid-ash diet. The nurse encourages the client to eat which food?

Cheese

A client is seen in the health care clinic and acute pyelonephritis is suspected. The nurse reviews the client's record and should expect to note which associated signs and symptoms documented? Select all that apply.

Chills General weakness Nausea and vomiting

A nurse is caring for a hospitalized client with a suspected diagnosis of tuberculosis (TB). Which finding does the nurse expect to note during data collection?

Chills and night sweats

A client's kidneys are retaining greater amounts of sodium. The nurse anticipates that the kidneys are also retaining greater amounts of which other substances?

Chloride and bicarbonate

A nurse has finished suctioning a client. The nurse would use which of the following parameters to best determine the effectiveness of suctioning?

Clear breath sounds

A nurse determines that which of the following clients is most likely to be a candidate for cardioversion?

Client with unstable rapid atrial fibrillation

A client with acquired immunodeficiency syndrome (AIDS) reports nausea, vomiting, and abdominal pain after beginning didanosine (Videx) therapy. The clinic nurse reinforces which instruction to this client?

Come to the health care clinic to be seen by the health care provider

A client with acquired immunodeficiency syndrome (AIDS) is taking didanosine. The client calls the nurse at the health care provider's office and reports nausea, vomiting, and abdominal pain. Which instruction should the nurse provide to the client?

Come to the office to be seen by the health care provider

A client has a halo vest that was applied following a C6 spinal cord injury. The nurse performs which of the following to determine whether the client is ready to begin sitting up?

Compares the client's pulse and blood pressure when both flat and sitting

This morning a client sustained a right proximal fibula and tibia fracture that was casted in a long leg plaster cast. During evening rounds, the nurse notes that the right lower extremity capillary refill is greater than 3 seconds and the toes are edematous and dusky. The client states that the pain medication is not working anymore and that the right foot feels like it is asleep. The nurse analyzes the data and determines that the client's symptoms are indicative of:

Compartment Syndrome

This morning a client sustained a right proximal fibula and tibia fracture that was casted in a long leg plaster cast. During evening rounds, the nurse notes that the right lower extremity capillary refill is greater than 3 seconds and the toes are edematous and dusky. The client states that the pain medication is not working anymore and that the right foot feels like it is asleep. The nurse analyzes the data and determines that the client's symptoms are indicative of:

Compartment syndrome

A nurse is caring for a client following the application of a plaster cast because of a fractured left radius. The nurse monitors the neurovascular status of the client's casted extremity because:

Compartment syndrome may lead to irreversible nerve and muscle tissue injury.

A nurse is assigned to care for a client with a diagnosis of detached retina. Which finding would indicate that bleeding has occurred as a result of retinal detachment?

Complaints of a burst of black spots or floaters

The nurse reads the chart of a client who has been diagnosed with stage 3 Lyme disease. Which sign/symptom supports this diagnosis?

Complaints of joint pain

A nurse is trying to communicate with a brain attack (stroke) client with aphasia. Which action by the nurse would be least helpful to the client?

Completing the sentences that the client cannot finish

A nurse is collecting data on a client with Parkinson's disease. Which finding indicates a serious complication of this disorder?

Congested cough and coarse rhonchi heard on auscultation

A nurse notes that the client's eyes are reddened, and the client states that an eye infection has been diagnosed. The nurse interprets that the client is most likely referring to infection of which of the following structures that provides a protective covering for the eye?

Conjunctiva

A client has epididymitis as a complication of a urinary tract infection (UTI). The nurse is giving the client instructions to prevent a recurrence. The nurse determines that the client needs further teaching if the client states the intention to do which?

Continue to take antibiotics until all symptoms are gone

The nurse is assisting in the care of a client diagnosed with systemic lupus erythematosus (SLE). The nurse should administer which prescribed medication that is needed to manage the condition?

Corticosteroid

The home care nurse is prescribing dressing supplies for a client who has an allergy to latex. The nurse asks the medical supply personnel to deliver which?

Cotton pads and silk tape

A clinic nurse periodically cares for a client diagnosed with acquired immunodeficiency syndrome. The nurse assesses for an early manifestation of Pneumocystis jiroveci infection by monitoring for which sign/symptom at each client visit?

Cough

A nurse is caring for a client who is suspected of having lung cancer. The nurse monitors the client for which most frequent early sign of lung cancer?

Cough

A nurse is performing nasotracheal suctioning of a client. The nurse determines that the client is adequately tolerating the procedure if which observation is made?

Coughing occurs with suctioning

A nurse is performing nasotracheal suctioning of a client. The nurse interprets that the client is adequately tolerating the procedure if which of the following observations are made?

Coughing occurs with suctioning.

A nurse is discussing signs of severe airway obstruction with a group of nursing students. Which one of the following signs would the nurse emphasize is one that indicates severe airway obstruction?

Cyanosis

31) During the prenatal visit, the client states that she has been experiencing heartburn frequently. The LPN provides instruction on the cause and prevention of heartburn. When she ask to verbalize understanding of the information, which of the following statements by the client indicates further instruction may be necessary? A. "The sphincter that normally prevents stomach contents from going back up into the esophagus is relaxed." B. "I should try to avoid drinking fluids while I'm eating." C. "Eating six or seven small meals a day may help my symptoms." D. "I'll eat enough to ensure that I am full at every meal."

D Explanation It suggests that the instruction might need to be reinforced on preventing stomach distention.

When talking with a pregnant client who is experiencing aching swollen, leg veins, the nurse would explain that this is most probably the result of which of the following? A. thrombophlebitis B. pregnancy induced hypertension C. pressure on blood vessels from the enlarging uterus D. the force of gravity pulling down on the uterus

D - Explanation Pressure of the growing uterus on blood vessels results in an increased risk for venous stasis in the lower extremities. Subsequently, edema and varicose vein formation may occur. Thrombophlebitis is an inflammation of the veins due to thrombus formation. Pregnancy-induced hypertension is not associated with these symptoms. Gravity plays only a minor role with these symptoms.

Pediazole is a suspension medication that contains 200 mg erythromycin and 600 mg sulfisoxazole per 5 mL. The physician orders Pediazole 4 mL PO every 12 hours. How many mg of sulfisoxazole is this client receiving in a 24-hour period? A. 160 mg B. 320 mg C. 480 mg D. 960 mg

D. Explanation 600 mg/ 5 mL = x mg/ 4 mL 2400 = 5x x= 2400/5 x= 480 mg per dose x 2 = 960 mg in 24 hours.

A client admitted to the hospital with a diagnosis of myocardial infarction (MI) tells the nurse that the pain likely resulted from the fried chicken sandwich that the client had for lunch. The nurse responds to the client, using the knowledge that:

Denial is a common occurrence early after MI.

A nurse is collecting data on a client with a diagnosis of right-sided heart failure. The nurse would expect to note which specific characteristic of this condition?

Dependent edema

A client is scheduled for intravenous pyelography (IVP). Which priority nursing action should the nurse take?

Determine a history of allergies

The nurse notes that a client's urinalysis report contains a notation of positive red blood cells (RBCs). The nurse interprets that this finding is unrelated to which item that is part of the client's medical record?

Diabetes mellitus

nurse is reviewing the record of a client with Crohn's disease. Which of the following stool characteristics would the nurse expect to see documented in the record

Diarrhea

A client with a possible hiatal hernia complains of frequent heartburn and regurgitation. Which sign or symptom would support this diagnosis?

Difficulty swallowing both liquids and solids

A nurse is collecting data from a client who has a history of untreated cataracts. The nurse checks the client for which associated manifestation

Difficulty with driving a car at night

The nurse is speaking with a client who underwent a minimally invasive procedure treatment for recurrent urolithiasis. Which instructions are appropriate to reinforce in the teaching plan? Select all that apply.

Drink at least 3000 mL of fluid each day Complete the full course of prescribed antibiotics Filter urine and collect any stones to take to the urological health care provider

A client is resuming a diet after partial gastrectomy. To minimize complications, the nurse would tell the client to avoid doing which of the following?

Drinking liquids with meals

A nurse has provided discharge instructions to a client with an application of a halo device. The nurse determines that the client needs further clarification of the instructions if the client states that he or she will:

Drive only during the daytime.

A nurse is caring for a client with osteoarthritis. The nurse collects data, knowing that which of the following is a clinical manifestation associated with the disorder?

Dull aching pain in the affected joints

A nurse is admitting a client with a possible diagnosis of chronic bronchitis. The nurse collects data from the client and notes that which of the following signs supports this diagnosis? Select all that apply.

Early onset cough Purulent mucus production Mild episodes of dyspnea

A nurse is providing dietary instructions to a client with peptic ulcer disease. The nurse tells the client to:

Eat anything that does not aggravate or cause pain.

A nurse documents that a client with a hiatal hernia is implementing effective health maintenance measures after the client reports doing which of the following

Eating low-fat or nonfat foods

A nurse is admitting a client with Guillain-Barré syndrome to the nursing unit. The client has an ascending paralysis to the level of the waist. Knowing the complications of the disorder, the nurse brings which of the following items into the client's room?

Electrocardiographic monitoring electrodes and intubation tray

A nurse is caring for a client with fresh application of a plaster leg cast. The nurse plans to prevent the development of compartment syndrome by:

Elevating the limb and applying ice to the affected leg

A client has the following laboratory values: pH of 7.55, HCO3 of 22 mm Hg, and a Pco2 of 30 mm Hg. What should the nurse do?

Encourage the client to slow down his breathing.

A nurse is assisting with developing a plan of care for the client with multiple myeloma. Which of the following is a priority nursing intervention for this client?

Encouraging fluids

A nurse is planning care for a client who displays confusion secondary to a neurological problem. Which approach by the nurse would be least helpful in assisting this client?

Encouraging multiple visitors at one time

A client with right leg hemiplegia is experiencing difficulty with mobility. The nurse determines that the family needs reinforcement of teaching if the nurse observes which of the following being done by the family?

Encouraging the client to stand unassisted on the leg

A nurse is caring for a client who has had an open reduction with internal fixation (ORIF) with a posterior approach. The client has been prescribed hip precautions. The nurse plans to implement which of the following in the care of the client? Select all that apply.

Ensure the client doesn't sit or stand for long periods of time. Ensure the client doesn't cross the legs past the midline of the body. Ensure the client uses assistive/adaptive devices with activities of daily living.

A client is at risk for complications of heart failure. What is the nurse's priority for early detection of the most likely cause of complications with this client?

Evaluating total body fluid

A nurse is caring for a client diagnosed with Bell's palsy 1 week ago. Which of the following data would indicate a potential complication associated with Bell's palsy?

Excessive tearing

A client recovering from a head injury is arousable and participating in care. The nurse determines that the client understands measures to prevent elevations in intracranial pressure (ICP) if the nurse observes the client doing which of the following activities?

Exhaling during repositioning

A client who has undergone cataract removal without an intraocular lens implant is visibly upset because his vision is still blurry. The nurse should do which of the following to provide realistic reassurance to this client?

Explain that vision will improve with adjustment to aphakic lenses

Which clinical manifestation is observed in the clonic phase of a seizure?

Extension spasms of the body

A nurse determines that motor function of which cranial nerve is intact if the client can perform this action? Refer to figure.

Facial

A nurse working the 3:00 to 11:00 PM shift notes that a client with coronary artery disease (CAD) has a prescription for serum lipid levels to be drawn in the morning. The nurse places the client on which dietary preparation to ensure accurate test results?

Fasting for 12 hours

A client has just been told by the health care provider about her diagnosis of breast cancer. The client responds, "Oh no, does this mean I'm going to die?" The nurse interprets that the client's initial reaction is one of:

Fear

A client is admitted to the hospital with severe jaundice and is having diagnostic testing. Because the client has no complaints of fatigue, the client is encouraged to ambulate in the hall to maintain muscle strength. The client paces around the room but will not enter the hall. Which of the following problems most likely is the reason for the client's reluctance to walk in the hall?

Feeling self-conscious about appearance

The nurse is collecting data from a client with epididymitis. The nurse should expect to note which signs and symptoms of this problem?

Fever, nausea and vomiting, and painful scrotal edema

A client presents to the urgent care center with a chemical burn of the right eye. The first priority of the nurse is to prepare the client for:

Flushing the right eye with copious amounts of sterile solution

A nurse in a health care clinic is assisting to test the client for accommodation. Initially, the nurse should ask the client to:

Focus on a distant object.

The nurse is assisting in preparing a plan of care for a client with acquired immunodeficiency syndrome (AIDS) who has nausea. Which dietary measure should the nurse include in the plan?

Foods that are at room temperature

A nurse is assisting a hospitalized client who is newly diagnosed with coronary artery disease (CAD) to make appropriate selections from the dietary menu. The nurse encourages the client to select which of the following meals?

Fresh strawberries, steamed vegetables, and baked fish

A licensed practical nurse (LPN) is assisting in insertion of a nasogastric (NG) tube for an adult client. The LPN helps determine the correct length to insert the tube by measuring

From the tip of the client's nose to the earlobe and then down to the xiphoid process

A nurse is assisting in caring for a postoperative client who had a pneumonectomy. The nurse monitors the client for which of the following adverse signs and symptoms, indicating acute pulmonary edema?

Frothy sputum

A nurse carries out a standard prescription for a stat electrocardiogram (ECG) on a client who has an episode of chest pain. The nurse should take which action next?

Give sublingual nitroglycerin (Nitrostat) per the health care provider's prescriptions.

The nurse is caring for a client who has been diagnosed as having a kidney mass. The client asks the nurse the reason for a renal biopsy, when other tests such as computed tomography (CT) and ultrasound are available. In formulating a response, the nurse incorporates the knowledge that a renal biopsy serves which purpose?

Gives specific cytological information about the lesion

A client who is recovering from a brain attack (stroke) has residual dysphagia. The licensed practical nurse instructs the nursing assistant to avoid which of the following at mealtime?

Giving the client thin liquids

A client with a brain attack (stroke) has residual dysphagia. When a diet prescription is initiated, the nurse avoids doing which of the following?

Giving the client thin liquids

A client with known coronary artery disease (CAD) begins to experience chest pain while getting out of bed. The nurse should take which action first?

Have the client stop and lie back down in bed.

A client newly diagnosed with chronic kidney disease has recently begun hemodialysis. Which are signs/symptoms of disequilibrium syndrome?

Headache, deteriorating level of consciousness, and twitching

A nurse is preparing to administer an intermittent tube feeding to a client with a nasogastric tube. The nurse checks the residual and obtains an amount of 200 mL. The nurse would:

Hold the feeding

A nurse is collecting neurological data on a post-stroke adult client. Which of the following techniques will the nurse perform to adequately check proprioception?

Hold the sides of the client's great toe and, while moving it, ask what position it is in.

Which conditions places the client at risk for developing acute postrenal failure?

Hydronephrosis

A nurse is told during shift report that a client is having occasional ventricular dysrhythmias. The nurse reviews the client's laboratory results, recalling that which electrolyte imbalance could be responsible for this development?

Hypokalemia

A nurse is providing preoperative instructions to a client scheduled for cataract surgery and prepares a written list of instructions for the client. Which statement by the client indicates a need for further instruction?

I can drink any liquids that I want to on the morning of the surgery."

A client who has undergone a barium enema is being readied for discharge from the ambulatory care unit. The nurse determines that the client has understood the discharge instructions if the client states:

I should take a laxative and my stool should return to normal color."

A nurse is caring for a client with a chest tube who accidentally disconnects the tube from the drainage system when trying to get out of bed. The nurse should take which action first?

Immerse the end of the tube in sterile saline

A nurse witnesses a client sustain a fall and suspects that the client's leg may be fractured. Which action is the priority?

Immobilize the leg before moving the client.

Which findings should cause the nurse to postpone administration of an immunization and do further data collection? Select all that apply.

Immune deficiency disease Familial history of severe allergic response to immunization

A client has sustained a closed fracture and has just had a cast applied to the affected arm. The client is complaining of intense pain. The nurse has elevated the limb, applied an ice bag, and administered an analgesic, which was ineffective in relieving the pain. The nurse interprets that this pain may be caused by:

Impaired tissue perfusion

A nurse has been assigned to a client with a hearing impairment. To enhance nurse-client communication, the nurse should plan to communicate with the client by speaking:

In a normal tone while facing the client

A client seeks medical attention for intermittent episodes in which the fingers of both hands become cold, pale, and numb. The client states that they then become reddened and swollen with a throbbing, achy pain. The nurse further collects data on the client to see whether these episodes occur with:

Ingestion of coffee or chocolate

A client being measured for crutches asks the nurse why the crutches cannot rest up underneath the arm for extra support. The nurse's response is based on the understanding that this could result in:

Injury to the brachial plexus nerves

A nurse is preparing a client for the administration of a Mantoux skin test. The nurse determines that which body area is the appropriate area for injection of the medication? Select all that apply.

Inner aspect of the forearm Dorsal aspect of the upper arm Away from heavy pigmentation

A client sustains a chemical eye injury from a splash of battery acid. The nurse prepares the client for which immediate measure?

Irrigating the eye with sterile normal saline

The nurse is providing instructions to a client with acquired immunodeficiency syndrome (AIDS) who is experiencing night fever and night sweats. The nurse advises the client to do which action to increase comfort while minimizing symptoms?

Keep liquids on the nightstand at home

This is a dark streak down the midline of the abdomen that may appear as the uterus is enlarging. The LPN correctly describes this to the pregnant woman as?

LINEA NIGRA

A nurse is assisting to care for a client who has sustained a nasal fracture. The nurse monitors for which priority finding specifically related to this injury?

Leakage of clear fluid from the nose.

A nurse is reviewing the laboratory results of a client who is receiving chemotherapy and notes that the platelet count is 10,000 cells/mm3. On the basis of this laboratory value, the priority action is to monitor which of the following?

Level of consciousness

A client has sought treatment in the ambulatory care clinic after an insect has become trapped in the external ear canal. The nurse prepares to assist the health care provider to instill which acceptable solutions into the ear to remove the insect? Select all that apply.

Lidocaine Mineral oil Ether solution

The nurse is reinforcing dietary instructions to a client diagnosed with acute glomerulonephritis. The nurse determines that the client understands the information presented if the client states the intention to do which action?

Limit protein intake

The nurse is caring for the client immediately after insertion of a permanent demand pacemaker via the right subclavian vein. The nurse prevents dislodgement of the pacing catheter by implementing which intervention?

Limiting movement and abduction of the right arm

A client with tuberculosis is being started on antitubercular therapy with isoniazid (INH). The nurse reviews the client's health care record to be sure that which of the following baseline studies have been completed before giving the client the first dose?

Liver enzymes

A nurse has assisted in the insertion of a Levin tube for gastrointestinal (GI) decompression. The nurse plans to set the suction to which of the following pressures?

Low and intermittent

A nurse has assisted with the insertion of a Levin tube for gastrointestinal (GI) decompression. The nurse expects that the health care provider will prescribe the suction setting at:

Low and intermittent

A client has been diagnosed with acute gastroenteritis. Which of the following diets should the nurse anticipate would be prescribed for the client

Low fiber

A nurse is preparing to provide a therapeutic environment for a client who recently had a myocardial infarction (MI). The nurse should alter the environment to ensure that it is:

Low stimulus, low stress

A client with hiatal hernia chronically experiences heartburn after meals. The nurse would teach the client to avoid which of the following, which is contraindicated with hiatal hernia?

Lying recumbent after meals

Which symptoms should the nurse expect and monitor for in clients with Stage 4 human immunodeficiency virus (HIV) infection? Select all that apply.

Lymphoma Kaposi sarcoma Candidiasis of the esophagus

The nurse is caring for a client who received a recent kidney transplant. Besides actual rejection of the transplant, which are some of the most important complications this client is at risk for? Select all that apply.

Malignancies Cardiovascular disease Susceptibility to infection Corticosteroids-related complicationa

A client with cancer is receiving chemotherapy and develops thrombocytopenia. Which intervention is a priority in the nursing plan of care?

Monitor the client for bleeding.

A nurse is caring for a client who was admitted to the hospital with a fractured right femur sustained from a fall 5 hours ago. The client's plan of care includes interventions related to monitoring for signs of fat embolism. The nurse provides appropriate care by:

Monitoring for signs of dyspnea

Which observations by the nurse caring for clients on a hospital medical-surgical unit should be immediately reported to the health care provider? Select all that apply.

New confused mental state and pulse rate of 106 bpm in a 72 year old client A volume of 105 mL of urine over 4 hours in the collection bag of a 1-day postoperative client

A client contacts the health care provider's office to report she is not feeling well, has burning with urination, and suspects she may have a urinary tract infection. The nurse instructs the client to collect a urine specimen for testing. Which urinalysis findings indicate the presence of a urinary tract infection? Select all that apply.

Nitrites, present White blood cells, 10 Leukoesterase, present

A nurse is assisting the health care provider in performing a caloric test on a client. Following instillation of warm water into the ear, the client complains of vertigo. The nurse documents the findings of this test as:

Normal

A nurse is reviewing the results of an eye examination on a client and notes that results from the tonometry test indicate an intraocular pressure of 20 mm Hg. The nurse interprets these findings as:

Normal intraocular pressure

A client who has just suffered a large flail chest is experiencing severe pain and dyspnea. The appropriate nursing action would be to:

Notify the registered nurse.

A client with a left arm fracture complains of severe, diffuse pain that is unrelieved with pain medication. On further data collection, the nurse notes that the client experiences more pain during passive motion of the left arm as compared with active motion. Based on these findings, the nurse should take which action?

Notify the registered nurse.

A nurse is assisting in planning care for a client scheduled for insertion of a tracheostomy. What equipment would the nurse plan to have at the bedside when the client returns from surgery?

Obturator

A client who has been taking indomethacin (Indocin) for gout has a prescription for guaiac testing of the stool. The nurse explains to the client that this test is necessary because it detects which of the following that may be caused or affected by this medication?

Occult blood

A nursing instructor asks a nursing student about the characteristics of Hodgkin's disease. The instructor determines that the student needs to read about the characteristics of this disease if the student states that which of the following is an associated characteristic?

Occurrence most often in older adults

The nurse notes that the client's physical examination record states the client's eyes moved normally through the six cardinal fields of gaze. The nurse interprets that this means that the client has normal:

Ocular movements

A nurse working in a long-term care facility is approached by the son of a resident, who wants his 78-year-old father to have a heating pad, because "his feet are always cold at night." The nurse should incorporate which of the following concepts when formulating a response to the family member?

Older adults often have slower neurological response times and are therefore more at risk for burns.

An older client is at risk for falls. When developing an individualized plan of care for this client, the nurse recalls that which concept is least relevant to maintenance of balance for the older client?

Older clients cannot think quickly enough to respond to emergencies.

A client who underwent a kidney transplant 6 months earlier is seen in the clinic for a routine monthly appointment. The nurse reviews how the client has been doing and observes for signs/symptoms of acute rejection. Which signs/symptoms suggest acute rejection of the transplanted kidney? Select all that apply.

Oliguria Elevation of blood pressure over baseline Abdominal tenderness on the side of the kidney transplant Elevation of serum blood urea nitrogen (BUN) and creatinine

A client arrives in the emergency department after an automobile crash. The client's forehead hit the steering wheel and a hyphema has been diagnosed. The nurse would prepare to position the client:

On bedrest in a semi-Fowler's position

A nurse in the recovery room area is preparing to care for a client following cataract extraction of the right eye. The nurse prepares to position the client:

On the left side with the head of the bed elevated

A nurse is assigned to care for a client following a cataract extraction. The nurse plans to position the client:

On the nonoperative side

A nurse is preparing a plan of care for the client who will be returning from surgery following a right lung wedge resection. Included in the plan of care is that in the postoperative period, the nurse should avoid positioning this client:

On the right side

A nurse is assigned to care for a client after a left pneumonectomy. Which one of the follow positions would be contraindicated for this client?

On the side

A licensed practical nurse (LPN) is providing follow-up teaching after a client underwent an upper gastrointestinal (GI) series. The nurse reminds the client that the stools will remain white for approximately:

One day

A client susceptible to motion sickness asks the nurse about the use of medication to prevent an occurrence. The nurse plans to incorporate into the discussion that the medication works effectively if it is taken at least:

One hour before a triggering event

A nurse is instructing a group of female clients about breast self-examination (BSE). The nurse would instruct the clients to perform the examination:

One week after menstruation begins

The nurse is caring for a client who is anxious and is experiencing dyspnea and restlessness from hypoxemia associated with pulmonary edema. Auscultation of the lungs reveals these breath sounds. (Click on the sound icon.) The nurse determines that these breath sounds usually are caused by which of the following?

Opening of small airways that contain fluid

The nurse is admitting a client to the nursing unit who has returned from the postanesthesia care unit following prostatectomy. The client has a three-way Foley catheter with continuous bladder irrigation. The nurse should maintain the flow rate of the continuous bladder infusion to maintain which urine output characteristic?

Pale yellow or slightly pink

A nurse is caring for a client who is developing pulmonary edema. The client exhibits respiratory distress, but the blood pressure is unchanged from the client's baseline. As an immediate action before help arrives, the nurse would:

Place the client in high-Fowler's position.

A nurse is reviewing the laboratory results of a client receiving chemotherapy for cancer. The nurse reports which abnormal result to the health care provider?

Platelet count, 40,000 cells/mm3

A client is admitted to the hospital with acute exacerbation of chronic obstructive pulmonary disease (COPD) and has an arterial blood gas test done. Which of the following results would the nurse expect to note?

Po2 of 60 mm Hg and Pco2 of 50 mm Hg

A nurse is providing instructions to a client with osteoporosis regarding appropriate food items to include in the diet. The nurse tells the client that which food item would provide the least amount of calcium?

Pork

nurse is reviewing the health care provider's prescriptions written for a client admitted with acute pancreatitis. Which health care provider prescription would the nurse verify if noted on the client's chart?

Position the client supine and flat

A client receiving parenteral nutrition (PN) has a history of congestive heart failure. The health care provider has prescribed furosemide (Lasix) 40 mg orally daily to prevent fluid overload. The nurse monitors which laboratory value to identify an adverse effect from this medication?

Potassium

A nurse is preparing to ambulate a postoperative client after cardiac surgery. The nurse plans to do which of the following to enable the client to best tolerate the ambulation?

Premedicate the client with an analgesic before ambulating.

Acetylsalicylic acid (Aspirin) is prescribed for a client before a percutaneous transluminal coronary angioplasty (PTCA). When the nurse takes the aspirin to the client, the client asks the nurse about its purpose. The nurse informs the client that the aspirin will:

Prevent the formation of clots.

A nurse is instructing a client about pursed lip breathing, and the client asks the nurse about its purpose. The nurse tells the client that the primary purpose of pursed lip breathing is to:

Promote carbon dioxide elimination

A client has just undergone lumbar puncture (LP). The nurse assists the client into which most optimal position if tolerated by the client?

Prone, with a pillow under the abdomen

A client admitted to the hospital with a neurological problem indicates to the nurse that magnetic resonance imaging (MRI) may be done. The nurse interprets that the client may be ineligible for this diagnostic procedure based on the client's history of:

Prosthetic valve replacement

A nurse is reinforcing instructions to a client following a total laryngectomy about caring for the stoma. Choose the instructions that the nurse provides to the client. Select all that apply.

Protect the stoma from water. Soaps should be avoided near the stoma. Wash the stoma daily using a washcloth. Apply a thin layer of petroleum jelly to the skin surrounding the stoma.

A nurse is caring for a client who will be undergoing surgical treatment for Ménière's disease. The nurse plans care, understanding that surgical treatment for this disorder is performed to:

Provide relief from accumulation of inner ear fluid in the endolymphatic sac

A client with acquired immunodeficiency syndrome (AIDS) is experiencing shortness of breath related to Pneumocystis jiroveci pneumonia. Which measure should the nurse suggest to assist the client in performing activities of daily living?

Provide supportive care with hygiene needs

A client is scheduled for a digital subtraction angiography. The nurse supports the client's understanding that the test is directed toward which outcome?

Providing information about the blood vessels

A nurse is preparing to instill an otic solution into the adult client's right ear. The nurse would include which action while performing this procedure? Select all that apply.

Pulling the auricle of the right ear upward Pulling the auricle of the right ear forward Warming the solution to room temperature Placing the client in a left side-lying position

A nurse is preparing to administer eardrops to an adult client. The nurse administers the eardrops by:

Pulling the pinna up and back

A client diagnosed with thrombophlebitis 1 day ago suddenly complains of chest pain and shortness of breath, and the client is visibly anxious. The nurse understands that a life-threatening complication of this condition is:

Pulmonary embolism

A nurse is performing an abdominal assessment on a client. The nurse interprets that which finding is abnormal and should be reported to the registered nurse (RN) or health care provider?

Pulsation between the umbilicus and pubis

A nurse assisting in caring for a client hospitalized with acute pericarditis is monitoring the client for signs of cardiac tamponade. The nurse determines that which finding is unrelated to possible cardiac tamponade?

Pulse rate of 58 beats per minute

The client with acquired immunodeficiency syndrome is diagnosed with cutaneous Kaposi's sarcoma. Based on this diagnosis, the nurse understands that this has been confirmed by which?

Punch biopsy of the cutaneous lesions

A nurse is planning to institute seizure precautions for a client who is being admitted from the emergency department. Which of the following measures should the nurse avoid in planning for the client's safety?

Putting a padded tongue blade at the head of the bed

A clinic nurse has given a client the materials needed to test the stool for occult blood as part of a routine screening for colorectal cancer. When the client asks the nurse whether there are any special precautions that must be followed in doing this test, the nurse tells the client to avoid eating which of the following for at least a day before performing the test?

Red meat

A nurse is preparing a client with a bowel tumor for surgery. The health care provider has informed the client that the surgery is palliative in the treatment of the tumor. The nurse understands that this type of surgery is performed to:

Reduce pain

A nurse is monitoring a client with a spinal cord injury who is experiencing spinal shock. Which of the following will provide the nurse with the best information about recovery from the spinal shock?

Reflexes

A nurse is assisting in caring for a client receiving chemotherapy. On review of the morning laboratory results, the nurse notes that the white blood cell count is extremely low, and the client is immediately placed on neutropenic precautions. The client's breakfast tray arrives, and the nurse inspects the meal and prepares to bring the tray into the client's room. Which of the following actions should the nurse take before bringing the meal to the client?

Remove the fresh orange from the breakfast tray.

A nurse has provided instructions to the client returning home after arthroscopy of the knee. The nurse determines that the client understands the instructions if the client states that he or she will:

Report fever or site inflammation to the health care provider.

During the early postoperative stage, the cataract extraction client complains of nausea and severe eye pain over the operative site. Which action by the nurse is appropriate

Report the client's complaints.

A client has sustained multiple fractures in the left leg and is in skeletal traction. The nurse has obtained an overhead trapeze for the client's use to aid in bed mobility. The nurse would pay particular attention to monitoring for which of the following high-risk areas for pressure and breakdown?

Right heel

A nurse is reviewing the health care record of a client with a diagnosis of otosclerosis. The nurse would expect to note documentation of which early symptom of this disorder?

Ringing in the ears

A nurse has given instructions to the client with Parkinson's disease about maintaining mobility. The nurse determines that the client understands the directions if the client states that he or she will:

Rock back and forth to start movement with bradykinesia.

A nurse is assigned to assist in caring for a client with a chest tube drainage system. In planning for the client, the nurse makes certain that which of the following is available, in the event that the drainage system needs to be changed?

Rubber-shod clamps

A nurse is assisting in caring for a client in the telemetry unit who is receiving an intravenous infusion of 1000 mL 5% dextrose with 40 mEq of potassium chloride. Which occurrence observed on the cardiac monitor indicates the presence of hyperkalemia?

ST segment depressions

A nurse is assisting in caring for a client in the telemetry unit and is monitoring the client for cardiac changes indicative of hypokalemia. Which occurrence noted on the cardiac monitor indicates the presence of hypokalemia?

ST-segment depression

The nurse caring for an older adult client understands that which of the following can increase disorientation in this client? Select all that apply.

Sedatives Anesthesia Physical restraints Analgesics

A nurse assigned to care for a client with cirrhosis reviews the medical record and notes that the client has difficulty maintaining an effective breathing pattern due to pressure on the diaphragm. The nurse plans care, knowing that which client position will best assist in facilitating breathing?

Semi-Fowler's

A nurse is assisting with caring for a client after a craniotomy. The nurse plans to position the client in a:

Semi-Fowler's position

A nurse is caring for the client who has suffered spinal cord injury. The nurse further monitors the client for signs of autonomic dysreflexia and suspects this complication if which of the following is noted?

Severe, throbbing headache

A client is having a lumbar puncture (LP) performed. The nurse would place the client in which position for the procedure?

Side-lying, with legs pulled up and head bent down onto the chest

A nurse reviews the laboratory results for a client diagnosed with leukemia who is receiving chemotherapy. The nurse notes that the white blood cell (WBC) count is 2000 cells/mm3. The nurse identifies the results as:

Signifying leukopenia

A nurse is providing instructions to a hospitalized client with a diagnosis of emphysema about positions that will enhance the effectiveness of breathing during dyspneic periods. Which position will the nurse instruct the client to assume?

Sitting on the side of the bed, leaning on an overbed table

The client with acquired immunodeficiency syndrome has raised, dark purplish lesions on the trunk of the body. The nurse anticipates that which procedure will be done to confirm whether these lesions are due to Kaposi's sarcoma?

Skin biopsy

A nurse has applied a hypothermia blanket to a client with a fever. The nurse should inspect the skin frequently to detect which complication of hypothermia blanket use?

Skin breakdown

A nurse is assisting in admitting a client to the emergency department with suspected carbon monoxide poisoning. The nurse understands that which of the following manifestations is least reliable for determining the oxygenation status of this client?

Skin color

A client calls the health care clinic and tells the nurse that he was bitten by a tick. The client is concerned and asks the nurse about the first signs of Lyme disease. Which is a characteristic of stage 1 of Lyme disease?

Skin rash

A client is diagnosed with stage 1 Lyme disease. The nurse checks the client for which hallmark characteristic of this stage?

Skin rash

A client has been placed in Buck's extension traction. The nurse can provide for countertraction by:

Slightly elevating the foot of the bed

A nurse is monitoring the respiratory status of a client who has suffered a fractured rib. The nurse monitors the client and understands that which manifestation is unrelated to the rib fracture?

Slow, deep respirations

The nurse is reinforcing dietary instructions to a client with renal calculi who must learn to eat an alkaline-ash diet. The nurse determines that the client has properly understood the information presented if the client chooses which selection from a diet menu?

Spinach, milk and a banana

The nurse is preparing to communicate with an older client who is hearing impaired. Which intervention should be implemented initially?

Stand in front of the client.

A nurse is reviewing the health care record of a client with a diagnosis of chronic pancreatitis. Which data noted in the record indicate poor absorption of dietary fats?

Steatorrhea

The nurse is caring for a hemodialysis client who has been receiving treatment for several years and is not a candidate for kidney transplant. The nurse knows that the majority of deaths of hemodialysis clients are related to which causes? Select all that apply.

Stroke Infectious complications Myocardial infarction (MI)

A male client with a history of ear problems telephones the ambulatory care nurse to cancel an appointment because he will be away on business. The client mentions that he will be flying during this trip. The nurse advises the client to engage in which of the following to prevent barotrauma during takeoff and landing? Select all that apply.

Sucking on a piece of hard candy Swallowing a few times Yawning occasionally Chewing gum

A nurse is caring for a client with an endotracheal tube attached to a ventilator. The high-pressure alarm sounds on the ventilator. The nurse prepares to perform which nursing intervention?

Suction the client.

A nursing student is collecting data on a client recently diagnosed with meningitis. The student expects to note which of the following signs and symptoms? Select all that apply.

Tachycardia Photophobia Red, macular rash Positive Kernig's sign

A nurse is providing a list of instructions to a client who is scheduled to have an electroencephalogram (EEG). Choose the instructions that the nurse places on the list. Select all that apply.

Tea and coffee are restricted on the day of the test. The test will take between 45 minutes and 2 hours. The hair should be washed the evening before the test.

A nurse is reviewing the health care provider's prescriptions for a client admitted to the hospital with a diagnosis of an acute attack of Ménière's disease. Which prescription, if noted on the client's chart, should the nurse question?

The administration of a vasoconstrictor

A nurse is collecting data on a client admitted to the hospital with suspected carbon monoxide poisoning and notes that the client behaves as if intoxicated. The nurse interprets that:

The behavior is likely the result of hypoxia.

A nurse is instructing a client to perform a testicular self-examination (TSE). Which instruction would the nurse provide to the client?

The best time for the examination is after a shower.

A client who is learning to use a cane is afraid it will slip with ambulation, causing a fall. The nurse provides the client with the greatest reassurance by telling the client that:

The cane has a flared tip with concentric rings to provide stability.

A nurse is assessing the chest tube drainage system of a postoperative client who had a right upper lobectomy. The closed drainage system has 300 mL of bloody drainage, and the nurse notes intermittent bubbling in the water-seal chamber. One hour following the initial assessment, the nurse notes that the bubbling in the water-seal chamber is now constant and the client appears dyspneic. Based on these findings, the nurse should first check:

The chest tube connections

A nurse is monitoring a client with a closed chest tube drainage system and notes fluctuation of the fluid level in the water-seal chamber during inspiration and expiration. On the basis of this finding, the nurse determines that:

The chest tube is functioning as expected.

A client's vision is tested with a Snellen chart. The results of the test are documented as 20/60. The nurse interprets this as:

The client can read at a distance of 20 feet what a client with normal vision can read at 60 feet.

A client with a T4 spinal cord injury is to be monitored for autonomic dysreflexia (hyperreflexia). Which finding is indicative of this complication?

The client complains of a headache and the blood pressure is elevated.

A nurse is collecting data from a client with varicose veins. Which finding would the nurse identify as an indication of a potential complication associated with this disorder?

The client complains of leg edema, and skin breakdown has started.

A nurse teaches a client how to use an incentive spirometer. Which observation would indicate the ineffective use of this equipment by the client?

The client is breathing through the nose.

A client with quadriplegia complains bitterly about the nurse's slow response to the call light and the rigidity of the therapy schedule. Which interpretation of this behavior would serve as a basis for planning nursing care?

The client is reacting to loss of control.

A client with acquired immunodeficiency syndrome has a respiratory infection from Pneumocystis jiroveci and a client problem of impaired gas exchange written in the plan of care. Which indicates that the expected outcome of care has not yet been achieved?

The client limits fluid intake

The use of peritoneal dialysis for the treatment of chronic kidney disease would be contraindicated for which clients?

The client with chronic severe emphysema

A nurse is planning to reinforce instructions to the client about proper use of a thoracolumbosacral orthosis (TLSO) after spinal fusion with instrumentation. The nurse plans to include which of the following teaching points in discussion with the client?

The device is applied before getting out of bed in the morning.

A nurse is caring for a client that is comatose and notes in the client's chart that the client is exhibiting decerebrate posturing. The nurse understands that decerebrate posturing is characterized by:

The extension of the extremities and pronation of the arms

A client being discharged from the hospital to home with a diagnosis of tuberculosis (TB) is worried about the possibility of infecting the family and others. The nurse determines that the client would get the most reassurance from the knowledge that:

The family will receive prophylactic therapy, and the client will not be contagious after 2 to 3 consecutive weeks of medication therapy.

A nurse is teaching a client who is to have a gallium scan about the procedure. The nurse should include which of the following items as part of the instructions?

The gallium will be injected intravenously 2 to 3 hours before the procedure.

A client who is prescribed zidovudine (Retrovir) has been diagnosed with severe neutropenia. The nurse anticipates which intervention should be implemented?

The medication will be temporarily discontinued

A nurse working in a long-term care facility is collecting data from a client experiencing chest pain. The nurse would interpret that the pain is likely a result of myocardial infarction (MI) if which of the following observations is made by the nurse?

The pain has not been unrelieved by rest and nitroglycerin tablets.

A client is admitted to the hospital for observation with a probable minor head injury after an automobile crash. The nurse would plan on leaving the cervical collar in place until:

The result of spinal x-rays is known.

The nursing student and clinical instructor are performing tracheotomy suction at the bedside of an adult client with a tracheostomy. Which action by the nursing student is incorrect, causing the clinical instructor to intervene?

The student suctions the client's tracheotomy tube for 15 seconds

A nurse checks the water seal chamber of a closed chest drainage system and notes fluctuations in the chamber. The nurse analyzes this finding as indicative of which of the following?

The system is functioning as expected

A client is admitted to the nursing unit following a lobectomy. The nurse caring for the client notes that, in the first hour after admission, the chest tube drainage was 75 mL. During the second hour, the drainage dropped to 5 mL. The nurse interprets that:

The tube may be occluded.

A nurse when inspecting the stoma of a client following an ureterostomy 6 hours ago, notes that the stoma appears pale in color. Which interpretation does the nurse make based on this finding?

The vascular supply to the stoma is insufficient.

A client had thoracic surgery 2 days ago and has a chest tube in place connected to a Pleur-Evac drainage system. The nurse notes continuous bubbling in the water seal chamber. The nurse determines that:

There is a leak in the system, which requires immediate investigation and correction.

A client with glaucoma has suffered significant eye damage prior to diagnosis and now has impaired vision. The nurse determines that the client needs further assistance in adapting to this situation if the client states that:

There is no difficulty driving at dusk.

Which statement by the spouse of a client with end-stage liver failure indicates the need for additional teaching by the multidisciplinary team regarding the management of pain?

This opioid will cause very deep sleep, which is what my husband needs."

A client is diagnosed with a disorder involving the inner ear. The nurse caring for the client understands that which of the following is the most common client complaint associated with a disorder involving the inner ear

Tinnitus

In order to assess the dorsalis pedis pulse of a client diagnosed with arterial vascular disease, the nurse palpates which anatomical location? Refer to figure.

Top of the foot

A client experiencing a pleural effusion had a thoracentesis. Analysis of the extracted fluid revealed a high red blood cell count. The nurse interprets that this result is consistent with:

Trauma

A client is diagnosed with nasal polyps, asthma, and an acetylsalicylic acid (aspirin) allergy. The nurse provides home care instructions, based on the knowledge that the client has:

Triad disease

A health care provider asks a nurse to obtain a Salem Sump tube for gastric intubation. The nurse selects which of the following tubes from the unit storage area?

Tube with a lumen and an air vent

The nurse is collecting data on a client with rheumatoid arthritis. The nurse looks at the client's hands and notes these characteristic deformities. The nurse identifies this deformity as which? Refer to figure.

Ulnar Drift

The nurse is assessing a client with suspected acute kidney injury. Which finding would support a diagnosis of acute intrarenal failure?

Urine analysis positive for casts and cellular debris

A client has a cerebellar lesion. The nurse determines that the client is adapting successfully to this problem if the client demonstrates proper use of which of the following items?

Walker

The nurse is preparing to administer eye drops. Select the interventions that the nurse takes to administer the drops. Select all that apply.

Wash hands. Put on gloves. Place the drop in the conjunctival sac. Pull the lower lid down against the cheek bone.

A nurse is reinforcing discharge instructions to a client going home after same-day eye surgery. During the postoperative period, the nurse stresses that the client may safely:

Watch television.

A nurse is collecting data from a client who is being seen in the health care clinic. The client is complaining of unrelieved back pain that has persisted over the past 3 months. The nurse determines that which of the following harmful effects can occur as a result of uncontrolled muscle pain?

Weakness

A client has experienced several episodes of sickle cell crisis. Which instruction should be included in the client's teaching plan to prevent recurrence?

Wear shoes and socks when walking outside to prevent damage to the feet.

A nurse is listening to the client's breath sounds and hears musical whistling noises on inspiration and expiration scattered throughout the right lung fields. The nurse interprets that this client has:

Wheezes

An older gentleman is brought to the emergency department by a neighbor who heard him talking and wandering in the street at 3 AM. The nurse should first determine which of the following about the client?

Whether this is a change in his usual level of orientation

A client has just returned from the cardiac catheterization laboratory. The left femoral vessel was used as the access site. After returning the client to bed and collecting initial data, the nurse places a sign above the bed stating that the client should remain on bedrest:

With the head of the bed elevated no more than 15 degrees

A client with glaucoma and an acute exacerbation of chronic obstructive pulmonary disease (COPD) has a new prescription to receive carteolol HCl (Ocupress) eye drops. Which action by the nurse is most appropriate?

Withhold the dose and notify the registered nurse.

A client has a fiberglass (nonplaster) cast applied to the lower leg. The client asks the nurse when he will be able to walk on the cast. The nurse replies that the client will be able to bear weight on the cast:

Within 20 to 30 minutes of application

A client is receiving a continuous intravenous (IV) infusion of heparin in the treatment of deep vein thrombosis. The nurse is told that the client's activated partial thromboplastin time (aPTT) level is 65 seconds and that the client's baseline before the initiation of therapy was 30 seconds. The nurse identifies these results as:

Within the therapeutic range

A nurse is providing instructions to a client regarding the use of ice packs to treat an eye injury. The nurse instructs the client to:

Wrap a plastic bag filled with ice with a pillowcase and place it on the eye.

A nurse in the outpatient unit is preparing a client who is scheduled for a laser trabeculoplasty for the treatment of primary open-angle glaucoma. Which of the following instructions should the nurse provide to the client?

You may return to work 1 or 2 days following the procedure."

The nurse is assigned to care for a client who has returned to the nursing unit following a left nephrectomy. The nurse places the highest priority on monitoring which data?

hourly urine output

The nurse is providing dietary instructions to a client with renal calculi, and the laboratory analysis has revealed that the calculus is composed of uric acid. The nurse tells the client that it would be helpful to make which dietary changes?

increase intake of legumes in the diet

A nurse is reviewing the arterial blood gas results of an assigned client. Which of the following arterial blood gases indicates metabolic alkalosis?

pH of 7.48, PCO2 of 40 mm Hg, HCO of 36 mEq/L

The nurse is assigned to care for a client who has just returned to the nursing unit after having hemodialysis for the first time. The nurse monitors the client carefully for which signs and symptoms of disequilibrium syndrome?

vomiting and headaches

Aluminum hydroxide is prescribed for the client with chronic kidney disease (CKD). When should the nurse instruct the client to take this medication?

with meals

A nurse is monitoring a client with a diagnosis of peptic ulcer. Which finding would most likely indicate perforation of the ulcer?

rigid, boardlike abdomen

A client has been diagnosed with chronic gastritis and has been told that there is too little intrinsic factor being produced. The nurse tells the client that which of the following will be prescribed to treat the problem?

vit B12 injections

The nurse is assisting in developing a plan of care for a pregnant client with acquired immunodeficiency syndrome (AIDS). The nurse determines that which concern is the priority for this client?

Development of an infection

A client with possible hiatal hernia complains of frequent heartburn and regurgitation. The nurse should gather further information about the presence of which signs or symptoms

Difficulty swallowing

A nurse is gathering data from a client with a history of untreated cataracts. The nurse asks the client about the presence of which of the following signs of a cataract

Difficulty with driving at night and blurred vision

In preparation for cataract surgery, the nurse is to administer cyclopentolate (Cyclogyl) eyedrops. The nurse administers the eyedrops, knowing that the purpose of this medication is to:

Dilate the pupil of the operative eye

A client with hepatic encephalopathy is receiving lactulose (Cephulac). The nurse determines that the medication is effective if which of the following is observed?

Client previously oriented to person only; can now state name, year, and present location

A nurse is caring for a client with Buerger's disease. Which finding would the nurse determine is a potential complication associated with this disease?

Numbness and tingling in the legs

A client has just undergone gastroscopy. Which of the following is the essential post-procedure nursing intervention

Check the gag reflex before giving oral foods or fluids.

A client is diagnosed with Haemophilus influenzae pneumonia. In addition to standard precautions, which of the following should be instituted immediately by the nurse?

Droplet precautions

A nurse is caring for a client with an intracranial aneurysm who was previously alert. Which finding would be an early indication that the level of consciousness (LOC) is deteriorating?

Drowsiness

A nurse is collecting admission data on the client with hepatitis. Which of the following findings would be a direct result of this client's condition

Drowsiness

The chest x-ray report for a client states that the client has a left apical pneumothorax. The nurse would monitor the status of breath sounds in that area by placing the stethoscope:

Just under the left clavicle

A nurse is preparing to perform an abdominal assessment on a client. The nurse places the client in which best position to perform the assessment? Refer to figure.

2

A nurse is caring for a client with emphysema who is receiving oxygen. The nurse checks the oxygen flow rate to ensure that it does not exceed:

2 L/min

A nurse is caring for a client after an autograft and grafting of a burn wound on the right knee. Which of the following would the nurse anticipate being prescribed for the client? 1. Placing the affected leg flat 2. Elevating and immobilizing the affected leg 3. Placing the affected leg in a dependent position 4. Immobilizing the client in a dependent position

2. Elevating and immobilizing the affected leg

A client with a major burn is admitted to the emergency department. The nurse anticipates that which of the following medication routes will be prescribed for analgesics for this client? 1. Oral 2. Intravenous 3. Intramuscular 4. Subcutaneous

2. Intravenous

A nurse reinforces instructions to a client diagnosed with impetigo. Which statement by the client indicates a need for further instructions? 1. "I need to continue with the antibiotics as prescribed." 2. "I can wash my laundry with other household members' items." 3. "I need to wash my hands thoroughly and frequently throughout the day." 4. "I need to separate my dishes and wash them separately from the dishes of other household members."

2. "I can wash my laundry with other household members' items."

A nurse reinforces instructions to a group of clients regarding measures that will assist with the prevention of skin cancer. Which statement by a client indicates the need for further instruction? 1. "I need to wear sunscreen when participating in outdoor activities." 2. "I need to avoid sun exposure before 10:00 ᴀᴍ and after 4:00 ᴘᴍ." 3. "I need to wear a hat, opaque clothing, and sunglasses when in the sun." 4. "I need to examine my body monthly for any lesions that may be suspicious."

2. "I need to avoid sun exposure before 10:00 ᴀᴍ and after 4:00 ᴘᴍ."

A nurse reinforces discharge instructions to a client following patch testing. Which statement by the client indicates the need for further instruction? 1. "I need to keep the test sites dry." 2. "If the patch comes off, I need to reapply it." 3. "I need to avoid activities that will cause me to sweat." 4. "I will return to the clinic in 2 days for the initial reading."

2. "If the patch comes off, I need to reapply it."

A client sustains a burn injury to the anterior right and left legs and perineal area. According to the rule of nines, the nurse would determine that this injury constitutes which of the following body percentages? 1. 10% 2. 19% 3. 23% 4. 37%

2. 19%

A nurse is assigned to care for a client with partial-thickness burns to 60% of her body surfaces. On the fourth day post-injury, the client's vital signs include an oral temperature of 102.8° F, pulse of 98 beats per minute, respirations of 24 breaths per minute, and blood pressure of 105/64 mm Hg. Parenteral nutrition is infusing at 82 mL/hr. Based on these data, the nurse plans to initially: 1. Recheck the vital signs in 1 hour. 2. Monitor the client for signs of infection. 3. Change the parenteral nutrition solution and IV tubing. 4. Determine when the client was last medicated for pain.

2. Monitor the client for signs of infection.

A nurse inspects a pressure ulcer on a client's sacrum and notes that the ulcer has partial-thickness skin loss and the formation of a blister. The nurse categorizes the ulcer as: 1. Stage I 2. Stage II 3. Stage III 4. Stage IV

2. Stage II

An emergency department nurse is caring for a client who sustained a burn injury to the anterior arms and anterior chest area. The client sustained the burn from a home fire that occurred in the basement. On data collection of the client, which of the following would indicate that the client sustained a respiratory injury as a result of the burn? 1. Clear breath sounds 2. Use of accessory muscles for breathing 3. Fear and anxiety 4. Complaints of pain

2. Use of accessory muscles for breathing

A nurse is caring for a client with a skin infection who is receiving amoxicillin (Amoxil) 500 mg every 8 hours. Which of the following indicates to the nurse that the client is experiencing a frequent side effect related to the medication? 1. Severe abdominal cramps 2. Vaginal drainage 3. Fever 4. Severe watery diarrhea

2. Vaginal drainage

A nurse reinforces discharge instructions regarding skin care to a client after the grafting of burn injuries of the left chest and left arm. Which statement by the client indicates the need for further instructions? 1. "I need to bathe using a mild soap and to rinse thoroughly." 2. "I need to avoid direct sunlight on the newly healed skin area." 3. "I should never wear warm clothing over the newly healed skin area." 4. "I need to avoid the use of lanolin products to the newly healed skin area."

3. "I should never wear warm clothing over the newly healed skin area."

A nurse reinforces instructions to a client who has complained of chronic dry skin and episodes of pruritus. Which of the following client statements indicates the need for further instructions? 1. "I should drink eight to ten glasses of water a day." 2. "I need to avoid using astringents on my skin." 3. "I should use a dehumidifier, especially during the winter months." 4. "I should limit myself to one shower per day and apply an emollient to my skin after the shower."

3. "I should use a dehumidifier, especially during the winter months."

A client calls the emergency department and tells the nurse that he has been cleaning a wooded area and that he came into direct contact with poison ivy shrubs. The client tells the nurse that he cannot see anything on the skin and asks the nurse what to do. The nurse makes which statement to the client? 1. "Come to the emergency department." 2. "Apply calamine lotion immediately to the exposed skin areas." 3. "Take a shower immediately, and lather and rinse several times." 4. "It is not necessary to do anything if you cannot see anything on your skin."

3. "Take a shower immediately, and lather and rinse several times."

A client is brought to the emergency department immediately following a smoke inhalation injury. The initial nursing action is to prepare the client to receive: 1. Pain medication 2. Oxygen via nasal cannula 3. 100% humidified oxygen by face mask 4. Endotracheal intubation

3. 100% humidified oxygen by face mask

The nurse is assessing her clients for skin breakdown. Which of the following clients would have the lowest priority for concern in the development of skin breakdown? 1. A client incontinent of urine and feces 2. A client with chronic nutritional deficiencies 3. A client with a lowered mental awareness status 4. A client who is unable to move about and is confined to bed

3. A client with a lowered mental awareness status

During the emergent phase after a major burn injury, which of the following abnormalities would the nurse expect to note? 1. Increased albumin and decreased hematocrit 2. Decreased hemoglobin and increased sodium 3. Increased hematocrit and increased potassium 4. Decreased hemoglobin and decreased potassium

3. Increased hematocrit and increased potassium Rationale: During the emergent phase of a burn injury, the client's hemoglobin and hematocrit will be elevated because of fluid loss. Sodium will be decreased because of trapping in edema fluid and loss through plasma leakage. Potassium will be increased because of disruption of the sodium-potassium pump, tissue destruction, and red blood cell hemolysis; and albumin will be low because of loss through the wound and increased capillary permeability.

A nurse is assisting with monitoring the functioning of a chest-tube drainage system in a client who just returned from the recovery room after a thoracotomy with wedge resection. Which findings would the nurse expect to note? Select all that apply.

3.50 mL of drainage in the drainage-collection chamber 4.The drainage system is maintained below the client's chest. 5.An occlusive dressing is in place over the chest-tube insertion site. 6.Fluctuation of water in the tube of the water-seal chamber during inhalation and exhalation

A nurse is providing cardiopulmonary resuscitation (CPR) to an adult cardiac arrest victim. What is the proper compression-to-ventilation ratio for one-person CPR?

30:2

A nurse is checking a client for the correct placement of a nasogastric (NG) tube. The nurse aspirates the client's stomach contents and checks their pH level. Which of the following pH values indicates the correct placement of the tube?

4

A nurse is planning to provide instructions to the client about how to stand on crutches. In the instructions, the nurse plans to tell the client to place the crutches:

8 inches to the front and side of the client's toes

A nurse is teaching a hospitalized client who has had aortoiliac bypass grafting about measures to improve circulation. The nurse tells the client to do which of the following?

Keep the ankles uncrossed.

A nurse is preparing a list of cast care instructions for a client who just had a plaster cast applied to his right forearm. Which instructions should the nurse include on the list? Select all that apply.

Keep the cast and extremity elevated. The cast needs to be kept clean and dry. Allow the wet cast 24 to 72 hours to dry.

A nurse is assisting in admitting a client who experienced seizure activity in the emergency department. The nurse avoids doing which of the following when managing this client's environment?

Keeping the bed position raised to the nurse's waist level

Family centered nursing care for women and newborn focuses on which of the following? A. Assisting individuals and families achieve their optimal health B. Diagnosing and treating problems promptly C. Preventing further complications from developing D. Conducting nursing research to evaluate clinical skills

A

A nurse has been reinforcing dietary teaching for a client with peptic ulcer disease who has a routine follow-up visit. Which behavior is the best indicator of a successful outcome for this client?

A decrease in sour eructation

Remove jewelry before the test

A client being seen in a health care provider's office has just been scheduled for a barium swallow the next day. The nurse writes down which of the following instructions for the client to follow before the test

A nurse determines that which of the following clients is at greatest risk for development of acute respiratory distress syndrome (ARDS)?

A client with pancreatitis and gram-negative sepsis

Assist the client in expressing feelings

A client with viral hepatitis states to the nurse, I am so yellow."" The nurse would appropriately:"

A nurse is caring for a client with a cerebral aneurysm who is on aneurysm precautions and is monitoring the client for signs of aneurysm rupture. The nurse understands that an early sign of rupture is which of the following?

A decline in the level of consciousness

A client in the emergency department is diagnosed with Bell's palsy. The nurse collecting data on this client expects to note which of the following?

A lag in closing the bottom eyelid

A nurse is completing the laboratory requisition that will accompany an arterial blood gas (ABG) specimen sent to the laboratory for analysis. The nurse understands that which of the following data will not be needed by the laboratory for adequate evaluation of the specimen?

A list of client allergies

A nurse is caring for several clients with respiratory disorders. Which client is at least risk for developing a tuberculosis infection?

A man who is an inspector for the U.S. Postal Service

A nurse is collecting data about how well a client with a gastrointestinal (GI) disorder is able to digest food. The nurse recalls that digestion is best defined as:

A mechanical and chemical process involving the breakdown of foods

A nursing student is caring for a client in the health care clinic who has been diagnosed with glaucoma. The nursing instructor asks the student to describe the type of medication that will likely be prescribed for the client to treat the eye disorder. Which statement by the student indicates an accurate understanding of the treatment for glaucoma?

A miotic agent will lower the pressure in the eye and increase the blood flow to the retina."

A nurse is assigned to administer the prescribed eye drops for a client preparing for cataract surgery. Which type of eye drops will the nurse expect to be prescribed?

A mydriatic medication

A client reports to the health care clinic to obtain testing regarding human immunodeficiency virus (HIV) status after being exposed to an individual who is HIV positive. The test results are reported as negative and the client tells the nurse that he feels so much better knowing that he has not contracted HIV. The nurse explains the test results to the client, providing which information?

A negative HIV test is not considered accurate during the first 6 months after exposure.

A client is admitted to the nursing unit after a left below-knee amputation following a crush injury to the foot and lower leg. The client tells the nurse, "I think I'm going crazy. I can feel my left foot itching." The nurse interprets the client's statement to be:

A normal response and indicates the presence of phantom limb sensation

A perforated eardrum is suspected in a client who was hit in the ear with a basketball. Which documented observation concerning an otoscopic examination supports this suspicion?

A round or oval darkened area on the eardrum

A nurse is assigned to care for a client with a detached retina. Which finding would the nurse expect to be documented in the client's record?

A sense of a curtain falling across the field of vision

A nurse notes that the health care provider has documented a diagnosis of presbycusis on the client's chart. The nurse understands that this condition is accurately described as:

A sensorineural hearing loss that occurs with aging

A nursing instructor asks a nursing student to list the functions of the amniotic fluid. The student responds correctly by stating that which of the following are functions of amniotic fluid? Select all that apply. A. Allows for fetal movement B. Is a measure of kidney function C. Surrounds, cushions, and protects the fetus D. Maintains the body temperature of the fetus E. Prevents large particles such as bacteria from passing to the fetus F. Provides an exchange of nutrients and waste products between the mother and the fetus

A, B, C, D

The nurse knows that there are psychological maternal changes that occurs during pregnancy in a primigravida patient. Select all the normal psychological maternal changes that happens throughout pregnancy. A. Ambivalence B. Breast tenderness C. Emotional lability D. Body image changes E. Bonding or relationship with the fetus F. Nausea and vomiting G. Syncope H. Urinary frequency

A, C, D, E

The home care nurse is collecting data from a client who has been diagnosed with an allergy to latex. In determining the client's risk factors associated with the allergy, the nurse questions the client about an allergy to which food items? Select all that apply.

Kiwi Bananas

A primigravida patient who is 12 weeks pregnant visits a helath promotion program in the community pertaining to the pregnancy care. A group of nursing student is educating the public about measures to prevent discomfort of pregnancy. The primigravida patient asks one of the student about measures on how to prevent heartburn she is experiencing throughout the day. Select all the necessary measures to prevent the primigravia patient's complaint. A. Eating small, frequent meals and avoiding fatty and spicy food B. Eating high fiber foods and increase drinking fluids C. Drinking milk between milk D. Arranging frequent rest periods throughout the day E. Sitting upright for 30 minutes after a meal F. Engaging in regular exercise

A, C, E

The camp nurse prepares to instruct a group of children about Lyme disease. Which information should the nurse include in the instructions?

Lyme disease is caused by a tick carried by deer

A client was just admitted to the hospital to rule out a gastrointestinal (GI) bleed. The client has brought several bottles of medications prescribed by different specialists. During the admission assessment, the client states, "Lately, I have been hearing some roaring sounds in my ears, especially when I am alone." Which medication would the nurse determine could be the cause of the client's complaint?

Acetylsalicylic acid (aspirin)

A nurse is assisting in caring for a client with an endotracheal tube attached to a ventilator when the high-pressure alarm sounds. The nurse checks the client and system for which most likely cause?

Accumulation of secretions in the client's lungs

A nurse is reviewing the medication list for an assigned client. The nurse becomes concerned because which of the following medications is the only one on the client's prescription sheet that does not have an ototoxic effect?

Acetaminophen (Tylenol)

A client with spinal cord injury becomes angry and belligerent whenever the nurse tries to administer care. The nurse should:

Acknowledge the client's anger and continue to encourage participation in care.

A nurse is assisting in preparing a client for a cardiac catheterization. The nurse understands that it is important to check the client for a history of:

Allergy to shellfish

A client sustains a contusion of the eyeball after a traumatic injury with a blunt object. The nurse takes which action immediately?

Applies ice to the affected eye

The nurse is assisting in developing a plan of care for a client with acquired immunodeficiency syndrome (AIDS) who is experiencing night fever and night sweats. Which nursing intervention should the nurse suggest including in the plan of care to manage this symptom?

Administer an antipyretic at bedtime

A nurse is reviewing the prescriptions of a client admitted to the hospital with a diagnosis of acute pancreatitis. Choose the interventions that the nurse would expect to be prescribed for the client.

Administer antacids, as prescribed,Encourage coughing and deep breathing,Administer anticholinergics, as prescribed

A nurse is reviewing the preoperative prescriptions of a client scheduled for a keratoplasty. Which of the following prescriptions, if noted in the client's chart, would the nurse question?

Administer medication to dilate the affected pupil.

A nurse is told that an assigned client will have the chest tubes removed. In preparation for the procedure, the nurse plans to:

Administer pain medication 15 to 30 minutes before the procedure.

A client is at risk for pulmonary embolism and is on anticoagulant therapy with warfarin sodium (Coumadin). The nurse is told that the client's prothrombin time is 18 seconds with a control of 11 seconds. The nurse plans to:

Administer the next dose of warfarin sodium.

A nurse is providing care of the client following a bone biopsy. Which action by the nurse is unnecessary in the care of this client?

Administering intramuscular opioid analgesics

A nurse notes that a client who is attached to a cardiac monitor suddenly develops atrial fibrillation at a rate of 130. The nurse notifies the registered nurse immediately and prepares the client for which initial intervention?

Administration of a calcium channel blocker

A nurse is assisting with conducting a health-promotion program to community members regarding testicular cancer. The nurse determines that further teaching is needed if a community member states that which of the following is a sign of testicular cancer?

Alopecia

A client is admitted to the hospital with a diagnosis of acute pancreatitis. The nurse plans care, knowing that which problem occurs with this disorder

Alteration in comfort related to abdominal pain

A nurse provides instructions to a client about the use of an incentive spirometer. The nurse determines that the client needs further instruction about its use if the client says she must:

After maximal inspiration, hold the breath for 10 seconds and then exhale

A client is scheduled for an endoscopic retrograde cholangiopancreatography (ERCP). The nurse includes which intervention in the plan of care for the client?

After the procedure, keep client nothing by mouth (NPO) until the gag reflex returns

A nurse has obtained a personal and family history from a client with a neurological disorder. Which finding in the client's history will have the least amount of added risk for neurological problems?

Allergy to pollen

An 84-year-old client in an acute state of disorientation was brought to the emergency department by the client's daughter. The daughter states that this is the first time that the client experienced confusion. The nurse determines from this piece of information that which of the following is unlikely to be the cause of the client's disorientation?

Alzheimer's disease

The nurse overhears the term "sundowning" used to describe the behavior of a client newly admitted to the nursing unit during the previous evening shift. The nurse interprets that this client most likely has a diagnosis of:

Alzheimer's disease

A client diagnosed with chronic kidney disease is being treated at home with continuous ambulatory peritoneal dialysis. The client notes that there is a decrease in the catheter outflow following the prescribed 6-hour dwell time and calls the nurse to report this occurrence. The nurse should reinforce instructing the client to take which action?

Ambulate in the home

The nurse is caring for a client with systemic lupus erythematosus (SLE) that is affecting the hematopoietic system. Based on this, which signs and symptoms should the nurse anticipate and collect data on? Select all that apply.

Anemia Splenomegaly Lymphadenopathy

A nurse in the emergency department is caring for a client with a fractured arm. The nurse understands that which item is not necessary before reduction of the fracture in the casting room?

Anesthesia consent

A nurse is listening to a health care provider explain the results of an eye examination to a client. The health care provider states that the client has glaucoma resulting from a congenitally narrow anterior chamber angle, which has suddenly become blocked by the base of the iris. The nurse interprets that the health care provider is describing which of the following types of glaucoma?

Angle-closure glaucoma

A nurse has a prescription to get the client out of bed to a chair on the first postoperative day after total knee replacement. The nurse plans to do which of the following to protect the knee joint?

Apply a knee immobilizer before getting the client up, and elevate the client's surgical leg while sitting.

A client who has undergone a colostomy several days ago is reluctant to leave the hospital and has not yet looked at the ostomy site. Which measures are most likely to promote coping?

Ask a member of the local ostomy club to visit with the client before discharge, Ask the enterostomal nurse specialist to consult with the client before discharge,Ask the client to begin doing one part of the ostomy care and increase tasks daily.

A nurse is assisting in developing a plan of care for a client following the surgical removal of an acoustic neuroma. Which assessment will be included in the plan of care for this specific intervention?

Assessment of cranial nerve VII (facial)

The nurse notes this dysrhythmia on the client's cardiac monitor (refer to figure). The nurse next reports that the client is experiencing which of the following?

Atrial fibrillation

The nurse is caring for a client with glaucoma. Which of the following medications, if prescribed for the client, would the nurse question?

Atropine sulfate (Isopto Atropine)

A nurse has just confirmed that a client has been scheduled for a mammogram for the following week. The nurse reinforces that the client should: Select all that apply.

Avoid applying skin lotion on the day of the test, Remove any necklaces before presenting for the procedure.

A client will undergo a barium swallow to determine whether the client has a hiatal hernia. The nurse provides pre-procedure instructions and tells the client to:

Avoid eating or drinking after midnight before the test

A client will undergo a barium swallow to confirm a diagnosis of a hiatal hernia. In preparation for the test, the nurse instructs the client to:

Avoid eating or drinking after midnight before the test.

A client is being discharged home after application of a plaster leg cast. The nurse determines that the client understands proper care of the cast if the client states to:

Avoid getting the cast wet

A nurse is assisting in preparing a list of instructions for an adult client who is being discharged following a tonsillectomy. Choose the instructions that the nurse should place on the list. Select all that apply.

Avoid hot fluids. Avoid rough foods. Rest for the next 24 hours.

A nurse is providing instructions to a client with angina pectoris about measures to reduce recurrence of chest pain. The nurse should stress to the client the importance of doing which of the following?

Avoiding exposure to either very hot or very cold weather

A clinic nurse is reinforcing instructions to a client with a diagnosis of pharyngitis. What intervention will the client be encouraged to perform?

Avoiding foods that are highly seasoned

Q.12) A nurse is collecting data during the admission asessment of a client who is pregnant with twins. The client also has 5 year old child. The nurse would document which gravida and para status on this client? A. G1P1 B. G2P1 C. G2P2 D. G3P2

B

Which of the following prenatal laboratory test values would the nurse consider as significant? A. Hematocrit 33.5% B. Rubella titer less than 1:8 C. White blood cells 8,000/mm3 D. One hour glucose challenge test 110 g/dL

B Explanation A rubella titer should be 1:8 or greater. Thurs, a finding of a titer less than 1:8 is significant, indicating that the client may not possess immunity to rubella. A hematocrit of 33.5% a white blood cell count of 8,000/mm3, and a 1 hour glucose challenge test of 110 g/dl are with normal parameters.

A pregnant patient asks the nurse Kate if she can take castor oil for her constipation. How should the nurse respond? A. "Yes, it produces no adverse effect." B. "No, it can initiate premature uterine contractions." C. "No, it can promote sodium retention." D. "No, it can lead to increased absorption of fat-soluble vitamins."

B Explanation Castor oil can initiate premature uterine contractions in pregnant women. It also can produce other adverse effects, but it does not promote sodium retention. Castor oils is not known to increase absorption of fat-soluble vitamins, although laxatives in general may decrease absorption if intestinal motility is increased.

During which of the following would the focus of classes be mainly on physiologic changes, fetal development, sexuality, during pregnancy, and nutrition? A. post partum phase B. first trimester C. second trimester D. third trimester

B Explanation First-trimester classes commonly focus on such issues as early physiologic changes, fetal development, sexuality during pregnancy, and nutrition. Some early classes may include pregnant couples. Second and third trimester classes may focus on preparation for birth, parenting, and newborn care.

The nurse is developing a teaching plan for a patient who is 8 weeks pregnant. The LPN should tell the patient that she can expect to feel the fetus move at which time? A. Between 10 and 12 weeks' gestation B. Between 16 and 20 weeks' gestation C. Between 21 and 23 weeks' gestation D. Between 24 and 26 weeks' gestation

B Explanation A pregnant woman usually can detect fetal movement (quickening) between 16 and 20 weeks' gestation. Before 16 weeks, the fetus is not developed enough for the woman to detect movement. After 20 weeks, the fetus continues to gain weight steadily, the lungs start to produce surfactant, the brain is grossly formed, and myelination of the spinal cord begins.

A client at 36 weeks' gestation is schedule for a routine ultrasound prior to an amniocentesis. After teaching the client about the purpose for the ultrasound, which of the following client statements would indicate to the nurse in charge that the client needs further instruction? A. The ultrasound will help to locate the placenta B. The ultrasound identifies blood flow through the umbilical cord C. The test will determine where to insert the needle D. The ultrasound locates a pool of amniotic fluid

B. Explanation Before amniocentesis, a routine ultrasound is valuable in locating the placenta, locating a pool of amniotic fluid, and showing the physician where to insert the needle. Color Doppler imaging ultrasonography identifies blood flow through the umbilical cord. A routine ultrasound does not accomplish this.

A 25-year-old client with diabetes type I visits the clinic to discuss her and her husband's desire to start a family. This diabetic client A. should be discouraged from becoming pregnant B. has a greater risk of complications during pregnancy C. should be informed about treatment for infertility D. will be able to carry out a completely normal pregnancy

B. Explanation Clients with DM are at greater risk for developing maternal and fetal complications during pregnancy.

The nurse is assessing a client who has frequent episodes of asthma. Which assessment finding is most closely associated with asthma?

Bilateral wheezing

During a lecture on reproduction, a student nurse asks the instructor what determines the sex of a fetus. Accurate information in response to this question would be: A. "The sex of the fetus is not determined until the eighth week of gestation." B. "The fertilization of the zygote is the point at which sex is determined." C. "Males have one less pair of chromosomes than females." D. "Sex is determined by the chromosomes contributed by the ovum."

B. Explanation The sex of the fetus is determined at the point that the sperm fertilizes the ovum to form the zygote. Sex is ultimately determined by the chromosome contributed by the sperm.

A nurse is evaluating goal achievement for a client in traction with impaired physical mobility. The nurse determines that the client has not successfully met all of the goals formulated if which of the following outcomes was noted?

Bowel movement every 5 days

A nurse is caring for a client with cancer receiving chemotherapy who has developed stomatitis. The nurse plans to give mouth care by using oral care agents and devices:

Based on the severity of stomatitis

A client with myocardial infarction (MI) has been transferred from the coronary care unit (CCU) to the general medical unit with cardiac monitoring via telemetry. The nurse assisting in caring for the client expects to note which type of activity prescribed?

Bathroom privileges and self-care activities

A nurse is providing client teaching regarding glaucoma. Which of the following are important to include in the teaching plan? Select all that apply.

Be sure to report halos of light or increased eye pain to your health care provider. Follow a low-sodium, minimal-caffeine diet with plenty of fiber.

A nurse is teaching a client who is newly diagnosed with a hiatal hernia about measures to prevent recurrence of symptoms. The nurse makes which statement to the client?

Be sure to sleep with your head elevated in bed."

A nurse is teaching a client with a newly diagnosed hiatal hernia about measures to prevent recurrence of symptoms. Which statement would be included in the teaching?

Be sure to sleep with your head elevated in bed."

A cervical radiation implant is placed in the client for treatment of cervical cancer. What activity order would the nurse most likely expect to note in the health care provider's prescriptions?

Bedrest

A client with liver cancer who is receiving chemotherapy tells the nurse that some foods on the meal tray taste bitter. The nurse would try to limit which of the following foods that is most likely to have this taste for the client?

Beef

A nurse is monitoring a client with a C5 spinal cord injury for spinal shock. Which of the following findings would be associated with spinal shock in this client? Select all that apply.

Bowel sounds are absent. The client's abdomen is distended. Respiratory excursion is diminished. Accessory muscles of respiration are areflexic.

A client has a history of left-sided heart failure. The nurse would look for the presence of which of the following to determine whether the problem is currently active?

Bilateral lung crackles

A client is experiencing double vision, or diplopia. The nurse interprets that this client is experiencing a loss of which of the following normal functions of the eye

Binocular vision

A client is diagnosed as having a bowel tumor, and several diagnostic tests are prescribed. The nurse understands that which test will confirm the diagnosis of malignancy?

Biopsy of the tumor

After a renal biopsy, the client complains of pain at the biopsy site, which radiates to the front of the abdomen. Which would this indicate?

Bleeding

A nurse is caring for a client who has developed compartment syndrome from a severely fractured arm. The client asks the nurse how this can happen. The nurse's response is based on the understanding that:

Bleeding and swelling cause increased pressure in an area that cannot expand.

A client complaining of chest pain has an as-needed (PRN) prescription for sublingual nitroglycerin (Nitrostat). Before administering the medication to the client, the nurse should first check the client's:

Blood pressure

A client with benign prostatic hypertrophy (BPH) undergoes a transurethral resection of the prostate (TURP) and is receiving continuous bladder irrigations postoperatively. Which are the signs/symptoms of transurethral resection (TUR) syndrome?

Bradycardia and confusion

A client is admitted to the surgical nursing unit following transurethral resection of the prostate (TURP) for benign prostatic hypertrophy. The client has a bladder irrigation infusing, and output is light cherry colored. The blood pressure is 134/82 mm Hg, the pulse is 84 beats per minute, and the client is afebrile with a respiratory rate of 18 breaths per minute. The licensed practical nurse (LPN) assisting in caring for the client collects assessment data 1 hour after admission to the nursing unit. The LPN notifies the registered nurse (RN) if which is noted on data collection?

Blood pressure of 102/50 mmHg, pulse 110 bpm

A nurse is beginning to ambulate a client with activity intolerance caused by bacterial endocarditis. The nurse determines that the client is best tolerating ambulation if which parameter is noted?

Blood pressure that increases from 114/82 to 118/86 mm Hg

A client with cancer has undergone a total abdominal hysterectomy and has a Foley catheter in place during the immediate postoperative period. The nurse would expect to note which of the following types of urinary drainage immediately following this surgery?

Blood tinged

A client is returned to the nursing unit following thoracic surgery with chest tubes in place. During the first few hours postoperatively, the nurse assisting in caring for the client checks for drainage and expects to note that it is:

Bloody

The nurse is caring for a hospitalized client following cystoscopy and is monitoring for signs of complications associated with the procedure. Which result noted in the first few hours following the procedure indicates the need to notify the registered nurse?

Bloody urine with clots

A client arrives at the ambulatory care clinic with low abdominal pain. A routine urine specimen reveals hematuria. The client does not have a fever. The nurse should next ask the client about a history of which condition?

Blow or trauma to the bladder or abdomen

A nurse is reviewing the health record of a client diagnosed with a cataract. The chief clinical manifestation that the nurse would expect to note in the early stages of cataract formation is:

Blurred vision

A nurse is caring for a client with a suspected diagnosis of aplastic anemia. Which of the following tests would the nurse anticipate to be performed to confirm the diagnosis?

Bone marrow aspiration

Latanoprost (Xalatan) drops are prescribed for the client with glaucoma. The client returns to the health care clinic for evaluation. Which of the following findings, if noted in the client, indicates a side effect associated with the use of these eye drops?

Brown pigmentation of the iris

The nurse is reviewing the medical record of a client who is suspected of having systematic lupus erythematosus (SLE). Which sign would the nurse expect to be documented in the record that is related to this diagnosis?

Butterfly rash on cheeks and bridge of the nose

The nurse is collecting data on a client who complains of fatigue, weakness, malaise, muscle pain, joint pain at multiple sites, anorexia, and photosensitivity. Systematic lupus erythematosus (SLE) is suspected. The nurse further checks for which manifestation that is also indicative of the presence of SLE?

Butterfly rash on the cheeks and bridge of the nose

The LPN is preparing to administer Solu-medrol 40 mg mixed in 150 mL of sodium chloride via intravenous piggyback. The medication is to be administered over 30 minutes. Using the tubing with a drop factor of 15 ggts/mL, what would the LPN calculate the rate to be in drops per minute? A. 40 B. 50 C. 75 D. 150

C

The hormone responsible for the development of the ovum during the menstrual cycle is? A. estrogen B. progesterone C. follicle stimulating hormone (Correct Answer) D. leutenizing hormone (Your Answer)

C

A client LMP began July 5. Her EDD should be which of the following? A. January 2 B. March 28 C. April 12 D. October 12

C Explanation To determine the EDD when the date of the client's LMP is known use Nagele rule. To the first day of the LMP, add 7 days, subtract 3 months, and add 1 year (if applicable) to arrive at the EDD as follows: 5 + 7 = 12 (July) minus 3 = 4 (April). Therefore, the client's EDD is April 12.

Heartburn and flatulence, common in the second trimester, are most likely the result of which of the following? A. increased plasma HCG levels B. decreased intestinal motility C. decrease gastric acidity D. elevated estrogen levels

C Explanation During the second trimester, the reduction in gastric acidity in conjunction with pressure from the growing uterus and smooth muscle relaxation, can cause heartburn and flatulence. HCG levels increase in the first, not the second, trimester. Decrease intestinal motility would most likely be the cause of constipation and bloating. Estrogen levels decrease in the second trimester.

Which of the following represents the average amount of weight gained during pregnancy? A. 12 to 2 lbs B. 15 to 25 lbs C. 25 to 35 lbs D. 25 to 40 lbs

C Explanation The average amount of weight gained during pregnancy is 25 to 35 lb. This weight gain consists of the following: fetus - 7.5 lb; placenta and membrane - 1.5 lb; amniotic fluid - 2 lb; uterus - 2.5 lb; breasts - 3 lb; and increased blood volume - 2 to 4 lb; extravascular fluid and fat - 4 to 9 lb. A gain of 12 to 22 lb is insufficient, whereas a weight gain of 15 to 25 lb is marginal. A weight gain of 25 to 40 lb is considered excessive.

The nurse identifies substance abuse behaviors exhibited by a pregnant client during an initial prenatal screening. While promoting a therapeutic and accepting environment, the care managment by the nurse would be MOST appropriate if focused on which of the following? A. Discouraging substance use during pregnancy B. Termination of the pregnancy at an early stage C. Eliminating substance use during pregnancy D. Setting boundaries with the client in regards to substance use

C. Explanation Use of substances during pregnancy can lead to severe fetal or neonatal abnormalities, complications, and death. The primary goal of nursing care should be prevention or elimination of substance use during pregnancy.

Cervical softening and uterine souffle are classified as which of the following? A. diagnostic signs B. presumptive signs C. probable signs D. positive signs

C. Explanation Cervical softening (Goodell sign) and uterine soufflé are two probable signs of pregnancy.Probable signs are objective findings that strongly suggest pregnancy. Other probable signs include Hegar sign, which is softening of the lower uterine segment; Piskacek sign, which is enlargement and softening of the uterus; serum laboratory tests; changes in skin pigmentation; and ultrasonic evidence of a gestational sac. Presumptive signs are subjective signs and include amenorrhea; nausea and vomiting; urinary frequency; breast tenderness and changes; excessive fatigue; uterine enlargement; and quickening.

A nurse checks a closed chest tube drainage system on a client who had a lobectomy of the left lung 24 hours ago. The nurse notes that there has been no chest tube drainage for the past hour. The nurse would first:

Check for kinks in the chest drainage system.

A nurse is caring for a client on a cardiac monitor who is alone in a room at the end of the hall. The client has a short burst of ventricular tachycardia (VT) followed by ventricular fibrillation (VF). The client suddenly loses consciousness. Which intervention should the nurse do first?

Call for help and initiate cardiopulmonary resuscitation (CPR).

A nurse is providing dietary instructions to a client with congestive heart failure (CHF). The nurse determines that the client understands the instructions if the client states that which of the following food items will be avoided?

Catsup

The nurse is caring for an older client who is on bedrest. The nurse plans which intervention to prevent respiratory complications?

Changing the client's position every 2 hours

A nurse observes that a client's nasogastric tube has suddenly stopped draining. The tube is connected to suction, the machine is on and functioning and all connections are snug. The tube is secured properly and does not appear to have been dislodged. After checking placement, the nurse gently flushes the tube with 30 mL of normal saline, but the tube still is not draining. The nurse analyzes this problem as:

Channels of gastric secretions may be bypassing the holes in the tube; turning the client will promote stomach emptying

A client has received instructions about an upcoming cardiac catheterization. The nurse determines that the client has the best understanding of the procedure if the client knows to report which of the following items?

Chest pain

A client is at risk of developing a pulmonary embolism. The nurse monitors for which of the following, which commonly is reported initially?

Chest pain that occurs suddenly

The home care nurse is assigned to care for a client who returned home from the emergency department following treatment for a sprained ankle. The nurse notes that the client was sent home with crutches that have rubber axillary pads and needs to reinforce instructions regarding crutch walking. On data collection, the nurse discovers that the client has an allergy to latex. Before providing instructions regarding crutch walking, the nurse should do which?

Cover the crutch pads with cloth

A client arrives at the emergency department for treatment of an injury to the eye after being hit by a baseball bat. On data collection, the nurse notes that the eye is bleeding. Which of the following nursing actions is appropriate?

Cover the eye with cold, sterile saline gauze.

A client with myocardial infarction suddenly becomes tachycardic, shows signs of air hunger, and begins coughing frothy, pink-tinged sputum. A nurse listens to breath sounds, expecting to hear bilateral:

Crackles

A nurse is reviewing the health record of a client with laryngeal cancer. The nurse would expect to note which most common risk factor for this type of cancer documented in the record?

Cigarette smoking

A nurse is caring for a client following craniotomy for removal of an acoustic neuroma. The nurse understands that assessment of which of the following cranial nerves would identify a complication specifically associated with this surgery?

Cranial nerve VII, facial nerve

A nursing instructor asks a nursing student about the points to document if the client has had a seizure. The instructor determines that the student needs to read about seizures and related documentation points if the student stated that it is important to document:

Client's diet in the 2 hours preceding seizure activity

A health care provider aspirates synovial fluid from a knee joint of a client with rheumatoid arthritis. The nurse reviews the laboratory analysis of the specimen and should expect the results to indicate which finding?

Cloudy synovial fluid

A nurse is assisting in the care of a client diagnosed with multiple myeloma who has been prescribed an intravenous solution. Which finding would indicate a positive response to this treatment?

Creatinine of 1 mg/dL

A nurse is giving the client with a left leg cast crutch-walking instructions using the three-point gait. The client is allowed touch-down of the affected leg. The nurse tells the client to advance the:

Crutches and the left leg, then advance the right leg

A nurse is developing a poster to use in teaching clients about the prevention of hearing loss. The nurse would diagram which of the following structures as part of the inner ear?

Cochlea

A client with a 4-day-old lumbar vertebral fracture is experiencing muscle spasms. The nurse avoids using which of the following in an effort to relieve the spasm?

Cold

A nurse caring for a client in the postoperative period following an enucleation, notes bloody staining on the surgical eye dressing. Which of the following is the appropriate initial action?

Contact the health care provider.

A gastrectomy is performed on a client with gastric cancer. In the immediate postoperative period the nurse notes bloody drainage from the nasogastric (NG) tube. The nurse plans to:

Continue to monitor the drainage

A nurse is assigned to assist with caring for a client who has a chest tube. The nurse notes fluctuations of the fluid level in the water-seal chamber. Based on this observation, which action would be appropriate?

Continue to monitor, because this is an expected finding.

The nurse is assigned to care for a client admitted to the hospital with a diagnosis of systemic lupus erythematosus (SLE). The nurse reviews the health care provider's prescriptions. Which medication does the nurse expect to be prescribed?

Corticosteroid

A client with a Sengstaken-Blakemore tube in place to treat esophageal varices suddenly becomes restless. The client's heart rate and blood pressure increase and the client has difficulty breathing. The important initial nursing action is to:

Cut the tube and pull it out

A client is pregnant with her third child. Medical history of the client indicates a previous precipitate labor and birth. Which of the following interventions would NOT be expected during labor of the present pregnancy? A. Use of magnesium sulfate B. Close monitoring of the fetus for hypoxia C. The nurse stays at the bedside constantly or as much as possible D. amnioinfusion will be performed

D Explanation Amnioinfusion is instillation of fluid into the amniotic sac within the uterus to treat oligohydraminios. This is not done to prevent precipitate labor and birth.

The LPN has initiated the administration of vancomycin via IV piggyback . In which of the following situations should the nurse recognize that the client may be experiencing a fatal reaction to this medication? A. The client start coughing B. The client complains of pain at the intravenous catheter insertion site C. The nurse hears the client snoring from the hall D. The nurse notices the client's neck and chest is bright red

D Explanation While administering vancomycin the LPN should know to monitor the client carefully for the development of Red Man Syndrome or anaphylactic shock. The common side effects of this medicine are pruritus, flushing and erythema to the head, neck, and upper body.

A client who has a cold is seen in the emergency department with an inability to void. Because the client has a history of benign prostatic hyperplasia (BPH), the nurse questions the client about use of which medication?

Decongestants

A nurse is monitoring a client with a head injury and notes that the client is assuming this posture. The nurse notifies the registered nurse immediately to report that the client is exhibiting: Refer to figure.

Decorticate posturing

A complete blood cell count is performed on a client with systemic lupus erythematosus (SLE). The nurse should suspect that which finding will be reported from this blood test?

Decrease of all cell types

A client with chronic obstructive pulmonary disease (COPD) who is beginning oxygen therapy asks the nurse why the flow rate cannot be increased to more than 2 L/min. The nurse responds that this would be harmful because it could:

Decrease the client's oxygen-based respiratory drive

The nurse is collecting data from a client who has had benign prostatic hyperplasia (BPH) in the past. To determine if the client is currently experiencing exacerbation of BPH, the nurse should ask the client about the presence of which early symptom?

Decreased force in the stream of urine

A nurse is caring for the client with a head injury secondary to a motor vehicle accident. The nurse observes the client's status regularly, monitoring closely for which changes in vital signs?

Decreasing pulse, decreasing respirations, increasing BP

A client is admitted with an arterial ischemic leg ulcer. The nurse expects to note that this ulcer has which of the following typical characteristics?

Deep and painful

A client who has just been told by the health care provider that she has breast cancer responds by stating, "Oh, no, this has to be a big mistake." The nurse interprets that the client's initial reaction is one of:

Denial

An emergency department client who complains of slightly improved but unrelieved chest pain for 2 days is reluctant to take a nitroglycerin sublingual tablet offered by the nurse. The client states, "I don't need that—my dad takes that for his heart. There's nothing wrong with my heart." Which of the following best describes the client's response?

Denial

A nurse is assisting a health care provider with the insertion of a chest tube. The nurse notes fluctuation of the fluid level in the water seal chamber after the tube is inserted. Based on this observation, the nurse plans to take which appropriate action?

Document the accurate functioning of the tube.

A client who sustained a closed head injury has a new onset of copious urinary output. Urine output for the previous 8-hour shift was 3300 mL, and 2800 mL for the shift before that. The findings have been reported to the health care provider, and the nurse anticipates a prescription for which of the following medications?

Desmopressin (DDAVP)

A nurse is preparing for the admission of a client with a suspected diagnosis of Guillain-Barré syndrome. Which clinical manifestation is considered a primary symptom of this syndrome?

Development of muscle weakness

A health care provider places a Miller-Abbott tube in a client who has a bowel obstruction. Six hours later, the nurse measures the length of the tube outside of the nares and notes that the tube has advanced 6 cm since it was first placed. Based on this finding, which action should the nurse take next?

Document the finding in the client's record.

A nurse is caring for a client following total hip replacement who has a Hemovac wound suction drain in place. At the end of the 8-hour shift, the nurse empties 45 mL of drainage from the wound-suction device. Based on this amount of drainage, which action is appropriate?

Document the findings.

A client brings the following medications to the clinic for her yearly physical. The nurse realizes which medication has been prescribed to treat heart failure?

Digoxin (Lanoxin)

A nursing student is caring for a client scheduled for cataract surgery. The student reviews the preoperative prescriptions with the nursing instructor and notes that cyclopentolate (Cyclogyl) eye drops are prescribed to be administered preoperatively. The nursing instructor asks the student about the action of the eye drops. The student appropriately responds by telling the instructor that the action of these eye drops is to:

Dilate the pupil of the operative eye.

A client with active tuberculosis (TB) demonstrates less-than-expected interest in learning about the prescribed medication therapy. The nurse suggests to the health care team that this client ultimately may need:

Directly observed therapy

A nurse has given the client with tuberculosis instructions for proper handling and disposal of respiratory secretions. The nurse determines that the client understands the instructions if the client verbalizes which of the following?

Discard used tissues in a plastic bag.

A client scheduled for a pulmonary angiography is fearful about the procedure and asks the nurse if the procedure involves significant pain and radiation exposure. The nurse gives a response to the client that provides reassurance, based on the understanding that:

Discomfort may occur with needle insertion, and there is minimal exposure to radiation

A nurse is suctioning a client through a tracheostomy tube. During the procedure, the client begins to cough, and the nurse notes the presence of an audible wheeze. The nurse attempts to remove the suction catheter from the client's trachea but is unable to do so. The nurse would first:

Disconnect the suction source from the catheter

A client with acquired immunodeficiency syndrome (AIDS) is taking zidovudine (Retrovir) 200 mg orally three times daily. The client reports to the health care clinic for follow-up blood studies, and the results of the blood studies indicate severe neutropenia. Which should the nurse anticipate to be prescribed for the client?

Discontinuation of the medication

A nurse is providing endotracheal suctioning to a client who is mechanically ventilated when the client becomes restless and tachycardic. What should the nurse do?

Discontinue suctioning until the client is stabilized and monitor vital signs.

A client has just undergone computed tomography (CT) scanning with a contrast medium. The nurse determines that the client understands postprocedure care if the client verbalizes that he or she will:

Drink extra fluids for the day.

A client has had a bone scan procedure. The nurse determines that the client understands the elements of follow-up care if the client states that he or she will:

Drink plenty of water for a day or two following the procedure.

A nurse is reviewing the record of a client admitted to the hospital for treatment of bladder cancer. Which risk factor related to this type of cancer would the nurse likely note in the client's record?

Drinks coffee and smokes cigarettes

A client with acquired immunodeficiency syndrome (AIDS) has become infected with histoplasmosis. The nurse monitors the client for which of the following signs and symptoms?

Dyspnea

A nurse is caring for a client hospitalized with acute exacerbation of chronic obstructive pulmonary disease (COPD). Which of the following would the nurse expect to note in this client? Select all that apply.

Dyspnea on exertion Presence of a productive cough Difficulty breathing while talking

The nurse suspects the client has a urinary tract infection (UTI). Which signs/symptoms suggest a UTI? Select all that apply.

Dysuria Hematuria Frequency Flank pain Cloudy urine

A male client has a tentative diagnosis of urethritis. The nurse collects data from the client knowing that which are signs/symptoms of this disorder?

Dysuria and penile discharge

A nurse is caring for a client scheduled for magnetic resonance imaging (MRI). Which instruction does the nurse reinforce to the client?

Earplugs can be worn if the noise from the machine is uncomfortable.

A client has just completed an information session about measures to minimize the progression of coronary artery disease (CAD). The nurse determines that the client indicates an initial understanding of lifestyle alterations if the client states an intention to:

Eat a diet that is low in fat and cholesterol.

A nurse is reinforcing dietary instructions to a client with peptic ulcer disease. The nurse encourages the client to:

Eat anything as long as it does not aggravate or cause pain

A nurse is caring for a client who sustained a spinal cord injury. While administering morning care, the client developed signs and symptoms of autonomic dysreflexia. The initial nursing action is to:

Elevate the head of the bed.

A nurse is assigned to care for a client with multiple traumas who is admitted to the hospital. The client has a leg fracture, and a plaster cast has been applied. In positioning the casted leg, the nurse should:

Elevate the leg on pillows continuously for 24 to 48 hours.

A nurse is instructing a client who had a herniorrhaphy about how to reduce postoperative swelling following the procedure. Which of the following would the nurse suggest to the client to prevent swelling?

Elevate the scrotum

The nurse is reviewing the client's record and notes that the health care provider has documented that the client has a renal disorder. Which laboratory results would indicate a decrease in renal function? Select all that apply.

Elevated serum creatinine level Decreased red blood cell (RBC) count Elevated blood urea nitrogen (BUN) level

A client, who had experienced significant blood loss in an automobile crash, was admitted to the hospital 2 days earlier. The nurse observes the client for which signs/symptoms that indicate acute kidney injury (AKI)? Select all that apply.

Elevated urine specific gravity Rising serum blood urea nitrogen (BUN) and creatinine levels Urine output averaging 25 mL per hour while receiving an intravenous infusion at 150 mL/hour

A nurse monitors the laboratory data on a client at risk for coronary artery disease. A fasting blood glucose reading of 200 mg/dL is recorded on the chart. The nurse analyzes this result as:

Elevated, signaling the presence of diabetes mellitus, a risk factor of coronary artery disease

A nurse is caring for a client after a radical mastectomy. Which nursing intervention would assist with preventing lymphedema of the affected arm?

Elevating the affected arm on a pillow above heart level

A client admitted to the hospital with coronary artery disease complains of dyspnea at rest. The nurse determines that which of the following items would be of most help to the client?

Elevating the head of the bed to at least 45 degrees

The nurse is collecting data on a client with chronic airflow limitation (CAL) and notes that the client has a "barrel chest." The nurse interprets that this client has which of the following forms of CAL?

Emphysema

A nurse is told that an assigned client will have a fenestrated tracheostomy tube inserted. The nurse prepares the client for the procedure, knowing that this type of tube:

Enables the client to speak

A client with ovarian cancer is scheduled to receive chemotherapy with cisplatin. The nurse assisting in caring for the client reviews the plan of care, expecting to note which intervention?

Encourage fluids.

A client receiving chemotherapy tells the nurse, "What will I ever do when my hair starts to fall out?" The appropriate response by the nurse should be to:

Encourage her to select a wig.

A nurse is assisting in developing a plan of care for a client who will be returning to the nursing unit following a cardiac catheterization via the femoral approach. Which nursing intervention will be included in the post-procedure plan of care?

Encourage the client to increase fluid intake.

A client has an arteriovenous (AV) shunt in place for hemodialysis. The nurse should take which priority precaution, knowing that bleeding is a potential complication?

Ensure that small clamps are attached to the AV shunt dressing

A client has just had skeletal traction applied following insertion of pins. The nurse should place highest priority on doing which of the following while caring for the client?

Ensure that the weights on the traction setup are hanging free.

Before administering an intermittent tube feeding through a nasogastric tube, the nurse checks for gastric residual volume. The nurse understands that the rationale for checking gastric residual volume before administering the tube feeding is to:

Evaluate absorption of the last feeding

A generally healthy 63-year-old man is seen in the health care provider's office for a routine examination. Which statement made by the client is important for the nurse to follow up on?

Everyone in my immediate family has died from gastrointestinal cancer.

A nurse is preparing the client for eye testing and the examiner is planning to test the eyes using the confrontational method. The nurse tells the client that this test is performed to:

Examine visual fields or peripheral vision.

A client undergoing diagnostic testing for cancer is scheduled for magnetic resonance imaging (MRI). The nurse reinforces to the client which of the following about the procedure?

Expect the MRI machine to make loud noises.

A client who is experiencing severe respiratory acidosis has a potassium level of 6.2 mEq/L. The nurse interprets that this result is:

Expected and indicates that acidosis has driven hydrogen ions into the cell, forcing potassium out

A client diagnosed with tuberculosis (TB) is distressed over the loss of physical stamina and fatigue. The nurse plans to tell the client that this is:

Expected and the client should very gradually increase activity as tolerated

A postoperative client with incisional pain complains to the nurse about completing respiratory exercises. The client is willing to do the deep breathing exercises but states that it hurts to cough. The nurse provides gentle encouragement and appropriate pain management to the client, knowing that coughing is needed to:

Expel mucus from the airways

A client newly diagnosed with renal failure will be receiving peritoneal dialysis. During the infusion of the dialysate, the client complains of abdominal pain. Which action by the nurse is appropriate?

Explain that the pain will subside after the first few exchanges

A nurse is trying to help the family of an unconscious client cope with the situation. Which intervention should the nurse plan to incorporate into the care routine for the client?

Explaining equipment and procedures on an ongoing basis

A nurse is caring for an unconscious client who is experiencing persistent hyperthermia with no signs and symptoms of infection. The nurse understands that there may be damage to the client's thermoregulatory center in the:

Hypothalamus

A sexually active 20-year-old client has developed viral hepatitis. Which of the following statements, if made by the client, would indicate a need for teaching

I can never drink alcohol again."

A client is seen in the ambulatory care office for a routine examination. Which statement by the client would be important for the nurse to follow up?

I just lost a family to gastrointestinal cancer."

A nurse provides home care instructions to a client after cataract removal and placement of an intraocular implant in the right eye. Which statement by the client indicates a need for further instruction?

I need to remove the eye dressing as soon as I get home and place a warm pack on my eye."

A nurse is reinforcing home care instructions to a client who has a hordeolum (stye) of the right eye. Which statement by the client indicates an understanding of the instructions?

I should apply antibiotic ointment as prescribed."

A nurse is collecting data on a client who was just admitted to the hospital with a diagnosis of coronary artery disease (CAD). The client reveals having been under a great deal of stress recently. Based on this finding, the nurse should:

Explore with the client the sources of stress in life.

A nurse reads a client's Mantoux skin test as positive. The nurse notes that previous tests were negative. The client becomes upset and asks the nurse what this means. The nurse's response is based on the understanding that the client has:

Exposure to tuberculosis

A nurse is assisting in gathering data on cranial nerve XII of a client who sustained a brain attack (stroke). The nurse understands that the client should be asked to:

Extend the tongue.

A nurse is assisting in caring for a client who sustained a traumatic head injury following a motor vehicle accident. The nurse documents that the client is exhibiting decerebrate posturing. The nurse bases this documentation on which observation?

Extension of the extremities and pronation of the arms

After an eye examination, a client has been diagnosed with acute angle-closure glaucoma. The nurse collecting data from the client asks the client about an accompanying history of which of the following?

Eye pain

A nurse is assigned to care for an adult client who had a brain attack (stroke) and is aphasic. Choose the appropriate interventions for communicating with the client. Select all that apply.

Face the client when talking. Speak slowly and maintain eye contact. Use gestures when talking to enhance words. Give the client directions using short phrases and simple terms.

A client reporting recent right eye discomfort is diagnosed with chalazion of the right eye. The nurse reinforces instructions to the client regarding care to the eye. Which statement by the client indicates an understanding of the measures?

I should apply warm packs to my eye."

Which are risk factors for systemic lupus erythematous (SLE)? Select all that apply.

Female gender African-American origin Being in the childbearing years

A client who has undergone a left heart catheterization using the right femoral approach is returned to the nursing unit. Thirty minutes later the client complains of numbness and tingling of the right foot. The pedal pulse is weak, and the foot is pale. The nurse notifies the registered nurse immediately because these symptoms are consistent with:

Femoral artery thrombus or hematoma

A nurse is assisting the health care provider in performing a lumbar puncture. The nurse prepares for the procedure by placing the client in which position?

Fetal position

A client is admitted to the hospital with possible rheumatic heart disease. The nurse collects data from the client and checks the client for which signs or symptoms?

Fever and sore throat

A nurse is providing discharge instructions to a client following surgical treatment for carpal tunnel syndrome. Which statement by the client would indicate a need for further instruction?

I should perform pronation and supination exercises of my wrist starting twenty-four hours after surgery."

The client is diagnosed with stage I of Lyme disease. The nurse should check the client for which characteristic of this stage?

Flu-like symptoms

A nurse is caring for a client with a spinal cord injury. The nurse prepares to place high-top sneakers on the client's feet to prevent the occurrence of:

Footdrop

A nurse is preparing for the admission of a client with a diagnosis of early stage Alzheimer's disease. The nurse assists in developing a plan of care, knowing that which of the following is a characteristic of early Alzheimer's disease?

Forgetfulness

A client seeks treatment in the emergency department for a lower leg injury. There is visible deformity to the lower aspect of the leg, and the injured leg appears shorter than the other. The area is painful, swollen, and beginning to become ecchymotic. The nurse interprets that this client has experienced a:

Fracture

nurse is collecting data on a client with a diagnosis of peptic ulcer disease. Which of the following is least likely associated with this disease?

History of the use of acetaminophen (Tylenol) for pain and discomfort

A client with Guillain-Barré syndrome has been asking many questions about the condition, and the nursing staff feels that the client is very discouraged about her condition. It is important for the nurse to include which of the following information in discussions with the client?

Generally, a vast number of people recover from this condition.

Which signs/symptoms would indicate to the nurse that a client is experiencing an anaphylactic reaction? Select all that apply.

Hives Stridor Dyspnea Urticaria Wheezing

A client who has laryngeal nodules is scheduled for outpatient surgery to have them removed. The nurse collects data on the client and expects the client to complain of which typical symptom associated with this condition?

Hoarseness

The nurse determines that the client diagnosed with Ménière's disease understands the discussed dietary instructions when the client states that which food will be avoided in the diet?

Hot dogs

A nurse has provided instructions to a client with a herniated lumbar disk about proper body mechanics and other items pertinent to low back care. The nurse determines that the client needs further instructions if the client verbalizes that he or she will:

Get out of bed by sitting straight up and swinging the legs over the side of the bed.

A clinic nurse is assisting in caring for a client whose chief complaint is the presence of flu-like symptoms. Which recommendation by the nurse is therapeutic? Select all that apply.

Get plenty of rest. Take antipyretics for fever. Increase intake of liquids.

A nurse is assisting in caring for a client with a Sengstaken-Blakemore tube. To prevent ulceration and necrosis of oral and nasal mucosa, the nurse plans to

Give frequent oral and nasal care.

A nurse is assisting a client who will wear a Holter monitor for continuous cardiac monitoring over the next 24 hours. The nurse takes which of the following actions to assist this client?

Gives the client a device holder to wear around the waist

A client who has undergone barium enema is being readied for discharge. The nurse determines that the client has understood discharge instructions when the client states:

I should take a laxative, and my stool will then return to a normal color.

A client with chlamydial infection has received instructions on self-care and prevention of further infection. The nurse determines that the client needs further teaching if the client states which?

I will use an antibiotic prophylactically to prevent symptoms of Chlamydia

A client with a spinal cord injury expresses little interest in food, and is very particular about the choice of meals that are actually eaten. The nurse interprets that:

Meal choices represent an area of client control and should be encouraged as much as is nutritionally reasonable.

A client is admitted to the hospital with a venous stasis leg ulcer. The nurse inspects the ulcer, expecting to note that the ulcer:

Has a brownish or "brawny" appearance

A nurse notices that a client with trigeminal neuralgia has been withdrawn, is having frequent episodes of crying, and is sleeping excessively. The best way for the nurse to explore issues with the client regarding this behavior is to:

Have the client express the feelings in writing.

A client presents to the urgent care center with epistaxis but no obvious facial injury. The nurse should take which action first?

Have the client sit down, lean forward, and apply pressure to the nose.

An adult client with a history of ear infections reports a right earache accompanied by a sensation of fullness. The client also reports nausea and has a temperature of 100.6° F. The nurse initially questions the client about whether:

He has had a recent upper respiratory infection (URI)

A nurse is assigned to assist with caring for a client after cardiac catheterization. The nurse plans to maintain bedrest with:

Head elevation of no more than 30 degrees

A nurse is caring for a client who has undergone craniotomy with a supratentorial incision. The nurse should plan to place the client in which position postoperatively?

Head of bed elevated 30 to 45 degrees, head and neck midline

A nurse is positioning the client with increased intracranial pressure (ICP). Which position should the nurse avoid?

Head turned to the side

A nurse is caring for a client with coronary artery disease, and a topical nitrate is prescribed for the client. The nurse provides medication instructions and tells the client that acetaminophen (Tylenol) is usually prescribed to be taken before the administration of the topical nitrate because:

Headache is a common side effect of nitrates.

A nurse is assisting the health care provider with performing a Weber tuning fork test on a client. The nurse understands that this test checks for:

Hearing loss

A nurse is preparing to care for a client with a diagnosis of brain attack (stroke). The nurse notes in the client's record that the client has anosognosia. The nurse plans care, knowing that the client will:

Neglect the affected side.

A nurse has been caring for a client with a Sengstaken-Blakemore tube. The health care provider arrives on the nursing unit and deflates the esophageal balloon. Following deflation of the balloon, the nurse should monitor the client most closely for which of the following

Hematemesis

A client with endometrial cancer is receiving doxorubicin (Adriamycin), an antineoplastic agent. The nurse would specifically collect data about which of the following?

Hematological laboratory values

A nurse is reviewing the history of a client with bladder cancer. The nurse would expect to note which most common symptom of this type of cancer as being documented in the client's record?

Hematuria

A client diagnosed with pernicious anemia asks the nurse what caused the deficiency. The nurse replies that it is likely a result of which of the following conditions that is part of the client's health history?

Hemigastrectomy

A nurse analyzes the results of laboratory studies performed on a client with peptic ulcer disease. Which of the following laboratory values would indicate a complication associated with the disease?

Hemoglobin 10.2 g/dL

After the deflation of the balloon of a client's Sengstaken-Blakemore tube, the nurse monitors the client closely for which esophageal complication

Hemorrhage

It has been determined that a client with hepatitis has contracted the infection from contaminated food. What type of hepatitis is this client most likely experiencing?

Hepatitis A

Of the following infection control methods, which would be the priority to include in the plan of care to specifically prevent hepatitis B in a client considered to be at high risk for exposure?

Hepatitis B vaccine

A nurse is assisting with the insertion of a nasogastric tube into a client. The nurse places the client in which position for insertion?

High Fowler's position

A nurse discusses the risk factors associated with gastric cancer as part of a health promotion program. The nurse determines that further discussion is necessary if a member attending the program states that which factor is a risk?

High meat and carbohydrate consumption

A nursing instructor asks a nursing student about the risk factors associated with osteoporosis. The instructor tells the student that she needs to read and learn about this disorder if the student states that which of the following is an associated risk factor?

High-calcium diet consumption

A client has asymptomatic diverticular disease. What type of diet should the nurse anticipate to be prescribed

High-fiber diet

A nurse is caring for the client diagnosed with tuberculosis (TB). Which of the following findings, if made by the nurse, would be inconsistent with the usual clinical presentation of tuberculosis?

High-grade fever

A client with suspected Guillain-Barré syndrome has a lumbar puncture performed. The cerebrospinal fluid (CSF) protein is 750 mg/dL. The nurse analyzes these results as:

Higher than normal supporting the diagnosis of Guillain-Barré

A client complains of stomach pain 30 minutes to 1 hour after eating. The pain is not relieved by further intake of food, although it is relieved by vomiting, and a gastric ulcer is suspected. The nurse should gather which of the following additional supportive data for this diagnosis from the client?

History of alcohol use, smoking, and weight loss

A client complains of stomach pain 30 minutes to 1 hour after eating. The pain is not relieved by further intake of food, although it is relieved by vomiting. A gastric ulcer is suspected. Which of the following data would further support this diagnosis?

History of alcohol use, smoking, and weight loss

The client with diabetes mellitus receiving peritoneal dialysis asks the nurse why it is important to leave the dialysate infused only for a specific amount of time. The nurse responds that not adhering to the dwell time can increase the risk of the client experiencing which complication?

Hyperglycemia

The nurse is instructing a client with diabetes mellitus about peritoneal dialysis. The nurse tells the client that it is important to maintain the dwell time for the dialysis at the prescribed time because of risk for which complication?

Hyperglycemia

The nurse is admitting a client with chronic kidney disease (CKD) to the nursing unit. The nurse monitors the client for which frequent cardiovascular sign that occurs in CKD?

Hypertension

A nurse is monitoring the respiratory status of a client following insertion of a tracheostomy. The nurse understands that oxygen saturation measurements obtained by pulse oximetry may be inaccurate if the client has which of the following coexisting problems?

Hypotension

A nurse suspects neurogenic shock in a client with complete transection of the spinal cord at the T3 (thoracic 3) level if which of the following clinical manifestations are observed?

Hypotension and bradycardia

A nurse is reviewing the medication record of a client with acute gastritis. Which of the following medications, if noted on the client's record, would the nurse question

Ibuprofen (Motrin)

A client with Parkinson's disease is experiencing a parkinsonian crisis. The nurse should immediately place the client:

In a quiet, dim room with respiratory and cardiac support available

The nurse is caring for a client after a supratentorial craniotomy in which a large tumor was removed from the left side. Choose the positions in which the nurse can safely place the client. Select all that apply.

In a semi-Fowler's position With the head in a midline position

To assess for the presence of the posterior tibialis pulse, the nurse should palpate which of the following areas?

In the groove behind the medial malleolus and the Achilles tendon

A nurse is caring for a client with acute pancreatitis and a history of alcoholism and is monitoring the client for complications. Which of the following data would be a sign of paralytic ileus?

Inability to pass flatus

The nurse is evaluating the data results of a client with sepsis and acute kidney injury with related azotemia and oliguria. Which are the primary features of azotemia and oliguria? Select all that apply.

Increase in serum creatinine Increase in blood urea nitrogen (BUN) Urine output less than 0.5 mL/kg/hour

A nurse is reviewing the laboratory results of a client who has been diagnosed with multiple myeloma. Which of the following would the nurse expect to specifically note with this diagnosis?

Increased calcium level

A nurse is caring for a comatose client at risk for fat embolism because of a fractured femur and pelvis sustained in a fall. Which of the following findings does the nurse identify as early signs of possible fat embolism?

Increased heart rate and adventitious breath sounds

A nurse is reviewing the laboratory results of a client with leukemia who has received a regimen of chemotherapy. Which laboratory value would the nurse specifically note as a result of the massive cell destruction that occurs with the chemotherapy?

Increased uric acid level

A client diagnosed with primary open-angle glaucoma has been prescribed pilocarpine ophthalmic drops. When the client asks the nurse how this medication lowers intraocular pressure, the nurse tells the client that the medication:

Increases the outflow of aqueous humor

A nurse is caring for a client with increased intracranial pressure (ICP). The nurse should monitor for which of the following trends in vital signs that would occur if ICP is rising?

Increasing temperature, decreasing pulse, decreasing respirations, increasing blood pressure (BP)

A nurse notes documentation that a postcraniotomy client is having difficulty with body image. The nurse determines that the client is still working on the postoperative outcome criteria if the client:

Indicates that facial puffiness will be a permanent problem

The nurse is assisting in identifying clients in the community at risk for latex allergy. Which client population is most at risk for developing this type of allergy?

Individuals with spina bifida

A nurse is assisting in caring for a client with pneumonia who suddenly becomes restless. Arterial blood gases are drawn, and the results reveal a PaO2 of 60 mm Hg. The nurse reviews the plan of care for the client and determines that which priority problem potentially exists for this client?

Ineffective oxygen and carbon dioxide exchange

The nurse is assisting in developing a plan of care for a client with immunodeficiency. The nurse understands that which problem is a priority for the client?

Infection

A client has Buck's extension traction applied to the right leg. The nurse should plan which intervention to prevent complications of the device?

Inspecting the skin on the right leg at least once every 8 hours

A nurse is planning care for a client with Bell's palsy. Which measure should be included in the plan?

Instill artificial tears and wear a patch over the affected eye at night.

A client arrives at the emergency department with a foreign body in the left ear that has been determined to be an insect. Which intervention would the nurse anticipate to be prescribed initially?

Instillation of mineral oil or diluted alcohol

A nurse is planning care for a client whose oxygenation is being monitored by a pulse oximeter. Which of the following is important to include to ensure accurate monitoring of the client's oxygenation status?

Instruct the client not to move the sensor

male client with chronic obstructive pulmonary disease (COPD) on bedrest is weaned from the ventilator before transferring to a medical unit. To adequately restore client strength before getting the client out of bed, what is the priority client activity for the nurse to incorporate in the plan of care?

Instruct the client to reposition himself.

A client in a long-term care facility who has a history of angina pectoris wants to go for a short walk outside with a family member. It is a sunny but chilly December day. The nurse should do which of the following to care for this client in a holistic manner?

Instruct the family member to dress the client warmly before going outside.

A nurse is teaching a local women's church group about the risks of cervical cancer. The nurse determines that further teaching is necessary if a group member states that which of the following is a risk factor?

Intercourse with circumcised males

A nurse is caring for a client after a Billroth II procedure. On review of the postoperative prescriptions, which of the following, if prescribed, would the nurse question and verify?

Irrigating the nasogastric (NG) tube

A nurse is working with a client who has been diagnosed with Prinzmetal's (variant) angina. The nurse plans to teach the client that this type of angina:

Is generally treated with calcium-channel blocking agents

A nurse has given simple instructions on preventing some of the complications of bedrest to a client who experienced a myocardial infarction. The nurse would intervene if the client were performing which of these activities, which would be contraindicated?

Isometric exercises of the arms and legs

Which statement by the nurse indicates an understanding of the diagnosis of presbycusis?

It is a sensorineural type of hearing loss that occurs with aging."

The family of an unconscious client with increased intracranial pressure is talking at the client's bedside. They are discussing the gravity of the client's condition and wondering if the client will ever recover. The nurse intervenes, based on the understanding that:

It is possible the client can hear the family.

A nurse is assisting with providing a teaching session to a community group regarding the risks and causes of bladder cancer. The nurse determines that additional teaching is needed if a member of the community group states which of the following regarding this type of cancer?

It most often occurs in women.

A nurse is caring for a client following a total hip replacement. The client has been diagnosed with iron deficiency anemia. The nurse instructs the client to increase intake of which of the following foods?

Lean beef and chicken liver

A nurse is assisting in planning stress management strategies for the client with irritable bowel syndrome. Which suggestion would the nurse give to the client?

Learn measures such as biofeedback or progressive relaxation.

A nurse should include which of the following in a teaching plan for a client who has peptic ulcer disease?

Learn to use stress reduction techniques

A nurse would include which of the following when reinforcing home care instructions for a client who has peptic ulcer disease?

Learn to use stress reduction techniques.

A nurse is administering a cleansing enema to a client with a fecal impaction. Before administering the enema, the nurse assists the client to which of the following positions?

Left Sims' position

A nurse is teaching the client about an upcoming colonoscopy procedure. The nurse would include in the instructions that the client will be placed in which of the following positions for the procedure?

Left Sims' position

A client has had skeletal traction applied to the right leg and has an overhead trapeze available for use. The nurse should monitor which of the following as a high-risk area for pressure and breakdown?

Left heel

A client receiving parenteral nutrition through a central intravenous line is exhibiting signs and symptoms of an air embolism. The nurse immediately places the client in which position?

Left side in Trendelenburg's

A client with right pleural effusion by chest x-ray is being prepared for a thoracentesis. The client experiences dizziness when sitting upright. The nurse assists the client to which of the following positions for the procedure?

Left side-lying with the head of the bed elevated 45 degrees

A nurse is caring for a client with a diagnosis of myocardial infarction (MI) and is assisting the client in completing the diet menu. Which of the following beverages does the nurse instruct the client to select from the menu?

Lemonade

A nurse provides instructions to a client after a liver biopsy. The nurse tells the client to:

Lie on the right side for 2 hours

The nurse is assisting a client who is new to a low-potassium diet to select food items from the menu. Which food item is lowest in potassium and should be recommended to the client on this dietary restriction?

Lima Beans

A client with spinal cord injury is prone to experiencing autonomic dysreflexia. The nurse should avoid which measure to minimize the risk of recurrence?

Limiting bladder catheterization to once every 12 hours

A nurse positions a client for a surgical procedure. Which position can likely lead to the potential for decreased lung expansion in the client?

Lithotomy

A nurse is reinforcing instructions to a client taking divalproex sodium (Depakote). The nurse tells the client to return to the clinic for follow-up laboratory studies related to which test?

Liver function studies

A client with an oral endotracheal tube attached to a mechanical ventilator is about to begin the weaning process. The nurse asks the health care provider whether this process should be delayed temporarily, based on administration of which of the following medications to the client in the last hour?

Lorazepam (Ativan)

A nurse is determining the need for suctioning in a client with an endotracheal tube (ETT) attached to a mechanical ventilator. Which observation by the nurse is inconsistent with the need for suctioning?

Low peak inspiratory pressure on the ventilator

A nurse is assisting with conducting a health-promotion program at a local school. The nurse determines that additional teaching is needed if a student identifies which of the following as a risk factor associated with cancer?

Low-fat and high-fiber diets

A nurse is assigned to care for a client hospitalized with Ménière's disease. The nurse expects that which of the following would most likely be prescribed for the client?

Low-sodium diet

A nurse is communicating with a client who is hard of hearing in both ears. To facilitate communication with this client, the nurse should:

Lower the voice pitch and face the client when speaking.

A client who has had a radical neck dissection begins to hemorrhage at the incision site. Which action by the nurse would be contraindicated?

Lowering the head of the bed to a flat position

A nurse is assisting in caring for a client with a suspected diagnosis of meningitis. The nurse reinforces to the client information regarding which diagnostic test that is commonly used to confirm this diagnosis?

Lumbar puncture

A client with infective endocarditis is at risk for heart failure. The nurse monitors the client for which of the following?

Lung crackles, peripheral edema, and weight gain

A client is being prepared for a thoracentesis. The nurse assigned to care for the client assists the client to which of the following positions for the procedure?

Lying in bed on the unaffected side with the head of the bed elevated 45 degrees

A nurse is caring for a client in the emergency department who has right lower quadrant abdominal pain. After noting a white blood cell count of 16,500/mm3, the nurse should question a prescription for which of the following?

MOM

A nurse is caring for a client within the first 24 hours following a total gastrectomy for gastric cancer. The nurse should focus interventions on which of the following during this time frame?

Maintaining a patent nasogastric (NG) tube

A nurse is planning to put aneurysm precautions in place for the client with a cerebral aneurysm. Which item would be included as part of the precautions?

Maintaining the head of the bed at 15 degrees

A nurse planning care for a client with hepatitis plans to meet the client's safety needs by:

Monitoring prothrombin and partial thromboplastin values

A nurse is caring for the client who has skeletal traction applied to the left leg. The client complains of severe left leg pain. The nurse checks the client's alignment in bed and notes that proper alignment is maintained. Which action should the nurse take next?

Notify the registered nurse.

A nurse observes that a client with Parkinson's disease has very little facial expression. The nurse attributes this piece of data to which of the following?

Mask-like facies is a component of Parkinson's disease.

A nurse is caring for a client with a long bone fracture who is at risk for fat embolism. The nurse specifically monitors for the earliest signs of this complication by checking the client's: Select all that apply.

Mental status Respiratory function

A nurse is reviewing the record of a client with a suspected diagnosis of Huntington's disease. Which documented early symptom supports this diagnosis?

Vertigo

A client has had a set of arterial blood gases drawn. The results are pH 7.34, Paco2 of 37, Pao2 of 79, HCO of 19. The nurse interprets that the client is experiencing:

Metabolic alkalosis

A nurse is reviewing the medical record of a client diagnosed with amyotrophic lateral sclerosis (ALS). Which initial clinical manifestation of this disorder supports this diagnosis?

Mild clumsiness

A client in the emergency department reports right lower quadrant abdominal pain. After noting a white blood cell count of 16,500/mm3, the nurse would question a prescription for which of the following

Milk of magnesia

A client was seen and treated in the emergency department for treatment of a concussion. The nurse determines that the family needs further discharge instructions if they say to bring the client back to the emergency department if which of the following occurs?

Minor headache

Which nursing interventions are appropriate for a client recovering from surgery for retinal detachment? Select all that apply.

Monitor for hemorrhage. Administer eye medications. Maintain the eye patch or shield. Assist with activities of daily living. Educate regarding symptoms of retinal detachment

A nurse is assisting in caring for a client with a diagnosis of bladder cancer who recently received chemotherapy. The nurse receives a telephone call from the laboratory who reports that the client's platelet count is 20,000/mm3. Based on this laboratory value, the nurse revises the plan of care and suggests including which intervention?

Monitor skin for the presence of petechiae.

A thymectomy via a median sternotomy approach is performed on a client with a diagnosis of myasthenia gravis. The nurse has assisted in developing a plan of care for the client and includes which of the following in the plan?

Monitor the chest tube drainage.

A nurse develops a plan of care for a client following a lumbar puncture. Which interventions should be included in the plan? Select all that apply.

Monitor the client's ability to void. Maintain the client in a flat position. Monitor the client's ability to move the extremities. Inspect the puncture site for swelling, redness, and drainage.

A nurse is preparing a plan of care to monitor for complications in a client who will be returning from the operating room following transsphenoidal resection of a pituitary adenoma. Which of the following does the nurse document in the plan as the priority nursing intervention for this client?

Monitor urine output.

A nurse is providing endotracheal suctioning to a client who is mechanically ventilated, when the client becomes restless and tachycardic. What should the nurse do?

Monitor vital signs and discontinue attempts at suctioning until the client is stabilized.

A client has just undergone a gastroscopy. Which action should be taken by the nurse as the essential post-procedure nursing intervention?

Monitoring for the gag reflex

A nurse is evaluating the client's use of a cane for left-sided weakness. The nurse would intervene and correct the client if the nurse observed that the client:

Moves the cane when the right leg is moved

A nurse is planning care for a client in spinal shock. Which of the following actions would be least helpful in minimizing the effects of vasodilation below the level of the injury?

Moving the client quickly as one unit

A nurse has given instructions to the client with Raynaud's disease about self-management of the disease process. The nurse determines that the client needs further instructions if the client states that:

Moving to a warmer climate should help.

A client with a herniated intervertebral lumbar disk complains of a knifelike, stabbing pain in the lower back, as well as pain radiating into the right buttock. The nurse interprets that the sharp, stabbing pain is probably a result of:

Muscle spasm in the area of the herniated disk

A nurse is collecting data from a client admitted to the hospital with a diagnosis of suspected gastric ulcer and is asking the client questions about pain. Which statement, if made by the client, would support the diagnosis of gastric ulcer?

My pain comes shortly after I eat, maybe a half hour or so later

Prescriptive eyeglasses are prescribed for a client with bilateral aphakia. When reinforcing teaching instructions regarding the glasses, the nurse determines the need for further instructions when the client states:

My peripheral vision will not be distorted."

A client with a nasal tumor is being admitted to the hospital. The nurse collects data about which primary symptom that the client is expected to exhibit?

Nasal obstruction

A client arrives in the emergency department with bleeding esophageal varices, and the health care provider states that a Sengstaken-Blakemore tube will need to be used to try to control the gastrointestinal hemorrhage. The nurse prepares for insertion of the tube via which of the following routes?

Nasogastric

Treatment measures have been implemented for a client with bleeding esophageal varices and have been unsuccessful. The health care provider states that a Sengstaken-Blakemore tube will be used to control the resulting hemorrhage. The nurse prepares for insertion via which of the following routes?

Nasogastric

A client with a tentative diagnosis of gastroesophageal reflux is going to undergo ambulatory pH monitoring. The nurse brings which of the following items to the bedside?

Nasogastric (NG) tube

A client is receiving radiation therapy to the brain because of a diagnosis of a brain tumor. Which side effect does the nurse expect the client to likely experience?

Nausea and vomiting

An adult client has increased fluid in the middle ear, which is causing vertigo. The nurse assesses this client for which associated signs and symptoms of this condition?

Nausea and vomiting

A client with end stage kidney disease (ESKD) begins peritoneal dialysis. The nurse observes for which signs/symptoms indicating peritonitis? Select all that apply.

Nausea and vomiting Abdominal tenderness Cloudy peritoneal effluent Oral temperature of 38 C

A nurse is reading the results of a Mantoux skin test on a client with no documented health problems. The site has no induration and a 1-mm area of ecchymosis. The nurse interprets that the result is:

Negative

A nurse is reading the results of the Mantoux skin test for a client who has no documented health problems. The site has no induration and a 1-mm area of ecchymosis. The nurse interprets that the result is:

Negative

A nurse has reinforced instructions to a client who is scheduled for a cataract extraction. Which statement by the client indicates a need for further instruction?

No eating or drinking for at least 18 hours before the surgery."

A nurse is caring for a client suspected of having appendicitis. Which of the following would the nurse anticipate will be prescribed for this client?

No oral intake of liquids or food

A client with Crohn's disease is seen by the health care provider, and a complete blood count (CBC) has been prescribed. The nurse provides instructions to the client who will be reporting to the laboratory in the morning to have the blood test drawn. The nurse gives the client which of the following information about this test

No special preparation is necessary.

The nurse is evaluating the assessment of a client's arteriovenous fistula being used for hemodialysis. Which findings would prompt the nurse to notify the health care provider immediately? Select all that apply.

No thrill palpated at fistula site No bruit auscultated at the fistula site Absent pulse distal to the arteriovenous fistula

A client has just undergone spinal fusion after suffering a herniated lumbar disk. The nurse should avoid which of the following to maintain client safety after this procedure?

Overhead trapeze

A nurse is observing a nursing student listening to the breath sounds of a client. The nurse intervenes if the student performs which incorrect procedure?

Places the stethoscope on the client's gown

The client has undergone funduscopic examination of the eye. The documented results indicate that the blood vessels are without tortuosity, narrowing, pulsation, or nicking. The nurse interprets that this report indicates which of the following?

Normal retinal examination

A nurse is checking the neurovascular status of a client who returned to the surgical nursing unit 4 hours ago after undergoing aortoiliac bypass graft. The affected leg is warm, and the nurse notes redness and edema. The pedal pulse is palpable and unchanged from admission. The nurse interprets that the neurovascular status is:

Normal, caused by increased blood flow through the leg

A client has a chest tube that is attached to a chest drainage system. The client asks the nurse, "Can the tube come out faster if you turn the wall suction up higher?" The nurse's response is based on the understanding that turning up the wall suction would:

Not increase the actual suction in the system but would cause more air to be pulled through the air vent and suction chamber to the suction source

A client is admitted to the emergency department following a fall from a horse. The health care provider (HCP) prescribes the insertion of an indwelling urinary catheter. The nurse notes blood at the urinary meatus while preparing for the procedure. Which action should the nurse take?

Notify the health care provider

A nurse is reviewing the laboratory results of a client with bladder cancer and bone metastasis and notes that the calcium level is 15 mg/dL. The nurse should take which appropriate action?

Notify the health care provider.

A nurse is caring for the client who has skeletal traction applied to the left leg. The client complains of severe left leg pain. The nurse checks the client's alignment in bed and notes that proper alignment is maintained. Which action should the nurse take next?

Notify the nurse

A client recovering from a craniotomy complains of a "runny nose." Based on the interpretation of the client's complaint, the best nursing action is to:

Notify the registered nurse.

A client with a left arm fracture exhibits loss of sensation in the left fingers, pallor, slow refill, and diminished left radial pulse. The nurse should take which of the following actions?

Notify the registered nurse.

A nurse in the ambulatory care unit is caring for a client following cataract extraction. The client suddenly complains of nausea and severe eye pain in the surgical eye. The nurse should take which of the following actions immediately?

Notify the registered nurse.

A nurse is caring for a client following enucleation. On data collection the nurse notes staining and bleeding on the dressing. The nurse should take which appropriate action?

Notify the registered nurse.

A nurse is caring for a client who has bilateral vocal cord paralysis. The client begins to experience severe dyspnea; the nurse listens to the client's breath sounds and hears this sound. (Click on the sound icon.) What intervention should the nurse take immediately?

Notify the registered nurse.

A nurse has provided instructions to a client in an arm cast about the signs and symptoms of compartment syndrome. The nurse determines that the client understands the information if the client states to report which early symptom of compartment syndrome?

Numbness and tingling in the fingers

The nurse interprets that the client who is prescribed zalcitabine (Hivid) is experiencing an adverse effect of this medication when which event is reported by the client?

Numbness in the legs

A client is recovering at home after suffering a brain attack (stroke) 2 weeks ago. A home caregiver tells the home health nurse that the client has some difficulty swallowing food and fluids. Which nursing action would be appropriate as a first action?

Observe the client feeding himself or herself.

A nurse is preparing a list of home care instructions regarding stoma and laryngectomy care to a client who had a laryngectomy. Choose the instructions that would be included in the list. Select all that apply.

Obtain a Medic-Alert bracelet, Prevent debris from entering the stoma, Avoid exposure to people with infections, Avoid swimming and use care when showering

The nurse is providing instructions to a client who will be self-administering eyedrops. To minimize the systemic effects that eyedrops can produce, the client is instructed to:

Occlude the nasolacrimal duct with a finger over the inner canthus for 30 to 60 seconds after instilling the drops

A client who has been prescribed indomethacin (Indocin) for gout is asked to provide a stool sample for guaiac testing. The nurse explains that the purpose of the test is to determine:

Occult blood

A client with viral hepatitis has no appetite, and food makes the client nauseated. Which nursing intervention would be appropriate

Offer small, frequent meals

A nurse is collecting data on a client with a diagnosis of meningitis and notes that the client is assuming this posture. (Refer to figure.) The nurse contacts the health care provider and reports that the client is exhibiting:

Opisthotonos

A nurse is providing instructions to a client scheduled for conization in 1 week for the treatment of microinvasive cervical cancer. The procedure has been explained by the health care provider, and the nurse is reviewing the complications associated with the procedure. The nurse determines that the client needs further instruction if the client states that which of the following is a complication of this procedure?

Ovarian perforation

In what area of the chest would the nurse expect to auscultate this breath sound? (Click on the sound icon.)

Over the peripheral lung fields

A nurse is caring for a client who has been admitted to the hospital with a diagnosis of angina pectoris. The client is receiving oxygen via nasal cannula at 2 L. The client asks the nurse why the oxygen is necessary. The nurse bases the response on which of the following?

Oxygen supply to the heart cells that is deficient results in angina pectoris pain.

A nurse is setting up the bedside unit for a client being admitted to the nursing unit from the emergency department with a diagnosis of coronary artery disease (CAD). The nurse should place highest priority on making sure that which of the following is available at the bedside?

Oxygen tubing and flowmeter

A client is admitted to the hospital with acute exacerbation of chronic obstructive pulmonary disease (COPD). Which of the following arterial blood gases support this diagnosis?

PO2 of 60 mm Hg and PCO2 of 50 mm Hg

A nurse is assisting a client who underwent radical neck surgery to get out of bed. The nurse provides the support to the client, who is afraid to move the head by doing which of the following?

Placing a hand behind the client's head

A client with retinal detachment is admitted to the outpatient nursing unit in preparation for a scleral buckling procedure. Which of the following would the nurse anticipate to be prescribed?

Placing an eye patch over the client's affected eye

The nurse is assisting in planning a diet for a client with acute kidney injury (AKI). The nurse plans to restrict which dietary component from this client's diet?

Potassium

A nurse is caring for a client with osteoarthritis. The nurse monitors the client, knowing that which of the following is a clinical manifestation associated with the disorder?

Pain that increases with activity and is relieved by rest

The nurse is urging a client to cough and deep breathe after a nephrectomy. The client tells the nurse, "That's easy for you to say! You don't have to do this." The nurse interprets that the client's statement is likely a result of which contributing factor?

Pain that is intensified because the location of the incision is near the diaphragm

A clinic nurse is reviewing the record of a client recently diagnosed with a cataract. Which clinical manifestation associated with this disorder should the nurse expect to be documented in the client's record?

Painless progressive loss of vision

A hemodialysis client with a left arm fistula is at risk for arterial steal syndrome. The nurse monitors this client for which signs/symptoms of this disorder?

Pallor, diminished pulse, and pain in the left hand

A nurse is using a stethoscope to listen to the client's heart and hears this sound. (Click on the sound icon.) To assist in identifying the sound, the nurse should take which initial best action?

Palpate the carotid pulse for a pulsation.

A nurse is assigned to care for a client who has a chest tube. The nurse is told to monitor the client for crepitus (subcutaneous emphysema). The nurse monitors the client for this complication by:

Palpating for the leakage of air into the subcutaneous tissues

The nurse is assessing the patency of an arteriovenous fistula in the left arm of a client who is receiving hemodialysis for the treatment of chronic kidney disease. Which finding indicates that the fistula is patent?

Palpation of a thrill over the fistula

A client with no history of respiratory disease is admitted to the hospital with respiratory failure. The nurse reviews the arterial blood gas reports for which of the following results, which are consistent with this disorder?

Pao2 49 mm Hg, Paco2 52 mm Hg

A postoperative client is using an incentive spirometer. The nurse observes the client inhale slowly with the mouthpiece placed between the teeth with the lips closed. The client inhales to the preset inspiratory goal and holds the breath for about 3 seconds, then exhales slowly. The client takes one breath and returns the incentive spirometer to the bedside. Based on this observation, what interpretation should the nurse make?

The client should be repeating the sequence 10 to 20 times in each session

A nurse is auscultating a client's heart sounds and hears these sounds. (Click on the sound icon.) The nurse identifies these as being produced during which phase of the cardiac cycle?

Passive filling phase of ventricles

A client with a brain attack (stroke) is experiencing residual dysphagia. The nurse would remove which of the following food items that arrived on the client's meal tray from the dietary department?

Peas

A nurse is evaluating the effects of care for the client with deep vein thrombosis. Which of the following limb observations would the nurse note as indicating the least success in meeting the outcome criteria for this problem?

Pedal edema that is 3+

A client is tentatively diagnosed with ovarian cancer. The nurse gathers data about which late manifestation of this disease?

Pelvic pain, anemia, and ascites

A gastric analysis is prescribed for a client with a suspected diagnosis of tuberculosis (TB). The nurse understands that the test is relevant in confirming this diagnosis because:

People can frequently swallow small amounts of sputum

A nurse is assisting in checking for Tinel's sign in a client suspected of having carpal tunnel syndrome (CTS). Which technique would the nurse expect to be used to elicit this sign?

Percuss the medial nerve at the wrist as it enters the carpal tunnel, and monitor for tingling sensations.

A nurse is assigned to assist the health care provider with the removal of a chest tube. The nurse instructs the client to do which of the following during this process?

Perform Valsalva's maneuver

A nurse is caring for a client with a head injury and is monitoring the client for signs of increased intracranial pressure (ICP). Which sign if noted in the client would the nurse report immediately?

The client vomits.

A client who had a prostatectomy has learned perineal exercises to gain control of the urinary sphincter. The nurse determines that the client needs further teaching if the client states that he will perform which action as part of these exercises?

Perform the Valsalva maneuver

A health care provider is about to remove a chest tube from a client. Once the dressing is removed and the sutures have been cut, the nurse assisting the health care provider asks the client to:

Perform the Valsalva maneuver

A nurse has provided instructions regarding specific leg exercises for the client immobilized in right skeletal lower leg traction. The nurse determines that the client needs further instruction if the nurse observes the client:

Performing active range of motion (ROM) to the right ankle and knee

A client is admitted to the hospital with a diagnosis of pericarditis. The nurse reviews the client's record for which manifestation that differentiates pericarditis from other cardiopulmonary problems?

Pericardial friction rub

A nurse is caring for a client who has a malignant lung neoplasm and has developed cardiopulmonary complications. (Click on the sound icon.) On auscultation, the nurse hears these breath sounds over the left lower sternal border (over the apical area) and interprets the sounds as which of the following?

Pericardial friction rub

A client with cancer is at risk for experiencing vena cava syndrome. The nurse would monitor this client for which of the following as an early sign of this oncological emergency?

Periorbital edema

A client who has been taking isoniazid (INH) for 1½ months complains to the nurse about numbness, paresthesias, and tingling in the extremities. The nurse interprets that the client is experiencing:

Peripheral neuritis

A nurse is participating in a health screening clinic and is preparing teaching materials about colorectal cancer. The nurse would plan to include which risk factor for colorectal cancer in the material?

Personal history of ulcerative colitis or gastrointestinal (GI) polyps

The use of peritoneal dialysis for the treatment of chronic kidney disease would be contraindicated for which client?

The client with severe emphysema

A nurse is observing a client with chronic obstructive pulmonary disease (COPD) performing the pursed-lip breathing technique. Which observation by the nurse would indicate accurate performance of this breathing technique?

The client's exhalation is twice as long as inhalation.

A client is hospitalized for the insertion of an internal cervical radiation implant. While giving care, the nurse finds the radiation implant in the bed. The nurse would immediately:

Pick up the implant with long-handled forceps and place into a lead container.

A client has been diagnosed with glaucoma. The nurse who is teaching the client principles of self-care would encourage the client to limit or refrain from which of the following usual activities on a repeated basis?

Picking objects up off the floor

A nurse is repositioning the client who has returned to the nursing unit following internal fixation of a fractured right hip. The nurse plans to use a:

Pillow to keep the right leg abducted during turning

A nurse is preparing a plan of care for a client in skeletal leg traction with an overbed frame. Which nursing intervention should be included in the plan of care to best assist the client with positioning in bed?

Place a trapeze on the bed to provide a means for the client to lift the hips off the bed.

A nurse is preparing a plan of care for a client being admitted to the hospital with a diagnosis of retinal detachment. Which of the following will the nurse include in the plan of care?

Place an eye patch over the affected eye.

A nurse is assisting in developing a postoperative plan of care for a client following a mastectomy. Choose the interventions that will be included in the plan of care. Select all that apply

Place the affected arm on a pillow, Assess the incision for signs of infection, Monitor and measure drainage in the collection device.

An older client with ischemic heart disease has experienced an episode of dizziness and shortness of breath. The nurse reviews the plan of care and notes documentation of decreased cardiac output, dyspnea, and syncopal episodes. The nurse plans to take which important action in the care of the client?

Place the client on a cardiac monitor.

A nurse is caring for a client with a nasogastric (NG) tube and tests the pH of the aspirate to determine correct placement. The nurse notes that the pH is 5 and determines that the:

Placement of the NG tube is accurate.

A client arrives in the emergency department with an episode of status asthmaticus. The nurse first:

Places the client in high-Fowler's position

A student nurse is assigned to assist in caring for a client with acute pulmonary edema who is receiving digoxin (Lanoxin) and heparin therapy. The nursing instructor reviews the plan of care formulated by the student and tells the student that which intervention is unsafe?

Restricting the client's potassium intake

A client seeking treatment for an episode of hyperthermia is being discharged to home. The nurse determines that the client needs clarification of discharge instructions if the client stated that he or she will:

Resume full activity level immediately.

A nurse has given the client with hepatitis instructions about postdischarge management during convalescence. The nurse determines that the client needs further teaching if the client states to:

Resume full activity level within 1 week

A nurse is assisting a health care provider in performing a caloric test on a client. Following instillation of warm water into the ear, the nurse notes that nystagmus does not occur. The nurse would document the findings of this test as:

Positive

When the nurse taps at the level of the client's facial nerve, the following response is noted (refer to figure). How should the nurse document this finding on the client record?

Positive Chvostek's sign

A client complains of pain in the lower back and pain and spasms in the hamstrings when the nurse attempts to extend the client's leg. (Refer to figure.) How should the nurse record this finding on the client's medical record?

Positive Kernig's sign

A nurse is collecting admission data on a client with Parkinson's disease. The nurse asks the client to stand with the feet together and the arms at the side and then to close the eyes. The nurse notes that the client begins to fall when the eyes are closed. Based on this finding, the nurse documents which of the following in the client's record?

Positive Romberg's test

A client with acute kidney injury (AKI) has been treated with sodium polystyrene sulfonate (Kayexalate) by mouth. The nurse evaluates this therapy as effective if which value is noted on follow-up laboratory testing?

Potassium, 4.9 mEq/L

A bone marrow aspiration is scheduled for a client suspected of having leukemia. The nurse prepares supplies for the procedure and plans to bring which of the following skin cleansing agents to the bedside before this procedure?

Povidone-iodine (Betadine)

A hypertensive client who has been taking metoprolol (Lopressor) has been prescribed to decrease the dose of the medication. The client asks the nurse why this must be done over a period of 1 to 2 weeks. In formulating a response, the nurse incorporates the understanding that abrupt withdrawal could:

Precipitate rebound hypertension

A nurse is checking the casted extremity of a client. The nurse would check for which of the following signs and symptoms indicative of infection?

Presence of a "hot spot" on the cast

A nurse is caring for a client with a resolved intestinal obstruction who has a nasogastric tube in place. The health care provider has now prescribed the nasogastric tube to be discontinued. To determine the client's readiness for discontinuation of the nasogastric tube, the nurse should check for:

Presence of bowel sounds in all four quadrants

A client with peptic ulcer disease has been prescribed misoprostol (Cytotec) and sucralfate (Carafate). The nurse teaches the client that these two medications will work primarily to:

Protect the gastric mucosa

A client with peptic ulcer disease has been prescribed to take misoprostol (Cytotec) and sucralfate (Carafate). The nurse teaches the client that these two medications will work primarily to:

Protect the gastric mucosa

A client is diagnosed with an immune deficiency. The nurse focuses on which nursing responsibility as the highest priority when providing care to this client?

Protecting the client from infection

The nurse is assisting in planning care for a client with a diagnosis of immune deficiency. The nurse should incorporate which as a priority in the plan of care?

Protecting the client from infection

A nurse is attempting to inspect the lacrimal apparatus of a client's eye. Because of its anatomical location, the nurse should do which of the following?

Retract the upper eyelid and ask the client to look down.

Cycloserine (Seromycin) is added to the medication regimen for a client with tuberculosis. Which of the following would the nurse suggest to include in the client teaching plan regarding this medication?

Return to the clinic weekly for serum drug levels.

A nurse notes bilateral 2+ edema in the lower extremities of a client with known coronary artery disease who was admitted to the hospital 2 days ago. Based on this finding, the nurse implements which action?

Reviews the intake and output records for the last 2 days

A nurse is monitoring for stoma prolapse in a client with a colostomy. The nurse would observe which of the following appearances in the stoma if prolapse occurred

Protruding and swollen

A nurse is planning measures to decrease the incidence of chest pain for a client with angina pectoris. The nurse should do which of the following to effectively accomplish this goal?

Provide a quiet and low-stimulus environment.

A nurse is caring for a client with a Sengstaken-Blakemore tube. To prevent ulceration and necrosis of oral and nasal mucosa, the nurse should plan to:

Provide frequent oral and nasal care on a regular basis.

A client with lung cancer receiving chemotherapy tells the nurse that the food on the meal tray tastes "funny." Which of the following is the appropriate nursing intervention?

Provide oral hygiene care frequently

A nurse is caring for a client with tuberculosis (TB) who is fearful of the disease and anxious about the prognosis. In planning nursing care, the nurse would incorporate which of the following as the best strategy to assist the client in coping with the disease?

Provide reassurance that continued compliance with medication therapy is the most proactive way to cope with the disease

The nurse is reinforcing instructions to a client about the types of fluids that assist in prevention and treatment of urinary tract infections (UTIs). The nurse instructs the client to consume which fluids? Select all that apply.

Prune juice Apple juice Cranberry juice

A nurse is collecting data on a client admitted to the hospital with hepatitis. Which data would indicate that the client may have liver damage?

Pruritus

A nurse is monitoring a client following a motor vehicle accident. The nurse determines the need to prepare for chest tube insertion when the client exhibits:

Shortness of breath and tracheal deviation

A nurse assists in developing a plan of care for a client admitted to the hospital with an acute myocardial infarction (MI). The nurse identifies that the priority problem during the acute phase would be:

The client's pain

A client who has experienced a brain attack (stroke) has partial hemiplegia of the left leg. The straight-leg cane formerly used by the client is not sufficient any longer. The nurse determines that the client could benefit from the somewhat greater support and stability provided by a:

Quad cane

A client has a prescription to have radial arterial blood gases (ABGs) drawn. Before drawing the sample, an Allen test will be performed. In performing the Allen test, the nurse assists to occlude the:

Radial and ulnar arteries, releases one, evaluates the color of the hand, and repeats the process with the other artery

A client with spinal cord injury suddenly experiences an episode of autonomic dysreflexia. After checking vital signs, the nurse immediately:

Raises the head of the bed and removes the noxious stimulus

A client is suspected of having systemic lupus erythematous. The nurse monitors the client, knowing that which is one of the initial characteristic signs/symptoms of systemic lupus erythematous?

Rash on the face across the nose and on the cheeks

A client with arthritis of the hands and fingers is having difficulty using a metered-dose inhaler (MDI). The nurse suggests to ask the health care provider for a prescription to use a(n):

Spacer

A nurse is collecting data on a client suspected of having Alzheimer's disease. The priority data would focus on which of the following characteristic of this disease?

Recent memory loss

An adult client had a cerebrospinal fluid (CSF) analysis after lumbar puncture. The nurse interprets that a negative value of which of the following is consistent with normal findings?

Red blood cells

A nurse is reviewing the record of a client with mastoiditis. The nurse would expect to note which documented characteristic regarding the results of the otoscopic examination?

Red, dull, thick, and immobile tympanic membrane

A nurse is providing care to a client with increased intracranial pressure (ICP). Which approach(es) may be beneficial in controlling the client's ICP from an environmental viewpoint? Select all that apply.

Reducing environmental noise Maintaining a calm atmosphere Allowing the client uninterrupted time for sleep

A postgastrectomy client who is being discharged from the hospital tells the nurse, "I hope my stomach problems are over. I need to get back to work right away. I've missed a lot of work and I'm really behind. If I don't get my act together, I may lose my job." Based on the client's statement, the nurse determines that at this time, it is appropriate to discuss:

Reducing stressors in life

A client with a suspected throat infection with Streptococcus needs to have a throat culture obtained. The nurse should do which of the following after obtaining the culture if the specimen cannot be delivered to the laboratory for at least an hour?

Refrigerate the specimen

A nurse reviews the care plan of a client with cancer undergoing chemotherapy. The nurse notes that the client has a concern about her appearance as a result of alopecia. The nurse plans to tell the client which of the following about hair loss and regrowth to assist the client in coping with this possible change?

Regrown hair may have a different color and texture.

A nurse is monitoring a client for bradypnea. Which is characteristic of this respiratory pattern?

Regular but abnormally slow

A nurse is reviewing the medical history of a client admitted to the hospital with a diagnosis of colorectal cancer. The nurse understands that which information documented in the medical history is an unassociated risk factor of this type of cancer?

Regular consumption of a high-fiber diet

A nurse provides information to a client following a gastrectomy about the signs and symptoms of pernicious anemia. The nurse understands that:

Regular monthly injections of vitamin B12 are used to treat this disorder

A nurse reinforces instructions to a client following a gastrectomy about the signs and symptoms of pernicious anemia, knowing that:

Regular monthly injections of vitamin B12 will prevent this complication

A client is admitted to the hospital with a bowel obstruction secondary to a recurrent malignancy, and the health care provider plans to insert a Miller-Abbott tube. When the nurse tries to explain the procedure, the client interrupts the nurse and states, "I don't want to hear about that. Just let the doctor do it." Based on the client's statement, the nurse determines that the best action is to:

Remain with the client and be silent.

A nurse has instructed the family of a brain attack (stroke) client who has homonymous hemianopsia about measures to help the client overcome the deficit. The nurse determines that the family understands the measures to use if they state that they will:

Remind the client to turn the head to scan the lost visual field.

A nurse notes that a hospitalized client has experienced a positive reaction to the Mantoux skin test. Which action by the nurse is the priority?

Report the findings.

A nurse is caring for a client after enucleation and notes the presence of bright red drainage on the dressing. The nurse takes which appropriate action?

Reports the finding to the registered nurse (RN)

A nurse is working in a tuberculosis (TB) screening clinic. The nurse understands that which population is at highest risk for TB?

Residents of a long-term care facility

A licensed practical nurse (LPN) is assisting in caring for a client with a diagnosis of myocardial infarction (MI). The client is experiencing chest pain that is unrelieved by the administration of nitroglycerin. The registered nurse administers morphine sulfate to the client as prescribed by the health care provider. Following administration of the morphine sulfate, the LPN plans to monitor:

Respirations and blood pressure

Arterial blood gases (ABG) are obtained on a client with pneumonia. The ABG results are pH, 7.50; PCO2, 30 mm Hg; HCO, 20 mEq/L; PO2, 75 mm Hg. The nurse interprets these results and determines that which of the following acid-base conditions exists?

Respiratory alkalosis

A nurse is caring for a client after pulmonary angiography via catheter insertion into the left groin. The nurse monitors for an allergic reaction to the contrast medium by noting the presence of:

Respiratory distress

An emergency department nurse is assigned to assist in caring for a client who has suffered a head injury following a motor vehicle accident. The nurse understands that the initial data collection should focus on which of the following?

Respiratory status

A nurse is planning therapeutic interventions for a client who experienced a rib fracture 2 days earlier. The nurse understands that which intervention should be included? Select all that apply.

Rest Local heat Analgesics

A nurse is caring for a client who begins to experience seizure activity while in bed. Which action by the nurse would be contraindicated?

Restrain the client's limbs.

A nurse is assisting with developing a plan of care for a client who is experiencing hematological toxicity as a result of chemotherapy. The nurse suggests including which of the following in the plan of care?

Restricting fresh fruits and vegetables in the diet

A nurse has assisted the health care provider with a liver biopsy, which was done at the bedside. Upon completion of the procedure, the nurse assists the client into which of the following positions?

Right side-lying with a small pillow or towel under the puncture site

An older client with advanced Alzheimer's disease is placed in balanced suspension traction, and the health care provider expects to internally fixate the client's femur in 1 week. Based on this information, the nurse determines that the first priority relates to addressing which of the following nursing diagnoses?

Risk for constipation

A nurse is caring for a client who is hearing-impaired and takes which approach to facilitate communication?

Speaks in a normal tone

A nurse is assisting in preparing a teaching plan for a client with Ménière's disease. The nurse places highest priority on teaching the client information related to:

Safety

A nurse is assisting in developing a plan of care for the client scheduled for cataract surgery. The nurse makes suggestions regarding the plan, knowing that which problem is specifically associated with this type of surgery?

Sensory perceptual alteration

A client has clear fluid leaking from the nose after a basilar skull fracture. The nurse determines that this is cerebrospinal fluid (CSF) if the fluid:

Separates into concentric rings and tests positive for glucose

A client with diabetes mellitus has had a right below-knee amputation. The nurse would be especially vigilant in monitoring for which of the following because of the client's history of diabetes mellitus?

Separation of wound edges

A nurse is collecting data from a client who is experiencing the typical clinical manifestations of tuberculosis (TB). The nurse would expect the client to report having symptoms of fatigue and cough that have been present for:

Several weeks to months

A client tells the nurse she completed an educational program to manage her stress incontinence but is now discouraged. Which information from the client indicates the need for further teaching? Select all that apply.

She performs the Kegel exercises every other day She quit drinking coffee with cream but drinks diet cola She has begun an exercise program that includes lifting weights

An client is brought to the emergency department via ambulance after sustaining a fall. An x-ray indicates that the client sustained a femoral neck fracture. The nurse should anticipate which of the following on inspection of the client's leg?

Shortening, adduction, and external rotation

An emergency department nurse is caring for a client who sustained a blunt injury to the chest wall. Which sign, if noted in the client, would indicate the presence of a pneumothorax?

Shortness of breath

A client has undergone subtotal gastrectomy and the nurse is preparing the client for discharge. Which item should be included when teaching the client about ongoing self-management?

Smaller, more frequent meals should be eaten

A nurse is attempting to communicate with a hearing-impaired client. Which of the following strategies by the nurse would be least helpful when talking to this client?

Smiling continuously during conversation

A client who has had spinal fusion and insertion of hardware is extremely concerned about the perceived lengthy rehabilitation period. The client expresses concerns about finances and the ability to return to work. The nurse understands that the client's needs could best be addressed by referral to the:

Social worker

A client is diagnosed with polycystic kidney disease, and the nurse provides information to the client about the treatment plan. The nurse determines that the client needs further teaching if the client states that which component is part of the treatment plan?

Sodium restriction

A nurse is gathering data on a client with a diagnosis of tuberculosis (TB). The nurse reviews the results of which diagnostic test that will confirm this diagnosis?

Sputum culture

A nurse is explaining how sound is conducted from the middle ear to the inner ear in teaching a client who is experiencing hearing loss. The nurse plans to use a diagram that illustrates how which of the following bones connects to the cochlea at the oval window?

Stapes

A nurse provides cast application instructions to a client who is going to have a plaster cast applied. The nurse determines that the client needs further instruction if the client states that:

The client may bear weight on the cast in 30 minutes.

A client with acute pyelonephritis is scheduled for a voiding cystourethrogram. Which information about this procedure should the nurse give to the client?

The client must void while the micturition process is filmed

A nurse is monitoring a client following cardioversion. Which of the following observations would be of highest priority to the nurse?

Status of airway

A nurse stops at the scene of an automobile accident to assist a victim. The victim complains of severe leg pain, is unable to get out of the automobile, and is frightened. The appropriate nursing action is to:

Stay with the Victim

A nurse is one of several people who witness a vehicle hit a pedestrian at a fairly low speed on a small street. The individual is dazed and tries to get up, and the leg appears fractured. The nurse would plan to:

Stay with the person and encourage the person to remain still.

A nurse is suctioning an adult client through a tracheostomy tube. During the procedure, the nurse notes that the client's oxygen saturation by pulse oximetry is 89%. The nurse would:

Stop the suctioning procedure.

A client has slight weakness in the right leg. Based on this information, the nurse determines that the client would benefit most from the use of a:

Straight-leg cane

A client is admitted to the hospital with possible rheumatic endocarditis. The nurse would check the client for signs and symptoms of concurrent:

Streptococcal infection

A long-term care nurse notes that a female client has leakage of urine when sneezing, coughing, or laughing. The nurse reports that this client has which type of incontinence?

Stress incontinence

A nurse is assisting in caring for the client immediately after removal of the endotracheal tube following radical neck dissection. The nurse interprets that which of the following signs experienced by the client should be reported immediately to the registered nurse (RN)?

Stridor

A nurse is preparing for the admission of a client with a prescription for seizure precautions. Which supplies will the nurse make available to this client? Select all that apply.

Suction machine Oxygen administration Padding for the side rails Prescribed diazepam (Valium)

A nurse is assisting in the care of a client diagnosed with rheumatic heart disease. When teaching the client about self-management of this health problem, the nurse reminds the client to alert his dentist about the condition because:

The client requires prophylactic antibiotics before treatment.

A licensed practical nurse (LPN) is helping a registered nurse (RN) conduct an abdominal assessment. The LPN assists the client into which of the following positions?

Supine with the head raised slightly and the knees slightly flexed

A nurse is preparing to care for a client who will be arriving from the recovery room after an above-the-knee amputation. The nurse ensures that which priority item is in the client's hospital room?

Surgical tourniquet

A nurse is monitoring a client for the early signs and symptoms of dumping syndrome. Which of the following symptoms indicate this occurrence?

Sweating and pallor

A nurse is reviewing the health care record of a client suspected of having mastoiditis. Which finding does the nurse expect to note if this disorder is present?

Swelling behind the ear

A nurse is preparing to teach a client how to safely use crutches. Before initiating the teaching the nurse collects data on the client. The priority data would include which of the following?

The client's vital signs, muscle strength, and previous activity level of the client

A client is being discharged from the ambulatory care unit following cataract removal. Which instruction from the discharge teaching plan should the nurse reinforce?

Take acetaminophen (Tylenol) if any discomfort occurs.

A nurse is providing home care instructions to a client following a gastric resection. The nurse includes which of the following in the instructions?

Take actions to prevent dumping syndrome

A licensed practical nurse (LPN) is preparing to assist a registered nurse (RN) with removing a nasogastric (NG) tube from the client. The LPN would instruct the client to do which of the following?

Take and hold a deep breath.

The client's B-type natriuretic peptide (BNP) level is 691 pg/mL. Which of the following should the nurse institute when providing care for the client?

Take daily weights and monitor trends.

A client with angina pectoris who was given a first dose of newly prescribed nitroglycerin (Nitrostat) sublingual tablet complains of slight dizziness and headache. The nurse takes which action first?

Takes the client's blood pressure

A nurse is teaching the client with myasthenia gravis about prevention of myasthenic and cholinergic crises. The nurse tells the client that this is most effectively done by:

Taking medications on time to maintain therapeutic blood levels

A client with no history of heart disease has experienced an acute myocardial infarction and has been given thrombolytic therapy with tissue plasminogen activator (t-PA). The nurse interprets that the client is likely experiencing a complication of this therapy if which of the following occurs?

Tarry stools

A client being seen in the emergency department for complaints of chest pain confides in the nurse about regular use of cocaine as a recreational drug. The nurse takes which important action in delivering holistic nursing care to this client?

Teaches about the effects of cocaine on the heart and offers referral for further help

The nurse is explaining about antigens and antibodies when the client asks where antibodies come from. Which is an appropriate response? Select all that apply.

Tears Spleen Saliva Blood serum Lymph nodes

A nursing student is developing information for use in a clinical conference about hearing disorders. In the presentation, the student plans to include the statement that the ear is housed in which of the following bones of the skull?

Temporal

A client arrives in the emergency department following an eye injury from a chemical solution. Which of the following is the initial nursing action?

Test the eye pH with litmus paper.

A nurse is told in report that a client has a positive Chvostek's sign. What other data would the nurse expect to find on data collection? Select all that apply.

Tetany Diarrhea Possible seizure activity Positive Trousseau's sign

A nurse has given the client instructions regarding crutch safety. The nurse determines that the client needs reinforcement of the instructions if the client states:

That crutch tips will not slip, even when wet

A client has had extensive surgery on the gastrointestinal tract and has been started on parenteral nutrition (PN). The client tells the nurse, "I think I'm going crazy...I feel like I'm starving and yet that bag is supposed to be feeding me." The best response of the nurse would be:

That is because the empty stomach sends signals to the brain to stimulate hunger.

Which of the following identifies the route of transmission of tuberculosis (TB)?

The airborne route

A client had a Miller-Abbott tube inserted 24 hours ago. The nurse is asked to check the client to determine whether the tube is in the appropriate location at this time. Which of the following findings would indicate adequate location of the tube?

The aspirate from the tube has a pH of 7.45.

A client with tuberculosis (TB) asks a nurse about precautions to take after discharge from the hospital to prevent transmitting infection of others. The nurse develops a response to the client's question, based on the understanding that:

The disease is transmitted by droplet nuclei.

A client suspected of having an abdominal tumor is scheduled for a computed tomography (CT) scan with dye injection. The nurse tells the client which of the following about the test?

The dye injected may cause a warm, flushing sensation.

A nursing student is preparing to assist with an ear irrigation on an assigned client who has a buildup of cerumen in the left ear. The nursing instructor asks the student about the procedure for the irrigation. The instructor determines that the student understands this procedure if the student tells the instructor that:

The irrigating solution is warmed to 100° F.

The spouse of a client with acute kidney injury secondary to heart failure asks the nurse how this could happen. The nurse plans to base a response in part on the fact that which statement is true?

The kidneys generally require and receive about 20% to 25% of the resting cardiac output

A nurse is preparing to perform chest physiotherapy (CPT) on a client. Before determining the correct position in which to place the client, the nurse must ascertain:

The lung areas involved

The nurse is assisting in planning a teaching session with a client diagnosed with urethritis caused by infection with Chlamydia. The nurse should plan to include which point in the teaching session?

The most serious complication of this infection is sterility

A client has been newly diagnosed with glaucoma. As part of the discharge instructions, the nurse should plan to include which of the following?

The need for lifelong medication therapy

A client has been diagnosed with open-angle glaucoma and asks the nurse to repeat the health care provider's explanation of the disorder. The nurse would tell the client that:

The pressure increases within the eye from excess fluid or blocking of drainage.

A client reports the chronic use of nasal sprays. The nurse provides instructions to this client about which piece of information related to chronic use of nasal sprays?

The protective mechanism of the nose may be damaged

A nurse is collecting data about how well a client with a gastrointestinal (GI) disorder is able to absorb food. While carrying out this function, the nurse recalls that absorption is best defined as:

The transfer of digested food molecules from the GI tract into the bloodstream

A client has a closed-chest tube drainage system in place. The fluid in the water seal chamber rises and falls during inspiration and expiration. The nurse interprets that:

The tube is patent

A nurse is assigned to assist in caring for a client who has a pneumothorax. The nurse notes continuous bubbling in the water seal chamber of the client's closed-chest drainage system. The nurse determines that which of the following is occurring?

There is an air leak somewhere in the system.

A client has a newly fractured fibula that is plaster casted in the emergency department. Because the client will need to use crutches, the nurse plans to teach the client which crutch-walking gait before discharge?

Three-point gait

A client is taking large doses of acetylsalicylic acid (aspirin) for rheumatoid arthritis. The nurse tells the client to report which of the following signs and symptoms of ototoxicity?

Tinnitus, hearing loss, dizziness, ataxia

The nurse is preparing a teaching plan for a client who is undergoing cataract extraction with intraocular implant. Which home care measures will the nurse include in the plan? Select all that apply.

To avoid activities that require bending over To place an eye shield on the surgical eye at bedtime To contact the surgeon if a decrease in visual acuity occurs To take acetaminophen (Tylenol) for minor eye discomfort

A nurse is providing home care instructions to a client following a fenestration procedure for the treatment of otosclerosis. The nurse tells the client:

To increase fluids and take a stool softener daily

A client just diagnosed with toxoplasmosis asks the nurse, "What is toxoplasmosis? How did I get it, and what do I have to do to get rid of it"? Which information should the nurse include in the reply? Select all that apply.

Toxoplasmosis is treated with sulfadiazine Pregnant people should not empty litter boxes Toxoplasmosis is an organisms found in rare pork Toxoplasmosis may cause a severe inflammatory response

A hospitalized client is dyspneic and has been diagnosed with left pneumothorax by chest x-ray. Which of the following signs or symptoms observed by the nurse clearly indicates that the pneumothorax is rapidly worsening?

Tracheal deviation to the right

nurse answers the call bell of a client who had insertion of an internal cervical radiation implant. The client states that the implant fell out, and the nurse sees it lying in the bed after moving back the sheet. Which of the following actions should the nurse take first?

Use a long-handled forceps to place the implant in a lead container.

A nurse is teaching the client with a below-the-knee amputation (BKA) measures to protect the residual limb, or stump. The nurse would be sure to include which of the following points in discussions with the client?

Use a mirror to inspect all areas of the residual limb.

A client with a tracheostomy gets easily frustrated when trying to communicate personal needs to the nurse. The nurse determines that which of the following methods for communication may be the easiest for the client?

Use a picture or word board.

A nurse is evaluating the effect of dietary counseling on the client with cholecystitis. The nurse determines that the client understands the instructions given if the client states that which food item is acceptable to include in the diet?

Turkey and lettuce sandwich

A nurse assigned to care for a hearing-impaired client should use which approach to communication in order to enhance communication and preserve the client's self-esteem? Select all that apply.

Turning down the volume on the radio or TV when talking Standing directly in front of the client while speaking Speaking slowly and clearly

A nurse has reviewed activity restrictions with a client who is being discharged following hip surgery and insertion of a femoral head prosthesis. The nurse determines that the client understands the material presented if the client states that it is acceptable to:

Use a raised toilet seat.

A nurse instructs the client in breast self-examination (BSE). The nurse instructs the client to lie down and examine the left breast. The nurse instructs the client that while examining the left breast, to place a pillow:

Under the left shoulder

Which intervention should be implemented for the older client with presbycusis who has a hearing loss?

Use low-pitched tones

A client with ascites is scheduled for a paracentesis. The nurse is assisting the health care provider in performing the procedure. Which of the following positions will the nurse assist the client to assume for this procedure?

Upright

A client with a history of prostatic hypertrophy has purchased the over-the-counter medication, diphenhydramine (Benadryl), to treat symptoms of a runny nose. The nurse explains to the client that this medication combined with prostatic hypertrophy could cause exacerbation of which symptom?

Urinary retentio

Which instruction is appropriate for the nurse to initially provide to a client who reports via telephone that he is certain an insect has flown into his ear because he can hear it "buzzing"?

Use a flashlight to coax the insect out of the ear.

The nurse is providing general information to a group of high school students about preventing human immunodeficiency virus (HIV) transmission. The nurse should inform the students that which behavior is unsafe?

Use of natural skin condoms

A client with prostatitis resulting from kidney infection has received instructions on management of the condition at home and prevention of recurrence. Which statement indicates that the client understood the instructions?

Use warm sitz baths and analgesics to increase comfort

A nurse is reinforcing information regarding chemotherapy with a client who has been diagnosed with cancer. The nurse tells the client that an advantage of continuous intravenous (IV) chemotherapy is that it:

Uses smaller doses to kill cancer cells, so it is less toxic to normal tissues

For a client diagnosed with pulmonary edema, the nurse establishes a goal to have the client participate in activities that reduce cardiac workload. Which of the following client activities will contribute to achieving this goal

Using a bedside commode for stools

A nurse is administering mouth care to an unconscious client. The nurse should avoid doing which of the following?

Using products with lemon or alcohol

A client with a diagnosis of rapid rate atrial fibrillation asks the nurse why the health care provider is going to perform carotid massage. The nurse responds that this procedure may stimulate the:

Vagus nerve to slow the heart rate

A client with angina complains that the anginal pain is prolonged and severe and occurs at the same time each day, most often in the morning. On further data collection, the nurse notes that the pain occurs in the absence of precipitating factors. This type of anginal pain is best described as:

Variant angina

The low-pressure alarm sounds on the ventilator. The nurse checks the client and then attempts to determine the cause of the alarm but is unsuccessful. Which initial action will the nurse take?

Ventilate the client manually.

The low-exhaled volume (low-pressure) alarm sounds on a ventilator. A nurse rushes to the client's room and checks the client to determine the cause of the alarm but is unable to do so. The immediate nursing action is to:

Ventilate the client with a resuscitation bag

A nurse is admitting a client to the nursing unit who is suspected of having tuberculosis (TB). The nurse plans to admit the client to a room that has:

Venting to the outside, six air exchanges per hour, and ultraviolet light

A nurse is assisting a health care provider with the insertion of an endotracheal tube (ETT). The nurse should plan to ensure that which of the following is done as a final measure to determine correct tube placement?

Verify placement by a chest x-ray

A nurse plans dietary measures for a client with osteomalacia, knowing that the client is deficient in which of the following?

Vit D

A client with angina pectoris has just been started on medication therapy with nitroglycerin (Nitrostat). In planning care for this client, the nurse would place priority on measuring:

Vital signs

A nurse is caring for a client with a diagnosis of chronic gastritis. The nurse anticipates that this client is at risk for which vitamin deficiency?

Vitamin B12

A nurse is providing discharge instructions to a client who has had a total gastrectomy. The nurse tells the client about the importance of returning to the health care clinic as scheduled for which priority assessment?

Vitamin B12 and folic acid studies

A client has been diagnosed with chronic gastritis and has been told that there is too little intrinsic factor being produced. The nurse tells the client about the need for:

Vitamin B12 injections

The nurse is caring for a 58-year-old client with renal failure who is on peritoneal dialysis. Which finding is considered most important by the nurse, requiring health care provider notification? Refer to chart.

WBC 15,000 cells/mL

The nurse prepares the client for ear irrigation as prescribed by the health care provider. In performing the procedure, the nurse:

Warms the irrigating solution to 98°

A client with leukemia who had a bone marrow aspiration is thrombocytopenic. The nurse gives which of the following instructions to the family as the client is discharged to home?

Watch the puncture site for bleeding for the next several days.

A client with diplopia has been taught to use an eye patch to promote better vision and prevent injury. The nurse determines that the client understands how to use the patch if the client states that he or she will:

Wear the patch continuously, alternating eyes each day.

The nurse prepares to give a bath and change the bed linens on a client with cutaneous Kaposi's sarcoma lesions. The lesions are open and draining a scant amount of serous fluid. Which should the nurse incorporate in the plan during the bathing of this client?

Wearing a gown and gloves

A nurse is assisting a client admitted to the hospital with pulmonary edema to prepare for discharge. The nurse would reinforce with the client the importance of complying with which of the following measures to prevent a recurrence?

Weigh self every morning before breakfast.

A client is admitted to an acute care facility with complications of celiac disease. Which question would be helpful initially in obtaining information for the nursing care plan?

What is your understanding of celiac disease?"

A nursing instructor asks a student about cochlear implants. The student understands that which clients are candidates for such a procedure? Select all that apply.

Who have a profound hearing loss in both ears Who received no benefit from conventional hearing aids

A client returns to the nursing unit after an above-the-knee amputation of the right leg. The nurse positions the client:

With the foot of the bed elevated

A client has just returned from the cardiac catheterization laboratory. The left femoral vessel was used as the access site. After returning the client to bed and conducting an initial assessment, the nurse assisting in caring for the client expects that the health care provider wrote a prescription for the client to remain on bedrest:

With the head of bed elevated no more than 30 degrees

A client with acute glomerulonephritis had a urinalysis sent to the laboratory. The report reveals that there is hematuria and proteinuria in the urine. The nurse interprets that these results are which?

consistent with glomerulonephritis

A client with a history of gastrointestinal upset has been diagnosed with acute diverticulitis. To aid the client in symptom management, the nurse suggests foods that are on which of the following diets

low-fiber diet


Set pelajaran terkait

NURS 3230 Chapter 31 Skin Integrity and Wound Care NCLEX

View Set

MCAT general and organic chemistry

View Set

Consumer Behavior: Chapter 8 Quiz Review

View Set

MTLB- MODULE 5 (HIV LAWS)- RA 8504

View Set

Ch04: Civil Liberties: Nationalizing the Bill of Rights

View Set

resistance training notes test 2

View Set

PED #1 (Chp 1, 2, 31, 32, 33, 34, 35, 36, 37, 38, 42)

View Set

Chapter 30: Anemia and Polycythemia Vera (QUESTIONS)

View Set